Download Current practice in internal medicine

Survey
yes no Was this document useful for you?
   Thank you for your participation!

* Your assessment is very important for improving the workof artificial intelligence, which forms the content of this project

Document related concepts

Dental emergency wikipedia , lookup

Prenatal testing wikipedia , lookup

Epidemiology of metabolic syndrome wikipedia , lookup

Adherence (medicine) wikipedia , lookup

Dysprosody wikipedia , lookup

List of medical mnemonics wikipedia , lookup

Transcript
Module №3. Current practice in internal medicine
Text test questions
1.
Which tissue(s) or organ(s) should be evaluated in a man who begins to have fluid secretion from the
breast?
A.
Posterior pituitary and testes
B.
Adrenal medulla and adrenal cortex
C.
*Hypothalamus and anterior pituitary
D.
Parathyroid and islets of Langerhans
E.
Non of above
2.
Which situation or condition is likely to result in an increased production of thyroid hormones?
A.
Starvation
B.
Dehydration
C.
Adequate sleep
D.
*Cold environmental temperature
E.
Non of above
3.
Which hormone prevents hypoglycemia by promoting the conversion of glycogen to glucose
(glycogenolysis) and the conversion of protein to glucose (gluconeogenesis)?
A.
Testosterone
B.
Somatostatin
C.
*Glucagon
D.
Insulin
E.
Non of above
4.
Which test results should the physician check to ascertain how well the patient is managing her
diabetes mellitus overall?
A.
Blood glucose level
B.
Glucose tolerance test
C.
*Glycosylated hemoglobin
D.
Radioimmunoassay of insulin
E.
Non of above
5.
The risk for which endocrine problem increases with aging?
A.
Hyperthyroidism
B.
*Diabetes mellitus
C.
Diabetes insipidus
D.
Cushing’s disease
E.
Non of above
6.
For what complications is the patient with hypercortisolism at greater risk?
A.
*Skin breakdown, infection, GI ulceration
B.
Anorexia, constipation, hypotension
C.
Kidney stones, weight loss, cataracts
D.
Diabetes insipidus, bradycardia, arthritis
E.
Non of above
7.
Which serum electrolyte values alert the physician to the possibility of hyperaldosteronism?
A.
*Serum sodium 150 mmol/L, serum potassium 2.5 mmol/L
B.
Serum sodium 140 mmol/L, serum potassium 5.0 mmol/L
C.
Serum sodium 130 mmol/L, serum potassium 2.5 mmol/L
D.
Serum sodium 130 mmol/L, serum potassium 7.5 mmol/L
E.
Non of above
8.
Which of the following statements regarding thyroid disorders is true?
A.
The presence of a “goiter” always indicates hypothyroidism.
B.
*The effects of thyroid dysfunction are found in all body tissues and organs.
C.
The hormones synthesized by the thyroid gland are not essential for life.
D.
The effects of hypothyroidism are less serious than those of hyperthyroidism.
E.
Non of above
9.
Which manifestation of Graves’ disease is unaffected by medical treatment for the hyperthyroidism?
A.
Weight loss
B.
Thinning hair
C.
*Exophthalmos
D.
Tachycardia
E.
Non of above
10.
Which statement regarding diabetes mellitus is true?
A.
Diabetes increases the risk for development of epilepsy.
B.
The cure for diabetes is the administration of insulin only
C.
*Diabetes increases the risk for development of cardiovascular disease.
D.
Carbohydrate metabolism is disturbed in diabetes, protein and lipid metabolism are normal.
E.
Non of above
11.
How does glucagon assist in maintaining blood glucose levels?
A.
Glucagon enhances the activity of insulin, restoring blood glucose levels to normal more quickly after
a high-calorie meal.
B.
Glucagon is a storage form of glucose and can be broken down for energy when blood glucose levels
are low.
C.
Glucagon converts the excess glucose into glycogen, lowering blood glucose levels in times of
excess.
D.
*Glucagon prevents hypoglycemia by promoting glucose release from liver storage sites.
E.
Non of above
12.
What is the physiologic basis for the polyuria manifested by individuals with untreated diabetes
mellitus?
A.
Inadequate secretion of antidiuretic hormone (ADH)
B.
Early-stage renal failure causing a loss of urine concentrating capacity
C.
Chronic stimulation of the detrusor muscle by the ketone bodies in the urine
D.
*Hyperosmolarity of the extracellular fluids secondary to hyperglycemia
E.
Non of above
13.
What is the basic underlying pathology of diabetes mellitus?
A.
A disruption of the cellular glycolytic pathway
B.
An inability of the liver to catabolize glycogen
C.
*A failure to synthesize and/or utilize insulin
D.
An inhibition of the conversion of protein to amino acids
E.
Non of above
14.
Which action should the physician teach the diabetic patient as being most beneficial in delaying the
onset of microvascular and macrovascular complications?
A.
*Controlling hyperglycemia
B.
Preventing hypoglycemia
C.
Restricting fluid intake
D.
Preventing ketosis
E.
Non of above
15.
With which therapy for diabetes mellitus is the patient not at risk for hypoglycemia?
A.
Regular insulin
B.
Lente insulin
C.
*Biguanides
D.
Sulfonylureas
E.
Non of above
16.
Which nutritional group should the physician teach the diabetic patient with normal renal function to
rigidly control to reduce the complications of diabetes?
A.
*Fats
B.
Fiber
C.
Proteins
D.
Carbohydrates
E.
Non of above
17.
Which clinical manifestation in a patient with uncontrolled diabetes mellitus should the physician
expect as a result of the presence of ketoacid in the blood?
A.
*Increased rate and depth of respiration
B.
Non of above
C.
Extremity tremors followed by seizure activity
D.
Oral temperature of 102° F (38.9° C)
E.
Severe orthostatic hypotension
18.
Which arterial blood gas values indicate to the physician that the patient is experiencing
ketoacidosis?
A.
pH 7.38, HCO3– 22 mEq/L, PCO2 38 mm Hg, PO2 98 mm Hg
B.
*pH 7.28, HCO3– 18 mEq/L, PCO2 28 mm Hg, PO2 98 mm Hg
C.
pH 7.48, HCO3– 28 mEq/L, PCO2 38 mm Hg, PO2 98 mm Hg
D.
pH 7.28, HCO3– 22 mEq/L, PCO2 58 mm Hg, PO2 88 mm Hg
E.
Non of above
19.
What is the priority intervention for the patient having Kussmaul respirations as a result of diabetic
ketoacidosis?
A.
Administration of oxygen by mask or nasal cannula
B.
Intravenous administration of 10% glucose
C.
Implementation of seizure precautions
D.
*Administration of intravenous insulin
E.
Non of above
20.
Which nutritional problem should the physician be more alert for in older adult patients with diabetes
mellitus?
A.
Obesity
B.
*Malnutrition
C.
Alcoholism
D.
Hyperglycemia
E.
Non of above
21.
Why is ketosis rare in patients with type 2 diabetes, even when blood glucose levels are very high
(higher than 900 mg/dL)?
A.
Ketosis is less prevalent among obese adults.
B.
People with type 2 diabetes have normal lipid metabolism.
C.
*There is enough insulin produced by type 2 diabetes to prevent fat catabolism but not enough to
prevent hyperglycemia.
D.
Oral antidiabetic agents do not promote the breakdown of fat for fuel (lipolysis), and exogenous
insulin spares carbohydrates at the expense of fats.
E.
Non of above
22.
Which laboratory value indicates inadequate functioning of a transplanted pancreas?
A.
Total white blood cell count <5000/mm3
B.
*50% decrease in urine amylase level
C.
Blood urea nitrogen >30 mg/dL
D.
Elevated bilirubin level
E.
Non of above
23.
Which of these factors can cause Addison’s disease
A.
Brain tumor
B.
Diabetes Insipidus
C.
Autoimmune destruction of thyroid gland
D.
Tumor acting on adrenal hormone
E.
*Tuberculosis
24.
Which of these hormones is not produced by adrenal cortex:
A.
Hydrocortisone.
B.
Aldosterone.
C.
*Epinephrine.
D.
Dehydroisoandrosterone.
E.
None above them
25.
The most important laboratory investigation in diagnostics of chronic adrenal insufficiency is:
A.
*The level of 17 – OCS in urine
B.
The level of 17 – CS in urine
C.
Glucose tolerance test
D.
The level of sodium and potassium in blood
E.
Test with ACTH
26.
The most prominent feature of pheochromocytoma is:
A.
Weight loss.
B.
*Hypertension.
C.
Nausea.
D.
Hyperpigmentation.
E.
Headache.
27.
In patient with well-defined hyperpituitarism can be found:
A.
*Fasting hyperglycemia; and diminished glucose tolerance
B.
Fasting hyperglycemia but increased glucose tolerance
C.
Normal blood-sugar and sugar tolerance
D.
Hypoglycemia
E.
High level of blood sugar
28.
Which of those laboratory data is metabolite of epinephrine:
A.
17 - OCS
B.
17 - CS
C.
*VAA
D.
Norepenephrine
E.
Metoxineepinephrine
29.
Localization of pathological process in patients with Cushing’s syndrome is:
A.
The anterior pituitary.
B.
The posterior pituitary.
C.
Hypothalamus.
D.
*The adrenal cortex.
E.
The adrenal medulla.
30.
Cushing`s syndrome is characterized by all signs, EXCEPT:
A.
Dysplastic obesity
B.
Stria ruberorum
C.
Hyperglycemia
D.
Osteoporosis
E.
*Hypotension
31.
Biological action of cortisone does NOT include:
A.
Activation of gluconeogenesis
B.
*Decreasing of glycogen synthesis in the liver
C.
Stimulation of glucose transfer to muscles
D.
Antiallergic and anti-inflammatory action
E.
Regulation of blood pressure
32.
In a person who suspected Cushing’s syndrome, the diagnosis of functioning adrenal carcinoma
would be suggested by all, EXCEPT:
A.
A palpable abdominal mass
B.
Markedly increased urinary excretion of 17-ketosteroids
C.
*High plasma levels of ACTH
D.
Failure to suppress 17-hydrocorticosteroid secretion with high-dose dexamethasone
E.
Failure to suppress urine free cortisol with low-dose dexamethasone
33.
These are signs of Addison’s disease EXEPT:
A.
Hyperpigmentation
B.
*Hyperglycemia
C.
Hypotension
D.
Hyperpotassiemia
E.
Dyspeptic signs
34.
All these hormones are glucocorticoids, EXCEPT:
A.
Corticosteron
B.
Cortisone
C.
Cortizol
D.
11-dehydrocorticosteron
E.
*Norepinephrine
35.
What sign will be present in patient with primary adrenal failure and absent in patient with secondary
one?
A.
Nausea
B.
Weight loss
C.
Hypotension
D.
Tachycardia
E.
*Hyperpigmentation
36.
Where is localized tumor in patient with pheochromocytoma?
A.
Zona glomerulosa
B.
Zona fasciculate
C.
Zona reticularis
D.
*Medulla
E.
Kidney
37.
Which hormone is overproduce in patients with pheochromocytoma?
A.
Aldosterone
B.
Renin
C.
*Epinephrine
D.
Cortisol
E.
Dophamine
38.
Pheochromocytoma is characterized by:
A.
Hypotension
B.
*Very high blood pressure
C.
High only systolic pressure
D.
High only diastolic pressure
E.
Increased pulse pressure
39.
A 11-yr-old girl has Conn's syndrome, which causes hyperreninism. What kind of electrolyte
disturbance is present here?
A.
Hyperkalaemia
B.
Hyperchloraemia
C.
Hyponatraemia
D.
Hypercalcaemia
E.
*Hypokalaemia
40.
In pheochromocytoma the following feature is characterized by all signs EXEPT:
A.
Hypertension
B.
One side tumor
C.
*Hypoglycemia
D.
High level of catecholamines
E.
Hyperglycemia
41.
Anabolic action of glucocorticoids can be found in such organs or tissues:
A.
*Liver
B.
Muscles
C.
D.
E.
42.
A.
B.
C.
D.
E.
43.
A.
B.
C.
D.
E.
44.
A.
B.
C.
D.
E.
45.
A.
B.
C.
D.
E.
46.
A.
B.
C.
D.
E.
47.
A.
B.
C.
D.
E.
48.
A.
B.
C.
D.
E.
49.
A.
B.
C.
D.
E.
50.
A.
Connective tissue
Fat tissue
Lymphatic tissue
Regulation of water and electrolyte balance in organism is made by:
Progesterone
Estrogen
*Aldosterone
Insulin
Parathyroid hormone
Biological effect of cortisol is NOT include:
Increasing of gluconeogenesis
Anti-inflammatory action
Antialergic action
Hyperglycemic action
*Hypotension
Mineralocorticoids have such biological effect:
*Excretion of potassium from organism
Excretion of sodium from organism
Excretion of chlorides from organism
Reabsorbtion of water
Excretion of phosporus from organism
Pathogenesis of hypotensive syndrome development in patients with Addison’s disease:
Dehydration
Decreased quantity of sodium
Decreased blood circulating volume
Decreased sensitivity of vessel’s adrenoreceptors for katecholamine action
*All of them
Hyperglycemic action of glucocoricoids will be due to all effects EXEPT:
Stimulation of gluconeogenesis
Activation of fat destruction
Decreased oxidation of glucose in tissues
*Increased reabsorbtion of glucose in the intestine
Decreased synthesis of glycogen in muscles
In primary hypocorticism disease the following feature is NOT seen:
Hypotension
*Hypertension
Weight loss
Hyperpigmentation
Hyponatremia
The best diagnostic test for pheochromocytoma is:
Ultrasound of abdomen
*Estimation of urinary metabolites
Phentolamine test
Biochemical blood analysis
None above them
The prominent feature of Addison’s disease is:
*Hypotension
Hypertension
Hypokalemia
Weight gain
Hypernatremia
We can found adrenal reserve by infusion of:
Glucocorticoids
B.
C.
D.
E.
51.
A.
B.
C.
D.
E.
52.
A.
B.
C.
D.
E.
53.
A.
B.
C.
D.
E.
54.
A.
B.
C.
D.
E.
55.
A.
B.
C.
D.
E.
56.
A.
B.
C.
D.
E.
57.
A.
B.
C.
D.
E.
58.
A.
B.
C.
D.
E.
59.
*ACTH
Metyrapone
Insulin
5 % glucose solution
The main cause of Ectopic ACTH Syndrome is:
Renal cell carcinoma
Lymphoma
*Bronchogenic carcinoma
Pituitary adenoma
None above them
Where is aldosterone synthesize?
*Zona glomerulosa
Zona fasciculate
Zona reticularis
Medulla
Kidney
Where are catecholamines synthesize?
Zona glomerulosa
Zona fasciculate
Zona reticularis
*Medulla
Kidney
Where are mineralocorticoids synthesize?
*Zona glomerulosa
Zona fasciculate
Medulla
Zona reticularis
Kidney
Where are glucocorticoids synthesize?
Zona glomerulosa
*Zona fasciculate
Zona reticularis
Medulla
Kidney
Where are androgens synthesize?
Zona glomerulosa
Zona fasciculate
*Zona reticularis
Medulla
Kidney
Where is epinephrine synthesized?
Zona glomerulosa
Zona fasciculate
Zona reticularis
*Medulla
Kidney
Where is cortisol synthesized?
Zona glomerulosa
*Zona fasciculate
Zona reticularis
Medulla
Kidney
Where is dehydroepiandrosterone synthesized?
A.
B.
C.
D.
E.
60.
A.
B.
C.
D.
E.
61.
A.
B.
C.
D.
E.
62.
A.
B.
C.
D.
E.
63.
A.
B.
C.
D.
E.
64.
A.
B.
C.
D.
E.
65.
A.
B.
C.
D.
E.
66.
A.
B.
C.
D.
E.
67.
A.
B.
C.
D.
E.
Zona glomerulosa
Zona fasciculate
*Zona reticularis
Medulla
Kidney
Cushing's syndrome is NOT seen in:
Pituitary adenoma
Adrenal cancer
Oat-cell carcinoma of lung
*Medulloblastoma
None of them
Which is FALSE about Conn's syndrome:
*Generalized edema
Low renin levels
Decreased water reabsorption
Decreased level of potassium
Hypertension
Primary hyperaldosteronism does NOT have:
*Ankle edema
Polyuria
High blood pressure
Hypokalemia
Low renin levels
Which of the following is NOT seen in hyperaldosteronism:
High blood pressure
Decreased level of potassium
*Increase level of calcium
Polyuria
Increased level of sodium
Conn's syndrome all true EXCEPT:
Increased level of sodium
*Fluid retention/edema
Hypertension
Decreased level of potassium
Hyperaldosteronemia
The main feature of hyperaldosteronismis:
Hyperkalemia
Hyponatremia
Decreased water reabsorption
*Hypokalemia
Hypotension
Which of the following if present would RULE OUT Conn's syndrome
Elevated serum sodium level
Periodic paralysis
*Albuminuria
Malignant hypertension
Decreased potassium level
The features of primary adrenal failure is/are:
Weight loss
Skin pigmentation
Hyponatremia
Hyperkalemia
*All of the above
68.
A.
B.
C.
D.
E.
69.
A.
B.
C.
D.
E.
70.
A.
B.
C.
D.
E.
71.
A.
B.
C.
D.
E.
72.
A.
B.
C.
D.
E.
73.
A.
B.
C.
D.
E.
74.
A.
B.
C.
D.
E.
75.
A.
B.
C.
D.
E.
76.
A.
B.
C.
D.
Detection of urinary is the most important in the diagnosis of pheochromocytoma:
HIAA
*VMA
17-ketosteroids
MHPG
17-oxy-ketosteroids
What diagnostic test can you recommend for pheochromocytoma?:
Ultrasound
*Estimation of urinary metanephrines
Estimation of urinary HIAA
Phenotolamine test
Cortisolemia
Diagnostic tesst for pheochromocytoma include scan with:
Strontium
Phosphorus
I131
*MIBG
I128
The most common cause of Cushing's syndrome is:
Pituitary adenoma
Adrenal adenoma
Ectopic ACTH
*Iatrogenic steroids
All of them
Signs of Cushing syndrome will be present in such diseases:
Adrenal carcinoma
Oat cell carcinoma of lung
Pituitary adenoma
Adrenal adenoma
*All of them
Clinical features of Cushing's syndrome include everything EXCEPT:
Insulin resistance
*Menorrhagia
Violaceous striae
Central obesity
Hypertension
Primary adrenal failure disease is characterized by:
*Hypoglycemia
Hypertension
Hypokalemia
Edema
Hypernatremia
The most earlier manifestation of Cushing's syndrome is?
*Loss of diurnal variation
Increased ACTH
Increased plasma Cortisol
Increased urinary metabolites of Cortisol
Truncal obesity
The most common cause of primary adrenal insufficiency in INDIA is:
Autoimmune
Surgery
Steroid withdrawal
*Tuberculosis
E.
Sarcoidosis
77.
Features of primary adrenal insufficiency include all of the following EXCEPT:
A.
Asthenia
B.
Hyperpigmentation
C.
*Hypertension
D.
Abdominal pain
E.
Tachycardia
78.
Regarding Addisonian pigmentation, all are true EXCEPT
A.
Involves moles and scars
B.
Involves palmer creases
C.
*Does not involve oral mucosa
D.
Decreased fibrosis
E.
Involves areola of mammary glands
79.
Pheochromocytoma predominantly secretes:
A.
Epinephrine
B.
*Norepinephrine
C.
Dopamine
D.
DOPA
E.
None above them
80.
Pheochromocytoma characterized by all these features, EXCEPT:
A.
Hypertensive paroxysm
B.
Headache
C.
Orthostatic hypotension
D.
*Wheezing
E.
Tachycardia
81.
Pheochromocytoma characterized by all these features, EXCEPT:
A.
Increased hematocrit
B.
Orthostatic hypotension
C.
*Low cortisol level
D.
Impaired glucose tolerance
E.
Palpitation
82.
What type of diabetes is absent in classification of diabetes mellitus due to recommendations of
WHO, 1999?
A.
Diabetes mellitus type 1
B.
Diabetes mellitus type 2.
C.
Diabetes connected with other status and syndromes.
D.
Gestation diabetes.
E.
*Tropical diabetes.
83.
Which clinical sign is ABSENT in patients with neuropathic diabetic foot?
A.
Pink color of the skin
B.
*Decreased temperature of the skin
C.
Normal pulsation on visceral vessels
D.
Absence of sensibility
E.
Can be edema
84.
Development of steroid diabetes includes such mechanism as:
A.
Increased absorption of glucose in gastrointestinal tract.
B.
Increased distraction of glycogen in muscles.
C.
*Increased glyconeogenesis.
D.
Changes of reabsorption of glucose in nephron
E.
Increased lipogenesis
85.
Which organ or tissues is NOT responsive to insulin:
A.
Liver.
B.
Muscle.
C.
*Brain.
D.
Connective tissue.
E.
Adipocytes
86.
What you have to EXCLUDE from the mechanisms of insulin resistance:
A.
Influence of contrainsular hormones
B.
Antibodies to insulin
C.
Changes of hormonal receptors
D.
*Antibodies to b-cells
E.
Some endocrine diseases
87.
The peculiarities of ischemic heart disease in diabetics include everything EXEPT?
A.
Cardiovascular changes tend to occur earlier in patients with DM when compared with individuals of
the same age.
B.
Frequency of myocardial infarction (MI) and mortality is higher in diabetics than that in nondiabetics
of the same age.
C.
Often can observe atypical forms (without pain).
D.
Diabetic patients have more complications of MI than nondiabetic ones.
E.
*Male : female = 10: 1
88.
Macroangiopathy includes:
A.
Hypertensive disease
B.
*Ischemic heart disease.
C.
Nephropathy
D.
Encephalopathy
E.
Retinopathy
89.
The ischemic diabetic foot include all signs EXEPT:
A.
Dry gangrene
B.
Cyanotic color of the skin
C.
*Absence of sensibility
D.
Distant ulcers
E.
Decreased pulsation on visceral vessels
90.
The absolute insulin resistance takes place in a case of:
A.
Steroid diabetes.
B.
Type 2 diabetes.
C.
Gestation diabetes.
D.
Glucogonoma.
E.
*Pancreatectomy.
91.
The most important method to establish the stage of long – term compensation in diabetics is
checking of:
A.
The level of fasting glycemia.
B.
The level of postprandial glycemia.
C.
The level of glucosurea.
D.
*The level of HbA1C
E.
The level of the insulin in the blood.
92.
The preproliferative diabetic retinopathy does NOT include:
A.
Exudates
B.
Hemorrhages
C.
Macular edema
D.
*Neovascularization
E.
Microaneurisms
93.
The most prominent feature of V stage of diabetic nephropathy (by Mogensen):
A.
Blood hypertension
B.
Proteinurea
C.
Hyperpotassemia
D.
*Decreased GFR
E.
Hypoproteinemia
94.
The results of the glucose tolerance test: a fasting serum glucose is 5,2 mmol/l, 2-hour postprandial
serum glucose is 7,5 mmol/l. What is your diagnosis?
A.
*Normal.
B.
Impaired glucose tolerance.
C.
Diabetes mellitus.
D.
Impaired fasting glycemia.
E.
Neither
95.
The results of the glucose tolerance test: a fasting serum glucose is 6,5 mmol/l, 2-hour postprandial
serum glucose is 14,6 mmol/l. What is your diagnosis?
A.
Normal.
B.
Impaired glucose tolerance.
C.
Impaired fasting glycemia.
D.
*Diabetes mellitus.
E.
Neither.
96.
The most sensitive investigation for detecting of diabetic nephropathy is:
A.
Serum creatinine level
B.
Creatinine clearance
C.
Glucose tolerance test
D.
*Urine albumin
E.
Ultrasonography
97.
What can NOT be found in patients with nonproliferative (background) diabetic retinopathy?
A.
*Retinal detachment
B.
Increased capillary permeability
C.
Cotton-wool spots
D.
Microaneurysms
E.
Dot and blot hemorrhages
98.
The biological effects of insulin include:
A.
*Inhibition of proteolysis.
B.
Inhibition of glycogenesis.
C.
Stimulation of glycogenolysis.
D.
Stimulation of glyconeogenesis.
E.
Activation of lipolysis
99.
Macroangiopathy, as a symptom of diabetes mellitus, most often destroys vessels of:
A.
*Brain
B.
Lungs
C.
Kidneys
D.
Bowel
E.
Retina
100. The actions of insulin includes which one of the following?
A.
*Increased utilization of glucose by muscle
B.
Increased hepatic output of glucose
C.
Increased lipolysis
D.
Increased amino acid flux from muscle due to protein breakdown
E.
Activated glycogenolysis
101. Which test can you recommened as the most indicative of average recent blood glucose levels:
A.
Fasting serum glucose level
B.
Random serum glucose level
C.
Oral glucose tolerance test
D.
*Serum level of hemoglobin A1C
E.
Urine glucose concentration
102. The cause of sudden vision loss in patient with diabetic retinopathy can be due to:
A.
Cataract
B.
C.
D.
E.
103.
A.
B.
C.
D.
E.
104.
A.
B.
C.
D.
E.
105.
A.
B.
C.
D.
E.
106.
A.
B.
C.
D.
E.
107.
A.
B.
C.
D.
E.
108.
A.
B.
C.
D.
E.
109.
A.
B.
C.
D.
E.
110.
A.
B.
C.
D.
E.
111.
Glaucoma
*Vitreous haemorrhage
Papilloedema
Iritis
Which organ is the place of insulin primarily action?
*Muscle
Brain
Kidney
Adrenals
Bones
Impaired glucose tolerance test can lead to development:
Neuropathy
Nephropathy
Retinopathy
Obesity
*Ischemic heart disease
Proinsulin can best characterized as:
*Immunologicaly similar to insulin
More active than in insulin
Smaller than insulin
None above them
All of them
Destraction of how many % of b-cells mass can lead to hyperglycemia:
40 %
60 %
80 %
*90 %
100 %
In pregnant diabetic, the risk of intrauterine mortality increases after the ----week of gestation:
12th
26th
30th
*36th
40th
Charcot’s joint in diabetes is NOT seen in:
None of them
Knee
Hip
Tarsal
*Elbow
Which statement is true about Hb A1C:
Absent in normal people
A mutant of haemoglobin
Is a result of enzymatic degradation of glucose
Diagnostic criteria of diabetes
*Indicates average levels of glucose in blood
Diabetes mellitus type 1 in 95 % of patients is characterized by presence of:
HLA B27
HLA B3
HLA B4
*HLA DR3
HLA A3
The diabetic nephropathy is NOT characterized by:
A.
B.
C.
D.
E.
112.
A.
B.
C.
D.
E.
113.
A.
B.
C.
D.
E.
114.
A.
B.
C.
D.
E.
115.
A.
B.
C.
D.
E.
116.
A.
B.
C.
D.
E.
117.
A.
B.
C.
D.
E.
118.
A.
B.
C.
D.
E.
119.
A.
B.
C.
D.
E.
Cardiovascular abnormalities can occur
Micro- and macro- albuminuria can occur
ACE inhibitors can reduce microalbuminurea
*insulin requirement becomes high
Changes of GFR can be found
What is true in diabetes mellitus type 2?
Isilinites of beta-cells
*Hyperplasia of beta -cells
Hyalisation of beta -cells
Degeneration of beta -cells
Atrophy of beta -cells
The high level of C-peptide can be found in a case of:
Glucagonoma
*Insulinoma
Gastrinoma
Hepatoma
In all of them
Which index can differentiate factitious hyperinsulinemia from insulinoma:
*C – peptides
Insulin antibodies
Serum glucose levels
None
All of them
An adequate metabolic control of diabetes CAN’T prevent development of:
Microscopic albuminuria
*Amyotrophy
Increase leakage through retinal vessels
Peripheral neuropathy
None above them
Development of retinopathy more likely can be seen in:
Type 1 DM of 4 years duration
*Type 2 DM of 8 years duration
Gestational diabetes
Juvenile diabetes started before puberty
Any of above
Localization of necrobiosis lipoidica diabeticorum is most often can be found on:
Forearms
Face
*Front of legs
Back of legs
Abdomen
Which of the following HLA important in type 1 DM:
HLA-A3
HLA-B-27
*HLA DRW3, DRW4
HLA-W2
HLA-B-4
Where can be seen HLA linkage:
*Type1 DM
Type 2 DM
Secondary diabetes due to pancreatic disease
Diabetes due to insulin antagonist
Gestation diabetes
120.
A.
B.
C.
D.
E.
121.
A.
B.
C.
D.
E.
122.
A.
B.
C.
D.
E.
123.
A.
B.
C.
D.
E.
124.
A.
B.
C.
D.
E.
125.
A.
B.
C.
D.
E.
126.
A.
B.
C.
D.
E.
127.
A.
B.
C.
D.
E.
128.
A.
B.
C.
D.
Which statement is true in diabetes mellitus type 2?
Insulinitis of beta- cells
Hyalinization of beta- cells
Atrophy of beta cells
*Intact beta- cells
Degeneration of beta cells
Which statement is true in Diabetic Nephropathy:
Hypotension is typical
Erytrocyturia always occur
ACE inhibitors can’t reduce microalbuminuria
Insulin requirement becomes high
*Insulin requirement becomes low
The level of urinary albumin excretion refers to Microalbuminuria:
*30-300 mg/24 hour
400-600 mg/24 hour
700-900 mg/24 hour
>100 mg/24 hour
>1000 mg/24 hour
When we can say about Dawn phenomenon?
*Early morning hyperglycemia
Early morning hypoglycemia
Hypoglycemia followed by hyperglycemia
High insulin levels
Night hyperglycemia
Which type diabetes is HLA associated:
*Type 1 diabetes
Type 2 diabetes
Tropic diabetes
Gestation diabetes
All of them
In a patient with type 2 DM which of the following condition is seen:
Ketosis commonly occurs on stopping treatment.
Hyperlipidemia never occurs
Pancreatic beta cells stop producing insulin
*Increased levels of insulin in blood, may be seen
Low level or absence of C – peptide level in blood
HBA1C level in blood explains:
Acute rise of sugar
*Long terms status of blood sugar
Hepatorenal syndrome
Degree of severity of anemia
Chronic pancreatitis
The main cause of lactic acidosis is
Insulin deficiency
Dehydration
Extreme activity
*Hypoxia
Intercurrent infection
What is the most severe adverse event of biguanides?
Allergy;
Gastrointestinal tract disorders;
*Lactoacidosis
Hypoglycemia
E.
129.
A.
B.
C.
D.
E.
130.
A.
B.
C.
D.
E.
131.
A.
B.
C.
D.
E.
132.
A.
B.
C.
D.
E.
133.
A.
B.
C.
D.
E.
134.
A.
B.
C.
D.
E.
135.
A.
B.
C.
D.
E.
136.
A.
B.
C.
D.
E.
137.
A.
B.
C.
Resistance
All of following are oral hypoglycemic agents EXEPT:
Sulfonilureas
Biguanides
Thiazolidinediones
*Thyrostatics
Glinides
The main cause of hyperosmolar coma is:
Insulin deficiency
*Dehydration
Extreme activity
Hypoxia
Intercurrent infection
All of the following are principles of treatment of DM 2, EXEPT:
Achievement and maintenance of normal or reasonable body weight.
Normalization of metabolism and achievement of DM compensation.
*First line of therapy is insulin.
Maintenance (preservation) of working capacity.
Prophylaxis of acute and chronic complications.
A patient with DKA has deep sighing respiration. The main cause of Kussmaul breathing is:
Hyperglycemia
*Metabolic acidosis
Dehydration
Hypokalemia
Ketonemia
The shortest period of the action is present in such group of preparations as:
Thiazolidinediones
Biguanides
Sulfonilureas
Acarbose
*Glinides
Hypoglycemic coma can be caused by:
Insulin deficiency
Dehydration
*Extreme activity
Metabolic acidosis
Hypokalemia
Which of the following is NOT the contraindication for sulfonilureas?
Diabetic with excessive weight
Gestation diabetes
Diabetic with progressive weight loss
*Moderate degree of type 2 DM
Pregnancy and lactation
All of following are oral hypoglycemic agents EXEPT:
Sulfonilureas
Biguanides
Thiazolidinediones
*Phtorchinolones
Glinides
The actions of insulin include which one of the following?
Increased glycogenolysis
Increased hepatic output of glucose
Decreased lipogenesis
D.
Increased amino acid flux from muscle due to protein breakdown
E.
*Increased utilization of glucose by muscle
138. Diabetes mellitus is NOT associated with such complication:
A.
Encephalopathy
B.
Myopathy
C.
Angiopathy
D.
*Myelitis
E.
Neuropathy
139. The frequent causes of hypoglycemia in hospitalized diabetic patients include all the following
EXCEPT
A.
Insulin
B.
Sulfonylureas
C.
*Adrenal insufficiency
D.
Alcohol
E.
Renal failure
140. The most effective correction of acidosis in diabetic acidosis is:
A.
IV bicarbonate
B.
IV Saline
C.
Oral bicarbonate
D.
*IV Insulin
E.
IV potassium
141. Hypoglycemic effect of intermediate acting insulin lasts for:
A.
*12-16 hours
B.
20 -30 hours
C.
0.5 – 1.0 hours
D.
4 – 6 hours
E.
6 – 8 hours
142. Hyperosmolar nonketotic coma can be precipitated in diabetic patient by:
A.
Metoprolol
B.
Metformin
C.
*Furosemide
D.
Thyroxin
E.
All of them
143. Minimum amount of carbohydrate required to prevent ketonuria in a case of diabetes is about:
A.
10 gm/daily
B.
25 gm/daily
C.
*100 gm/daily
D.
150 gm/daily
E.
200 gm/daily
144. Which of the following oral anti-diabetic drugs can be used in patients with renal failure:
A.
Tolbutamide
B.
Chlorpropamide
C.
*Gliquidone
D.
Glipizide
E.
Metformin
145. Commonest cause of lactic acidosis during treatment of diabetes is:
A.
Diuretics
B.
Insulin treatment
C.
Sulfonylurea preparations
D.
*Biguanides
E.
Nonsulfonylurea insulin stimulators
146. Dawn phenomenon refers to:
A.
*Early morning hyperglycemia followed by hypoglycaemia
B.
C.
D.
E.
147.
A.
B.
C.
D.
E.
148.
A.
B.
C.
D.
E.
149.
A.
B.
C.
D.
E.
150.
A.
B.
C.
D.
E.
151.
A.
B.
C.
D.
E.
152.
A.
B.
C.
D.
E.
153.
A.
B.
C.
D.
E.
154.
A.
B.
C.
D.
E.
155.
Hypoglycaemia followed by hyperglycaemia
Early morning hypoglycaemia
High insulins levels
None above them
Insulin resistance is associated with all of the following status, EXEPT:
Acanthosis nigricans
Lipodystrophy
*Gout
Obesity
Metabolic syndrome
In which case the dose of insulin in stable diabetics can be reduced:
Thyrotoxicosis
Steroid therapy
Pregnancy
*Chronic renal failure
None above them
A patient with BMI 34 serum glucose 26 mmol/l, urinary ketones 4+ requires:
*Insulin
Glibenclamide
Metformin
Glimepiride
Acarbose
Which of the following is a contraindication for starting sulfonylurea therapy:
*Total pancreatectomy
DM type 2
Diabetes after 60 years
Decreased weight
None
Muscle weakness, bradycardia, hypotension, tall peaked T waves on ECG can be explained like:
Hypokalaemia
*Hyperkalaemia
Hypocalcaemia
Hypercalcaemia
Hyponatraemia
Muscle weakness and ectopic beats, flattened or inverted T waves on ECG can be explained like:
*Hypokalaemia
Hyperkalaemia
Hypocalcaemia
Hypercalcaemia
Hyponatraemia
Somogi phenomenon characterized by :
*Early morning hyperglycemia
Early morning hypoglycemia
Hypoglycemia followed by hyperglycemia
High insulin levels
Night hyperglycemia
Deaths from lactic acidosis in diabetes mellitus are associated with therapy:
Glibenclamide
Gliquidone
Chlorpropamide
*Phenformin
Glipizide
Diabetic ketoacidosis characterized by:
A.
Low serum potassium
B.
*Increased anion gap
C.
Metabolic alkalosis
D.
Respiratory acidosis
E.
Hypoglycemia
156. Diabetic coma is associated with:
A.
Ketosis
B.
*Hyperglycemia with hyperosmolar ketosis
C.
Simple hyperglycemia
D.
Hyperkalaemia
E.
Hyperosmolarity
157. A patient with BMI > 30 (kg/m2), serum glucose 24 mmol/L, urinary ketones ++++, he requires:
A.
*Insulin
B.
Glibenclamide
C.
Metformin
D.
Phenformin
E.
Oral bicarbonate
158. The following statements concerning diabetic ketoacidosis are correct EXCEPT:
A.
Pyrexia is uncommon even in the presence of infection
B.
*Leucocytosis is highly suggestive of associated infection
C.
Dehydration is present in all patients
D.
Low-dose insulin therapy is the treatment of choice
E.
Ketone bodies are present in urine
159. In patients with nonketotic hyperosmolar coma the level of glycemia is :
A.
Slightly elevated
B.
Mildly elevated
C.
Moderately elevated
D.
May be decreased
E.
*Grossly elevated
160. Hyperosmolar hyperglycemic non-ketonic coma is characterized by such level of glycemia:
A.
*55 mmol/1
B.
20 mmol/1
C.
30 mmol/1
D.
5 mmol/1
E.
25 mmol/1
161. Oral anti-diabetic drug of choice in renal failure is
A.
*Gliquidone
B.
Glimepiride
C.
Glipizide
D.
Metformin
E.
Pioglitasone
162. Hypoglycemic unawareness that occurs in diabetic patients when transferred from oral hypoglycemic
agents to insulin, is due to :
A.
*Autonomic neuropathy
B.
Insulin resistance
C.
Allergy
D.
Somogi phenomenon
E.
Down phenomenon
163. Which of the following diseases is characterized by hypoglycemia?
A.
Cushing's syndrome
B.
Acromegaly
C.
Hypothyroidism
D.
*Hypopitutarism
E.
Pheochromocytoma
164. Hypoglycemia can be seen in the following conditions, EXCEPT:
A.
Uremia
B.
*Acromegaly
C.
Addison's disease
D.
Hepatocellular failure
E.
Insulinoma
165. A 49-yr-old woman complains of perioral paraesthesia, carpopedal spasm and generalised seizures.
Put diagnosis of metabolic disturbances, please.
A.
Hypokalaemia
B.
Hyperkalaemia
C.
*Hypocalcaemia
D.
Hypercalcaemia
E.
Hyponatraemia
166. An 9-yr-old girl of Pakistan origin presents with bow legs. What kind of electrolyte disturbance is
present here?
A.
Hyperkalaemia
B.
Hyperchloraemia
C.
*Hypocalcaemia
D.
Hypernatraemia
E.
Hypokalaemia
167. A 63-yr-old woman is found to have a large toxic nodular goitre. What treatment of thyroid disorder
can you recommend?
A.
Radioactive iodine
B.
*Subtotal thyroidectomy
C.
Metaprolol
D.
Carbimazole
E.
Radiotherapy
168. A 16-year-old woman, complains of a soft diffuse swelling that moves with swallowing. Which
diagnostic procedure would you prescribe?
A.
Thyroid scan
B.
*TSH and T4
C.
Serum TSH
D.
Radioactive iodine uptake
E.
FNAC
169. A 22-year-old pregnant is diagnosed with Graves' disease shortly. Appropriate therapy includes:
A.
*Propylthiouracil therapy with the goal of maintaining her thyroid function tests in the highnormal
B.
Radioactive iodine to ablate her thyroid gland or slightly high range
C.
Methimazole therapy
D.
A beta blocker
E.
Propylthiouracil therapy with care taken to maintain her thyroid function tests in the midnormal range
170. The most important regulator of serum 1,25(OH)2 vitamin D concentration is
A.
Serum calcium
B.
Serum magnesium
C.
Serum 25(OH) vitamin D
D.
TSH
E.
*Parathyroid hormone
171. The most common presentation of primary hyperparathyroidism is:
A.
Bone fracture
B.
Increased serum creatinine
C.
Osteitis fibrosa cystica
D.
*Asymptomatic hypercalcemia
E.
Kidney stones
172. Which of the following conditions is LEAST likely to be associated with a low serum 25(OH)
vitamin D level?
A.
Dietary deficiency of vitamin D
B.
Chronic severe cholestatic liver disease
C.
Sedentary life-style
D.
*Chronic renal failure
E.
High-dose glucocorticoid therapy
173. All the following can be indications for parathyroidectomy in patients with hyperparathyroidism
EXCEPT
A.
*Advanced age
B.
Kidney stones
C.
Osteoporosis
D.
Calcium level >2.9 mmol/L
E.
Decreased creatinine clearance
174. Which of the following acts directly on the thyroid gland?
A.
ADH
B.
FSH
C.
ACTH
D.
*TSH
E.
TRH
175. Which of the following produces calcitonin?
A.
A (alpha) cells
B.
B (beta) cells
C.
*C (parafollicular) cells
D.
D (delta) cells
E.
F cells
176. Which of the following increases bone resorbtion?
A.
Calcitonin
B.
*Parathyroid hormone
C.
Thyroxin
D.
All of them
E.
Neither
177. To remove onset of tetany you will order
A.
*Calcium chloride
B.
Diuretics
C.
Prednisolone
D.
Potassium citrate
E.
Magnesium sulphate
178. The treatment of Grave’s disease usually include:
A.
Sulfonylureas
B.
Diuretics
C.
Narcotic analgesics
D.
Antidepressants
E.
*Thyrostatics
179. Which of the following is the most active form of thyroid hormone:
A.
Thyroxine (T4)
B.
*Triiodothyronine (T3)
C.
Thyrotropin
D.
Thyroglobulin
E.
Thyroid peroxidase
180. Which of the following is the main component of the colloid of the thyroid follicle?
A.
Thyroxine (T4)
B.
Triiodothyronine (T3)
C.
Thyrotropin
D.
*Thyroglobulin
E.
Thyroid peroxidase
181. Which of the following is the main product of the thyroid gland?
A.
*Thyroxine (T4)
B.
Triiodothyronine (T3)
C.
Thyrotropin
D.
Thyroglobulin
E.
Thyroid peroxidase
182. Which of the following is NOT associated with hypoparathyroidism:
A.
Muscle cramps
B.
Cataracts
C.
High serum phosphorus
D.
*Renal calculi
E.
Positive Chvostek sign
183. Such results of investigations as positive Chvostek sign and elevated serum parathyroid hormone
(PTH) level are associated with:
A.
*Pseudohypoparathyroidism
B.
Primary hypoparathyroidism
C.
Primary hyperparathyroidism
D.
Osteoporosis
E.
Hypercalcemia of malignancy
184. The principal role of the kidneys in the synthesis of the active form of vitamin D is:
A.
Conversion of 7-dehydrocholesterol to vitamin D3
B.
Absorption of vitamin D2
C.
Conversion of vitamin D to 25-OH vitamin D
D.
Absorption of vitamin D3
E.
*Conversion of 25-OH vitamin D to 1,25-(OH)2 vitamin D
185. Which one of the following is associated with increased bone resorbtion?
A.
Estrogens
B.
Calcitonin
C.
Bisphosphonates
D.
*Hyperparathyroidism
E.
Hypothyroidism
186. Bilateral subtotal of thyroidectomy for Graves’ disease most likely can cause hypoparathyroidism as
a result as:
A.
Loss of thyrocalcitonin
B.
Reduction in parathyroid stimulating hormone
C.
Removal of the four parathyroids
D.
*Infarction of the parathyroids
E.
Secondary hypopituitarism
187. Which of the following is the most accurate description of the findings in the hypoparathyroidism?
A.
The low serum calcium, spontaneous fractures
B.
*Low blood serum calcium, high blood serum phosphorus, tetany
C.
High blood serum calcium, tetany
D.
High blood serum calcium, low blood serum phosphorus
E.
None
188. Which of the following is the most characteristic eye sign in dysthyroid status?
A.
*Exophthalmus
B.
Ptosis
C.
Optic neuropathy
D.
Myopathy
E.
Retinopathy
189.
A.
B.
C.
D.
E.
190.
A.
B.
C.
D.
E.
191.
A.
B.
C.
D.
E.
192.
A.
B.
C.
D.
E.
193.
A.
B.
C.
D.
E.
194.
A.
B.
C.
D.
E.
195.
A.
B.
C.
D.
E.
196.
A.
B.
C.
D.
E.
197.
A.
B.
C.
D.
Thyrotoxicosis can be differentiated from anxiety neurosis clinically by:
Tachycardia
*Sleeping pulse rate
Moist hands
Ankle's oedema
Wide pulse pressure
Which of the following is the feature of hyperthyroidism?
Pericardial effusion
Menorrhagia
Delayed ankle relaxation
Constipation
*Heat intolerance
Which one of the following is NOT a feature of thyrotoxicosis:
Wide pulse pressure
Brisk tendon reflexes
Pretibial myxoedema
*Coarse dry skin
Sinus tachycardia
Cardiac manifestations of Grave's disease would include all of the following EXCEPT:
Wide pulse pressure
Atrial fibrillation
Pleuropericardial scratch
*Aortic insufficiency
Tachycardia
All clinical signs can be seen in ophthalmic Grave's disease EXEPT:
Lid retraction
*Frequent blinking
Poor convergence
Upper lid "lad" on down gaze
Wide opened eyes
Which of the following you will NOT prescribe in treatment of the thyroid crisis:
*Iodine
Iodides
Carbimazole
Propranolol
Glucocorticoids
The most common cause of thyrotoxicosis is:
*Grave's disease
Follicular adenoma
Multinodular goiter
Subacute thyroiditis
Iatrogenic
All of the following are used in the management of uncomplicated thyrotoxicosis EXCEPT:
Propylthiouracil
*Vasodilators
Radioactive iodine
Lugol's iodine
Propranolol
Best antithyroid drug to be used to pregnancy is:
Carbimazole
Thiamazole
*Propylthiouracil
Radiocative I
E.
198.
A.
B.
C.
D.
E.
199.
A.
B.
C.
D.
E.
200.
A.
B.
C.
D.
E.
201.
A.
B.
C.
D.
E.
202.
A.
B.
C.
D.
E.
203.
A.
B.
C.
D.
E.
204.
A.
B.
C.
D.
E.
205.
A.
B.
C.
D.
E.
206.
A.
B.
C.
Non of above
The most common cause of thyroid crisis is:
*Thyroid surgery
131I administration
Administration of Beta blockers
Thyrostatic therapy
All of the above
Treatment of choice of thyrotoxicosis during pregnancy is:
Anti-thyroid drugs
Resection of thyroid
*Propranolol
Radio iodine
Sedative drugs
The place of 25-hydroxycholecalciferol formation is:
*Liver
Kidney
Intestines
Pancreas
Brain
Which of the following is a feature of nephrocalcionsis:
*Primary hyperparathyroidism
Medullary cystic kidney
Vitamin C intoxication
Pseudohypoparathyroidism
Primary hypothyroidism
True statement about Hypercalcemia :
Radioiodine therapy of the primary cause is effective
Malignancy does not produce hypercalcemia
*I.V. fluid with Furosemide is given
Pamidronate is not effective.
I.V. fluid with Calcium chloride is effective
Such radiological findings as "hour-glass" vertebrae and "triturated pelvis" are seen in:
Thyrotoxicosis
Myxedema
Cretinism
*Hyperparathyroidism
Hypothyroidism
Hypercalcemia may be treated by:
IV pamidronate
IV fluids + furosemide
Glucocorticoids
None above them
*All of them
All statements are true in hyperparathyroidism EXCEPT:
May cause hypercalcemia
*Commonly occurs after thyroidectomy
Solitary adenoma in parathyroid is a common cause
Nephrolithiasis is common
Frequent fractures
Hypercalcaemia is NOT treated by:
Furosemide
Bisphosphonats
Plicamycin
D.
E.
207.
A.
B.
C.
D.
E.
208.
A.
B.
C.
D.
E.
209.
A.
B.
C.
D.
E.
210.
A.
B.
C.
D.
E.
211.
A.
B.
C.
D.
E.
212.
A.
B.
C.
D.
E.
213.
A.
B.
C.
D.
E.
214.
A.
B.
C.
D.
E.
215.
A.
B.
*Thiazides
IV fluids
The main cause of hypercalcemic crisis is :
*Parathyroid adenoma
CA Breast
Parathyroid hyperplasia
Paget's disease
Hyperthyroidism
The true statement of tertiary hyperparathyroidism is:
Primary hyperparathyroidism with decrease Ca+*level
*Secondary hyperparathyroidism with chief cell adenoma
Secondary hyperparathyroidism following intestinal malabsorption
Metastasis with normal phosphate level
Secondary hyperparathyroidism following chronic renal failure
Secondary hyperparathyroidism is associated with:
Parathyroid adenoma
Marked hypercalcaemia
*Chronic renal failure
Parathyroidectomy
All of them
All signs are associated with tetany EXEPT:
Chvostek’s sign
Trousseau's sign
Erb's sign
*Cole's sign
None above them
Which signs are associated with tetany EXEPT:
Chvostek’s sign
Trousseau's sign
Erb's sign
*All of them
None above them
All of the following are associated with Thyroid storm, EXCEPT:
*Development of thyroiditis
Surgery for thyrotoxicosis
Stressful illness in thyrotoxicosis
I131 therapy for thyrotoxicosis
All of them
A 32 year old young male shows an asymptomatic hypercalcemia. It can be caused by:
Occult primary malignancy
*Primary Hyperparathyroidism
Familial hypocalciuria
Hyper-nephroma
Hodgkin's lymphoma
Osteoporosis may be seen in all EXCEPT:
Hyperparathyroidism
*Hypoparathyroidism
Thyrotoxicosis
Sedentary lifestyle
Hypercorticism
Primary hyperparathyroidism most likely caused by:
Multiple parathyroid adenomas
*Solitary parathyroid adenoma
C.
Adrenal hyperplasia
D.
Ectopic PTH production
E.
None above them
216. Osteoporosis may be seen in such cases:
A.
Hyperparathyroidism
B.
Thyrotoxicosis
C.
Sedentary lifestyle
D.
Hypercortizism
E.
*All of them
217. A 32-yr-old woman presents with weight gain, constipation, lethargy and flaky rash. Put diagnosis,
please.
A.
De Quervain's thyroiditis
B.
*Hypothyroidism
C.
Hashimoto's thyroiditis
D.
Graves' disease
E.
Follicular carcinoma
218. Patient D. presents elevated serum T4 and increased radioactive iodine uptake. Put diagnosis, please.
A.
Non-toxic goiter
B.
Hashimoto's thyroiditis
C.
Subacute thyroiditis
D.
Hypothyroidism
E.
*Graves' disease
219. Patient R. with a neck painless mass presents normal T3 and T4. Put diagnosis, please.
A.
*Non-toxic goiter
B.
De Quervain's thyroiditis
C.
Subacute thyroiditis
D.
Hypothyroidism
E.
Graves' disease
220. A 72-yr-old woman with tingling and numbness in her fingers, obstipation and deafness. Put
diagnosis, please.
A.
Cerebral malignancy
B.
Hashimoto's thyroiditis
C.
Subacute thyroiditis
D.
*Hypothyroidism
E.
Graves' disease
221. A 21-yr-old female presents with elevated serum T4 and low radioactive iodine uptake. Put
diagnosis, please.
A.
Non-toxic goiter
B.
Hashimoto's thyroiditis
C.
*Subacute thyroiditis
D.
Hypothyroidism
E.
Graves' disease
222. A 43-yr-old woman presents with a hard, nodular midline neck mass. Blood tests reveal the presence
of antibodies to thyroglobulin. Put diagnosis, please.
A.
Thyroglossal cyst
B.
*Hashimoto's thyroiditis
C.
Acute thyroiditis
D.
Graves' disease
E.
Subacute thyroiditis
223. A 43-yr-old woman presents with a solitary nodule in the right thyroid lobe. FNAC suggests
follicular adenoma.What treatment of thyroid disorder can you recommend?
A.
*Total thyroid lobectomy
B.
Ablative dose of radioactive iodine
C.
Chemotherapy
D.
Thyroxine
E.
Subtotal thyroidectomy
224. A 21-yr-old man presents with a 4 cm solid mass in the left thyroid lobe. FNAC reveals papillary
carcinoma.What treatment of thyroid disorder can you recommend?
A.
Total thyroid lobectomy
B.
Ablative dose of radioactive iodine
C.
Chemotherapy
D.
*Total thyroidectomy and removal of central group of lymph nodes
E.
Subtotal thyroidectomy
225. A 16-yr-old girl presents with a 1 cm solitary thyroid nodule and diarrhea. FNAC is reported as
malignant. Serum calcitonin is raised. What treatment of thyroid disorder can you recommend?
A.
Total thyroid lobectomy
B.
Ablative dose of radioactive iodine
C.
Chemotherapy
D.
*Total thyroidectomy and removal of central group of lymph nodes
E.
Subtotal thyroidectomy
226. A 32-yr-old woman presents with a 2 cm thyroid nodule. FNAC suggests a colloid nodule. What can
you recommend?
A.
Total thyroidectomy and removal of central group of lymph nodes
B.
Ablative dose of radioactive iodine
C.
Chemotherapy
D.
*Reassure and repeat FNAC in 1 yr
E.
Subtotal thyroidectomy
227. A 18-yr-old student presents with a neck swelling. On examination the swelling moves up with
swallowing and protrusion of the tongue. Put diagnosis, please.
A.
*Thyroglossal cyst
B.
Hashimoto's thyroiditis
C.
Thyroid carcinoma
D.
Graves' disease
E.
Multinodular goitre
228. A 18-yr-old girl complains of anterior neck mass. It moves upward upon protrusion of tongue.
Thyroid radionucleotide scan shows no uptake in the midline. Put diagnosis, please.
A.
*Thyroglossal cyst
B.
Hashimoto's thyroiditis
C.
Thyroid carcinoma
D.
Graves' disease
E.
Multinodular goitre
229. Which hormone acts indirectly on the thyroid gland:
A.
ADH
B.
FSH
C.
ACTH
D.
TSH
E.
*TRH
230. A 37-yr-old man presents with a hard, nodular midline neck mass that moves upward on swallowing.
Thyroid radionucleotide scan shows cold spots. Put diagnosis, please.
A.
Thyroglossal cyst
B.
Hashimoto's thyroiditis
C.
*Thyroid carcinoma
D.
Graves' disease
E.
Multinodular goitre
231. What point, concerning myxedema coma is correct ?
A.
Feeling warm may provoke myxedema coma
B.
*Hypothermia is common
C.
Cause is – increased sensitivity to T 3 and T4 receptors.
D.
Hyperfunction of adrenal glands
E.
Thyroid cancer can provoke coma
232. Which substance is the active form of thyroid hormone:
A.
Reverse triiodothyronine (rT3)
B.
*Triiodothyronine (T3)
C.
Thyrotropin
D.
Thyroglobulin
E.
Thyroid peroxidase
233. Which of the following can help to put a diagnosis of subclinical hypothyroidism for a patient?
A.
Low thyroid hormone levels but no symptoms
B.
Classic symptoms of hypothyroidism but normal thyroid function test results
C.
Low free T4 levels but normal serum thyroid-stimulating hormone (TSH) level
D.
Low serum TSH but normal free T4 levels
E.
*Increased serum TSH but normal free T4 levels
234. Which substances aren’t components of the colloid of the thyroid follicle?
A.
Thyroxine (T4)
B.
Triiodothyronine (T3)
C.
*Thyrotropin
D.
Thyroglobulin
E.
Thyroid peroxidase
235. Patient T. presents very high concentration of antithyroid peroxidase antibody, it is most closely
associated with:
A.
Graves' disease
B.
Hypothyroidism
C.
Subacute thyroiditis
D.
*Chronic autoimmune thyroiditis
E.
Estrogen therapy
236. Which of the substances is the main product of the thyroid follicle?
A.
*Thyroxine (T4)
B.
Triiodothyronine (T3)
C.
Thyrotropin
D.
Thyroglobulin
E.
Thyroid peroxidase
237. Congenital hypothyroidism is NOT characterized by:
A.
Sexual retardation
B.
Growth retardation
C.
Retardation of bone age
D.
Mental retardation
E.
*Signs of hypopituitarism
238. The main cause of secondary hypothyroidism is:
A.
Inflammation of the thyroid gland
B.
*Hypopituitarism
C.
Thyrostatic therapy
D.
Iodine deficiency status
E.
Disorders of cells sensitivity to thyroid hormones
239. A young male patient presents a single painless nodule in the thyroid gland. The most advisable
management is:
A.
Observation for evidences of enlargement or until other nodules appear
B.
*Simple excision
C.
The administration of radioactive iodine
D.
Thyroidectomy –total
E.
240.
A.
B.
C.
D.
E.
241.
A.
B.
C.
D.
E.
242.
A.
B.
C.
D.
E.
243.
A.
B.
C.
D.
E.
244.
A.
B.
C.
D.
E.
245.
A.
B.
C.
D.
E.
246.
A.
B.
C.
D.
E.
247.
A.
B.
C.
D.
E.
248.
A.
B.
C.
Thyroxine
Which method or preparation is not used in patients with hypothyroid coma?
Levothyroxine
Corticosteroid therapy
Decreasing of hypoventilation
*Rehydration
Treatment the disorders which lead to the development of coma
All signs can be found in patients with hypothyroidism, EXEPT:
Dry scin
Hypercholesterolemia
Constipation
*Hyperglycemia
Edema
Patient F., 29 yr. old, presents with subacute thyroiditis. Treatment will include:
Antibiotics
Vitamins
Antithyroid drugs
Iodine preparations
*Corticosteroids
Endemic goiter as a rule is presented by:
Acute thyroiditis
*Diffuse goiter
Hyperthyroid
Solitary nodule
Subacute thyroiditis
In endemic goiter such statement will be true:
Investigations invariably show some degree of hypothyroidism
There is a clear evidence of an increased incidence of carcinoma of the thyroid
*Administration of thyroxine will often cause the goitre to shrink
Administration of iodine occasionally precipitates hyperthyroidism
The incidence after puberty is much higher in females
Radioiodine uptake in endemic goitre will show such result:
Normal
*Increased
Decreased
Erratic
Absent
A 42-yr-old woman presents thyrotoxicosis with decreased radioiodine uptake. Put diagnosis, please.
Acute infectious thyroiditis
Hashimoto's thyroiditis
Grave's disease
Toxic multinodular goiter
*De Quervain's thyroiditis
For chronic (autoimmune) thyroiditis all are true EXCEPT:
*Maximum incidence in children
Neutrophilic infiltration
Painless
None of the above
All of them
Subacute granulomatous thyroiditis is NOT characterized by:
Raise T4 levels
Raised ESR
Pain
D.
*High Radio iodine uptake
E.
Leucosytosis
249. A 72-yr-old-female was admitted to the hospital in unconscious status. Myxoedema coma was
diagnosed. What treatment can you prescribe?
A.
Hydrocortisone
B.
IV dextrose
C.
T3 injections
D.
*All of the above
E.
None above them
250. In patients with hypothyroidism such feature may NOT be seen:
A.
Cold intolerance
B.
Deafness
C.
Pericardial effusion
D.
Pretibial myxoedema
E.
*Diarrhea
251. Hypothyroidism is associated with the following clinical problems, EXCEPT:
A.
Menorrhagia
B.
Early abortions
C.
Galactorrhoea
D.
*Thromboembolism
E.
Constipation
252. All may be features of myxoedema, EXCEPT:
A.
Carpal tunnel syndrome
B.
Pericarditis
C.
Menorrhagia
D.
*Weight loss
E.
Creaky voice
253. Which of the following are NOT related to hypothyroidism?
A.
Coronary atherosclerosis
B.
Type III hyperlipoproteinaemia
C.
Pericardial effusion
D.
*Often cardiac tamponade
E.
Hypotension
254. Which is NOT a feature of hypothyroidism:
A.
Myxoedema
B.
Carpal tunnel syndrome
C.
Weight gian
D.
*Increased appetite
E.
Hoarseness of voice
255. Which amongst the following is the earliest indicator of hypothyroidism?
A.
Decreased serum T4
B.
Decreased serum T3
C.
Increased relaxation phase of deep tendon reflexes
D.
*High serum TSH
E.
High serum TRH
256. Which indicator can be used for monitoring of primary hypothyroidism treatment:
A.
T3
B.
T4
C.
*TSH
D.
TRH
E.
LH
257. Average replacement dose of Thyroxine in treatment of hypothyroidism is:
A.
*0.1-0.2 mg
B.
C.
D.
E.
258.
A.
B.
C.
D.
E.
259.
A.
B.
C.
D.
E.
260.
A.
B.
C.
D.
E.
261.
A.
B.
C.
D.
E.
262.
A.
B.
C.
D.
E.
263.
A.
B.
C.
D.
E.
264.
A.
B.
C.
D.
E.
265.
A.
B.
C.
D.
E.
266.
0.3-0.4 mg
1-2 mg
3-4 mg
100-200 mg
Presentation of hypothyroidism includes:
*Menorrhagia
Oligomenorrhoea
Amenorrhea
All of them
None above them
Which of the following is used in the treatment of the thyroid malignancy:
*I131
I125
Tc"
P32
MIBS
Which of the following is tumor marker of medullar carcinoma of thyroid gland:
T3
T4
TSH
*Calcitonin
PTH
Thyroid carcinoma with pulsative vascular skeletal metastasis is seen in:
Papillary
*Follicular
Medullary
Anaplastic
None above them
Thyroid carcinoma associated with hypocalcemia is
Follicular carcinoma
*Medullary carcinoma
Anaplastic carcinoma
Papillary carcinoma
All of them
Most common Thyroiditis is:
Reidel 's Thyroiditis
Subacute lymphocytic Thyroiditis
*Hashimoto's Thyroiditis
De Quervain's Thyroiditis
Acute Thyroiditis
All features are present in DeQuervan's disease, EXEPT:
*Autoimmune in etiology
Increased ESR
Tends to regress spontaneously
Painful & associated with enlargements of thyroid
Virus etiology
Screening method for medullary carcinoma thyroid is:
*Serum calcitonin
Serum calcium
Serum alkaline phosphate
Serum acid phosphatase
None above them
The most likely cause for the eating disorder anorexia nervosa is:
A.
B.
C.
D.
E.
267.
A.
B.
C.
D.
E.
268.
A.
B.
C.
D.
E.
269.
A.
B.
C.
D.
E.
270.
A.
B.
C.
D.
E.
271.
A.
B.
C.
D.
E.
272.
A.
B.
C.
D.
E.
273.
A.
B.
C.
D.
E.
274.
A.
B.
C.
D.
E.
Decreased levels of luteinizing hormonereleasing hormone (LHRH)
Decreased levels of growth hormone
Decreased levels of insulin-like growth factor I (somatomedin C)
*Psychiatric disorder
Decreased levels of serum thyroxine
Which of the following inhibits growth hormone secretion from the anterior pituitary gland?
Serotonin
Growth hormone-releasing hormone
Dopamine
Arginine
*Somatostatin
Target is the renal collecting tubules for:
*ADH
Dopamine
TSH
Somatostatin
Oxytocin
Targets include both ovaries and testes:
All of them
Dopamine
*FSH
Prolactin
Oxytocin
Target cells found in the adenohypophysis for:
FSH
Prolactin
*TRH
Oxytocin
None of the above
Target is the secretory cells of the mammary glands:
Dopamine
Somatotropin
Somatostatin
*Prolactin
Vasopressin
Developing of gigantism is conditioned by:
*Overwhelming secretion of GH in adolescence
Overwhelming secretion of GH in old age
Overwhelming secretion of GH in adults
Overwhelming secretion of somatostatin in adolescence
Lack of sensitivity in tissues to GH
Which of the following NOT the feature of diabetes insipidus?
*Hyperglycemia.
Polyuria.
Polydipsia.
Low urine osmolality.
High serum osmolality.
Which of the following are the features of diabetes insipidus?
*All of them.
Polyuria.
Polydipsia.
Low urine osmolality.
High serum osmolality.
275.
A.
B.
C.
D.
E.
276.
A.
B.
C.
D.
E.
277.
A.
B.
C.
D.
E.
278.
A.
B.
C.
D.
E.
279.
A.
B.
C.
D.
E.
280.
A.
B.
C.
D.
E.
281.
A.
B.
C.
D.
E.
282.
A.
B.
C.
D.
E.
283.
A.
B.
C.
D.
Which of the following is NOT associated with obesity:
Hypertension
Diabetes mellitus
*Hyperuricemia
Atherosclerosis
Dyslipidemia
All of these signs may be present in patient with pituitary insufficiency, EXEPT:
Hypotension.
*Hyperpigmentation.
Weight loss.
Hypogonadism.
Hypothyroidism.
Most distressing symptom of prolactinoma is:
Sexual disorders
*Visual defects
Priapism
Galactorrhoea
None above them
Which of the following may play a role in cardiac enlargement in acromegaly:
Diabetes mellitus
Hypertension
Increased work demand
*Direct pituitary hormone action on the heart
None above them
Treatment of diabetes insipidus include:
Loop diuretics
Oxytocin
*Desmopressin
Glibenclamide
Metformin
Treatment of diabetes insipidus include:
*Thiazides
Oxytocin
Bromcriptine
Glibenclamide
Thiamazole
Laboratory signs of central diabetes insipidus are:
Low plasma and low urine osmolality
High plasma and high urine osmolality
Low plasma and high urine osmolality
*Low urine and high plasma osmolality
None above them
Which of the following is NOT characterized by weight gain?
Hypothyroidism
Craniopharyngioma
Isulinoma
*Hyperthyroidism
Acromegaly
Which of the following is NOT the component of Laurence-Moon-Biedl syndrome?
Mental retardation
Digital anomalies
Hypogonadism
*Asthenic body built
E.
Obesity
284. The main cause of death in patients with morbid obesity is/are:
A.
*CVS complication
B.
Pickwickian syndrome
C.
Hypothyroid crisis
D.
Cushing syndrome
E.
Diabetic ketoacidosis
285. Klinefelter's syndrome is characterized by such chromosomal anomaly:
A.
*47 XXY
B.
45 XO
C.
47 XXX
D.
46 XX
E.
45 XXY
286. Weight gain can be present in all these cases, EXCEPT:
A.
Cushing syndrome
B.
Insulin secreting tumour
C.
*Pheochromocytoma
D.
Hypothyroidism
E.
Hypothalamic syndrome
287. Which of the following is characterized by polyurea with low fixed specific gravity:
A.
Diabetes mellitus
B.
*Diabetes insipidus
C.
CRF (chronic renal failure)
D.
Competitive water drinking
E.
Inappropriate secretion of ADH
288. Which of the following is NOT the sign of inappropriate ADH:
A.
*Hypoosmolar urine
B.
Water intoxication
C.
Expanded fluid volume
D.
Hypomagnesemia
E.
Hyperosmolar urine
289. Obesity is associated with all of the following EXCEPT:
A.
Hypothyroidism
B.
*Adrenal insufficiency
C.
Hypogonadism
D.
Cushing's syndrome
E.
Insulin secreting tumour
290. A patient F. presents with hypertension, abdominal strias, decreased level of potassium, and
hyperglycemia. The MOST appropriate screening test to rule out Cushing`s syndrome, is
A.
Retroperitoneal air study
B.
Response to intravenous ACTH
C.
Urinary 17-hydrocorticosteroid excretion
D.
Adrenal venography
E.
*Measurement of plasma cortisol after 1 mg of dexamethasone at midnight of previous night
291. A 42-yr-old female presents with hypertension, palpitations and sweating; 24 hr urinary VMA is
elevated. Put diagnosis, please.
A.
Parathyroid adenoma
B.
*Pheochromocytoma
C.
Prolactinoma
D.
Corticosteroma
E.
Follicular carcinoma thyroid
292. A 42-yr-old man has hypertension, hyperglycaemia, myopathy, thinning of the skin, buffalo hump
and truncal obesity. Prescribe investigations, please.
A. *Dexamethasone suppression test
B. ACTH stimulation test
C. Serum aldosterone
D. T3, T4 and TSH levels
E. Look at old radiographs
293. A 26-year-old male patient presents with hypertension, investigation - Na-148 mEq/1 K-1.45 mEq/I,
most probable diagnosis is:
A.
Pheochromocytoma
B.
*Conn's syndrome
C.
Cushing's syndrome
D.
Renal parenchymal disease
E.
Essential hypertension
294. Patient L., 49 years old, height 163 cm, weight 76 kg, level of fast (on an empty stomach) glucose is
6,3-7,2-8,7 mmol/l. What is the possible diagnosis?
A.
Normal.
B.
Impaired glucose tolerance.
C.
Diabetes mellitus type 1
D.
Impaired fasting glycemia.
E.
*Diabetes mellitus type 2
295. Diabetic K., 31 y-r-old, is pregnant. What laboratory data we have to control in pregnant diabetes,
EXCEPT fast glycemia?
A.
The level of postprandial glycemia.
B.
The level of glucosurea.
C.
*The level of glycated Hb
D.
The level of the insulin in the blood.
E.
The level of cholesterol.
296. 47 y-r-old obese female complains of thirst. The results of the glucose tolerance test: a fasting serum
glucose is 5,7 mmol/l, 2-hour postprandial serum glucose is 7,4 mmol/l. Put diagnosis, please?
A.
Normal.
B.
Impaired glucose tolerance.
C.
Diabetes mellitus.
D.
*Impaired fast glucose tolerance.
E.
Neither
297. A 31-year-old female is diagnosed with diabetes mellitus. Type 1 diabetes mellitus is usually
associated with which one of the following?
A.
Onset in middle age
B.
Obesity
C.
Insulin resistance
D.
*Requirement for insulin therapy
E.
Absence of autoimmune manifestations
298. A 37-year-old male is diagnosed with diabetes mellitus. Very high serum level of anti-islet cell
antibodies is most closely associated with:
A.
*Type 1 diabetes mellitus
B.
Type 2 diabetes mellitus
C.
Impaired fasting glucose
D.
Impaired glucose tolerance
E.
Diabetic ketoacidosis
299. A 47 y-r-old overweight man complains of paradontosis. The most sensitive test for diabetes mellitus:
A.
Fasting serum glucose level
B.
Random serum glucose level
C.
*Oral glucose tolerance test
D.
Serum level of hemoglobin A1C
E.
Urine glucose concentration
300. A 32-year-old female, insulin dependent diabetic, presents with failure to pass urine. Which
complication of diabetes can be found?
A.
Hyperglycaemia
B.
Hypoglycaemia
C.
Urinary tract infection
D.
Kidney calculi
E.
*Autonomic neuropathy
301. A 62-year-old diabetic on treatment for last 6 years, presents with calf pain exacerbated by
movement. Which complication of diabetes can be found?
A.
*Intermittent claudication
B.
Hypoglycaemia
C.
Atherosclerosis
D.
Somatic neuropathy
E.
Autonomic neuropathy
302. A 14-yr-old girl presents with 3 weeks of weight loss, polyuria and polydipsia. Prescribe
investigations, please.
A.
*Fasting blood glucose
B.
Urinary ketones
C.
T3, T4 and TSH levels
D.
HbAlc levels
E.
C-peptide levels
303. A patient has type 1 DM diagnosed at age of 14 years. The most reliable indicator for diabetic
nephropathy is:
A.
Urine albumin <30 mg per day in 3 consecutive samples
B.
*Urinary protein >550 mg per day for 3 consecutive samples
C.
Development of diabetic retinopathy
D.
Hematuria
E.
Leucocyturia
304. A 37 years old female has fasting and postprandial blood sugar within normal limit but urine sugar is
2 plus (++). The diagnosis is:
A.
*Renal Glycosuria
B.
Pancreatic failure
C.
Alimentary glycosuria
D.
High carbohydrate diet taken in the morning.
E.
Gestation diabetes
305. Patient L., 36 y-r-old, complains of polyuria, thirst. Plasma glucose concentration is 12,4 mmol/l.,
HLA DR3. HLA linkage is seen in :
A.
*Diabetes mellitus type 1
B.
Diabetes mellitus type 2
C.
Secondary diabetes due to pancreatic disease
D.
Diabetes due to insulin antagonist
E.
Gestation diabetes
306. A 71-year-old diabetic on treatment with Metformin presents with severe epigastric pain, drowsiness
and confusion. Which complication of diabetes can be found?
A.
Hyperglycaemia
B.
Hypoglycaemia
C.
*Lactic acidosis
D.
Gastitis
E.
Autonomic neuropathy
307. A 25 year old diabetic is admitted in a comatose state. His plasma glucose level is 2 mmol/l.What
treatment can you recommend?
A.
Insulin sliding scale, Heparin, 0.9% saline
B.
Insulin sliding scale, Heparin and 0.45% saline
C.
Insulin sliding scale, 0.9% Na and potassium replacement
D.
Insulin sliding scale, 0.45% Na and potassium replacement
E.
*50 ml of 50% dextrose IV
308. A 42-yr-old diabetic actor is started on Propanolol for stage fright. He collapses after a day shooting.
He has not changed his insulin regime. What treatment can you recommend?
A.
Insulin sliding scale, Heparin, 0.9% saline
B.
Insulin sliding scale, Heparin and 0.45% saline
C.
Insulin sliding scale, 0.9% Na and potassium replacement
D.
Insulin sliding scale, 0.45% Na and potassium replacement
E.
*50 ml of 50% dextrose IV
309. A 30-yr-old man with diabetes mellitus and severe renal failure (serum creatinine 0,7 mmol/l) has a
blood glucose concentration of 24 mmol/l. What treatment can you recommend?
A.
*Insulin—increased dosage
B.
Insulin—reduced dosage
C.
Metformin
D.
Gliquidon
E.
Enterosorbtion
310. A 51-yr-old diabetic collapsed on a long flight. On examination there was, sweating pallor and
tachycardia. What is the cause of syncope?
A.
Orthostatic hypotension
B.
*Hypoglycaemia
C.
Stokes-Adams syndrome
D.
Transient ischaemic attack
E.
Diabetic ketoacidosis
311. A 63-yr-old female is brought into the emergency department after collapsing at home. Her thighs
show evidence of lipoatrophy and her shins of Necrobiosis lipoidica. Prescribe investigations, please.
A.
*Blood glucose
B.
Ultrasound of abdomen
C.
T3, T4 and TSH levels
D.
HbAlc levels
E.
CT scan of the head
312. A 62-yr-old woman presents with stiff joints, myopathy and constipation. Plain radiographs reveal a
right calculus and, evidence of osteitis fibrosa cystitica. Put diagnosis, please.
A.
*Parathyroid adenoma
B.
Phaeochromocytoma
C.
Prolactinoma
D.
Corticosteroma
E.
Follicular carcinoma thyroid
313. A 51-yr-old woman complains of severe abdominal pain, nausea, vomiting, constipation, polyuria
and polydipsia. Put diagnosis of metabolic disturbances, please.
A.
Hypokalaemia
B.
Hyperkalaemia
C.
Hypocalcaemia
D.
*Hypercalcaemia
E.
Hyponatraemia
314. A 67-yr-old woman with a history of chronic renal failure presents with fits and fainting. She
complains of cramps in her limbs and circumoral numbness. Prescribe investigations, please.
A.
Serum glucose
B.
*Serum calcium
C.
Serum potassium
D.
Blood pressure standing and lying down
E.
Urinalysis
315. A 26-yr-old pregnant woman is found to have thyrotoxicosis due to Grave's disease during second
trimester of her pregnancy. What treatment of thyroid disorder can you recommend?
A.
Radioactive iodine
B.
Total thyroidectomy
C.
Propanolol
D.
*Carbimazole
E.
Potassium iodide
316. A 11-yr-old girl presents with thyrotoxicosis. A radioisotope scan shows an enlarged thyroid with
uniform uptake throughout. What treatment of thyroid disorder can you recommend?
A.
Radioactive iodine
B.
Subtotal thyroidectomy
C.
Propanolol
D.
*Carbimazole
E.
Glucocorticoids
317. What percentage of those with diabetes need insulin injections, oral antidiabetic medications, or both,
to control blood glucose levels?
A.
25%
B.
50%
C.
65%
D.
*75%
E.
Non of above
318. All of the following are true statements about insulin preparations EXCEPT:
A.
*intermediate-acting insulin has an onset of action of 15 to 30 minutes.
B.
insulin is supplied in U-50, U-100, and U-500 concentration.
C.
human insulin is the preferred insulin source.
D.
clear insulin that appears cloudy should not be used.
E.
Non of above
319. If a patient needed to mix two different types of insulin, which should he or she draw up first?
A.
*Regular
B.
Intermediate-acting
C.
NPH lente
D.
Ultra lente
E.
Non of above
320. What body mass index (BMI) should older adults have?
A.
Less than 21
B.
Between 20 and 30
C.
*Between 18.5 and 24.9
D.
Greater than 27
E.
None of above
321. Skin fold measurements are used to estimate which of the following?
A.
*Percentage of body fat
B.
Degree of obesity
C.
Body mass index
D.
Degree of malnutrition
E.
None of above
322. An older adult client has a serum albumin level of 2.9 g/dL. What interpretation should the physician
make?
A.
The patient has no protein depletion.
B.
*The patient has mild protein depletion.
C.
The patient has moderate protein depletion.
D.
The patient has severe protein depletion.
E.
None of above
323. What medication would predispose the patient to obesity?
A.
Lovastatin
B.
Metoprolol
C.
Lanoxin (Digoxin)
D.
*Dexamethasone
E.
None of above
324. What medication might the physician anticipate to stimulate the patient’s appetite?
A.
None of above
B.
Diphenhydramine (Benadryl)
C.
Methylprednisolone (Prednisone)
D.
Triamcinolone hexacetonide (Aristocort)
E.
*Cyproheptadine hydrochloride (Periactin)
325. What would be the effect on the patient's hormone response to a naturally occurring hormone if the
patient were taking a drug that “blocked” that hormone's receptor site?
A.
The patient’s response would indicate greater hormone metabolism.
B.
*The patient’s response would resemble decreased hormone activity.
C.
The patient’s response would resemble increased hormone activity.
D.
The patient’s response would be unchanged.
E.
None of above
326. How does a tropic hormone differ from other hormones?
A.
Tropic hormones are synthetic and are given to patients who have a hormone deficiency.
B.
Tropic hormones are exclusively involved in the production of sex hormones.
C.
*Tropic hormones stimulate other endocrine glands to secrete hormones.
D.
Tropic hormones are not under negative feedback control.
E.
None of above
327. hat would be the expected clinical manifestation for a patient who has excessive production of
melanocyte-stimulating hormone?
A.
Hypoglycemia and hyperkalemia
B.
Irritability and insomnia
C.
Increased urine output
D.
*Darkening of the skin
E.
None of above
328. What is the major hormone secreted by the adrenal medulla?
A.
Dopamine
B.
*Epinephrine
C.
Norepinephrine
D.
Aldosterone
E.
None of above
329. Which test results should the physician check to ascertain how well the patient is managing her
diabetes mellitus overall?
A.
Blood glucose level
B.
Glucose tolerance test
C.
*Glycated hemoglobin
D.
Radioimmunoassay of insulin
E.
None of above
330. The risk for which endocrine problem increases with aging?
A.
Hyperthyroidism
B.
*Diabetes mellitus
C.
Diabetes insipidus
D.
Cushing’s disease
E.
None of above
331. Which clinical manifestations alert the physician to the possibility of anterior pituitary
hyperfunction?
A.
*Enlarged hands and feet, heat intolerance
B.
Bradycardia, hypotension, and somnolence
C.
Chronic constipation and darkening of the skin
D.
Hyponatremia, hyperkalemia, and hypercalcemia
E.
None of above
332. Which patient responses demonstrate to the physician that treatment for diabetes insipidus is
effective?
A.
Urine output is increased; specific gravity is increased.
B.
Urine output is increased; specific gravity is decreased.
C.
*Urine output is decreased; specific gravity is increased.
D.
Urine output is increased; specific gravity is normal.
E.
None of above
333. Which of the following clinical manifestations alerts the physician to the possibility of side effects of
desmopressin acetate (DDAVP) therapy, taken nonparenterally by a patient with diabetes insipidus?
A.
Fibrosis at the injection site
B.
Orthostatic hypotension
C.
Decreased urine output
D.
*Nasal ulceration
E.
None of above
334. Which patient is at greatest risk for the development of the syndrome of inappropriate antidiuretic
hormone secretion (SIADH)?
A.
48-year-old man with an acute myocardial infarction
B.
38-year-old woman taking oral contraceptives
C.
68-year-old woman with diabetes mellitus
D.
*68-year-old man with chronic emphysema
E.
None of above
335. Which medication should the physician be prepared to administer to a patient with the syndrome of
inappropriate antidiuretic hormone (SIADH) secretion?
A.
Morphine
B.
*Demeclocycline
C.
Dextrose 5% in water
D.
Tricyclic antidepressants
E.
None of above
336. What dietary alterations should the physician make for the patient with Cushing's disease?
A.
High protein, high carbohydrate, low potassium
B.
*Low carbohydrate, high calorie, low sodium
C.
Low protein, high carbohydrate, low calcium
D.
High carbohydrate, low potassium, fluid restriction
E.
None of above
337. For what complications is the patient with hypercortisolism at greater risk?
A.
*Skin breakdown, infection, GI ulceration
B.
Anorexia, constipation, hypotension
C.
Kidney stones, weight loss, cataracts
D.
Diabetes insipidus, bradycardia, arthritis
E.
None of above
338. Which clinical manifestation indicates to the physician that the patient's adrenocortical insufficiency
is of primary origin rather than secondary origin?
A.
Weight loss
B.
Red, beefy tongue
C.
Orthostatic hypotension
D.
*Increased skin pigmentation
E.
None of above
339. Which statement made by the patient after a bilateral adrenalectomy indicates a need for further
clarification regarding medications?
A.
“I will take my cortisol replacement with food.”
B.
“I will avoid aspirin or aspirin-containing products.”
C.
“If I have any kind of stress, I will increase my doses of cortisol.”
D.
*“If I have nausea or vomiting, I will skip the medication until I am better.”
E.
None of above
340. Which drug(s) would the physician expect to administer to a patient to prevent gastrointestinal
ulceration from hypercortisolism?
A.
Metoclopramide (Reglan)
B.
Clarithromycin (Biaxin)
C.
*Omeprazole (Prilosec)
D.
Vitamins and iron
E.
None of above
341. Which serum electrolyte values alert the physician to the possibility of hyperaldosteronism?
A.
*Serum sodium 150 mmol/L, serum potassium 2.5 mmol/L
B.
Serum sodium 140 mmol/L, serum potassium 5.0 mmol/L
C.
Serum sodium 130 mmol/L, serum potassium 2.5 mmol/L
D.
Serum sodium 130 mmol/L, serum potassium 7.5 mmol/L
E.
None of above
342. The patient with hyperaldosteronism is being treated with spironolactone therapy. What precautions
should the physician teach this patient?
A.
*“Avoid salt substitutes.”
B.
“Avoid adding salt to food.”
C.
“Avoid excessive exposure to sunlight.”
D.
“Avoid acetaminophen and acetaminophen-containing products.”
E.
None of above
343. Which clinical manifestation change indicates to the physician that the therapy for the patient with
hyperaldosteronism is effective?
A.
The serum calcium level (total) has increased from 8.6 to 9.0 mg/dL.
B.
The urine output has decreased from 25 mL/hr to 15 mL/hr.
C.
*The systolic blood pressure has decreased by 24 mm Hg.
D.
The fasting blood glucose level is 86 mg/dL.
E.
None of above
344. Which of the following statements regarding thyroid disorders is true?
A.
The presence of a “goiter” always indicates hypothyroidism.
B.
*The effects of thyroid dysfunction are found in all body tissues and organs.
C.
None of above
D.
The hormones synthesized by the thyroid gland are not essential for life.
E.
The effects of hypothyroidism are less serious than those of hyperthyroidism.
345. Which clinical manifestation alerts the physician to the possibility of Graves’ disease as the cause of
hyperthyroidism?
A.
Weight loss
B.
*Exophthalmus
C.
Menstrual irregularities
D.
Increased heart rate and blood pressure
E.
None of above
346. In collaboration with the dietician, what dietary modification should the physician suggest for the
patient with hyperthyroidism?
A.
Decrease calories and proteins and increase carbohydrates.
B.
Eliminate carbohydrates and increase proteins and fats.
C.
*Increase calories, proteins, and carbohydrates.
D.
No dietary modification is needed.
E.
None of above
347. For which patient with hyperthyroidism is radioactive iodine therapy contraindicated?
A.
18-year-old man with asthma
B.
*28-year-old woman who is pregnant
C.
48-year-old man with type 2 diabetes mellitus
D.
68-year-old woman with mild congestive heart failure
E.
None of above
348. Which manifestation of Graves’ disease is unaffected by medical treatment for the hyperthyroidism?
A.
Thinning hair
B.
*Exophthalmos
C.
Weight loss
D.
Tachycardia
E.
None of above
349. Which statement made by the patient alerts the physician to the possibility of hypothyroidism?
A.
“My sister has thyroid problems.”
B.
“I seem to feel the heat more than other people.”
C.
“Food just doesn't taste good without a lot of salt.”
D.
*“I am always tired, even when I get 10 or 12 hours of sleep.”
E.
None of above
350. Which medication should the physician be prepared to administer to a patient with bradycardia as a
result of hypothyroidism?
A.
Atropine sulfate
B.
*Levothyroxine sodium
C.
Propranolol
D.
Epinephrine
E.
None of above
351. Which clinical manifestation indicates to the physician that treatment for the patient with
hypothyroidism is effective?
A.
The patient’s total white blood cell count is 6000 cells/mm3.
B.
The patient is thirsty.
C.
The patient’s weight has been the same for 3 weeks.
D.
*The patient has had a bowel movement every day for 1 week.
E.
None of above
352. Which patient is at greatest risk for hyperparathyroidism?
A.
28-year-old patient with pregnancy-induced hypertension
B.
*45-year-old patient receiving dialysis for end-stage renal disease
C.
55-year-old patient with moderate congestive heart failure after myocardial infarction
D.
60-year-old patient on home oxygen therapy for chronic obstructive pulmonary disease
E.
None of above
353. Which action should the physician teach the diabetic patient as being most beneficial in delaying the
onset of microvascular and macrovascular complications?
A.
*Controlling hyperglycemia
B.
Preventing hypoglycemia
C.
Restricting fluid intake
D.
Preventing ketosis
E.
None of above
354. With which patient should the physician be alert for undiagnosed diabetes mellitus?
A.
25-year-old white male
B.
45-year-old African American man
C.
25-year-old African American woman
D.
*45-year-old Native American woman
E.
None of above
355. With which therapy for diabetes mellitus is the patient not at risk for hypoglycemia?
A.
Regular insulin
B.
Lente insulin
C.
*Biguanides
D.
Sulfonylureas
E.
None of above
356. What intervention should the physician teach the patient with diabetes who uses an insulin infusion
pump to prevent the complication of infection?
A.
“Test your urine daily for the presence of ketone bodies.”
B.
“Use buffered insulin to prevent crystal formation.”
C.
“Keep the insulin frozen until you fill the pump.”
D.
*“Change the needle every 3 days.”
E.
None of above
357. What intervention should the physician suggest to the diabetic patient who self-injects insulin to
prevent or limit local irritation at the injection site?
A.
“Massage the site for 1 full minute after injection.”
B.
“Do not reuse needles.”
C.
“Try to make the injection deep enough to enter muscle.”
D.
*“Allow the insulin to warm to room temperature before injection.”
E.
None of above
358. Which nutritional group should the physician teach the diabetic patient with normal renal function to
rigidly control to reduce the complications of diabetes?
A.
*Fats
B.
Fiber
C.
Proteins
D.
Carbohydrates
E.
None of above
359. Which clinical manifestation in a patient with uncontrolled diabetes mellitus should the physician
expect as a result of the presence of ketoacid in the blood?
A.
*Increased rate and depth of respiration
B.
Extremity tremors followed by seizure activity
C.
Oral temperature of 102° F (38.9° C)
D.
Severe orthostatic hypotension
E.
None of above
360. Which arterial blood gas values indicate to the physician that the patient is experiencing
ketoacidosis?
A.
pH 7.38, HCO3– 22 mEq/L, PCO2 38 mm Hg, PO2 98 mm Hg
B.
*pH 7.28, HCO3– 18 mEq/L, PCO2 28 mm Hg, PO2 98 mm Hg
C.
pH 7.48, HCO3– 28 mEq/L, PCO2 38 mm Hg, PO2 98 mm Hg
D.
pH 7.28, HCO3– 22 mEq/L, PCO2 58 mm Hg, PO2 88 mm Hg
E.
None of above
361. What is the priority intervention for the patient having Kussmaul respirations as a result of diabetic
ketoacidosis?
A.
Administration of oxygen by mask or nasal cannula
B.
Intravenous administration of 10% glucose
C.
Implementation of seizure precautions
D.
*Administration of intravenous insulin
E.
None of above
362. Which nutritional problem should the physician be more alert for in older adult patients with diabetes
mellitus?
A.
Obesity
B.
*Malnutrition
C.
Alcoholism
D.
Hyperglycemia
E.
None of above
363. Which action should the physician suggest to reduce insulin needs in the patient with diabetes
mellitus?
A.
Reducing intake of water and other liquids to no more than 2 L/day
B.
Eating animal organ meats high in insulin
C.
Taking two 1-hour naps daily
D.
*Walking 1 mile each day
E.
None of above
364. Which action should the physician suggest to the patient who has been having difficulty with
hypoglycemia to decrease the rate of insulin absorption from the injection site?
A.
Massaging the injection site
B.
Exercising within 1 hour of insulin injection
C.
Injecting into muscle rather than subcutaneous tissue
D.
*Using refrigerated insulin without warming it to room temperature
E.
None of above
365. The diabetic patient has severe peripheral neuropathy, resulting in numbness and reduced sensation.
Which intervention should the physician teach the patient to prevent injury as a result of this complication?
A.
“Examine your feet daily.”
B.
“Rotate your insulin injection sites.”
C.
“Wear white socks instead of colored socks.”
D.
*“Use a bath thermometer to test water temperature.”
E.
None of above
366. Which change in clinical manifestations in a patient with long-standing diabetes mellitus alerts the
physician to the possibility of renal dysfunction?
A.
Loss of tactile perception
B.
The presence of glucose in the urine
C.
The presence of ketone bodies in the urine
D.
*A sustained increase in blood pressure from 130/84 to 150/100
E.
None of above
367. For the diabetic patient with microalbuminuria, what dietary modification should the physician
suggest?
A.
Decreased percentage of total calories derived from carbohydrates
B.
*Decreased percentage of total calories derived from proteins
C.
Decreased percentage of total calories derived from fats
D.
Decreased total caloric intake
E.
None of above
368. Which statement made by the diabetic patient who has a urinary tract infection indicates correct
understanding regarding antibiotic therapy?
A.
“If my temperature is normal for 3 days in a row, the infection is gone and I can stop taking my
medicine.”
B.
“If my temperature goes above 100° F (37.8° C) for 2 days, I should take twice as much medicine.”
C.
*“Even if I feel completely well, I should take the medication until it is gone.”
D.
“When my urine no longer burns, I will no longer need to take the antibiotics.”
E.
None of above
369. Why is ketosis rare in patients with type 2 diabetes, even when blood glucose levels are very high
(higher than 900 mg/dL)?
A.
Ketosis is less prevalent among obese adults.
B.
People with type 2 diabetes have normal lipid metabolism.
C.
*There is enough insulin produced by type 2 diabetes to prevent fat catabolism but not enough to
prevent hyperglycemia.
D.
Oral antidiabetic agents do not promote the breakdown of fat for fuel (lipolysis), and exogenous
insulin spares carbohydrates at the expense of fats.
E.
None of above
370. Which clinical manifestation indicates to the physician that the therapy for the patient with
hyperglycemic, hyperosmolar, nonketotic syndrome (HHNS) needs to be adjusted?
A.
The patient's serum potassium level increased from 2.8 mEq/L to 3.2 mEq/L.
B.
The patient's blood osmolarity has decreased from 350 mOsm to 330 mOsm.
C.
*The patient's score on the Glasgow Coma Scale is unchanged from 3 hours ago.
D.
The patient's urine has remained negative for ketone bodies for the past 3 hours.
E.
None of above
371. If a person needed to mix two different types of insulin, which should he or she draw up first?
A.
*Regular
B.
Intermediate-acting
C.
NPH lente
D.
Ultra lente
E.
None of above
372. What are the causes of hypocalcemia?
A.
Inadequate intake of calcium and vitamin D, in which inadequate levels of vitamin D inhibit
intestinal absorption of calcium
B.
hypoparathyroidism as a result of injury, disease, or surgery that decreases or eliminates secretion of
parathyroid hormone (PTH), which is necessary for calcium absorption and normal serum calcium levels
C.
malabsorption or loss of calcium from the GI tract, caused by increased intestinal motility from
severe diarrhea or laxative abuse; can also result from inadequate levels of vitamin D or PTH, or a reduction
in gastric acidity, decreasing the solubility of calcium salts
D.
*All of above
E.
None of above
373. What are the symptoms of hypocalcemia?
A.
Perioral paresthesia
B.
Twitching
C.
Carpopedal spasm
D.
*All of above
E.
None of above
374. Hormone that acts directly on the thyroid gland is:
A.
*TSH
B.
TRH
C.
FSH
D.
Thyroxine
E.
Triiodothyronine
375. Myxoedema is characterized by all next features EXCEPT:
A.
Carpal tunnel syndrome
B.
*Tachycardia
C.
Menorrhagia
D.
Hypotension
E.
Creaky voice
376. DeQuervan's thyroiditis is characterized by all next features EXCEPT
A.
*Increased radioactive iodine uptake
B.
Pain
C.
Increased ESR
D.
Fever
E.
Increased lymphocyte level
377. Serum T4 is decreased and low radioactive iodine uptake. Diagnosis?
A.
Subacute thyroiditis
B.
Non-toxic goiter
C.
*Hypothyroidism
D.
Graves' disease
E.
Non of above
378. Primary adrenal insufficiency is characterized by all the next features EXCEPT:
A.
Asthenia
B.
Hyperpigmentation
C.
*Hypertension
D.
Abdominal pain
E.
Tachycardia
379. Hypothyroidism is characterized by all the next features EXCEPT:
A.
Growth retardation
B.
*Pretibial myxoedema
C.
Menorrhagia
D.
Carpal-tunnel syndrome
E.
Creaky voice
380. Hashimoto's thyroiditis is characterized by all the next features EXCEPT:
A.
*Maximum incidence in children
B.
Neutrophilic infiltration
C.
Pain
D.
None of the above
E.
All of them
381. Hypofunction of thyroid gland is associated with the following clinical problems, EXCEPT:
A.
Menorrhagia
B.
Early abortions
C.
Galactorrhoea
D.
*Thromboembolism
E.
Dry skin
382. Endemic goiter is characterized by:
A.
Investigations invariably show some degree of hypothyroidism
B.
There is a clear evidence of an increased incidence of carcinoma of the thyroid
C.
*Administration of thyroxine will often cause the goitre to shrink
D.
Administration of iodine occasionally precipitates hyperthyroidism
E.
The incidence after puberty is much higher in females
383. What is NOT correct in characteristics of myxoedema?
A.
Slow pulse
B.
*Hypertension
C.
Hypotension
D.
Dry skin
E.
Constipation
384. What hormone determination is used for the monitoring of treatment of secondary hypothyroidism:
A.
T3
B.
*T4
C.
TSH
D.
TRH
E.
LH
385. The most common cause of primary adrenal insufficiency is:
A.
*Autoimmune
B.
Surgery
C.
Steroid withdrawal
D.
Tuberculosis
E.
Sarcoidosis
386. Investigaion of choice to distinguish between COPD with emphysema and bronchial asthma is:
A. *Allergy test to pollens
B. Methacholine provocative test
C. Chest X-ray
D. Arterial blood gas analysis
E. All of the above
387. A criterion for the diagnosis of asthma is:
A. *15 % reversibility in spirography
B. 5% reversibility in spirography
C. 10% reversibility in spirography
D. X-ray changes
E. Sputum changes
388. A drug is to be delivered by a nebuliser. The size of a droplet for its humidification (in mcm) is:
A. *less than 5
B. 5-10
C. 10-15
D. 15-20
E. All of the above
389. According recommendations of WHO daily dose of inhaled glucocortisteroids in the case of mild
persistant bronchial asthma is:
A. *200-500 mkg
B. 150-300 mkg
C. 800-2000 mkg
D. More than 2000 mkg
E. Glucocorticosteroids are not recommended
390. All medicines listed below are used in bronchial asthma, except:
A. *Morphine
B. Salbutamol
C. Aminophylline
D. Steroids
E. Ipratropium
391. All of the following are useful for treating acute bronchial asthma except:
A. *Sodium chromoglycate inhalation
B. 100% Oxygen
C. Hydrocortisone infusion
D. IV aminophylline
E. All of the above
392. All of the following drugs useful in the treatment of a patient with acute bronchial asthma except:
A. *Montelukast
B. Ipratropium
C. Salbutamol
D. Hydrocortisone
E. All of the above
393. All of the following features are seen in the viral pneumonia except:
A. Presence of interstitial inflammation
B. *Predominance of alveolar exudates
C. Bronchiolitis
D. Multinucleate giant cells in the bronchiolar wall
E. All features are present
394. All of the following statements about Leukotriene modifiers in the management of bronchial asthma
are true except:
A. *May be used for acute asthma
B. May be used for exercise induced asthma
C. Zileuton is Leukotriene modifier
D. May uncover Churg Strauss syndrome
E. All of the above
395. Aspirin sensitive asthma is associated with :
A. *Nasal polyps
B. Extrinsic asthma
C. Urticaria
D. Obesity
E. All of the above
396. Aspirin-sensitive asthma is associated with:
A. *Nasal polyp
B. Obesity
C. Urticaria
D. Extrinsic asthma
E. Nothing of the above
397. Asthma is precipitated by all except:
A. *Gold
B. Aspirin
C. Alcohol
D. Suxamethonium
E. Nothing of the above
398. Attack of bronchial asthma differs from heart asthma by such sign
A. *By expiration dyspnea
B. By sudden development of attack
C. By orthostatic position of the patient
D. By duration of the attack
E. By inspiration dyspnea
399. Blocking these can worsen the asthma
A. *β2 -adrenoreceptors
B. α-adrenoreceptors
C. Mast cells
D. Neutrophils
E. Eosinophils
400. Bronchial asthma is associated with raised levels of
A. *Leukotrienes
B. Thromboxane
C. Antinuclear antibodies
D. Corticosteroids
E. All of the above
401. Bronchial asthma patient on artificial ventilation requires :
A. *An IE ratio 1: 2.5
B. A low inspiratory flow
C. An equal IE ratio of 1:1
D. An inverese ratio ventilation
E. Nothing of the above
402. Bronchodylation activity is specific for
A. *Berotec
B. Anuzol
C. Inderal
D. Paracetamol
E. Azafen
403. Drug of choice for inhalation in acute asthma not responding to salbutamol is:
A. *Ipratropium bromide
B. Atropine sulfate
C. Isopropamide
D. Hyoscine methyl bromide
E. All of the above
404. Consider the following statement: Early onset of extrinsic episodic asthma is characterized by: 1.
Family history of eczema or rhinitis 2. Development of an early and late asthmatic reaction mediated by
mast cells 3. T lymphocytes that release cytokine like interleukin-4. Which of these statements are correct ?
A. *1,2 and 3
B. 1 and 2
C. 2 and 3
D. 1 and 3
E. Nothing of the above
405. Consider the following statements: Life threatening features of acute-severe asthma in children
include: 1. Altered sensorium 2. Pulsus paradoxus 3. Audible wheeze in both inspiration and expiration 4.
Oxygen saturation 92-95%. Which of these statements is/are correct?
A. *1 and 2
B. 1 only
C. 2 and 4
D. 1, 2, 3 and 4
E. Nothing of the above
406. Contraindication for glucocorticosteroid therapy of bronchial asthma is:
A. *Bleeding from gastric ulcer
B. Chronic persistant hepatitis
C. Exacerbation of respiratory-heart failure
D. Emphysema of lungs
E. Respiratory failure
407. Sharko-Leiden crystals are seen in:
A. *Bronchial asthma
B. Bronchiectasis
C. Chronic bronchitis
D. Wegners granulomatosis
E. All of the above
408. Curschmann's spirals in sputum are seen in:
A. *Bronchial asthma
B. Bronchiectasis
C. Chronic bronchitis
D. Wegners granulomatosis
E. All of the above
409. Curshmann's spirals in sputum is seen in:
A. *Asthma
B. Tuberculosis
C. Bronchitis
D. Bronchiectasis
E. Pneumonia
410. Decreased maximum mid-expiratory flow rate indicates obstruction in:
A. *Small airways
B. Trachea
C. Large airways
D. Trachea & bronchi both
E. Nothing of the above
411. Drug of choice in asthma with heart disease is
A. *Ipratropium
B. Rimiterol
C. Terbutaline
D. Cromolyn sodium
E. All of the above
412. Exercise induced asthma is not precipitated by :
A. *Swimming in hot water
B. High altitude climb and exercises
C. Cycling in cold weather
D. Swimming in cold water
E. All of the above
413. FEV1/FVC is reduced in case of:
A. *Asthma
B. Pleural effusion
C. Lung fibrosis
D. All of the above
E. Nothing of the above
414. For the patient who has sustained tracheobronchial trauma, which assessment finding alerts the
physician to the possibility of tracheal lacerations?
A. Hypertympanic sound on affected side
B. *Subcutaneous emphysema over the trachea
C. Hypotension and decreased capillary refill
D. Deviation of the trachea to the affected side
E. Non of above
415. Find common changes in spirography for patients with bronchial asthma
A. *FEV1, FVC, Typhno index are low
B. FEV1, FVC, Typhno index are high
C. FEV1, FVC, Typhno index are normal
D. FEV1, FVC are high, Typhno index is low
E. FEV1, FVC are low, Typhno index is high
416. Find common changes in sputum for patients with asthma
A. *Eosinophils
B. T cells
C. Mast cells
D. Neutrophils
E. IgM
417. General medical contraindications to admittance to work related to the dangerous harmful substances
and harmful industrial factors, are the following, except:
A. Chronic diseases of lungs with the evident pulmonary-cardiac insufficiency
B. *Essential hypertension of II stage
C. Bronchial asthma of severe form with the evident disturbance of breathing and circulation of blood
D. Active forms of tuberculosis of any localization
E. Stomach and duodenal ulcer with chronic recurrent course and susceptibility to complications
418. How does antibiotic therapy increase the risk of infection?
A. Most intravenous antibiotics contain high concentrations of glucose, providing an ideal medium for
bacterial proliferation
B. The associated diarrhea causes ulcerations to form in the intestinal tract, creating a new portal of entry
C. *Antibiotic therapy also kills off normal flora, which provide a means of protection from infection
D. The resulting dead bacteria provide a rich culture medium for viruses
E. Non of above
419. How does prostacycline therapy reduce the pathologic problems of primary pulmonary hypertension?
A. *Increasing lung blood flow by dilating pulmonary blood vessels
B. Improving ventilation by increasing the contractility of the diaphragm
C. Increasing gas exchange by changing the elasticity of the alveolar
D. Protecting the heart from development of cor pulmonale by increasing cardiac muscle strength
E. Non of above
420. How does the drug oseltamivir (Tamiflu) prevent influenza or shorten the duration of illness?
A. *Preventing the virus from entering respiratory cells
B. Boosting the patient’s immune system
C. Inhibiting the virus from multiplying
D. Directly killing the virus
E. Non of above
421. How long is the usual course of drug treatment for a patient with active tuberculosis (TB)?
A. 7 to 10 days
B. 6 weeks
C. *6 months
D. 2 years
E. Non of above
422. How many stages of severity of bronchial asthma do you know?
A. *4
B. 1
C. 2
D. 3
E. There are no stages of severity
423. In bronchial asthma glucocorticoids
A. *Reduce airway inflammation
B. Act as potent bronchodilators
C. Inhibit degranulation of mast cells
D. Block the action of humoral mediators
E. Nothing of the above
424. In patient K. 21 years old was diagnosed persistent bronchial asthma. Prophylaxis of attacks of
dyspnea can be provided with:
A. *Beclomethasone
B. Theophillin
C. Nedokromil sodium
D. Antihystamine medicines
E. Atrovent
425. In severe bronchial asthma, it is present:
A. *Hyperresonant chest with prolonged expiration
B. Infrequent rhonchi and absent breath sounds
C. Increased fremitus and crackles
D. Decreased fremitus and crepitation
E. All of the above
426. In the time of attack of bronchial asthma in lungs are heard
A. *Dry loud wheezes
B. Moist loud wheezes
C. Moist soundless wheezes
D. Voice of pleural friction
E. Crepitation
427. In what specific way does the respiratory system function contribute to acid-base balance?
A. Prevents excessive loss of hydrogen ions by evaporation
B. Increases the potassium ion content in bronchial secretions
C. *Removes carbon dioxide generated as a result of metabolism
D. Maintains body water levels through mucous membrane function
E. Non of above
428. In which patient with chronic airflow limitation would you expect to find dependent edema?
A. 25-year-old with cold-induced asthma
B. 65-year-old with exercise-induced asthma
C. *55- year-old with chronic bronchitis
D. 45-year-old with moderate emphysema
E. Non of above
429. What is the most necessary in order to diagnose bronchial asthma:
A. *To notice the asthmatic attack of dyspnea
B. To find allergic antibodies
C. To find the presence of expiration dyspnea
D. To find the presence of signs of obstructive respiratory failure
E. To find the presence of eosinophylia in blood
430. One of the following is not an indicator of the severity of asthma:
A. *Systolic hypertension
B. Use of accessory muscles
C. Pulsus paradoxus
D. Cyanosis
E. All of the above
431. Patient L., 25 years old, has persistent bronchial asthma. Prophylaxis of attacks of dyspnea can be
provided with:
A. *Fluticasone
B. Theophylline
C. Salbutamol
D. Antihystamine medicines
E. Atrovent
432. Patient of 52 has severe attack of expiratory dyspnea, with severe dry cough, distant wheezes,
palpitations. What medicine is the best one for the first aid?
A. *Salbutamol
B. Strofantin
C. Lasolvan
D. Atrovent
E. Prednisone
433. Patient with mild persistent bronchial asthma developed exacerbation. Your recommendations?
A. *To increase dose of inhaled glucocorticosteroid, β2-agonist of short action
B. To increase dose of β2-agonist of short action, do not change dose of inhaled glucocorticosteroid
C. To provide test with bronchodylator, and then to increase dose of inhaled glucocorticosteroid and β2agonist
D. To provide provocative test with hystamine, and then to increase the dose of inhaled glucocorticosteroid
and β2-agonist
E. To increase dose of inhaled glucocorticosteroid, β2-agonist, and then to provide provocational test with
hystamine
434. Patient`s diagnosis is “bronchial asthma, persistent form of moderate severity”. What medicine would
you give advantage for the planned treatment of disease?
A. *Budesonid-forte
B. Intal
C. Euphylline orally
D. Salbutamol
E. Berotec
435. Poor prognostic indicators in acute severe asthma include all of the following EXCEPT:
A. *PEFR less than 20% of previous value
B. Pulsus paradoxus
C. Hypocarbia
D. Use of accessory respiratory muscles
E. Nothing of above
436. Primary prophylaxis of bronchial asthma has to be provided:
A. *In case of bronchial asthma in relatives
B. For all healthy people
C. In case of any allergic diseases
D. In case of chronic non-obstructive bronchitis
E. In case of frequent colds
437. Pulmonary surfactant is secreted by:
A. Type I pneumocytes
B. *Type II pneumocytes
C. Lymphocytes
D. Bronchial epithelial cells
E. Tracheal epithelial cells
438. Silent chest is seen in
A. *Very severe asthma
B. Chronic bronchitis
C. Emphysema
D. Bronchiectasis
E. All of the above
439. Silicatosis is:
A. *Asbestosis
B. Siderosis
C. Aluminosis
D. Bisinosis
E. All of the above
440. Spirals of Kurshman in patients with bronchial asthma may be found at the laboratory test of:
A. *Sputum
B. Urine
C. Biopsy of pulmonary tissue
D. Gastric juice
E. Pleural exudates
441. Steroid inhalation side effect include:
A. *Oropharyngeal candidiasis
B. Retention of fluid
C. Sedation
D. Palpitations
E. Nothing of the above
442. The FEV1 is the volume of:
A. *air forcefully expired during the first second after a full breath and normally comprises > 75% of the
VC
B. air normally expired during the first second after a full breath and normally comprises > 75% of the VC
C. air forcefully expired during the first minute after a full breath and normally comprises > 75% of the VC
D. air normally expired during the first minute after a full breath and normally comprises > 75% of the VC
E. air forcefully expired during the second minute after a full breath and normally comprises > 75% of the
VC
443. The following are interstitial lung diseases except:
A. *Bronchial asthma
B. Sarcoidosis
C. Fibrosisng alveolitis
D. Pneumoconiosis
E. Nothing of the above
444. The most predictive and dangerous side effect of propranolol that makes it to be avoided in known
patient of COPD is induction of:
A. *Acute asthmatic attack
B. Respiratory failure
C. Glaucoma
D. Pleural effusion
E. All of the above
445. The patient diagnosed with moderate stage of COPD says there is no sense in stopping smoking now
because the damage is done. Which response is the best rationale for encouraging this patient to stop
smoking?
A. “The damage will be reversed.”
B. *“The COPD will progress more slowly.”
C. “Your risk for asthma development, which would further reduce your lung function, will be decreased.”
D. “You will be less likely to lose excessive amounts of weight and will have a more normal appearance.”
E. Non of above
446. The patient with manifestations of a respiratory infection is suspected of having SARS. In addition to
standard precautions, what other infection control precautions should the physician use until the diagnosis is
certain?
A. Airborne precautions
B. Droplet precautions
C. *Airborne precautions and contact precautions
D. Droplet precautions and contact precautions
E. Non of above
447. The substance released by the mast cell activation and causes bronchospasm.
A. *Histamine
B. Leukotrienes
C. IgM
D. IgG
E. IgE
448. This parameter of respiratory function is usually unaffected in asthma.
A. *Vital capacity
B. PEFR
C. FVC
D. FEV1
E. FEV1/FVC
449. Use of disodium cromoglycate as a preventive measure has been found to be of value in :
A. *Exercise induced asthma
B. Pneumonia
C. Chronic bronchitis
D. Fanner's lung
E. Nothing of the above
450. What is necessary to make during periodic medical examination to all workers, who contact with
occupational dust?
A. General blood analysis
B. Biochemical blood analysis
C. ECG
D. Ultrasound examination
E. *Roentgenography of chest organs
451. What is not specific for severe persistent bronchial asthma?
A. *Severe vomiting
B. Long duration of attacks
C. Development of status asthmaticus
D. Development of acute heart failure
E. Necessarity of usage of glucocorticosteroids
452. What is the main purpose of a negative-pressure ventilator?
A. Healing diseased lung tissue
B. Relieving hypoxemia by opening obstructed airways
C. *Assisting ventilation to healthy lungs by mimicking normal chest pressures
D. Delivering an individualized preset tidal volume to the lower respiratory tract
E. Non of above
453. What is the major difference in pathophysiology between asthma and COPD?
A. Asthma is a restrictive disorder and COPD is an obstructive disorder
B. *COPD results in permanent airflow obstruction and asthma is a condition of reversible airflow
obstruction
C. COPD is caused by chronic exposure to inhalation irritants and the major cause of asthma is cigarette
smoking
D. Asthma is the result of an inflammatory process and COPD is a result of inflammatory, infectious, and
hyperresponsiveness processes
E. Non of above
454. What is the mechanism of reduced gas exchange in a patient who has atelectasis?
A. Airway obstruction
B. *Reduced alveolar surface area
C. Failure of pulmonary circulation to fully perfuse lung tissue
D. Increased bronchial secretions filling the alveoli with fluid rather than with air
E. Non of above
455. What is the most important intervention for the patient with acute respiratory distress syndrome?
A. Antibiotic therapy
B. Bronchodilators
C. *Oxygen therapy
D. Diuretic therapy
E. Non of above
456. What is the physiologic consequences of CO2 narcosis?
A. Excessive sleepiness in the patient with hypercarbia
B. *Failure of rising blood levels of CO2 to trigger more rapid and deeper respirations
C. A change in the ventilation-perfusion ratio, in which ventilation exceeds perfusion
D. Increase in the percentage of oxygen delivered to the patient does not result in an increased PaO2
E. Non of above
457. What is the primary cause for the increased incidence in lung cancer among women during the past
20 years?
A. Women now have a longer life expectancy than men
B. *Cigarette smoking among women increased dramatically 50 years ago
C. More women now work in the industrial setting than in previous decades
D. Increased use of oral contraceptives combined with air pollution has driven the incidence of lung cancer
up in women
E. Non of above
458. What is the role of surfactant in pulmonary function?
A. *Reduces alveolar surface tension, improving gas exchange
B. Increases alveolar surface tension, improving gas exchange
C. Dilates pulmonary blood vessels, decreasing pulmonary vascular resistance
D. Relaxes bronchial smooth muscle, decreasing pulmonary vascular resistance
E. Non of above
459. What part of the pulmonary system is most affected by asthma?
A. Alveoli
B. Larynx
C. Pharynx
D. *Airways
E. Non of above
460. Which assessment finding in a patient who has been medicated during an asthma attack indicates to
you that the therapy should be modified?
A. Peak expiratory rate flow 10% below expected value
B. Presence of bilateral tactile fremitus
C. *Suprasternal retraction on inhalation
D. Trachea at the midline
E. Non of above
461. Which assessment finding in a patient with severe dyspnea indicates to you that the respiratory
problem is chronic?
A. Wheezing on exhalation
B. Productive cough
C. *Clubbed fingers
D. Cyanosis
E. Non of above
462. Which assessment finding should you expect in the patient with COPD?
A. *Decreased vocal fremitus
B. Grossly bloody sputum
C. Loss of the gag reflex
D. Tracheal deviation
E. Non of above
463. Which β2-agonist is not given for acute bronchial asthma?
A. *Salmeterol
B. Salbutamol
C. Terbutaline
D. Methyl xanthine
E. All of the above
464. Which blood gas value indicates that the patient is experiencing hypercarbia?
A. pH = 7.33
B. Bicarbonate = 20 mEq/L
C. *PaCO2 = 60 mm Hg
D. PaO2 = 80 mm Hg
E. Non of above
465. Which clinical change indicates the patient's COPD is becoming worse?
A. Pulse pressure has increased from 35 mm Hg to 40 mm Hg
B. *The patient has had an unplanned weight loss of 15 pounds
C. The patient's average respiratory rate has decreased from 34 to 33
D. The patient's PaCO2 is 58 mm Hg this month compared with 64 mm Hg last month
E. Non of above
466. Which clinical manifestation alerts you to the presence of hypoventilation when you are monitoring a
patient with chronic lung disease and hypercarbia who is receiving oxygen therapy?
A. Coarse crackles and wheezes on auscultation
B. *Slow, shallow respirations
C. Pulse oximetry of 90%
D. Clubbing of the fingers
E. Non of above
467. Which clinical manifestation in a patient with chronic bronchitis indicates to you a worsening of the
patient's respiratory condition?
A. Fatigue
B. Cachexia
C. *Confusion
D. Slow capillary refill
E. Non of above
468. Which clinical manifestation in a patient with long-standing COPD alerts you to the possibility of cor
pulmonale?
A. Pursed-lip breathing occurs when the patient is at rest
B. The patient's neck muscles are enlarged and prominent
C. The patient's ECG shows tall, peaked T waves and an absent U wave
D. *Jugular venous distention is present when the patient is in a sitting position
E. Non of above
469. Which complication is most frequent in patients with silicosis?
A. Tromboembolia of pulmonary artery
B. Bronchial asthma
C. Pneumonia
D. Cancer of lungs
E. *Tuberculosis
470. Which diagnostic indicator confirms the presence of active tuberculosis?
A. Positive tuberculine test
B. The presence of calcified lesions on chest x-ray
C. *The presence of M. tuberculosis in a sputum culture
D. The combined clinical manifestations of weight loss, night sweats, fever, and cough productive of
mucopurulent bloody sputum
E. Non of above
471. Which disorder is an example of a "restrictive" pulmonary disease?
A. Asthma
B. Bronchitis
C. Lung cancer
D. *Pulmonary fibrosis
E. Non of above
472. Which drug is more commonly used as therapy for COPD than as therapy for asthma?
A. Theophylline (Theo-Dur)
B. Montelukast (Singulair)
C. *Guaifenesin (Organidin)
D. Salmeterol (Serevent)
E. Non of above
473. Which laboratory value indicates to you that the patient may have allergic asthma?
A. *Eosinophil count of 13%
B. Total white blood cell count of 100%
C. Total absence of macrophages in the differential white blood cell count
D. Band neutrophil count of 32% and a segmented neutrophil count of 22%
E. Non of above
474. Which manifestations in a patient receiving oxygen therapy at 60% for more than 24 hours alerts you
to the possibility of oxygen toxicity?
A. *Increased dyspnea
B. Decreased rate and depth of respiration
C. Wheezing on inhalation and exhalation
D. Increased excretion of thick, white, frothy sputum
E. Non of above
475. Which of the following patients could be expected to require mechanical ventilation long-term?
A. 27-year-old with status asthmaticus
B. 45-year-old with morphine overdose
C. *24-year-old with muscular dystrophy
D. 65-year-old with bilateral bacterial pneumonia
E. Non of above
476. Which patient is a greatest risk for contracting SARS?
A. *The 30-year-old physician providing direct care to patients with SARS
B. The 50-year-old farmer working directly with cows and pigs
C. The 60-year-old patient with type 2 diabetes mellitus and renal insufficiency
D. The 70-year-old patient residing in an assisted living environment
E. Non of above
477. Which patient is a greatest risk for developing nosocomial pneumonia?
A. *The 60-year-old patient receiving mechanical ventilation
B. The 40-year-old patient receiving antibiotics for a surgical wound infection
C. The 60-year-old patient in traction for a fractured femur who also has a cold
D. The 40-year-old patient with type 2 diabetes who has a 50 pack-year smoking history
E. Non of above
478. Which patient is at greatest risk for acute respiratory distress syndrome?
A. The 62-year-old with COPD who has pneumonia
B. *The 22-year-old who received 10 units of blood after a motor vehicle crash
C. The 78-year-old with chronic congestive heart failure and pulmonary edema
D. The 24-year-old with asthma who has not taken any of her asthma medications for 2 weeks
E. Non of above
479. Which patient should the physician caution to avoid taking over-the-counter decongestants for
manifestations of a cold or flu?
A. 25-year-old man with a latex allergy
B. *45-year-old woman with hypertension
C. 32-year-old woman who is taking oral contraceptives
D. 65-year-old man who has had type 1 diabetes mellitus for 20 years
E. Non of above
480. Which set of arterial blood gas values indicates early pneumonia as the respiratory problem?
A. *pH 7.35, HCO3– 22 mEq/L, PCO2 45 mm Hg, PO2 86 mm Hg
B. pH 7.30, HCO3– 22 mEq/L, PCO2 60 mm Hg, PO2 92 mm Hg
C. pH 7.32, HCO3– 17 mEq/L, PCO2 25 mm Hg, PO2 98 mm Hg
D. pH 7.30, HCO3– 28 mEq/L, PCO2 65 mm Hg, PO2 75 mm Hg
E. Non of above
481. Which set of arterial blood gases would the physician expect to find in a patient who developed a
pulmonary embolism 15 minutes ago?
A. pH 7.30, HCO3– 22 mEq/L, PCO2 60 mm Hg, PO2 66 mm Hg
B. pH 7.38, HCO3– 22 mEq/L, PCO2 45 mm Hg, PO2 96 mm Hg
C. *pH 7.47, HCO3– 23 mEq/L, PCO2 25 mm Hg, PO2 82 mm Hg
D. pH 7.30, HCO3– 28 mEq/L, PCO2 65 mm Hg, PO2 75 mm Hg
E. Non of above
482. Which set of arterial blood gases would the physician expect to find in a patient who developed a
pulmonary embolism 6 hours ago?
A. *pH 7.30, HCO3– 22 mEq/L, PCO2 60 mm Hg, PO2 66 mm Hg
B. pH 7.38, HCO3– 22 mEq/L, PCO2 45 mm Hg, PO2 96 mm Hg
C. pH 7.47, HCO3– 23 mEq/L, PCO2 25 mm Hg, PO2 82 mm Hg
D. pH 7.30, HCO3– 28 mEq/L, PCO2 65 mm Hg, PO2 75 mm Hg
E. Non of above
483. Which type of acid-base imbalance is most common among patients with COPD?
A. *Respiratory acidosis
B. Respiratory alkalosis
C. Metabolic acidosis
D. Metabolic alkalosis
E. Non of above
484. With which of the following theophylline has an antagonistic interaction?
A. Histamine receptors.
B. Bradykinin receptors.
C. *Adenosine receptors.
D. Imidazoline receptors.
E. Beta-adrenoreceptors
485. According recommendations of WHO daily dose of inhaled glucocortisteroids in the case of mild
persistant bronchial asthma is:
A. *200-500 mkg
B. 150-300 mkg
C. 800-2000 mkg
D. More than 2000 mkg
E. Glucocorticosteroids are not recommended
486. All the following statements are true about chronic obstructive lung disease except:
A. *Decreased diffusion capacity
B. Decreased FEV1
C. Decreased MEFR
D. Increased RV
E. Nothing of the above
487. All the following lung volumes can be measured by a simple spirometer except:
A. *Residual volume
B. Vital capacity
C. Tidal volume
D. Forced vital capacity
E. Nothing of the above
488. Alveolar-arterial tension gradient increases in all states except:
A. *Hypoventilation
B. Diffusion defects
C. R-L shunt
D. Ventilation perfusion abnormality
E. All of the above
489. Alveolar-arterial tension gradient increases in all states except:
A. *Hypoventilation
B. Diffusion defects
C. R-L shunt
D. Ventilation perfusion abnormality
E. Nothing of the above
490. Bronchodylation activity is specific for
A. *Berotec
B. Anuzol
C. Inderal
D. Paracetamol
E. Azafen
491. Decreased maximum mid-expiratory flow rate indicates obstruction in:
A. *Small airways
B. Trachea
C. Large airways
D. Trachea & Bronchi both
E. Nothing of the above
492. False about COPD is that it can be characterized by:
A. *Decreased diffusion capacity
B. Decreased FEV1
C. Decreased midexpiratory peaks flow rate
D. Increased residual capacity
E. Nothing of the above
493. FEV1 ratio is decreased in all except:
A. *Interstitial lung disease
B. Bronchiectasis
C. Emphysema
D. Chronic bronchitis
E. Nothing of the above
494. Hemoptysis is an important symptom common for all states except:
A. *Alpha1-antitrypsin deficiency
B. Idiopathic hemosiderosis
C. Bronchial adenoma
D. Pulmonary TB
E. All of the above
495. High oxygen tension in alveoli develops due to:
A. *Ventilation perfusion mismatch
B. Right to left shunt
C. Inappropriate gas exchange
D. Bronchial asthma
E. All of the above
496. How many stages of severity of bronchial asthma do you know?
A. *4
B. 1
C. 2
D. 3
E. There are no stages of severity
497. If FEV1, is 1.3 L and FVC is 3.1 L in an adult man it suggests diagnosis of
A. *Obstructive lung disease
B. Normal lung function
C. Restrictive lung disease
D. None of the above
E. All of the above
498. In chronic obstructive disease the following pulmonary function abnormalities are present except:
A. *Decreased diffusing capacity
B. Decreased FEV1
C. Decreased maximum expiratory flow rate
D. Increased residual volume
E. Nothing of the above
499. Obstructive type of insufficiency of pulmonary ventilation is caused by:
A. *Narrowing of bronchi
B. Decreasing of diaphragm mobility
C. Arrising of pleuritis
D. Decreasing of breathing muscles strength
E. Violation of perfusion
500. One of the following features is not an indicator of the severity of asthma:
A. *Systolic hypertension
B. Use of accessory muscles
C. Pulsus paradoxus
D. Cyanosis
E. All of the above
501. Patient L., 25 years old, has persistent bronchial asthma. Prophylaxis of attacks of dyspnea can be
provided with:
A. *Fluticasoni
B. Theophillin
C. Salbutamol
D. Antihystaminic medicines
E. Atrovent
502. Patient of 52 has severe attack of expiration dyspnea, with severe dry cough with heared on distance
wheezes, palpitation. What preparation is the best one for the first aid?
A. *Salbutamol
B. Strofantin
C. Lasolvan
D. Atrovent
E. Prednisone
503. Poor prognostic indicators in acute severe asthma include all of the following EXCEPT
A. *PEFR less than 20% of previous value
B. Pulsus paradoxus
C. Hypocarbia
D. Use of accessory muscles of respiration
E. Nothing of the above
504. Restrictive lung disease is associated with:
A. *Interstitial lung disease
B. High residual lung volume
C. High PCO2
D. FEV1 below 50 %
E. Nothing of the above
505. Silent chest is seen in
A. *Very severe asthma
B. Chronic bronchitis
C. Emphysema
D. Bronchiectasis
E. All of the above
506. The hall mark of generalised obstructive lung disease is
A. *Reduced timed vital capacity
B. Reduced tidal volume
C. Reduced residual volume
D. Reduced vital capacity
E. All of the above
507. The most common cause of lowering of lower lungs borders is:
A. *Pulmonary emphysema
B. Bronchial asthma
C. Chronic pulmonary artery hypertension
D. Pneumothorax
E. Enteroptosis
508. The most predictive and dangerous side effect of propranolol that makes it to be avoided in known
patient of COPD is induction of :
A. *Acute asthmatic attack
B. Respiratory failure
C. Glaucoma
D. Pleural effusion
E. All of the above
509. The severity of COPD and the potential response to bronchodilator can be adequately assessed by:
A. *Simple spirometry (± flow-volume loop) before and after inhalation of bronchodilator
B. X-ray examination
C. Bronchscopy
D. Simple spirometry (± flow-volume loop) after inhalation of bronchodilator
E. Nothing of above
510. This parameter of respiratory function is usually unaffected in asthma:
A. *Vital capacity
B. PEFR
C. FVC
D. FEV1
E. FEV1/FVC
511. Ventilation/perfusion ratio is highest in:
A. *Apex of lung
B. Mid zone
C. Lower zone
D. Hilus
E. Nothing of the above
512. Volume of air taken into the lungs in normal respiration is known as:
A. *Tidal volume
B. Vital capacity
C. Timed vital capacity
D. Inspiratory reserve volume
E. All of the above
513. Which manifestations in a patient receiving oxygen therapy at 60% for more than 24 hours alerts you
to the possibility of oxygen toxicity?
A. *Increased dyspnea
B. Decreased rate and depth of respiration
C. Wheezing on inhalation and exhalation
D. Increased excretion of thick, white, frothy sputum
E. Non of above
514. α1- Antitrypsin deficiency is associated with:
A. *Panaciner-emphysema
B. Centriaciner emphysema
C. Irregular emphysema
D. Paraseptal-emphysema
E. All of the above
515. All characteristics are decreased in infiltrative lung disease, except:
A. *Alveolar arterial difference in PaO2
B. Vital capacity
C. Total lung capacity
D. Lung compliance
E. Nothing of the above
516. All of the following features are seen in the viral pneumonia except:
A. Presence of interstitial inflammation
B. *Predominance of alveolar exudates
C. Bronchiolitis
D. Multinucleate giant cells in the bronchiolar wall
E. All features are present
517. Alveolar hypoventilation seen in all except :
A. *Lobar pneumonia
B. COPD
C. Kyphoscoliosis
D. Bulbar poliomyelitis
E. Nothing of the above
518. Breath sounds are decreased in following except:
A. *Lobar pneumonia
B. Pneumothorax
C. Pleural effusion
D. Atelectasis
E. Nothing of the above
519. Bulging fissure sign is characteristic of pneumonia due to
A. *Klebsiella
B. Anaerobic bacteria
C. Mycoplasma
D. Pseudomonas
E. Nothing of the above
520. Feature of restrictive lung disease is:
A. *FEV1/FVC increases and compliance decreases
B. FEV1/FVC decreases and compliance decreases
C. FEV1/FVC increases and compliance increases
D. FEV1/FVC decreases and compliance increases
E. Nothing of the above
521. FEV1 ratio is decreased in all except :
A. *Interstitial lung disease
B. Bronchiectasis
C. Emphysema
D. Chronic bronchitis
E. Nothing of the above
522. For the patient who has sustained tracheobronchial trauma, which assessment finding alerts the
physician to the possibility of tracheal lacerations?
A. Hypertympanic sound on affected side
B. *Subcutaneous emphysema over the trachea
C. Hypotension and decreased capillary refill
D. Deviation of the trachea to the affected side
E. Non of above
523. General medical contraindications to admittance to work related to the dangerous harmful substances
and harmful industrial factors, are the following, except:
A. Chronic diseases of lungs with the evident pulmonary-cardiac insufficiency
B. *Essential hypertension of II stage
C. Bronchial asthma of severe form with the evident disturbance of breathing and circulation of blood
D. Active forms of tuberculosis of any localization
E. Stomach and duodenal ulcer with chronic recurrent course and susceptibility to complications
524. How does antibiotic therapy increase the risk of infection?
A. Most intravenous antibiotics contain high concentrations of glucose, providing an ideal medium for
bacterial proliferation
B. The associated diarrhea causes ulcerations in the intestinal tract, creating a new portal of entry
C. *Antibiotic therapy also kills normal flora, which provide a means of protection from infection
D. The resulting dead bacteria form a rich culture for development of viruses
E. Non of above
525. How does prostacycline therapy reduce the pathologic problems of primary pulmonary hypertension?
A. *Increasing lung blood flow by dilating pulmonary blood vessels
B. Improving ventilation by increasing the contractility of the diaphragm
C. Increasing gas exchange by changing the elasticity of the alveolar
D. Protecting the heart from development of cor pulmonale by increasing cardiac muscle strength
E. Non of above
526. How does the drug oseltamivir (Tamiflu) prevent influenza or shorten the duration of illness?
A. *Preventing the virus from entering respiratory cells
B. Boosting the patient’s immune system
C. Inhibiting the virus from multiplying
D. Directly killing the virus
E. Non of above
527. How long is the usual course of drug treatment for a patient with active tuberculosis?
A. 7 to 10 days
B. 6 weeks
C. *6 months
D. 2 years
E. Non of above
528. In pneumonia due to Mycoplasma all of the following are true except:
A. *Bilateral infiltration on chest X-ray
B. Myalgia
C. Dry cough
D. Pleuritic chest pain
E. All of the above
529. In restrictive lung disease :
A. *FEV1/FVC is high
B. FVC is high
C. FEV1 is high
D. All of the above
E. Nothing of the above
530. In what specific way does the respiratory system function contribute to acid-base balance?
A. Prevents excessive loss of hydrogen ions by evaporation
B. Increases the potassium ion content in bronchial secretions
C. *Removes carbon dioxide generated as a result of metabolism
D. Maintains body water levels through mucous membrane function
E. Non of above
531. In which patient with chronic airflow limitation would you expect to find dependent edema?
A. 25-year-old with cold-induced asthma
B. 65-year-old with exercise-induced asthma
C. *55- year-old with chronic bronchitis
D. 45-year-old with moderate emphysema
E. Non of above
532. Most common sign of aspiration pneumonitis:
A. *Tachypnoe
B. Bronchospasm
C. Cyanosis
D. Crepitations
E. Nothing of the above
533. Mycoplasma infection simulates:
A. *Viral pneumonia
B. Pnemuococcal pneumonia
C. Pulmonary oedema
D. Pulmonary infarction
E. Nothing of the above
534. Paradoxical breathing is charateristic of:
A. *Pneumonia
B. Pneumothorax
C. Atelectasis
D. Pleurisy
E. All of the above
535. Patient of 40 years old has pneumonia. On his chest X-ray film there is local shadow in some
segments of right lower lobe. What data at percussion and auscultation may be founded in this area?
A. *Shortened percussion sound, moist rales
B. Band-box percussion sound, vesicular breathing
C. Percussion sound is not changed. Dry wheezes
D. Tympanic sound. Amphoric breathing
E. Percussion sound is not changed. Vesicular breathing
536. Pneumothorax is a possible complication of
A. *Staphylococcal pneumonia
B. Pneumococcal pneumonia
C. Klebsiella pneumonia
D. Viral pneumonia
E. All of the above
537. Primary atypical pneumonia is caused by:
A. *Mycoplasma
B. Mycobaterium kansaii
C. Photochromogens
D. Pneumocystis carinii
E. Nothing of the above
538. Pulmonary surfactant is secreted by:
A. Type I pneumocytes
B. *Type II pneumocytes
C. Lymphocytes
D. Bronchial epithelial cells
E. Tracheal epithelial cells
539. Restrictive lung disease is associated with :
A. *Interstitial lung disease
B. High residual lung volume
C. High PCO2
D. FEV1 below 50 %
E. Nothing of the above
540. Silicatosis is:
A. *Asbestosis
B. Siderosis
C. Aluminosis
D. Bisinosis
E. All of the above
541. The major sign of hypoventilation is
A. *Cyanosis
B. Hypoxia
C. Hypercapnia
D. Dyspnea
E. All of the above
542. The most common community acquired infection is :
A. *Streptococcal pneumoniae
B. Hepatitis A
C. Cholera
D. Meningitis
E. Nothing of the above
543. The patient is taking enalapril (Vasotec), an angiotensin-converting enzyme (ACE) inhibitor, for
hypertension. Which respiratory side effect should you teach the patient to expect?
A. Wheezing on exhalation
B. Increased nasal stuffiness
C. Chest pressure or pain
D. *Persistent dry cough
E. Non of above
544. The patient with manifestations of a respiratory infection is suspected of having SARS. In addition to
standard precautions, what other infection control precautions should the physician use until the diagnosis is
certain?
A. Airborne precautions
B. Droplet precautions
C. *Airborne precautions and contact precautions
D. Droplet precautions and contact precautions
E. Non of above
545. The typical feature of interstitial lung disease is:
A. *End inspiratory rales
B. Expiratory rales
C. Inspiratory rhonchi
D. Expiratory rhonchi
E. All of the above
546. Amphoric breathing is seen in:
A. *Tubercular cavity
B. Pleural effusion
C. Dry pleurisy
D. Pneumonia infiltration
E. All of the above
547. Virus causing pneumonia are all except:
A. *Mumps
B. Cytomegalovirus
C. Measles
D. Retrovirus
E. Nothing of the above
548. What is necessary to make during periodic medical examination to all workers, who contact with
occupational dust?
A. General blood analyses
B. Biochemical blood analyses
C. ECG
D. Ultrasound examination
E. *Roentgenography of chest organs
549. What is the main purpose of a negative-pressure ventilator?
A. Healing diseased lung tissue
B. Relieving hypoxemia by opening obstructed airways
C. *Assisting ventilation to healthy lungs by mimicking normal chest pressures
D. Delivering an individualized preset tidal volume to the lower respiratory tract
E. Non of above
550. What is the major difference in pathophysiology between asthma and COPD?
A. Asthma is a restrictive disorder and COPD is an obstructive disorder
B. *COPD results in permanent airflow obstruction and asthma is a condition of reversible airflow
obstruction
C. COPD is caused by chronic exposure to inhalation irritants and the major cause of asthma is cigarette
smoking
D. Asthma is the result of an inflammatory process and COPD is a result of inflammatory, infectious, and
hyperresponsive process
E. Non of above
551. What is the mechanism of reduced gas exchange in a patient who has atelectasis?
A. Airway obstruction
B. *Reduced alveolar surface area
C. Failure of pulmonary circulation to fully perfuse lung tissue
D. Increased bronchial secretions filling the alveoli with fluid rather than with air
E. Non of above
552. What is the most important intervention for the patient with acute respiratory distress syndrome?
A. Antibiotic therapy
B. Bronchodilators
C. *Oxygen therapy
D. Diuretic therapy
E. Non of above
553. What is the physiologic consequences of CO2 narcosis?
A. Excessive sleepiness in the patient with hypercarbia
B. *Failure of rising blood levels of CO2 to trigger more rapid and deeper respirations
C. A change in the ventilation-perfusion ratio, in which ventilation exceeds perfusion
D. Increase in the percentage of oxygen delivered to the patient does not result in an increased PaO2
E. Non of above
554. What is the primary cause for the increased incidence in lung cancer among women during the past
20 years?
A. Women now have a longer life expectancy than men
B. *Cigarette smoking among women increased dramatically 50 years ago
C. More women now work in the industrial setting than in previous decades
D. Increased use of oral contraceptives combined with air pollution has driven the incidence of lung cancer
up in women
E. Non of above
555. What is the role of surfactant in pulmonary function?
A. *Reduces alveolar surface tension, improving gas exchange
B. Increases alveolar surface tension, improving gas exchange
C. Dilates pulmonary blood vessels, decreasing pulmonary vascular resistance
D. Relaxes bronchial smooth muscle, decreasing pulmonary vascular resistance
E. Non of above
556. What part of the pulmonary system is most affected by asthma?
A. Alveoli
B. Larynx
C. Pharynx
D. *Airways
E. Non of above
557. Which assessment finding in a patient who has been medicated during an asthma attack indicates to
you that the therapy should be modified?
A. Peak expiratory rate flow 10% below expected value
B. Presence of bilateral tactile fremitus
C. *Suprasternal retraction on inhalation
D. Trachea at the midline
E. Non of above
558. Which assessment finding in a patient with severe dyspnea indicates to you that the respiratory
problem is chronic?
A. Wheezing on exhalation
B. Productive cough
C. *Clubbed fingers
D. Cyanosis
E. Non of above
559. Which assessment finding should you expect in the patient with COPD?
A. *Decreased vocal fremitus
B. Grossly bloody sputum
C. Loss of the gag reflex
D. Tracheal deviation
E. Non of above
560. Which blood gas value indicates that the patient is experiencing hypercarbia?
A. pH = 7.33
B. Bicarbonate = 20 mEq/L
C. *PaCO2 = 60 mm Hg
D. PaO2 = 80 mm Hg
E. Non of above
561. Which clinical change indicates the patient's COPD is becoming worse?
A. Pulse pressure has increased from 35 mm Hg to 40 mm Hg
B. *The patient has had an unplanned weight loss of 15 pounds
C. The patient's average respiratory rate has decreased from 34 to 33
D. The patient's Pa CO2 is 58 mm Hg this month compared with 64 mm Hg last month
E. Non of above
562. Which clinical manifestation alerts you to the presence of hypoventilation when you are monitoring a
patient with chronic lung disease and hypercarbia who is receiving oxygen therapy?
A. Coarse crackles and wheezes on auscultation
B. *Slow, shallow respirations
C. Pulse oximetry of 90%
D. Clubbing of the fingers
E. Non of above
563. Which clinical manifestation in a patient with chronic bronchitis indicates to you a worsening of the
patient's respiratory condition?
A. Fatigue
B. Cachexia
C. *Confusion
D. Slow capillary refill
E. Non of above
564. Which clinical manifestation in a patient with long-standing COPD alerts you to the possibility of cor
pulmonale?
A. Pursed-lip breathing occurs when the patient is at rest
B. The patient's neck muscles are enlarged and prominent
C. The patient's ECG shows tall, peaked T waves and an absent U wave
D. *Jugular venous distention is present when the patient is in a sitting position
E. Non of above
565. Which clinical manifestation in an older patient with pneumonia indicates that the disease is
responding to the therapeutic regimen?
A. The patient does not have a cough
B. Urine output is 900 ml for the day
C. *Pulse oximetry shows an oxygen saturation of 90%
D. Tactile fremitus is increased over the affected lung fields
E. Non of above
566. Which complication is most frequent in patients with silicosis?
A. Tromboembolia of pulmonary artery
B. Bronchial asthma
C. Pneumonia
D. Cancer of lungs
E. *Tuberculosis
567. Which diagnostic indicator confirms the presence of active tuberculosis?
A. Positive tuberculine test
B. The presence of calcified lesions on chest x-ray
C. *The presence of M. tuberculosis in a sputum culture
D. The combined clinical manifestations of weight loss, night sweats, fever, and cough productive of
mucopurulent bloody sputum
E. Non of above
568. Which disorder is an example of a "restrictive" pulmonary disease?
A. Asthma
B. Bronchitis
C. Lung cancer
D. *Pulmonary fibrosis
E. Non of above
569. Which drug is more commonly used as therapy for COPD than as therapy for asthma?
A. Theophylline (Theo-Dur)
B. Montelukast (Singulair)
C. *Guaifenesin (Organidin)
D. Salmeterol (Serevent)
E. Non of above
570. Which manifestations in a patient receiving oxygen therapy at 60% for more than 24 hours alerts you
to the possibility of oxygen toxicity?
A. *Increased dyspnea
B. Decreased rate and depth of respiration
C. Wheezing on inhalation and exhalation
D. Increased excretion of thick, white, frothy sputum
E. Non of above
571. Which of the following patients could be expected to require mechanical ventilation longterm?
A. 27-year-old with status asthmaticus
B. 45-year-old with morphine overdose
C. *24-year-old with muscular dystrophy
D. 65-year-old with bilateral bacterial pneumonia
E. Non of above
572. Which one of the following organisms causes pneumatocele ?
A. *Staphylococcus aureus
B. Klebsiella pneumoniae
C. Haemophilus influenzae
D. Streptococcus pneumoniae
E. All of the above
573. Which patient is a greatest risk for contracting SARS?
A. *The 30-year-old physician providing direct care to patients with SARS
B. The 50-year-old farmer working directly with cows and pigs
C. The 60-year-old patient with type 2 diabetes mellitus and renal insufficiency
D. The 70-year-old patient residing in an assisted living environment
E. Non of above
574. Which patient is a greatest risk for developing nosocomial pneumonia?
A. *The 60-year-old patient receiving mechanical ventilation
B. The 40-year-old patient receiving antibiotics for a surgical wound infection
C. The 60-year-old patient in traction for a fractured femur who also has a cold
D. The 40-year-old patient with type 2 diabetes who has a 50 pack-year smoking history
E. Non of above
575. Which patient is at greatest risk for acute respiratory distress syndrome?
A. The 62-year-old with COPD who has pneumonia
B. *The 22-year-old who received 10 units of blood after a motor vehicle crash
C. The 78-year-old with chronic congestive heart failure and pulmonary edema
D. The 24-year-old with asthma who has not taken any of her asthma medications for 2 weeks
E. Non of above
576. Which patient is at greatest risk for developing a “community-acquired” pneumonia?
A. The 40-year-old first-grade teacher
B. *The 60-year-old smoker who is also an alcoholic
C. The 75-year-old with exercise-induced wheezing
D. The 35-year-old aerobics instructor who skips meals and eats only vegetables
E. Non of above
577. Which patient should the physician caution to avoid taking over-the-counter decongestants for
manifestations of a cold or flu?
A. 25-year-old man with a latex allergy
B. *45-year-old woman with hypertension
C. 32-year-old woman who is taking oral contraceptives
D. 65-year-old man who has had type 1 diabetes mellitus for 20 years
E. Non of above
578. Which set of arterial blood gas values indicates early pneumonia as the respiratory problem?
A. *pH 7.35, HCO3– 22 mEq/L, PCO2 45 mm Hg, PO2 86 mm Hg
B. pH 7.30, HCO3– 22 mEq/L, PCO2 60 mm Hg, PO2 92 mm Hg
C. pH 7.32, HCO3– 17 mEq/L, PCO2 25 mm Hg, PO2 98 mm Hg
D. pH 7.30, HCO3– 28 mEq/L, PCO2 65 mm Hg, PO2 75 mm Hg
E. Non of above
579. Which set of arterial blood gases would the physician expect to find in a patient who developed a
pulmonary embolism 15 minutes ago?
A. pH 7.30, HCO3– 22 mEq/L, PCO2 60 mm Hg, PO2 66 mm Hg
B. pH 7.38, HCO3– 22 mEq/L, PCO2 45 mm Hg, PO2 96 mm Hg
C. *pH 7.47, HCO3– 23 mEq/L, PCO2 25 mm Hg, PO2 82 mm Hg
D. pH 7.30, HCO3– 28 mEq/L, PCO2 65 mm Hg, PO2 75 mm Hg
E. Non of above
580. Which type of acid-base imbalance is most common among patients with COPD?
A. *Respiratory acidosis
B. Respiratory alkalosis
C. Metabolic acidosis
D. Metabolic alkalosis
E. Non of above
581. With which of the following theophylline has an antagonistic interaction?
A. Histamine receptors.
B. Bradykinin receptors.
C. *Adenosine receptors.
D. Imidazoline receptors.
E. Beta-adrenoreceptors
582. Common X-ray finding of Staphylococcal pneumonia:
A. *Bilateral hilar lymphadenopathy
B. Involvement of lower lobes
C. Dense homogenous consolidation
D. Visible air bronchogram
E. Nothing of the above
583. Above the cavity of abscess it is possible to hear such breathing
A. *Bronchial
B. Vesicular
C. Unclear
D. Hard
E. Weakened
584. Acute respiratory failure does not happen due to:
A. *Pb poisoning
B. Porphyria
C. Myasthenia gravis
D. Polio
E. Nothing of the above
585. All of the following are true about type I respiratory failure except:
A. *Normal A-a gradient
B. Decreased Pa O2
C. Decreased Pa CO2
D. Normal Pa CO2
586. All the following may be the causes of hemorrhagic pleural effusion except:
A. *Congestive heart failure
B. Pulmonary embolism
C. Lung cancer
D. Tuberculosis
E. Severe chest trauma
587. Alveolar hypoventilation is observed in:
A. *Bronchiectasis
B. Guillain-Barre syndrome
C. Status asthmaticus
D. Chronic bronchitis
E. Nothing of the above
588. Acute abscess and gangrene of lungs belong to
A. *Pyogenic diseases of lungs
B. Chronic unspecific diseases of lungs
C. Congenital pathology of lungs
D. Obstructive diseases of lungs
E. Systemic diseases
589. Best method for detecting minimal bronchiectasis is:
A. *CT scan
B. Bronchogram
C. Radionuclide lung scan
D. Chest X-ray
E. Nothing of the above
590. Best position to reveal small pleural effusions on chest X-ray is
A. *Lateral decubitus view
B. AP view
C. PA view
D. Lateral view
E. All of the above
591. Bilateral malignant pleural effusion is most often seen in:
A. *Cancer of lung
B. Cancer of breast
C. Mesothelioma
D. Lymphoma
E. Nothing of the above
592. Bilateral pleural effusion is seen in:
A. Congestive cardiac failure
B. Nephritic syndrome
C. Constrictive pericarditis
D. *All of the above
E. Nothing of the above
593. Bronchiectasis is most common in such lung lobe:
A. *Left lower
B. Right middle
C. Right upper
D. Left upper
E. Nothing of the above
594. In which lung lobe bronchiectasis is most commonly present:
A. *Left lower lobe
B. Right upper lobe
C. Right middle lobe
D. Left upper lobe
E. All of the above
595. Causes of haemorrhagic pleural effusion are all except:
A. *Bronchial adenoma
B. Pulmonary infarction
C. Mesothelioma
D. Tuberculosis
E. Nothing of the above
596. Cavity in case of lung abscess is
A. *Separated from pulmonary tissue and connected with a bronchus
B. Connected with a bronchus
C. Not connected with a bronchus
D. Connected with interstitium of lungs
E. Not separated from pulmonary interstitium
597. Chronic abscess is a result of
A. *Incomplete treatment of acute abscess
B. Initial gangrene of lungs
C. Presence of chronic bronchitis
D. Initially chronic disease
E. Joining of chronic pneumonia
598. Complication of bronchiectasis are all except:
A. *Bronchiogenic carcinoma
B. Cerebral abscess
C. Lung abscess
D. Amyloidosis
E. Nothing of the above
599. Complication of gangrene of lungs is
A. *Ichorous empyema
B. Pulmonary infarction
C. Exudative pleurisy
D. Pneumonia with abscesses
E. Dry pleurisy
600. Cyanosis is seen if :
A. Reduced HB 5 gm %
B. SulfHB0.5gm%
C. Meth HB 1.5 gm %
D. *All of the above
E. Nothing of the above
601. Dilatation of bronchi in patients with bronchiectasis may have such form
A. *Cylindric
B. Oval
C. Pyramidal
D. Cubic
E. Conic
602. Dilatation of large bronchi with destruction of bronchial wall is seen in:
A. *Bronchiectasis
B. Emphysema
C. Pneumonia
D. Squamous cell carcinoma lung
E. All of the above
603. False statement about type I respiratory failure
A. *Normal A-a gradient
B. Decreased Pa O2
C. Decreased Pa CO2
D. Normal Pa CO2
E. All of the above
604. In case of bronchiectasis there is
A. *Dilatation of bronchi
B. Narrowing of bronchi
C. Destruction of pulmonary tissue
D. Obstructive emphysema of lungs
E. Destruction of tissue of bronchial wall
605. In Kartagener's syndrome all the next changes are seen except:
A. *Cystic fibrosis
B. Dextrocardia
C. Sinusitis
D. Absence of cilia
E. All of the above
606. In the case of gangrene of lungs temperature is
A. *Hectic
B. Decreased
C. Normal
D. Subfebrile
E. Remittent
607. In the period of exacerbation of bronchiectasis there is:
A. *Increased amount of purulent sputum
B. Presence of peripheral oedema
C. Increase of dyspnea
D. Increase of cough
E. Increase of pain in a thorax
608. In type - II respiratory failure, there is:
A. *Low pO2 and high pCO2
B. Low pO2 and low pCO2
C. Normal pO2 and high pCO2
D. Low pO2 and normal pCO2
E. All of the above
609. In what specific way does the respiratory system function contribute to acid-base balance?
A. Prevents excessive loss of hydrogen ions by evaporation
B. Increases the potassium ion content in bronchial secretions
C. *Removes carbon dioxide generated as a result of metabolism
D. Maintains body water levels through mucous membrane function
E. Non of above
610. In which patient with chronic airflow limitation would you expect to find dependent edema?
A. 25-year-old with cold-induced asthma
B. 65-year-old with exercise-induced asthma
C. *55- year-old with chronic bronchitis
D. 45-year-old with moderate emphysema
E. Non of above
611. Possible complication of bronchiectasis
A. *Amyloiodosis of kidneys
B. Myocardial infarction
C. Pulmonary infarction
D. Chronic glomerulonephritis
E. Bronchial asthma
612. Kartagener's syndrome is not associated with:
A. *Sinusitis
B. Situs inversus
C. Subluxation of lens
D. Bronchiectasis
E. All of the above
613. Low glucose in pleural effusion is seen in all except
A. *Dressler's syndrome
B. Rheumatoid arthritis
C. Empyema
D. Malignant pleural effusion
E. Nothing of the above
614. Lung abscess in children may be produced by all of the following organisms except:
A. *Pneumococcus
B. Entamoeba histolytica
C. Klebsiella pneumoniae
D. Staphylococcus aureus
E. All of the above
615. Lung abscess is least likely complication of:
A. *Bronchopneumonia
B. Lobar pneumonia
C. Malignancy
D. Bronchiectasis
E. All of the above
616. Lung abscesses are least likely to be caused by:
A. *Metastatic malignancy
B. Histoplasma
C. Staphylococcus
D. Primary carcinoma
E. All of the above
617. The most likely precursor to bronchiectasis is:
A. *Necrotising pneumonia
B. Tuberculosis
C. Carcinoma
D. Bronchial adenoma
E. All of the above
618. The most unlikely cause of pleural effusion is:
A. *Thyroid tumors
B. Bronchial carcinoma
C. Carcinoma breast
D. Lymphoma
E. Nothing of the above
619. Patient 60 years old, who is in a hospital with left-side exudative pleurisy, develops rapid
accumulation of exudate after each punction. What disease may be possible?
A. *Blastomatous process
B. Dresler syndrome
C. Postpneumonic pleurisy
D. Tuberculosis of pleura
620. Peculiarity of sputum in the case of abscess
A. *Two-lays
B. Homogenous
C. Odourless
D. Foamy
E. Dense
621. Persistent coarse crepitations in the chest are common for:
A. *Bronchiectasis
B. Pulmonary TB
C. Pulmonary oedema
D. Cavity in lung
E. All of the above
622. Pleural effusion in rheumatoid arthritis is typically associated with the following features except:
A. *Glucose > 60 mg/dl
B. Protein > 3 gm/ml
C. Pleural fluid to serum protein ratio of > 0.6
D. Pleural fluid to serum LDH ratio of > 0.6
E. All of the above
623. Pleural fluid low in glucose in seen in all except:
A. *Tuberculosis
B. Rheumatoid arthritis
C. Mesothelioma
D. Empyema
E. Nothing of the above
624. Respiratory failure type I consists of
A. *Low Pa O2, normal or low Pa CO2
B. Raised Pa CO2, low Pa O2
C. Pa O2, low Pa O2
D. Pa O2 and PaCO2- high
E. All of the above
625. Sputum examination of a patient with bronchiectasis most likely will reveal:
A. *Pseudomonas
B. Pneumococcus
C. Klebsiella
D. Anaerobic bacteria
E. All of the above
626. Sputum in the case of gangrene of lungs is
A. *Three-layered
B. Homogenous
C. Two-layered
D. Multi-layered
E. Foamy
627. Stony dull note on percussion is characteristic of :
A. *Pleural effusion
B. Consolidation
C. Pleurisy
D. Tuberculous cavity
E. All of the above
628. The most common aerobic organism causing lung abscess is
A. *Staphylococcus
B. Pneumococcus
C. Klebsiella
D. coli
E. Nothing of the above
629. The most frequent cause of peripheral cyanosis is:
A. *Slowed circulation through the skin
B. Interatrial septal defect
C. Methaemoglobinaemia
D. Interstitial pulmonary fibrosis
E. All of the above
630. Transudative pleural effusion is present in all except:
A. *Pulmonary embolism
B. Liver cirrhosis
C. Chronic heart failure
D. Chronic kidney failure
E. Nothing of the above
631. Type II respiratory failure is seen in :
A. *COPD with cor pulmonale
B. Chronic renal failure
C. Adult repiratory distress syndrome
D. Pulmonary fibrosing alveolitis
E. All of the above
632. What is the main purpose of a negative-pressure ventilator?
A. Healing diseased lung tissue
B. Relieving hypoxemia by opening obstructed airways
C. *Assisting ventilation to healthy lungs by mimicking normal chest pressures
D. Delivering an individualized preset tidal volume to the lower respiratory tract
E. Non of above
633. What is the mechanism of reduced gas exchange in a patient who has atelectasis?
A. Airway obstruction
B. *Reduced alveolar surface area
C. Failure of pulmonary circulation to fully perfuse lung tissue
D. Increased bronchial secretions filling the alveoli with fluid rather than with air
E. Non of above
634. What is the most important intervention for the patient with acute respiratory distress syndrome?
A. Antibiotic therapy
B. Bronchodilators
C. *Oxygen therapy
D. Diuretic therapy
E. Non of above
635. Which assessment finding in a patient with severe dyspnea indicates to you that the respiratory
problem is chronic?
A. Wheezing on exhalation
B. Productive cough
C. *Clubbed fingers
D. Cyanosis
E. Non of above
636. Which blood gas value indicates that the patient is experiencing hypercarbia?
A. pH = 7.33
B. Bicarbonate = 20 mEq/L
C. *PaCO2 = 60 mm Hg
D. PaO2 = 80 mm Hg
E. Non of above
637. Which clinical manifestation alerts you to the presence of hypoventilation when you are monitoring a
patient with chronic lung disease and hypercarbia who is receiving oxygen therapy?
A. Coarse crackles and wheezes on auscultation
B. *Slow, shallow respirations
C. Pulse oximetry of 90%
D. Clubbing of the fingers
E. Non of above
638. Which clinical manifestation in a patient with long-standing COPD alerts you to the possibility of cor
pulmonale?
A. Pursed-lip breathing occurs when the patient is at rest.
B. The patient's neck muscles are enlarged and prominent.
C. The patient's ECG shows tall, peaked T waves and an absent U wave.
D. *Jugular venous distention is present when the patient is in a sitting position.
E. Non of above
639. Which disorder is an example of a "restrictive" pulmonary disease?
A. Asthma
B. Bronchitis
C. Lung cancer
D. *Pulmonary fibrosis
E. Non of above
640. Which manifestations in a patient receiving oxygen therapy at 60% for more than 24 hours alerts you
to the possibility of oxygen toxicity?
A. *Increased dyspnea
B. Decreased rate and depth of respiration
C. Wheezing on inhalation and exhalation
D. Increased excretion of thick, white, frothy sputum
E. Non of above
641. Which of the following is NOT a complication of bronchiectasis :
A. *Lung cancer
B. Lung abscess
C. Amylodosis
D. Empyema
E. All of the above
642. Which of the following patients could be expected to require mechanical ventilation longterm?
A. 27-year-old with status asthmaticus
B. 45-year-old with morphine overdose
C. *24-year-old with muscular dystrophy
D. 65-year-old with bilateral bacterial pneumonia
E. Non of above
643. Which patient is a greatest risk for contracting SARS?
A. *The 30-year-old physician providing direct care to patients with SARS
B. The 50-year-old farmer working directly with cows and pigs
C. The 60-year-old patient with type 2 diabetes mellitus and renal insufficiency
D. The 70-year-old patient residing in an assisted living environment
E. Non of above
644. Which patient is at greatest risk for acute respiratory distress syndrome?
A. The 62-year-old with COPD who has pneumonia
B. *The 22-year-old who received 10 units of blood after a motor vehicle crash
C. The 78-year-old with chronic congestive heart failure and pulmonary edema
D. The 24-year-old with asthma who has not taken any of her asthma medications for 2 weeks
E. Non of above
645. A 30 year old man had road traffic accident and sustained fracture of femur. Two days later he
developed sudden breathlessness. The most probable cause can be:
A. Pneumonia
B. Congestive heart failure
C. Bronchial asthma
D. *Fat embolism
E. Acute bronchitis
646. Man, 48 y.o., has severe attack of expiratory dyspnea, with intensive dry cough, distance wheezes,
palpitations. What medicine is the best one for the first aid?
A. *Salbutamol
B. Theophyllin
C. Lasolvan
D. Atrovent
E. Prednisone
647. What onset is typical for pleuropneumonia?
A. *Acute
B. Latent
C. Fulminant
D. Gradual
E. Non of the above
648. What onset is typical for bronchopneumonia?
A. Acute
B. Latent
C. Fulminant
D. *Gradual
E. Non of the above
649. The patient is a 48-year-old woman who has never smoked and has just been diagnosed with lung
cancer. Which type is she most likely to have?
A. *Adenocarcinoma
B. Large cell carcinoma
C. Small cell carcinoma
D. Squamous cell carcinoma
E. Non of above
650. The patient is taking enalapril (Vasotec), an angiotensin-converting enzyme (ACE) inhibitor, for
hypertension. Which respiratory side effect should you teach the patient to expect?
A. Wheezing on exhalation
B. Increased nasal stuffiness
C. Chest pressure or pain
D. *Persistent dry cough
E. Non of above
651. The patient who has experienced blunt trauma to the chest is at risk for developing a hemothorax.
Which would the physician expect to find in a patient with a hemothorax?
A. Hemoptysis
B. Paradoxical chest movements
C. *Percussion dullness on affected side
D. Hypertympanic sound on affected side
E. Non of above
652. Which clinical manifestation in an older patient with pneumonia indicates that the disease is
responding to the therapeutic regimen?
A. The patient does not have a cough
B. Urine output is 900 ml for the day
C. *Pulse oximetry shows an oxygen saturation of 90%
D. Tactile fremitus is increased over the affected lung fields
E. Non of above
653. Which patient is at greatest risk for developing a “community-acquired” pneumonia?
A. The 40-year-old first-grade teacher
B. *The 60-year-old smoker who is also an alcoholic
C. The 75-year-old with exercise-induced wheezing
D. The 35-year-old aerobics instructor who skips meals and eats only vegetables
E. Non of above
654. Your patient with asthma is receiving aminophylline intravenously. Which manifestation alerts you to
the possibility of aminophylline toxicity?
A. Pulse oximetry of 93%
B. *Increased restlessness
C. Hourly urine output of 45 ml
D. Heart rate increase from 72 to 84 beats per minute
E. Non of above
655. A 55-yr-old smoker with a long history of recurrent chest infection presents with haemoptysis and
greenish sputum. On examination he has clubbing and coarse crepitations over the bases of both lungs. What
is the cause?
A. *Bronchiectasis
B. Pulmonary infarction
C. Bronchogenic carcinoma
D. Pneumonia
E. Foreign body inhalation
656. A middle-aged smoker presents with chronic cough and phlegm. His sputum is tenacious but not
yellow or blood stained. His chest is hyperinflated. His arterial carbon dioxide is high and is arterial oxygen
is low. What changes may be found on spyrogram?
A. *FEV1, FVC, Typhno index are low
B. FEV1, FVC, Typhno index are high
C. FEV1, FVC, Typhno index are normal
D. FEV1, FVC are high, Typhno index is low
E. FEV1, FVC are low, Typhno index is high
657. An old woman complains of wheeze, dyspnoea and cough. She cannot sleep at night because of a
chronic cough. Her CXR suggests hyperinflation. Spyrography: FEV1, FVC, Typhno index are low. What is
the previous diagnosis?
A. *COPD
B. Bronchogenic carcinoma
C. Emphysema
D. Respiratory failure
E. Bronchial asthma
658. Patient D., 60 years old. Carcinoma of right main bronchus was suspected. What is the nesessary
method of investigation?
A. *Fiberoptic bronchoscopy
B. Sputum culture
C. Mediastinoscopy
D. Spyrography
E. General blood analysis
659. The 82-year-old patient has a pulmonary infection. Which action addresses the age-related change of
increased vascular resistance to blood flow through pulmonary vasculature in this patient?
A. Encouraging the patient to turn, cough, and deep breathe every hour
B. *Assessing the patient's level of consciousness
C. Raising the head of the bed
D. Humidifying the oxygen
E. Non of above
660. The patient who has experienced blunt trauma to the chest is at risk for developing a hemothorax.
Which would the physician expect to find in a patient with a hemothorax?
A. Hemoptysis
B. Paradoxical chest movements
C. *Percussion dullness on affected side
D. Hypertympanic sound on affected side
E. Non of above
661. A 10-yr-old boy with cystic fibrosis presents with pneumonia. What is the most common cause of
pneumonia in patients with cystic fibrosis?
A. *Pseudomonas aeroginosa
B. Chlamydia psittacci
C. Coxiella burnetti
D. Staphylococcus aureus
E. Pneumocystis carinii
662. A 12-yr-old boy with cystic fibrosis presents with a chest infection. The boy also suffers from mild
renal failure. Choose the most appropriate treatment
A. *Ciprofloxacin
B. Erythromycm
C. Tetracycline
D. Flucoxacillin
E. Isoniazid
663. A 20-yr-old woman presents with a week's history of fever, rigors arid productive rusty cough. CXR
shows right lower lobe consolidation. What is the previous diagnosis?
A. *Pneumonia
B. COPD
C. Emphysema
D. Bronchial asthma
E. Pulmonary fibrosis
664. A 20-yr-old woman presents with a week's history of fever, rigors arid productive rusty cough. CXR
shows right lower lobe consolidation. Prescribe treatment.
A. *Azitromycin
B. Co-trimoxazole
C. Prednisolone
D. Penicillin
E. Salbutamol inhaler
665. A 25-yr-old man has a 3-day history of shivering, general malaise and productive cough. The chest
X-ray shows right lower lobe consolidation. What is the possible diagnosis?
A. *Pneumonia
B. Emphysema
C. COPD
D. Lung abscess
E. Respiratory failure
666. A 25-yr-old man has a 3-day history of shivering, general malaise and productive cough. The chest
X-ray shows right lower lobe consolidation. What is the most possible cause of the disease?
A. *Streptococcus pneumoniae
B. Mycobacterium tuberculosis
C. Coxiella burnetti
D. Pneumocystis carinii
E. Bacteroides fragilis
667. A 30 year old man had road traffic accident and sustained fracture of femur. Two days later he
developed sudden breathlessness. The most probable cause of his state:
A. Pneumonia
B. Congestive heart failure
C. Bronchial asthma
D. *Fat embolism
E. Acute bronchitis
668. A 30-yr-old man with HIV presents with a productive cough and haemoptysis. CXR shows a round
ball in the right upper lobe surmounted by a dome of air. Which infection is the most possible cause of his
state?
A. *Aspergillosis
B. Chlamydia psittacci
C. Coxiella burnetti
D. Staphylococcus aureus
E. Pneumocystis carinii
669. A 30-yr-old woman has just returned from a holiday in Cyprus. She complains of a dry cough, fever,
malaise. A CXR shows patchy consolidation. Which infection is the most possible cause of her state?
A. *Legionella pneumoniae
B. Haemophilus influenzae
C. Chlamydia trachomatis
D. Pneumocystis carinii
E. Klebsiella pneumoniae
670. A 30-yr-old woman presents with a 3-day history of pyrexia, rigors and sweats. This was preceded by
a 1 week history of cough and increasing dyspnea. She had a swinging fever and dullness to percussion at
the left lung base. Previous diagnosis?
A. *Pneumonia
B. Emphysema
C. Tuberculosis
D. Bronchial carcinoma
E. Mesothelioma
671. A 35-yr-old pet shop owner presents with high fever, excruciating headache and a dry hacking cough.
The CXR shows patchy consolidation. Which infection is the most possible cause of his state?
A. *Chlamydia psittaci
B. Coxiella burnetti
C. Staphylococcus aureus
D. Pneumocystis carinii
E. Aspergillosis
672. A 37-yr-old man presents with dyspnoea, cough, weight loss and night sweats. His CXR shows
diffuse bilateral infiltrates. Which infection is the most possible cause of his state?
A. *Chlamydia psittaci
B. Mycoplasma pneumoniae
C. Pneumocystis carinii
D. coli
E. Pseudomonas aeruginosa
673. A 41-yr-old drug abuser presents with fever, cough and breathlessness. This was preceded by viral
influenza. Chest radiograph shows multiple abscesses. Which infection is the most possible cause of his
state?
A. *Staphylococcus aureus
B. Crytococcus
C. Streptococcus pneumoniae
D. Legionella pneumoniae
E. Mycobacterium avium
674. A 45-yr-old farmer presents with flu-like illness anorexia and dry cough. His CXR shows patchy
consolidation. Which infection is the most possible cause of his state?
A. *Mycoplasma pneumoniae
B. Pneumocystis carinii
C. Chlamydia psittaci
D. coli
E. Pseudomonas aeroginosa
675. A 70-yr-old woman presents with confusion and productive cough. CXR shows right lower lobe
consolidation. What is the possible cause of the disease?
A. *Streptococcus pneumoniae
B. Chlamydia psittaci
C. Staphylococcus aureus
D. Pneumocystis carinii
E. Mycobacterium tuberculosis
676. A 70-yr-old woman presents with confusion and productive cough. CXR shows right lower lobe
consolidation. What is the possible diagnosis?
A. *Pneumonia
B. COPD
C. Emphysema
D. Pulmonary embolism
E. Asthma
677. A chronic smoker with COPD develops a fever. He reports bringing up green phlegm on coughing.
Which infection is the most possible cause of his state?
A. *Haemophilus influenzae
B. Mycoplasma tuberculosis
C. Haemophilus influenzae
D. Pneumocystis carinii
E. Klebsiella pneumoniae
678. A person who has high fever, tachycardia, haemoptysis and a lobar consolidation on CXR has:
A. *Lobar pneumonia
B. Pleurisy
C. Pulmonary oedema
D. Pulmonary infarction
E. All of the above
679. A young male homosexual with Kaposi's sarcoma complains of increasing breathlessness and a dry
cough. The X-ray shows right lower lobe consolidation. Which infection is the most possible cause of his
state?
A. *Pneumocystis carinii
B. Mycoplasma tuberculosis
C. Haemophilus influenzae
D. Chlamydia trachomatis
E. Klebsiella pneumoniae
680. Drug abuser, a 41-yr-old man, presents with fever, cough and breathlessness. This was preceded by
viral influenza. Chest radiograph shows multiple abscesses. What is the most possible etiology of disease?
A. *Staphylococcus aureus
B. Cryptococcus
C. Streptococcus pneumoniae
D. Legionella pneumoniae
E. Mycobacterium avium
681. Patient R., 48 y.o., complains for sharp pain in the right part of his chest at deep breathing and cough.
Pain in the chest which relates to disease of respiratory system, is typical for
A. Bronchiectatic disease
B. Asthma
C. Emphysema
D. Exudative pleurisy
E. *Dry pleurisy
682. Patient W., 62 y.o., suffers from morning cough with expectoration of large volume of greenish
sputum. The sputum is better expelled in a certain position of patient’s body. Such kind of cough is typical
for
A. *Bronchiectatic disease
B. Asthma
C. Emphysema
D. Pneumonia
E. Pleurisy
683. Drug abuser, a 41-yr-old man, presents with fever, cough and breathlessness. This was preceded by
viral influenza. Chest radiograph shows multiple abscesses. What is the most possible etiology of disease?
A. *Staphylococcus aureus
B. Cryptococcus
C. Streptococcus pneumoniae
D. Legionella pneumoniae
E. Mycobacterium avium
684. Patient has fever, dry cough, dyspnea increases. Most of the time she lies on one side. Auscultation
and percussion data let to suspect an exudative pleurisy. What method of investigation will confirm a
diagnosis?
A. *Chest X-ray
B. Pneumotachometry
C. Spyrometry
D. Bronchography
E. Bronchoscopy
685. Patient is ill with COPD for 20 years. He usually products approximately 0,5 l of purulent sputum
every day with the maximal production in the morning; fingers are like "drumsticks", nails are like "sentinel
glass". What disease is possible?
A. *Bronchiectasis
B. Pneumonia
C. Chronic bronchitis
D. Gangrene of lungs
E. Tuberculosis
686. Patient R., 48 y.o., complains for the sharp pain in the right part of his chest at deep breathing and
cough. Chest pain in case of respiratory system disease, is mostly typical for
A. Bronchiectatic disease
B. Asthma
C. Emphysema
D. Exudative pleurisy
E. *Dry pleurisy
687. Patient S. has bronchiectatic disease. After cough suddenly he develops dyspnea, dizziness, feels pain
in thorax, from mouth with cough some foamy blood is expectorated. Objectively: condition of the patient is
severe, skin is pale, pulse - 110/min, breathing rate - 30/min. Patient has to be hospitalized:
A. *To the surgical department
B. Home treatment is possible
C. To the pulmonological department
D. To the cardiological department
E. To the emergency department
688. Patient of 47 years old, complaints on cough with production of purulent sputum 0,5 l per day, pain in
the left lower part of thorax, periodical increasing of temperature. What method of investigation may be
informative for diagnosis?
A. *Bronchography
B. X-ray of lungs
C. Bronchoscopy
D. Pleural punction
E. Sputum analysis
689. Patient V. 32 y.o., has chronic pulmonary abscess for 5 years, with frequent exacerbations. She stays
in a hospital for determination of the most effective treatment. Her diagnosis is: abscess of upper lobe of
right lung, phase of remission. What treatment is the most effective?
A. *Surgical
B. Antibiotics
C. Broncholytics
D. Physiotherapy
E. Resort treatment
690. The 82-year-old patient has a pulmonary infection. Which action addresses the age-related change of
increased vascular resistance to blood flow through pulmonary vasculature in this patient?
A. Encouraging the patient to turn, cough, and deep breathe every hour
B. *Assessing the patient's level of consciousness
C. Raising the head of the bed
D. Humidifying the oxygen
E. Non of above
691. Which clinical manifestation in a patient with chronic bronchitis indicates to you a worsening of the
patient's respiratory condition?
A. Fatigue
B. Cachexia
C. *Confusion
D. Slow capillary refill
E. Non of above
692. A 37-yr-old man who has had recurrent chest infections since a serious bout of influenza 3-yr ago
presents with chronic productive cough. His sputum is tenacious and blood stained. On auscultation you find
crackling. What changes may be founded on X-ray?
A. *Round shadow with horizontal level of fluid
B. Infiltration
C. Round shadow
D. Signs of fibrosis and hyperpneumatization
E. Signs of fibrosis and local infiltration
693. A 70-yr-old alcoholic man presents with sudden onset of purulent productive cough. CXR shows
consolidations at left upper lobe. Which infection is the most possible cause of his state?
A. *Klebsiella
B. Chlamydia psittacci
C. Staphylococcus aureus
D. Pneumocystis carinii
E. Aspergillosis
694. Pancreatic calcification may be seen in the following cases except:
A. *primary diabetes mellitus
B. severe protein-calorie malnutrition
C. hereditary pancreatitis,
D. posttraumatic pancreatitis,
E. hyperparathyroidism
695. What is the only major technique that provides a direct view of the pancreatic duct?
A. *Endoscopic retrograde cholangiopancreatography
B. Sonography,
C. CT,
D. Plain radiography
E. Laparoscopy.
696. Ultrasonography can provide important information in patients with all of the following except:
A. *perforated duodenal ulcer
B. acute pancreatitis,
C. chronic pancreatitis,
D. pancreatic calcification,
E. pseudocyst.
697. In acute pancreatitis, the pancreas:
A. *is characteristically enlarged
B. has scattered calcification
C. has ductal decompression
D. is replaced
E. no changes
698. Most lesions of pancreas on CT are characterized by all of the following except:
A. *the presence of ductal decompression
B. enlargement of the pancreatic outline,
C. distortion of the pancreatic contoura
D. a fluid filling that has a different attenuation coefficient than normal pancreas
E. none of above
699. The differential diagnosis of chronic pancreatitis should include all of the following disorders except:
A. *paroxysmal atrium fibrillatium
B. perforated peptic ulcer;
C. acute cholecystitis and biliary colic;
D. acute intestinal obstruction;
E. mesenteric vascular occlusion
700. The differential diagnosis of chronic pancreatitis includes:
A. *All of above
B. myocardial infarction;
C. dissecting aortic aneurysm;
D. pneumonia;
E. peptic ulcer.
701. The secretin-cholecystokinin test is useful in the evaluation of patients with suspected chronic
pancreatitis. Which statement regarding this test is correct?
A. *Secretion of pancreatic enzymes may be measured.
B. Those with chronic pancreatitis usually have a high bicarbonate output after stimulation.
C. In patients with early chronic pancreatitis, enzyme output is relatively more deranged than failure to
achieve an adequate bicarbonate concentration.
D. Endocrine hormone output after stimulation is an end-point of the test.
E. The test can distinguish between chronic pancreatitis and pancreatic carcinoma.
702. Which is the common cause of chronic pancreatitis?
A. *Alcohol
B. Autoimmune diseases
C. Gall stones
D. None of above
E. Tumour of stomach
703. According to ERCP, what are found to have associations with chronic pancreatitis?
A. *Biliary obstruction and Pancreatic duct abnormalities
B. Pancreatic duct abnormalities
C. Biliary obstruction
D. None of above
E. Tumour of pancreas
704. It is supposed that gall stones can result in chronic pancreatitis because they may induce:
A. *Inflammation and stenosis or obstruction of ampulla of Vater
B. Fever
C. Abdominal pain
D. None of above
E. Peptic ulcer
705. What patients with gall stones tend to induce pancreatitis?
A. *Those patients who have common pancreaticobiliary channels in the ampullary of Vater
B. Those patients who do not have common pancreaticobiliary channels
C. Anyone who has gall stones
D. None of above
E. All of above
706. Which of the following is not true regarding blood supply of pancreas?
A. *Postero superior pancreaticoduodenal artery is a branch of Superior mesenteric artery.
B. Body and tail of pancreas is supplied by Splenic artery
C. It receives blood supply from coeliac trunk and superior mesenteric artery.
D. All major pancreatic arteries lie posterior to pancreatic ducts.
E. None of above
707. Most Common Cause of death in early acute Pancreatitis is
A. *Respiratory Failure
B. Cardiac failure
C. Renal Failure
D. Uncontrolled Coagulopathy
E. None of above
708. All of the following have been used in management of Acute Pancreatitis except
A. *Interleukin-10
B. Quamatel
C. Somatostatin
D. Peritoneal dialysis
E. None of above
709. Which of the following is not true for ectopic pancreas
A. *Islet tissue is present in all the organs where ectopic pancreas is present.
B. Ectopic pancreas appears as a submucosal irregular nodule in the wall
C. stomach and duodenum are the most common site
D. Ulceration, bleeding and obstruction are the most common symptoms
E. None of above
710. Which feature of pancreatic pseudocyst on CT suggests secondary infection?
A. *Gas formation in the cyst.
B. Calcification of the cyst wall.
C. Heterogeneous attenuation of cyst contents.
D. Enhancement of the cyst wall after IV contrast injection.
E. Extrapancreatic extension.
711. Which imaging technique or procedure is most specific for differentiating a mucinous cystic tumor
from a microcystic adenoma?
A. *Percutaneous cyst aspiration.
B. Endoscopic retrograde cholangiopancreatography (ERCP).
C. MR cholangiopancreatography.
D. CT.
E. Endoscopic sonography.
712. Which organ also functions as an endocrine gland
A. *Pancreas
B. Gall bladder
C. Liver
D. Spleen
E. Duodenum
713. Which organ releases digestive substances but does not produce them
A. *Gall bladder
B. Pancreas
C. Liver
D. Spleen
E. Duodenum
714. Which organ functions in detoxification of the blood
A. *Liver
B. Gall bladder
C. Pancreas
D. Spleen
E. Duodenum
715. Which organ releases the digestive hormone secretin
A. *Duodenum
B. Gall bladder
C. Liver
D. Spleen
E. Pancreas
716. Which organ releases a buffer that neutralizes acid from the stomach
A. *Pancreas
B. Gall bladder
C. Liver
D. Spleen
E. Duodenum
717. Which organ produces bile salts
A. *Liver
B. Gall bladder
C. Pancreas
D. Spleen
E. Duodenum
718. Markers of Cytolitic syndrome
in liver cirrhosis
A. *↑AST, ↑ALT, ↑GGT, ↑Bilirubin
B. ↑Bilirubin, ↑Alkaline phosphatase, ↑GGT, ↑cholesterol
C. ↓Albumine, ↓prothrombin, ↓cholesterol, ↓fibrinogen
D. ↑ESR, ↑‫ﻻ‬-globulins, ↑timol test ↑Le, ↑C-react. protein
E. None of above
719. Markers of Cholestatic syndrome in liver cirrhosis
A. *↑Bilirubin, ↑Alkaline phosphatase, ↑GGT, ↑cholesterol
B. ↑AST, ↑ALT, ↑GGT, ↑Bilirubin
C. ↓Albumine, ↓prothrombin, ↓cholesterol, ↓fibrinogen
D. ↑ESR, ↑‫ﻻ‬-globulins, ↑timol test ↑Le, ↑C-react. protein
E. None of above
720. Markers of Liver cellular insufficiency syndrome in liver cirrhosis
A. *↓Albumine, ↓prothrombin, ↓cholesterol, ↓fibrinogen
B. ↑AST, ↑ALT, ↑GGT, ↑Bilirubin
C. ↑Bilirubin, ↑Alkaline phosphatase, ↑GGT, ↑cholesterol
D. ↑ESR, ↑‫ﻻ‬-globulins, ↑timol test ↑Le, ↑C-react. protein
E. None of above
721. Markers of Mesenchyme-inflammatory syndrome in liver cirrhosis
A. *↑ESR, ↑‫ﻻ‬-globulins, ↑timol test ↑Le, ↑C-react. protein
B. ↑AST, ↑ALT, ↑GGT, ↑Bilirubin
C. Bilirubin, ↑Alkaline phosphatase, ↑GGT, ↑cholesterol
D. ↓Albumine, ↓prothrombin, ↓cholesterol, ↓fibrinogen
E. None of above
722. Markers of Hypersplenism syndrome
in liver cirrhosis
A. *anemia, thrombocytopenia, leukocytopenia
B. ↑AST, ↑ALT, ↑GGT, ↑Bilirubin
C. ↑Bilirubin, ↑Alkaline phosphatase, ↑GGT, ↑cholesterol
D. ↓Albumine, ↓prothrombin, ↓cholesterol, ↓fibrinogen
E. None of above
723. Markers of Cholestatic syndrome in hepatitis
A. *All of above
B. ↑cholesterol
C. ↑Bilirubin
D. ↑Alkaline phosphatase
E. ↑GGT
724. Marker of Cholestatic syndrome
in hepatitis
A. *↑Alkaline phosphatase
B. All of above
C. ↓fibrinogen
D. ↑AST
E. ALT
725. Markers of Cholestatic syndrome in hepatitis are all except
A. *ALT
B. ↑cholesterol
C. ↑Bilirubin
D. ↑Alkaline phosphatase
E. ↑GGT
726. Markers of Liver cellular insufficiency syndrome in hepatitis
A. *All of above
B. ↓Albumine
C. ↓cholesterol
D. ↓prothrombin
E. ↓fibrinogen
727. Marker of Liver cellular insufficiency syndrome in hepatitis
A. *↓Albumine
B. ↑ALT
C. ↑GGT
D. ↑Bilirubin
E. ↑AST
728. Markers of Liver cellular insufficiency syndrome in hepatitis are all except
A. *↑ALT
B. ↓Albumine
C. ↓cholesterol
D. ↓prothrombin
E. ↓fibrinogen
729. Markers of Hypersplenism syndrome
in liver cirrhosis
A. *All of above
B. anemia
C. thrombocytopenia
D. leukocytopenia
E. None of above
730. Markers of Hypersplenism syndrome
in liver cirrhosis
A. *anemia, thrombocytopenia, leukocytopenia
B. ↑ALT
C. ↑GGT
D. ↑Bilirubin
E. ↑AST
731. HEMOCHROMATOSIS is characterized by:
A. *Characterized by excessive deposition of iron in liver
B. Characterized by excessive deposition of copper in liver
C. results in cirrhosis and emphysema
D. None of above
E. All of above
732. WILSON DISEASE is characterized by:
A. *Characterized by excessive deposition of copper in liver
B. Characterized by excessive deposition of iron in liver
C. results in cirrhosis and emphysema
D. None of above
E. All of above
733. What is true about α1-ANTITRYPSIN DEFICIENCY:
A. *results in cirrhosis and emphysema
B. Characterized by excessive deposition of iron in liver
C. Characterized by excessive deposition of copper in liver
D. None of above
E. All of above
734. Pathologic Mallory stain findings characterizes
A. *Alcoholic hepatitis
B. Autoimmune Hepatitis
C. Cryptogenic Hepatitis
D. Chronic Viral Hepatitis B
E. Chronic Viral Hepatitis C
735. Chronic hepatitis is generally defined as disease that has lasted for:
A. *6 months or longer
B. 1 months
C. 2 months
D. 2,5 months
E. 3 months
736. What is true about Autoimmune Hepatitis
A. *All of above
B. It is four times more common in women than in men
C. There is a rise in IgG, a positive anti-smooth-muscle antibody present in 60%.
D. Antinuclear antibody and liver-kidney microsomal antibody (LKM-1) may be present.
E. Response rate to corticosteroids therapy is 70-90%.
737. Autoimmune Hepatitis is characterized by
A. *There is a rise in IgG, a positive anti-smooth-muscle antibody present in 60%
B. It is most aggressive when it presents in the firsr and second decades
C. All of above
D. It is more common in men than in women
E. None of above
738. What is not true about Autoimmune Hepatitis
A. *It is more common in men than in women
B. It can be associated with other autoimmune diseases
C. There is a rise in IgG, a positive anti-smooth-muscle antibody present in 60%.
D. Antinuclear antibody and liver-kidney microsomal antibody (LKM-1) may be present.
E. Response rate to corticosteroids therapy is 70-90%.
739. DRUG-INDUCED CHRONIC HEPATITIS may be caused by
A. *All of above
B. Paracetamol
C. Isoniazid
D. Oestrogens
E. Antibiotics
740. What is true about CRYPTOGENIC HEPATITIS
A. *Unknown cause
B. It is four times more common in women than in men
C. There is a rise in IgG, a positive anti-smooth-muscle antibody present in 60%.
D. Antinuclear antibody and liver-kidney microsomal antibody (LKM-1) may be present.
E. Response rate to corticosteroids therapy is 70-90%.
741. Chronic hepatitis is characterized by next clinical syndromes
A. *All of above
B. Cholestatic
C. Hepatomegaly
D. Dispeptic
E. None of above
742. Chronic hepatitis is not characterized by next clinical syndrome
A. *Portal hypertension
B. Cholestatic
C. Hepatomegaly
D. Dispeptic
E. Pain
743. Liver cirrhosis is characterized by next clinical syndromes
A. *All of above
B. Cholestatic
C. Hepatomegaly
D. Dispeptic
E. Miocardial infarction
744. What is true regarding congenital hypertrophic pyloric stenosis :
A. *Hypochloremic alkalosis
B. More common in girls
C. Hellers myotomy is the procedure of choice.
D. Most often manifests at birth
E. None of above
745. A child presents with massive hemetemesis and systemic hypotension. He has no fever or other
significant history. Examination reveal massive splenomegaly but no hepatomegaly. Likely diagnosis is
A. *Non-cirrhotic portal fibrosis
B. Bleeding duodenal ulcer
C. Oesophageal varices
D. Hepatocellular carcinoma
E. None of the above
746. A 19 year old male has presented with repeated episodes of hematemesis. There is no history of
jaundice or liver decompensation. On examination the significant findings include splenomegaly (8 cms
below costal margin), and presence of esophageal varices. There is no ascites or peptic ulceration. The
liver function tests are normal. The most likely diagnosis isA. *Non cirotic portal fibrosis
B. Extahepatic portal venous obstruction
C. Cirrhosis
D. Hepatic venous outflow tract obstruction
E. None of the above
747. Ingestion of arsenic causes
A. *Non cirrhotic portal fibrosis
B. Hepatic adenoma
C. Hepatic carcinoma
D. Hepatic cirrhosis
E. None of the above
748. Commonest cause of Budd Chiary syndrome is:
A. *Paroxysmal nocturnal hemoglobinuria
B. Hepatocellular carcinoma
C. Right ventricular failure
D. Renal cell carcinoma
E. None of above
749. Hepatic-encephalopathy may be precipitated by all of the following except:
A. *Hyperkalemia
B. Barbiturates
C. Anemia
D. Hypothyroidism
E. Hypokalemia
750. In a case of hypertrophic pyloric stenosis, the metabolic disturbance is
A. *Metabolic alkalosis with paradoxical aciduria
B. Metabolic acidosis
C. Respiratory alkalosis
D. Metabolic alkalosis with alkaline urine
E. None of above
751. Skip granulomatous lesions are seen in
A. *Crohn's disease
B. Ulcerative colitis
C. Whipple's disease
D. Reiter's disease
E. None of above
752. Ingestion of arsenic causes
A. *Non cirrhotic portal fibrosis
B. Hepatic adenoma
C. Hepatic carcinoma
D. Hepatic cirrhosis
E. None of the above
753. Hepatic-encephalopathy may be precipitated by all of the following except:
A. *Hyperkalemia
B. Barbiturates
C. Anemia
D. Hypothyroidism
E. Hypokalemia
754. In patients with acute liver failure, the best prognostic indicator is:
A. *Factor V estimation
B. Serum alpha feto protein
C. Serum bilirubin
D. Serum albumin
E. None of above
755. Which of the following statements is true regarding a-fetoprotein :
A. *High levels are seen in hepatic carcinoma
B. Pre-operative high level indicates worse prognosis
C. High level are seen in stomach carcinoma
D. Levels may be increased in hepatitis
E. None of above
756. Gall bladder stone formation in influenced by all except:
A. *Leucocytosis
B. Hyperalimentation
C. Primary biliary cirrhosis
D. Clofibrate therapy
E. None of above
757. Peptic ulcer should be differentiated with
A. Cholecystitis
B. Gastritis
C. Myocardial infarction
D. Pancreatitis
E. *All of above
758. Which is not a paraneoplastic syndrome for Hepatocellular Carcinoma
A. *Hyperglycemia
B. Hypoglycemia
C. Erythrocytosis
D. Hypercalcemia
E. None of above
759. Which of the following structures do not form the external part of anal canal mechanism?
A. *Internal Sphincter
B. Levator Ani
C. Pubo rectalis
D. External Sphincter
E. None of above
760. Most common site for cholangiocarcinoma is
A. *Hepatic duct bifurcation
B. Intrahepatic
C. Lower End of CBD (Common Bile Duct)
D. Lower 1/3rd of CBD (Common Bile Duct)
E. None of above
761. Which of the following is not a congenital abnormality associated with Juvenile polyps
A. *Macrocephaly
B. Meckel's diverticulum
C. Malrotation
D. Mesenteric lymphangioma
E. None of above
762. A 47-yr-old agricultural worker complains of a chronic cough, purulent sputum and abdominal
distention. He has just arrived in England from Spain where he was picking grapes. Choose the single most
likely diagnosis from the list of options above.
A. *Tuberculosis
B. Cirrhosis
C. Malabsorption
D. Pancreatitis
E. Peptic ulcer
763. A 32-year-old mn has bloody diarrhea of 1 day’s duration associated with abdominal pain, fever, and
small, frequent stools. He was previously well.Which of the following pathogens is most likely causing this
patient’s acute illness?
A. *Campylobacter jejuni
B. Cryptosporidium
C. Giardia lambli
764. Which of the following statements regarding eosinophilic enteritis is correct?
A. *It may be difficult to distinguish from regional enteritis.
B. It affects only the small intestine.
C. The majority of patients have a history of food allergies or asthma.
D. Treatment with glucocorticoids is not indicated.
765. There are following diagnostic tests for chronic hepatitis B except:
A. *HDV RNA
B. IgG anti-HBc
C. HBeAg
D. HBV DNA
E. HBsAg
766. There are following diagnostic tests for chronic cryptogenic hepatitis except:
A. *all negative
B. HBsAg
C. IgG anti-HBc,
D. HBeAg
E. all positive,
767. For Primary biliary cirrhosis select the circulating antibodies with which it is most closely
associated:
A. *antibodies to mitochondria
B. antibodies to native DNA;
C. antibodies to smooth muscle cells;
D. antibodies to acetylcholine receptors;
E. antibodies to parietal cells.
768. Liver involvement in systemic diseases is the following except:
A. *Ischemic hepatitis
B. Sarcoidosis
C. Amyloidosis
D. Celiac disease
E. Tuberculosis
769. The stage of chronic hepatitis is based on the all of following except:
A. *no fibrosis
B. mild fibrosis
C. necrosis.
D. moderate fibrosis
E. severe fibrosis, including bridging fibrosis,
770. Morphologic characteristics of chronic active hepatitis mainly include all of the following except:
A. *severe fibrosis, including bridging fibrosis.
B. a dense mononuclear infiltrate of the portal tracts,;
C. destruction of the hepatocytes at the periphery of the lobule,;
D. connective tissue septa surrounding portal tracts and extending from the portal zones into the
lobule,isolating parenchymal cells into clusters and enveloping bile ducts;
E. evidence of hepatocellular regenerationѕ"rosette" formation, thickened liver cell plates, and regenerative
"pseudolobules";
771. Causes of Cirrhosis and/or Chronic Liver Disease are all of the following except:
A. *Herpes virus.
B. Brucellosis,
C. Capillariasis,
D. Toxoplasmosis,
E. Viral hepatitis C,
772. Hemochromatosis is associated with all of the following clinical manifestations EXCEPT:
A. *gout
B. cirrhosis
C. diabetes mellitus
D. gonadotropin deficiency
E. cardiomyopathy
773. For Primary biliary cirrhosis select the circulating antibodies with which it is most closely
associated:
A. *antibodies to mitochondria;
B. antibodies to acetylcholine receptors;
C. antibodies to native DNA;
D. antibodies to smooth muscle cells;
E. antibodies to parietal cells.
774. There are all of the following causes of noncirrhotic hepatic fibrosis except:
A. *All of them.
B. Intrahepatic phlebosclerosis and fibrosis;
C. Portal and splenic vein sclerosis;
D. Portal and splenic vein thrombosis;
E. Schistosomiasis;
775. A typical battery of blood tests used for initial assessment of liver disease includes measuring all of
the following except:
A. *urea
B. levels of serum alanine and aspartate aminotransferases
C. alkaline phosphatase
D. direct and total serum bilirubin
E. albumin.
776. In which of the following causes of fatty liver is microvesicular fat seen in biopsy specimens of liver?
A. *Acute fatty liver of pregnancy
B. None of above
C. Total parenteral nutrition
D. Prolonged intravenous hyperalimentation
E. Carbon tetrachloride poisoning
777. Chronic active hepatitis is most reliably distinguished from chronic persistent atitis by the presence
of:
A. *characteristic liver histology
B. hepatitis B surface antigen in the serum
C. antibody to hepatitis B core antigen in the serum
D. a significant titer of anti-smooth-muscle antibody
E. extrahepatic manifestations
778. Which one of the following statements about hepatitis B e antigen (HBeAg) is LEAST accurate?
A. *The absence of HBeAg in the serum rules out chronic infection caused by the hepatitis B virus.
B. The presence of HBeAg in the serum is correlated with infectiousness.
C. HBeAg can be detected transiently in the sera of patients ill with acute hepatitis B infection.
D. HBeAg is immunologically distinct from HBsAg but is genetically related to HBcAg.
E. The disappearance of HBeAg from the serum may be a harbinger of resolution of acute hepatitis B
infection.
779. Which of the following statements regarding delta hepatitis virus (HDV) is correct?
A. *HDV can infect only persons infected with hepatitis B virus (HBV).
B. HDV is a defective DNA virus.
C. The HDV genome is partially homologous with HBV DNA.
D. HDV infection has been found only in limited areas of the world.
E. Simultaneous infection with HDV and HBV results in an increased risk of the development of chronic
hepatitis.
780. A 56-year-old man presents with fatigue and teacolored urine. Physical examination reveals icteric
sclerae but is otherwise unremarkable. Which of the following conditions is LEAST likely to account for
these findings?
A. *Autoimmune hemolytic anemia
B. Gallbladder cancer
C. Primary biliary cirrhosis
D. Pancreatic cancer
E. Viral hepatitis
781. Which of the following cell types is most directly responsible for the excess deposition of collagen in
the liver typically seen in alcoholic cirrhosis?
A. *Stellate cell
B. Hepatocyte
C. Kupffer cell
D. Cannilicular cell
E. None of above
782. Which of the following conditions are known to predispose to the formation of cholesterol
gallstones?
A. *Surgical resection of the ileum
B. Hypercholesterolemia
C. Autoimmune hemolytic anemia
D. Sickle cell anemia
E. None of above
783. The Marseiles-Rome classification of 1988 includes all of the following except:
A. *Chronic autoimmune pancreatitis
B. Chronic calcifying pancreatitis
C. Chronic obstructive pancreatitis
D. Chronic inflammatory pancreatitis
E. None of above
784. Which of the following could falsely depress the serum amylase level in a patient suspected of having
acute pancreatitis?
A. *Hypertriglyceridemia
B. Hypercholesterolemia
C. Hypocalcemia
D. Associated pleural effusion
E. None of above
785. A 61-yr-old man presents with a 2-yr history of pain in the right upper quadrant exacerbated by eating
rich foods. Choose the single most likely investigation from the list of options above.
A. *Upper GI endoscopy
B. Oesophageal manometry
C. Motility studies
D. Mesenteric angiography
E. Barium enema
786. A 49-year-old white man is evaluated because of a 2-month history of dysphagia for solid foods and a
7-kg (15-Ib) weight loss. He has had heartburn for many years that is relieved by antacids. Which of the
following should be done next?
A. *Upper endoscopy
B. Esophageal manometry
C. Ambulatory 24-hour esophageal pH monitoring
D. Abdominal USG
E. Upright Chest X-ray
787. A 24-yr-old female presents with fever and right upper quadrant pain. On examination she has adnexal
tenderness and purulent cervical discharge. Choose the single most likely investigation from the list of options
below.
A. *Endocervical smear and culture
B. Barium enema
C. Abdominal X-ray
D. Upright Chest X-ray
E. Abdominal USG
788. A 41-yr-old man says he cannot finish his stool and that what he does pass is streaked with blood. He says
he has always been regular. He wants to know if a laxative will help. Choose the single most likely diagnosis
A. *Adhesions
B. Food poisoning
C. Rectal carcinoma
D. Irritable bowel syndrome
E. None of above
789. A healthy young woman presents with ulcer near her anus. It is painless, macular and hard. Choose the
single most likely diagnosis from the list of options above.
A. *Syphilis
B. Ulcerative colitis
C. Crohn's disease
D. Anaemia
E. Haemorrhoids
790. A 21-yr-old woman with a history of grumbling appendix presents with persisting diarrhoea. On
physical examination you find the liver to be slightly enlarged. She also complains of flushing and
difficulty in breathing. Choose the single most likely diagnosis from the list of options above.
A. *Carcinoid syndrome
B. Pancreatic adenoma
C. Peptic ulcer
D. Bowel cancer
E. Crohn's disease
791. A 45-yr-old woman has melanotic skin pigmentation, pruritus, hepatospleno-megaly and dark urine.
She develops jaundice 5 yrs after onset. Choose the single most likely diagnosis from the list of options
above.
A. *Primary biliary cirrhosis
B. Leptospirosis
C. Haemolytic anaemia
D. Gallstones
E. Hepatitis B associated with primary hepatocellular carcinoma
792. A 60-yr-old man presents with jaundice, hepatomegaly, nocturnal abdominal pain radiating through
to the back and weight loss. Choose the single most likely diagnosis from the list of options above.
A. *Pancreatic carcinoma
B. Leptospirosis
C. Haemolytic anaemia
D. Gallstones
E. Hepatitis B associated with primary hepatocellular carcinoma
793. A 45-yr-old man develops deep jaundice, abdominal pain, hypoglycaemia, fever. Liver biopsy shows
Mallory's hyaline and collagen deposition. Choose the single most likely diagnosis from the list of options
above.
A. *Alcoholic hepatitis
B. Leptospirosis
C. Primary biliary cirrhosis
D. Gallstones
E. Hepatitis B associated with primary hepatocellular carcinoma
794. A 39 -year-old woman for 1 year complained of attacks of right subcostal pain after fatty meal. Last
week the attacks have repeated every day and become more painful. What diagnostic study would you
recommend?
A. *Ultrasound examination of the gallbladder
B. Blood cell count
C. Ultrasound study of the pancreas
D. X-ray examination of the gastrointestinal tract
E. Liver function tests
795. A 49-year-old white man is evaluated because of a 2-month history of dysphagia for solid foods and a
7-kg (15-Ib) weight loss. He has had heartburn for many years that is relieved by antacids. Which of the
following should be done next?
A. *Upper endoscopy
B. Esophageal manometry
C. Ambulatory 24-hour esophageal pH monitoring
D. Barium swallow
E. USD
796. A 50 -year-old woman for 1 year complained of attacks of right subcostal pain after fatty meal. Last
week the attacks have repeated every day and become more painful. What diagnostic study would you
recommend?
A. *Ultrasound examination of the gallbladder
B. Liver function tests
C. X-ray examination of the gastrointestinal tract
D. Ultrasound study of the pancreas
E. Blood cell count
797. A 39-year-old man with chronic alcoholism is admitted to the hospital with acute pancreatitis. On the
third hospital day sudden, complete blindness develops in the left eye. The most likely explanation is
A. *Purtscher’s retinopathy
B. transient ischemic attack (transient monocular blindness)
C. occlusion of the retinal vein
D. acute glaucoma
E. None of above
798. A 39-year-old man with chronic alcoholism is admitted to the hospital with acute pancreatitis. On the
third hospital day sudden, complete blindness develops in the left eye. The most likely explanation is
A. *Purtscher’s retinopathy
B. transient ischemic attack (transient monocular blindness)
C. occlusion of the retinal vein
D. acute glaucoma
E. None of above
799. A 51-year-old woman with long-standing reflux esophagitis has developed improvement in his
heartburn. However, he also complains of progressive dysphagia after swallowing both liquids and solids, a
chest x-ray shows absence of the gastric air bubble and an air-fluid level in the mediastinum. An infusion of
cholecystokinin during esophageal manometry would show
A. *increase in lower esophageal sphincter pressure
B. no change in lower esophageal sphincter pressure
C. fall in lower esophageal sphincter pressure
D. rise in upper esophageal sphincter pressure
E. fall in upper esophageal sphincter pressure
800. There are following diagnostic tests for chronic cryptogenic hepatitis except:
A.
*all negative
B.
HBsAg,
C.
IgG anti-HBc,
D.
HBeAg
E.
all of them,
801. For Primary biliary cirrhosis select the circulating antibodies with which it is most closely associated:
A.
*antibodies to mitochondria
B.
antibodies to acetylcholine receptors;
C.
antibodies to native DNA;
D.
antibodies to smooth muscle cells;
E.
antibodies to parietal cells.
802. 40-year-old woman for 1 year complained of attacks of right subcostal pain after fatty meal. Last
week the attacks have repeated every day and become more painful. What diagnostic study would you
recommend?
A.
*Ultrasound examination of the gallbladder
B.
Blood cell count
C.
Ultrasound study of the pancreas
D.
X-ray examination of the gastrointestinal tract
E.
Liver function tests
803. Causes chronic hepatitis are all of the following except:
A.
*Budd-Chiari syndrome
B.
viral hepatitis A
C.
viral hepatitis B
D.
viral hepatitis C
E.
cryptogenic hepatitis
804. Liver involvement in systemic diseases is the following except:
A.
*Ischemic hepatitis
B.
Sarcoidosis
C.
Amyloidosis
D.
Celiac disease
E.
Tuberculosis
805. There are following diagnostic tests for chronic hepatitis B except:
A.
*HDV RNA
B.
HBsAg,
C.
IgG anti-HBc,
D.
HBeAg,
E.
HBV DNA
806. There are following diagnostic tests for chronic cryptogenic hepatitis except:
A.
*all negative
B.
HBsAg,
C.
IgG anti-HBc,
D.
HBeAg
E.
anti-HBc,
807. All of the following statements concerning transmission of viral hepatitis are true EX¬CEPT
A.
*A causes chronic infection
B.
children are at considerably less risk for hepatitis B infection than spouses of an infected individual
C.
in developing countries, vertical trans¬mission (mother to child) is the major route of transmission
vertical transmission results most often from transplacental infection
D.
in developing countries, vertical trans¬mission (mother to child) is the major route of transmission
vertical transmission results most often from transplacental infection
E.
Maternal-infant transmission occurs with both hepatitis B
808. Hemochromatosis is associated with all of the following clinical manifestations EXCEPT:
A.
*gout
B.
cirrhosis
C.
diabetes mellitus
D.
gonadotropin deficiency
E.
cardiomyopathy
809. Typical physical findings in liver disease are all of the following except:
A.
*heart failure
B.
palmar erythema,
C.
excoriations
D.
ascites
E.
edema
810. A child presents with massive hemetemesis and systemic hypotension. He has no fever or other
significant history. Examination reveals massive splenomegaly but no hepatomegaly. Likely diagnosis is
A.
*Non-cirrhotic portal fibrosis
B.
Bleeding duodenal ulcer
C.
Oesophageal varices
D.
Hepatocellular carcinoma
E.
No one mentioned
811. A nursing student has just completed her hepatitis B vaccine series. On reviewing her laboratory
studies (assuming she has no prior exposure to hepatitis B), you expect
A.
Positive test for hepatitis B surface antigen
B.
*Antibody against hepatitis B surface antigen (anti-HBS) alone
C.
Antibody against hepatitis core antigen (anti-HBC)
D.
Antibody against both surface and core antigen
E.
Antibody against hepatitis E antigen
812. A typical battery of blood tests used for initial assessment of liver disease includes measuring all of
the following except:
A.
*urea
B.
levels of serum alanine and aspartate aminotransferases
C.
alkaline phosphatase
D.
direct and total serum bilirubin
E.
albumin.
813. A typical battery of blood tests used for initial assessment of liver disease includes measuring all of
the following except:
A.
*urea
B.
Aminotransferases
C.
alkaline phosphatase
D.
direct and total serum bilirubin
E.
albumin.
814. A typical battery of blood tests used for initial assessment of liver disease includes measuring all of
the following except:
A.
*urea
B.
Aminotransferases
C.
Gamma-glutamyl transferase
D.
direct and total serum bilirubin
E.
Prothrombin time.
815. A typical battery of blood tests used for initial assessment of liver disease includes measuring all of
the following except:
A.
*urea
B.
Aminotransferases
C.
Gamma-glutamyl transferase
D.
direct and total serum bilirubin
E.
Serum sodium.
816. A typical battery of blood tests used for initial assessment of liver disease includes measuring all of
the following except:
A.
*urea
B.
Aminotransferases
C.
Gamma-glutamyl transferase
D.
direct and total serum bilirubin
E.
Coagulation test.
817. A typical battery of blood tests used for initial assessment of liver disease includes measuring all of
the following except:
A.
*urea
B.
Aminotransferases
C.
Gamma-glutamyl transferase
D.
General blood analysis
E.
Coagulation test.
818. A typical battery of blood tests used for initial assessment of liver disease includes measuring all of
the following except:
A.
*urea
B.
Aminotransferases
C.
Gamma-glutamyl transferase
D.
General blood analysis
E.
Coagulation test.
819. A typical battery of blood tests used for initial assessment of liver cirrhosis includes measuring all of
the following except:
A.
*urea
B.
Aminotransferases
C.
alkaline phosphatase
D.
direct and total serum bilirubin
E.
albumin.
820. A typical battery of blood tests used for initial assessment of liver cirrhosis includes measuring all of
the following except:
A.
*urea
B.
Aminotransferases
C.
Gamma-glutamyl transferase
D.
direct and total serum bilirubin
E.
albumin.
821. A typical battery of blood tests used for initial assessment of liver cirrhosis includes measuring all of
the following except:
A.
*urea
B.
Aminotransferases
C.
Gamma-glutamyl transferase
D.
direct and total serum bilirubin
E.
Prothrombin time.
822. A typical battery of blood tests used for initial assessment of liver cirrhosis includes measuring all of
the following except:
A.
*urea
B.
Aminotransferases
C.
Gamma-glutamyl transferase
D.
direct and total serum bilirubin
E.
Serum sodium.
823. A typical battery of blood tests used for initial assessment of liver cirrhosis includes measuring all of
the following except:
A.
*urea
B.
Aminotransferases
C.
Gamma-glutamyl transferase
D.
direct and total serum bilirubin
E.
Coagulation test.
824. A typical battery of blood tests used for initial assessment of liver cirrhosis includes measuring all of
the following except:
A.
*urea
B.
Aminotransferases
C.
Gamma-glutamyl transferase
D.
General blood analysis
E.
Coagulation test.
825. A typical battery of blood tests used for initial assessment of liver cirrhosis includes measuring all of
the following except:
A.
*urea
B.
Aminotransferases
C.
Gamma-glutamyl transferase
D.
General blood analysis
E.
Coagulation test.
826. A typical battery of blood tests used for initial assessment of liver cirrhosis includes measuring all of
the following except:
A.
*urea
B.
Aminotransferases
C.
Gamma-glutamyl transferase
D.
autoantibodies
E.
Coagulation test.
827. A typical battery of blood tests used for initial assessment of liver cirrhosis includes measuring all of
the following except:
A.
*urea
B.
Aminotransferases
C.
Gamma-glutamyl transferase
D.
autoantibodies
E.
Ferritin and transferrin saturation.
828. A typical battery of blood tests used for initial assessment of liver cirrhosis includes measuring all of
the following except:
A.
*urea
B.
Aminotransferases
C.
Gamma-glutamyl transferase
D.
autoantibodies
E.
copper and ceruloplasmin.
829. A typical battery of blood tests used for initial assessment of liver cirrhosis includes measuring all of
the following except:
A.
*urea
B.
Aminotransferases
C.
Gamma-glutamyl transferase
D.
autoantibodies
E.
Immunoglobulin levels.
830. A typical battery of blood tests used for initial assessment of liver cirrhosis includes measuring all of
the following except:
A.
*urea
B.
Aminotransferases
C.
Gamma-glutamyl transferase
D.
autoantibodies
E.
Cholesterol and glucose.
831. A typical battery of blood tests used for initial assessment of liver cirrhosis includes measuring all of
the following except:
A.
*urea
B.
Aminotransferases
C.
Gamma-glutamyl transferase
D.
autoantibodies
E.
Alpha 1-antitrypsin.
832. A typical battery of blood tests used for initial assessment of liver disease includes measuring all of
the following except:
A.
*urea
B.
levels of serum alanine and aspartate aminotransferases
C.
alkaline phosphatase
D.
direct and total serum bilirubin
E.
albumin.
833. As a consequence of severe liver damage, hepatic amino acid handling is deranged. In this situation,
plasma levels of which of the following are likely to be lower than normal?
A.
*Urea
B.
Ammonia (NH ) 3
C.
Ammonium (NH ) _
D.
Alanine
E.
Glycine
834. Autoimmune Hepatitis is characterized by
A.
*There is a rise in IgG, a positive anti-smooth-muscle antibody present in 60%
B.
It is most aggressive when it presents in the firsr and second decades
C.
All mentioned
D.
It is more common in men than in women
E.
No one mentioned
835. Causes of Cirrhosis and/or Chronic Liver Disease are all of the following except:
A.
*Viral hepatitis A.
B.
Autoimmune,
C.
Viral hepatitis C,
D.
Viral Hepatitis B
E.
Viral Hepatitis B with hepatitis D
836. Causes of Cirrhosis and/or Chronic Liver Disease are all of the following except:
A.
*Viral hepatitis E.
B.
Autoimmune,
C.
Viral hepatitis C,
D.
Viral Hepatitis B
E.
Viral Hepatitis B with hepatitis D
837. Causes of Cirrhosis and/or Chronic Liver Disease are all of the following except
A.
*Herpes virus
B.
Brucellosis,
C.
Capillariasis,
D.
Toxoplasmosis,
E.
Viral hepatitis C,
838. Causes of Liver Cirrhosis are all of the following except:
A.
*Viral hepatitis A.
B.
Autoimmune,
C.
Viral hepatitis C,
D.
Viral Hepatitis B
E.
Alcoholic liver disease
839. Causes of Liver Cirrhosis are all of the following except:
A.
*Viral hepatitis A.
B.
Autoimmune,
C.
Viral hepatitis C,
D.
Non-alcoholic steatohepatitis
E.
Alcoholic liver disease
840. Causes of Liver Cirrhosis are all of the following except:
A.
*Viral hepatitis A.
B.
Primary biliary cirrhosis,
C.
Viral hepatitis C,
D.
Non-alcoholic steatohepatitis
E.
Alcoholic liver disease
841. Causes of Liver Cirrhosis are all of the following except:
A.
*Viral hepatitis A.
B.
Primary biliary cirrhosis,
C.
Viral hepatitis C,
D.
Non-alcoholic steatohepatitis
E.
Primary sclerosing cholangitis
842. Causes of Liver Cirrhosis are all of the following except:
A.
*Viral hepatitis A.
B.
Hereditary hemochromatosis,
C.
Viral hepatitis C,
D.
Non-alcoholic steatohepatitis
E.
Primary sclerosing cholangitis
843. Causes of Liver Cirrhosis are all of the following except:
A.
*Viral hepatitis A.
B.
Hereditary hemochromatosis,
C.
Wilson's disease,
D.
Non-alcoholic steatohepatitis
E.
Primary sclerosing cholangitis
844. Causes of Liver Cirrhosis are all of the following except:
A.
*Viral hepatitis A.
B.
Hereditary hemochromatosis,
C.
Wilson's disease,
D.
Alpha 1-antitrypsin deficiency
E.
Primary sclerosing cholangitis
845. Causes of Liver Cirrhosis are all of the following except:
A.
*Viral hepatitis A.
B.
Hereditary hemochromatosis,
C.
Cardiac cirrhosis,
D.
Alpha 1-antitrypsin deficiency
E.
Primary sclerosing cholangitis
846. Causes of Liver Cirrhosis are all of the following except:
A.
*Viral hepatitis A.
B.
Hepatotoxic drugs or toxins,
C.
Cardiac cirrhosis,
D.
Alpha 1-antitrypsin deficiency
E.
Primary sclerosing cholangitis
847. Chronic active hepatitis is most reliably distinguished from chronic persistent hepatitis by the
presence of
A.
*characteristic liver histology
B.
hepatitis B surface antigen in the serum
C.
antibody to hepatitis B core antigen in the serum
D.
a significant titer of anti-smooth-muscle antibody
E.
extrahepatic manifestations
848. Chronic active hepatitis is most reliably distinguished from chronic persistent hepatitis by the
presence of
A.
*characteristic liver histology
B.
extrahepatic manifestations
C.
hepatitis B surface antigen in the serum
D.
antibody to hepatitis B core antigen in the serum
E.
a significant titer of anti-smooth-muscle antibody
849. Chronic hepatitis is characterized by following clinical syndromes
A.
*All mentioned
B.
Cholestatic
C.
Hepatomegaly
D.
Dyspeptic
E.
No one mentioned
850. Chronic hepatitis is generally defined as disease that has lasted for:
A.
*6 months or longer
B.
1 months
C.
2 months
D.
2,5 months
E.
3 months
851. Chronic hepatitis is not characterized by following clinical syndrome
A.
*Portal hypertension
B.
Cholestatic
C.
Hepatomegaly
D.
Dyspeptic
E.
Hypochondriac pain
852. Complications of liver cirrhosis are all of the following except
A.
*Hyperuricemia
B.
Bruising and bleeding
C.
Jaundice due to decreased processing of bilirubin
D.
Itching (pruritus)
E.
Hepatic encephalopathy
853. Complications of liver cirrhosis are all of the following except
A.
*Hyperuricemia
B.
Bruising and bleeding
C.
Jaundice due to decreased processing of bilirubin
D.
Itching (pruritus)
E.
Sensitivity to medication
854. Complications of liver cirrhosis are all of the following except
A.
*Hyperuricemia
B.
Bruising and bleeding
C.
Jaundice due to decreased processing of bilirubin
D.
Itching (pruritus)
E.
Hepatocellular carcinoma
855. Complications of liver cirrhosis are all of the following except
A.
*Hyperuricemia
B.
Bruising and bleeding
C.
Jaundice due to decreased processing of bilirubin
D.
Itching (pruritus)
E.
Esophageal varices
856. Complications of liver cirrhosis are all of the following except
A.
B.
C.
D.
E.
857.
A.
B.
C.
D.
E.
858.
A.
B.
C.
D.
E.
859.
A.
B.
C.
D.
E.
860.
A.
B.
C.
D.
E.
861.
A.
B.
C.
D.
E.
862.
A.
B.
C.
D.
E.
863.
A.
B.
C.
D.
E.
864.
A.
B.
C.
D.
E.
*Hyperuricemia
Bruising and bleeding
Jaundice due to decreased processing of bilirubin
Itching (pruritus)
Portal hypertension
Complications of liver cirrhosis are all of the following except
*Hyperuricemia
Bruising and bleeding
Jaundice due to decreased processing of bilirubin
Itching (pruritus)
Ascites
Complications of liver cirrhosis are all of the following except
*Hyperuricemia
Bruising and bleeding
Jaundice due to decreased processing of bilirubin
leading to infection
Ascites
DRUG-INDUCED CHRONIC HEPATITIS is not be caused by
*silymarin
Paracetamol
Isoniazid
Oestrogens
Antibiotics
For Primary biliary cirrhosis select the circulating antibodies with which it is most closely associated:
*antibodies to mitochondria;
anantibodies to Phospholipid;
antibodies to native DNA;
antibodies to smooth muscle cells;
antibodies to parietal cells.
Gall bladder stone formation in caused by all except:
*Leucocytosis
Hyperalimentation
Primary biliary cirrhosis
Clofibrate therapy
None of mentioned
Hemochromatosis is associated with all of the following clinical manifestations EXCEPT:
*gout
liver cirrhosis
diabetes mellitus
gonadotropin deficiency
cardiomyopathy
Hemochromatosis is characterized by:
*Characterized by excessive deposition of iron in liver
Characterized by excessive deposition of copper in liver
results in cirrhosis and emphysema
No one mentioned
All mentioned
Hepatic-encephalopathy may be aggravated by all of the following except:
*Hyperkalemia
Barbiturates
Alkalosis
Hypothyroidism
Hyponatraemia
865.
A.
B.
C.
D.
E.
866.
A.
B.
C.
D.
E.
867.
A.
B.
C.
D.
E.
868.
A.
B.
C.
D.
E.
869.
A.
B.
C.
D.
E.
870.
A.
B.
C.
D.
E.
871.
A.
B.
C.
D.
E.
872.
A.
B.
C.
D.
E.
873.
A.
B.
C.
D.
Hepatic-encephalopathy may be aggravated by all of the following except:
*Hyperkalemia
Renal failure
Alkalosis
Hypoxia
Dehydration
Hepatic-encephalopathy may be aggravated by all of the following except:
*Hyperkalemia
Renal failure
Alkalosis
Constipation
Dehydration
Hepatic-encephalopathy may be aggravated by all of the following except:
Hypokalemia
*Ascites
Alkalosis
Alcohol intoxication
Dehydration
Hepatic-encephalopathy may be precipitated by all of the following except:
*Hyperkalemia
Barbiturates
Anemia
Hypothyroidism
Hyponatraemia
In patients with acute liver failure, the best prognostic indicator is:
*Factor V estimation
Serum alpha feto protein
Serum bilirubin
Serum albumin
Level of GGT
In which of the following causes of fatty liver is microvesicular fat seen in biopsy specimens of liver?
*Acute fatty liver of pregnancy
Jejunoileal bypass for morbid obesity
Total parenteral nutrition
Prolonged intravenous hyperalimentation
Carbon tetrachloride poisoning
Ingestion of arsenic causes
*Non cirrhotic portal fibrosis
Hepatic adenoma
Hepatic carcinoma
Hepatic cirrhosis
No one mentioned
Liver cirrhosis is characterized by following clinical syndromes
*All mentioned
Cholestatic
Hepatomegaly
Dyspeptic
Hemorrhagic syndrome
Many patients have Budd-Chiari syndrome as a complication of:
*Paroxysmal nocturnal hemoglobinuria
Hepatocellular carcinoma
Right ventricular failure
Renal cell carcinoma
E.
874.
A.
B.
C.
D.
E.
875.
A.
B.
C.
D.
E.
876.
A.
B.
C.
D.
E.
877.
A.
B.
C.
D.
E.
878.
A.
B.
C.
D.
E.
879.
A.
B.
C.
D.
E.
880.
A.
B.
C.
D.
E.
881.
A.
B.
C.
D.
E.
F.
882.
A.
Pneumonia
Marker of Cholestatic syndrome in hepatitis is
*Alkaline phosphatase
All of above
vfibrinogen
↑AST
↑ALT
Marker of Cytolitic syndrome in hepatitis is
*↑ALT
All of mentioned
↑Alkaline phosphatase
↑cholesterol
↓fibrinogen
Marker of Liver cellular insufficiency syndrome in hepatitis is
*↓Albumine
↑ALT
↑GGT
↑Bilirubin
↑AST
Markers of Hypersplenism syndrome in liver cirrhosis are all of the following except
*leukocytosis, ↑ESR
thrombocytopenia
anemia+ thrombocytopenia
anemia+ thrombocytopenia +leukocytopenia
anemia+ ↑ESR
Markers of Hypersplenism syndrome in liver cirrhosis are
*anemia, thrombocytopenia, leukocytopenia
↑AST, ↑ALT, ↑GGT, ↑Bilirubin
↑Bilirubin, ↑Alkaline phosphatase, ↑GGT, ↑cholesterol
↓Albumine, ↓prothrombin, ↓cholesterol, ↓fibrinogen
No one mentioned
Markers of Hypersplenism syndrome in liver cirrhosis
*All mentioned
anemia
thrombocytopenia
leukocytopenia
No one mentioned
Markers of Hypersplenism syndrome in liver cirrhosis
*anemia, thrombocytopenia, leukocytopenia
↑ALT, ↑GGT
↑GGT, ↑Bilirubin
↑Bilirubin, ↑ALT, ↑GGT
↑AST, ↑ALT, ↑GGT
Markers of Mesenchyme-inflammatory syndrome in liver cirrhosis are
*↑ESR,
G-globulins, ↑timol test ↑Le, ↑C-react. protein
↑AST, ↑ALT, ↑GGT, ↑Bilirubin
↑Bilirubin, ↑Alkaline phosphatase, ↑GGT, ↑cholesterol
↓Albumine, ↓prothrombin, ↓cholesterol, ↓fibrinogen
No one mentioned
Morphologic characteristics of chronic active hepatitis mainly include all of the following except:
*severe fibrosis, including bridging fibrosis
a dense mononuclear infiltrate of the portal tracts;
B.
destruction of the hepatocytes at the periphery of the lobule;
C.
connective tissue septa surrounding portal tracts and extending from the portal zones into the
lobule, isolating parenchymal cells into clusters and enveloping bile ducts;
D.
evidence of hepatocellular regenerationѕ"rosette" formation, thickened liver cell plates, and
regenerative "pseudolobules";
883. Pathologic Mallory stain findings characterizes
A.
*Alcoholic hepatitis
B.
Autoimmune Hepatitis
C.
Cryptogenic Hepatitis
D.
Chronic Viral Hepatitis B
E.
Chronic Viral Hepatitis C
884. Peptic ulcer should be differentiated with
A.
*All mentioned
B.
Cholecystitis
C.
Gastritis
D.
Miocardial infarction
E.
Pancreatitis
885. Pruritus, elevation of alkaline phosphatase and positive antimitochondrial antibody test are
characteristic signs of
A.
*Primary biliary cirrhosis
B.
Sclerosing cholangitis
C.
Anaerobic liver abscess
D.
Hepatoma
E.
Hepatitis C
886. Risk factor of Budd-Chiari syndrome is:
A.
*Hormonal contraception
B.
Hepatocellular carcinoma
C.
Right ventricular failure
D.
Renal cell carcinoma
E.
Pneumonia
887. Signs of liver cirrhosis are all of the following except
A.
*Erythema nodosum
B.
Spider angiomata
C.
Palmar erythema
D.
Nail changes
E.
Hypertrophic osteoarthropathy
888. The stage of chronic hepatitis is based on the all of following except:
A.
*no fibrosis,
B.
mild fibrosis,
C.
necrosis
D.
moderate fibrosis,
E.
severe fibrosis, including bridging fibrosis,
889. There are all of the following causes of noncirrhotic hepatic fibrosis except:
A.
*All of them
B.
Intrahepatic phlebosclerosis and fibrosis;
C.
Portal and splenic vein sclerosis;
D.
Portal and splenic vein thrombosis;
E.
Schistosomiasis;
890. There are cholestatic diseases all of the following except:
A.
*gall stone,
B.
malignant obstruction
C.
primary biliary cirrhosis,
D.
drug-induced liver diseases,
E.
891.
A.
B.
C.
D.
E.
892.
A.
B.
C.
D.
893.
A.
B.
C.
D.
E.
894.
A.
B.
C.
D.
E.
895.
A.
B.
C.
D.
E.
896.
A.
B.
C.
D.
E.
897.
A.
B.
C.
D.
E.
898.
A.
B.
C.
D.
E.
899.
A.
B.
C.
passive congestion
There are following diagnostic tests for chronic hepatitis B except:
*HDV RNA
IgG anti-HBc,
HBeAg,
HBV DNA
HBsAg,
Typical physical findings in liver disease are all of the following except:
*hypertension
icterus,
hepatomegaly,
hepatic tenderness,
splenomegaly
What is not true about Autoimmune Hepatitis
*It is more common in men than in women
It can be associated with other autoimmune diseases
There is a rise in IgG, a positive anti-smooth-muscle antibody present in 60%.
Antinuclear antibody and liver-kidney microsomal antibody (LKM-1) may be present.
Response rate to corticosteroids therapy is 70-90%.
What is not true about CRYPTOGENIC HEPATITIS
*All mentioned
It is four times more common in women than in men
There is a rise in IgG, a positive anti-smooth-muscle antibody present in 60%.
Antinuclear antibody and liver-kidney microsomal antibody (LKM-1) may be present.
Response rate to corticosteroids therapy is 70-90%.
What is true about L1-ANTITRYPSIN DEFICIENCY:
*It results in cirrhosis and emphysema
Characterized by excessive deposition of iron in liver
Characterized by excessive deposition of copper in liver
No one mentioned
All mentioned
Which answer below represents an osmotic laxative?
*Lactulose.
Psyllium.
Magnesium citrate.
Castor oil.
Lubiprostrone.
Which is not a paraneoplastic syndrome for Hepatocellular Carcinoma
*Hyperglycemia
Hypoglycemia
Erythrocytosis
Hypercalcemia
None of mentioned
Which of the following is an important physiologic function of bile acids?
*Facilitating absorption of dietary fats
Conjugation with toxic substances, thus allowing
Allowing the excretion of hemoglobin breakdown
Aiding the absorption of vitamin B12
Maintaining appropriate intestinal pH
Which of the following statements is true regarding alpha-fetoprotein- L3?
*High levels are seen in hepatic carcinoma
Pre-operative high level indicates worse prognosis
High level are seen in stomach carcinoma
D.
Levels may be increased in hepatitis
E.
No one mentioned
900. Which of the following statements regarding cryptogenic hepatitis is true?
A.
*No specific test,
B.
HBsAg
C.
IgG anti-HBc,
D.
HBeAg
E.
anti-HBs,
901. Which of the following statements regarding delta hepatitis virus (HDV) is correct?
A.
*HDV can infect only persons infected with hepatitis
B.
HDV is a defective DNA virus.
C.
The HDV genome is partially homologous with
D.
HDV infection has been found only in limited areas
E.
Simultaneous infection with HDV and HBV results
902. Which of the following statements regarding liver cirrhosis isn’t true?
A.
*caused by viral hepatitis A
B.
is generally irreversible condition
C.
treatment focuses on preventing progression and complications
D.
caused by alcoholism
E.
caused by hepatitis B and C
903. Which of the following statements regarding liver cirrhosis isn’t true?
A.
*caused by viral hepatitis A
B.
is generally irreversible condition
C.
treatment focuses on preventing progression and complications
D.
caused by fatty liver disease
E.
ascites is the most common complication
904. Which of the following statements regarding primary biliary cirrhosis (PBC) is correct?
A.
*Rheumatoid arthritis, CREST syndrome, and scleroderma
B.
A positive anti-pus antibody test is present in more
C.
Glucocorticoid treatment is helpful.
D.
The majority of these patients are men.
E.
Administration of D-penicillamine appears to be an
905. Which of the following statements regarding the prophylaxis of viral hepatitis is true?
A.
*HBIg and hepatitis B vaccine can be effectivelyadministered simultaneously.
B.
Although immune globulin (Ig) is effective in preventingclinically apparent type A hepatitis, not allIg
preparations have adequate anti-HAV titers tobe protective.
C.
If given soon enough after exposure to hepatitis B,hepatitis immune globulin (HBIg) is effective
inpreventing infection.
D.
Hepatitis B vaccine is ineffective in preventingdelta hepatitis infection in persons who are notHBsAg
carriers.
E.
Ig prophylaxis after needle-stick, sexual, or perinatalexposure to hepatitis C is effective in
preventinginfection
906. Which one of the following is the cause of Non Cirrhotic portal fibrosis?
A.
*All of mentioned.
B.
Intrahepatic phlebosclerosis and fibrosis;
C.
Portal and splenic vein sclerosis;
D.
Portal and splenic vein thrombosis;
E.
Schistosomiasis;
907. Which one of the following statements about hepatitis B e antigen (HBeAg) is LEAST accurate?
A.
*The absence of HBeAg in the serum rules out chronic infection caused by the hepatitis B virus.
B.
The presence of HBeAg in the serum is correlated with infectiousness.
C.
HBeAg can be detected transiently in the sera of patients ill with acute hepatitis B infection.
D.
HBeAg is immunologically distinct from HBsAg but is genetically related to HBcAg.
E.
The disappearance of HBeAg from the serum may be a harbinger of resolution of acute hepatitis B
infection.
908. Wilson’s disease is characterized by:
A.
*Characterized by excessive deposition of copper in liver
B.
Characterized by excessive deposition of iron in liver
C.
results in cirrhosis and emphysema
D.
No one mentioned
E.
All mentioned
909. All of the following statements stand true for telengiectasia of colon except:
A.
May be seen in person with systemic scleroderma
B.
May be seen in person more than 60 years of age
C.
Common site is caecum
D.
May be seen in person less than 40 years of age
E.
*50% involve rectum
910. As more severe Crohn's disease develops, aphthous ulcers don’t become
A.
*"collar-button" ulcers
B.
enlarged
C.
deeper
D.
occasionally connected to one another
E.
forming linear ulcers
911. Best test for Small intestine malabsorption of carbohydrates is :
A.
Shilling test
B.
Lund meal test
C.
Follacin test
D.
None of above
E.
*D-Xylose test
912. Colonoscopy shows segmental areas of inflammation. SBFT shows nodular thickening of the
terminal ileum. What is the most likely diagnosis?
A.
Ulcerative colitis
B.
*Crohn’s disease
C.
Ischemic colitis
D.
Diverticulitis
E.
Amebic colitis
913. Complications of Crohn’s disease include all of the following except
A.
*strictures occurring in the rectum
B.
intestinal obstruction
C.
massive hemorrhage
D.
malabsorption
E.
severe perianal disease
914. Complications of Crohn’s disease include all of the following except
A.
*strictures occurring in the rectum
B.
fistula formation
C.
free perforation
D.
generalized peritonitis
E.
intraabdominal abscess
915. Complications of ulcerative colitis include
A.
*strictures occurring in the rectum
B.
intestinal obstruction
C.
toxic megacolon
D.
generalized peritonitis
E.
intraabdominal abscess
916. Crohn’s disease affects the lips:
A.
*Rarely
B.
Sometimes
C.
Frequently
D.
Never
E.
Invariably
917. Crohn’s disease is a disease that usually involves
A.
the rectum
B.
the mouth
C.
the anus
D.
small bowel disease alone
E.
*the terminal ileum
918. CT findings in Crohn’s disease include all of the following except
A.
*absence of small bowel thickening
B.
mural thickening >2 cm
C.
homogeneous wall density
D.
mural thickening of small bowe
E.
adenopathy
919. CT findings in ulcerative colitis include
A.
*absence of small bowel thickening
B.
mural thickening >2 cm
C.
homogeneous wall density
D.
mural thickening of small bowe
E.
mesenteric fat stranding
920. Deep ulcerations in ulcerative colitis can appear as
A.
*"collar-button" ulcers
B.
enlarged
C.
deeper
D.
occasionally connected to one another
E.
forming linear ulcers
921. Hyperpigmentation, hepatomegaly, polyarthritis, hyperglycemia are the signs of
A.
Insulin-dependent diabetes mellitus
B.
Pancreatic carcinoma
C.
Addison’s disease
D.
*Hemochromatosis
E.
Chronic Hepatitis C
922. In more advanced Crohn's disease all of the following may be detected except
A.
Strictures
B.
Fistulas
C.
inflammatory masses
D.
abscesses
E.
*Signet ring cells on gastric biopsy
923. Intestinal hypomotility is seen in all the following except:
A.
Parkinsonism
B.
Amyloidosis
C.
Diabetes
D.
Scleroderma
E.
*All of mentioned
924. Macrophages containing large quantities of undigested and partial digested bacteria in intestine are
seen in
A.
Amyloidosis
B.
Immunoapproliferative small instetinal disease
C.
Vibro cholerae infection
D.
Crohn’s disease
E.
*Whipple's disease
925. Mechanical obstruction of the colon is most commonly caused by
A.
adhesions
B.
*carcinoma
C.
volvulus
D.
hernia
E.
sigmoid diverticulitis
926. Metabolic changes associated with excessive vomitting includes the following:
A.
Metabolic acidosis
B.
Hyperchloremia
C.
Decreases bicarbonates
D.
Hypercalciemia
E.
*Hypokalemia
927. Most common Central nervous system manifestation of Whipple's disease is:
A.
Seizure
B.
Supranuclear ophthalmoplegia
C.
Cerebellar ataxia
D.
Coma
E.
*Dementia
928. Patient presents with diarrhea, weight loss, and RLQ pain, a tender mass is noted in the RLQ on
exam; the stool is guaiac-positive. What is the most likely diagnosis?
A.
Ulcerative colitis
B.
*Crohn’s disease
C.
Ischemic colitis
D.
Diverticulitis
E.
Amebic colitis
929. Pseudopolyps are features of
A.
Crohn's disease
B.
Celiac sprue
C.
Whipple's disease
D.
Reiter's disease
E.
*Ulcerative colitis
930. Sigmoidoscopy shows inflamed, friable mucosa from rectum to midsigmoid. What is the most likely
diagnosis?
A.
*Ulcerative colitis
B.
Crohn’s disease
C.
Ischemic colitis
D.
Diverticulitis
E.
Amebic colitis
931. Skip granulomatous lesions are seen in
A.
Whipple's disease
B.
Ulcerative colitis
C.
Reiter's disease
D.
Coeliac disease
E.
*Crohn's disease
932. Skip granulomatous lesions of bowel are seen in
A.
*Crohn's disease
B.
Ulcerative colitis
C.
Whipple's disease
D.
Reiter's disease
E.
Irritable bowel disease
933. Skip granulomatous lesions of bowel are seen in
A.
*Crohn's disease
B.
Celiac disease
C.
D.
E.
934.
A.
B.
C.
D.
E.
935.
A.
B.
C.
D.
E.
936.
A.
B.
C.
D.
E.
937.
A.
B.
C.
D.
E.
F.
G.
H.
I.
J.
938.
A.
B.
C.
D.
E.
939.
A.
B.
C.
D.
E.
940.
A.
B.
C.
D.
E.
941.
A.
B.
Whipple's disease
Reiter's disease
Irritable bowel disease
Subacute ischemic colitis can best be described by which of the following statements?
*Cobblestoning is noted on contrast studies.
Patients present with an acute abdomen.
Involvement of the rectum is common.
Symptoms and signs of nonocclusive ischemic colitis rarely resolve.
Angiography is the definitive diagnostic procedure.
The disease of gastrointestinal tract when aphthoid ulcerations are seen in ileal is
*Crohn's disease
Ulcerative colitis
Whipple's disease
Reiter's disease
Celiac sprue
The earliest macroscopic findings of colonic Crohn's disease are
*aphthous ulcers.
Strictures
Fistulas
inflammatory masses
abscesses
The early roentgenographic findings in the small bowel include thickened folds are typical in
Gastro-oesophageal reflux disease
*Crohn's disease
Ulcerative colitis
Irritable bowel syndrome
Celiac sprue
Gastro-oesophageal reflux disease
Crohn's disease
Ulcerative colitis
Irritable bowel syndrome
Celiac sprue
The endoscopic changes - aphthoid ulcerations are typical for
Gastro-oesophageal reflux disease
*Crohn's disease
Ulcerative colitis
Irritable bowel syndrome
Celiac sprue
The histological features of coeliac disease include all of the following, except:
Crypt hyperplasia
Lymphocyte infiltration of crypts
Increase in inflammatory cells in lamina propyria
Blunting of villi
*Increase in thickness of the mucosa
The most likely histological abnormality associated with celiac sprue (gluten-sensitive enteropathy) is
Signet ring cells on gastric biopsy
Mucosal inflammation and crypt abscesses on sigmoidoscopy
*Villous atrophy and increased lymphocytes in the lamina propria on small bowel biopsy
Small, curved gram-negative bacteria in areas of intestinal metaplasia on gastric biopsy
Edema and basal hyperplasia
The roentgenographic changes - "string sign" are typical in
Gastro-oesophageal reflux disease
*Crohn's disease
C.
Ulcerative colitis
D.
Irritable bowel syndrome
E.
Celiac sprue
942. Toxic megacolon is most commonly associated with
A.
Reiter's disease
B.
Crohn's disease
C.
Whipple's disease
D.
Coeliac disease
E.
*Ulcerative colitis
943. Ulcerative colitis is a mucosal disease that usually involves
A.
*the rectum
B.
the mouth
C.
the anus
D.
small bowel disease alone
E.
the terminal ileum
944. Ulcerative colitis:
A.
is more common in smokers than in non-smokers
B.
is treated first-line with infliximab
C.
commonly involves the jejunum
D.
is associated in most patients with psoriasis
E.
*is associated with HLA B27
945. What extrabowel sing of ulcerative colitis may be in patient if insidious onset of progressive fatigue,
pruritus, and jaundice have developed?
A.
Bile duct tumor
B.
Choledocholithiasis
C.
Congenital polycystic liver
D.
Primary biliary cirrhosis
E.
*Primary sclerosing cholangitis
946. What imaging modality is most appropriate for patient with diverticulitis?
A.
Ultrasound of the abdomen.
B.
*CT scan with and without contras
C.
Colonoscopy.
D.
Barium enema.
E.
Plain upright abdominal x-ray.
947. What is drug of choice for mild activity of ulcerative colitis:
A.
Vitamin B
B.
Prednisolone
C.
Mercaptopurine
D.
Misoprostol
E.
*5-amino salicylic acid
948. What is the most likely diagnosis in 72-year-old male with of severe abdominal pain after eating? His
past medical history is positive for peripheral vascular disease and hypertension.
A.
Small bowel obstruction.
B.
Diverticulitis.
C.
*Mesenteric ischemia.
D.
Viral gastroenteritis.
E.
Ulcerative colitis.
949. What is the next step in management of patient with peritonitis after obtaining a sample of the
peritoneal fluid and send it for culture and analysis?
A.
Obtain CT scan of the abdomen.
B.
Perform a complete therapeutic pericentesis.
C.
Start IV dexamethasone.
D.
*Start empiric antibiotics.
E.
Barium radiology
950. What street drug has the side effect of constipation, which may border on obstipation in severe cases?
A.
*Methamphetamines.
B.
Heroin.
C.
Smokable cocaine (Crack).
D.
Phencylidine.
E.
Inhalents.
951. Which ANSWER below represents an osmotic laxative?
A.
Euphyllin.
B.
Magnesium sulfat.
C.
Castor oil.
D.
Lubiprostrone.
E.
*Lactulose.
952. Which modality below comprises an acceptable colon cancer screening strategy for average-risk
patients?
A.
Fecal occult blood testing (FOBT) on three consecutive stools semi-annually.
B.
FOBT on at least five stools over 2 years.
C.
Single-contrast barium enema every 5 years.
D.
*Double-contrast barium enema every 5 years.
E.
Flexible sigmoidoscopy every 7 years.
953. Which of the following conditions does not cause secretory diarrhea?
A.
Medullary carcinoma of the thyroid
B.
Resection of the distal ileum
C.
Zollinger-Ellison syndrome
D.
*Somatostatinoma
E.
Carcinoid tumor
954. Which of the following represents an incorrect mechanism of action for the laxative noted?
A.
Psyllium increases stool bulk.
B.
Docusate salts lower the surface tension of stool.
C.
*Sorbitol stimulates colonic motor activity.*
D.
Cisapride enhances intestinal transit.
E.
Castor oil stimulates intestinal secretion.
955. Which of the following statements describing Meckel’s diverticulum is correct?
A.
*It is the most common congenital anomaly of the digestive tract
B.
Mechanical obstruction resulting from stricture may occur
C.
In young adults inflammatory complications may produce a clinical syndrome indistinguishable from
gastroenteritis.
D.
It is usually present in the jejunum.
E.
Barium studies are valuable in the diagnosis of diverticula associated with gastrointestinal bleeding.
956. Which of the following statements regarding eosinophilic enteritis is correct?
A.
*It may be difficult to distinguish from regional enteritis.
B.
It affects only the small intestine.
C.
The majority of patients have a history of food allergies or asthma.
D.
Treatment with glucocorticoids is not indicated.
E.
Peripheral blood eosinophilia is rare.
957. Which of the following structures does not form the external part of anal canal mechanism?
A.
*Internal Sphincter
B.
Levator Ani
C.
Pubo rectalis
D.
External Sphincter
E.
None of mentioned
958. Which of the following tests allows comprehensive evaluation of the small intestine and provision of
treatment without sedation or significant risk to the patient?
A.
Small bowel enteroclysis
B.
Mesenteric arteriography
C.
Technetium labeled bleeding scan
D.
Capsule endoscopy
E.
*None of the above
959. Which of the following tests will be most helpful in the evaluation of a patient with lower
gastrointestinal bleeding who presents with massive hemorrhage and a negative nasogastric lavage?
A.
Colonoscopy
B.
Radionuclide bleeding scan
C.
Upper endoscopy
D.
Capsule endoscopy
E.
*Arteriography
960. Which one isn’t detected in more advanced Crohn's disease?
A.
Strictures
B.
Fistulas
C.
inflammatory masses
D.
abscesses
E.
*Signet ring cells on gastric biopsy
961. Which one of the following medications is not useful in the disease of gastrointestinal tract when
deep, punched-out ulcers in ileal are seen?
A.
metronidazole
B.
mesalamine
C.
6-mercaptopurine
D.
Cyclosporine
E.
*Clarithromycin
962. Which one of the following medications is not useful in the disease with patchy, specific lesions of
bowel?
A.
metronidazole
B.
mesalamine
C.
6-mercaptopurine
D.
Cyclosporine
E.
*Clarithromycin
963. Which one of these extraintestinal complications of inflammatory bowel disease is LEAST likely to
be associated with ulcerative colitis?
A.
Pericholangitis
B.
Pyoderma gangrenosum
C.
Arthritis
D.
Uveitis
E.
*Oxalate kidney stones
964. Which statement regarding acute bleeding from colonic diverticula is correct?
A.
Diverticulitis usually is present.
B.
The source of hemorrhage is more likely to be on the left side than on the right side of the colon.
C.
Bleeding usually becomes life-threatening.
D.
*Bleeding usually becomes life-threatening.
E.
It is an uncommon cause of acute lower GI bleeding
965. The triad originally described by Zollinger-Ellison syndrome is characterized by:
A.
Peptic ulceration, gastric hypersecretion, beta cell tumour
B.
Peptic ulceration, achlorhydria, non beta cell tumour
C.
Peptic ulceration, achlorhydria, beta cell tumour
D.
Duodenal ulceration, achlorhydria, beta cell tumour
E.
*Peptic ulceration, gastric hypersecretion, non beta cell tumour
966. Ultrasonography can provide important information in patients with all of the following except:
A.
*perforated duodenal ulcer
B.
acute pancreatitis,
C.
chronic pancreatitis,
D.
pancreatic calcification,
E.
pseudocyst.
967. Ultrasound examination of the gallbladder is a useful diagnostic examination, the incidence of its
false-positive results is:
A.
*2%
B.
51%
C.
96%
D.
5-10%
E.
10%
968. What change can suggest severe pancreatic exocrine insufficiency?
A.
Steatorrhea
B.
marked excretion of fecal fat
C.
*decreased serum trypsinogen level
D.
elevated serum amylase
E.
elevated serum lipase levels
969. What imaging modality may be limited in patient with cholecystitis?
A.
MRCP.
B.
CT scan.
C.
*Ultrasonography.
D.
Esophogastroduodenoscopy (EGD).
E.
HIDA scan.
970. What is the best test for acute pancreatitis?
A.
*Lipase
B.
Serum amylase activity
C.
Urine amylase activity
D.
Plasma Bilirubine
E.
Alkaline phosphatase activity
971. What is the only major technique that provides a direct view of the pancreatic duct?
A.
*Endoscopic retrograde cholangiopancreatography
B.
Sonography
C.
CT
D.
Plain radiography
E.
Laparoscopy.
972. What patients with gall stones tend to induce pancreatitis?
A.
*Those patients who have common pancreatico-biliary channels in the ampullary of Vater
B.
Those patients who do not have common pancreatico-biliary channels
C.
Anyone who has gall stones
D.
None of the listed
E.
All of the listed
973. Which answer below represents an osmotic laxative?
A.
*Lactulose.
B.
Psyllium.
C.
Magnesium citrate.
D.
Castor oil.
E.
Lubiprostrone.
974. Which drug is not prescribed in case of chronic cholecystitis, acute phase:
A.
*Pangrol
B.
Analgin
C.
No-shpa
D.
Amoxicillin
E.
Platyphyllin
975.
A.
B.
C.
D.
E.
976.
A.
B.
C.
D.
E.
977.
A.
B.
C.
D.
E.
978.
A.
B.
C.
D.
E.
979.
A.
B.
C.
D.
E.
980.
A.
B.
C.
D.
E.
981.
A.
B.
C.
D.
E.
982.
A.
B.
C.
D.
E.
983.
A.
B.
C.
D.
Which drug is not prescribed in case of chronic pancreatitis:
*Dicloberl
Creon
Pangrol
Lansoprazol
Panzinorm
Which drug is not prescribed in case of dyskinesia of bile ducts, hypertonic type:
*Chophitol
Platyphyllin
No-shpa
Duspatalin
Meteospasmil
Which drug is not prescribed in case of dyskinesia of bile ducts, hypotonic type:
Chophitol
Cerucal
Motilium
Domperidon
*Meteospasmil
Which drug is not prescribed in case of dyskinesia of bile ducts:
Chophitol
Hepabene
Motilium
Domperidon
*Aspirin
Which drug is not prescribed in case of exocrine insufficiency of pancreas:
*Prednisolon
Creon
Pangrol
Festal
Panzinorm
Which drug is not prescribed in case of steatorrhea:
*Corticosteroids
Creon
Pangrol
Festal
Panzinorm
Which drug is prescribed in case of chronic cholecystitis, acute phase:
Pangrol
Creon
*No-shpa
Festal
Panzinorm
Which drug should be administrated if steatorrhea is present:
Corticosteroids
Vitamins A, K, D
Azatioprine
Riboxin
*Panzinorm
Which drug should be administrated in case of dyskinesia of bile ducts, hypertonic type:
*No-shpa
Vitamins A, K, D
Chophitol
Cerucal
E.
Pangrol
984. Which drug should be administrated in case of dyskinesia of bile ducts, hypotonic type:
A.
No-shpa
B.
Vitamins A, K, D
C.
*Chophitol
D.
Duspatalin
E.
Pangrol
985. Which drug should be administrated in case of endocrine insufficiency of pancreas:
A.
Aspirin
B.
Vitamins A, K, D
C.
Azatioprine
D.
*Insulin
E.
Pangrol
986. Which drug should be administrated in case of exocrine insufficiency of pancreas:
A.
Aspirin
B.
Vitamins A, K, D
C.
Azatioprine
D.
Insulin
E.
*Pangrol
987. Which entity is most difficult to diagnose using MR cholangiopancreatography (MRCP)?
A.
Congenital anomaly
B.
Traumatic duct injury
C.
Adenocarcinoma
D.
*Acute pancreatitis
E.
Intraductal papillary mucinous neoplasm (IPMN
988. Which feature of pancreatic pseudocyst on CT suggests secondary infection?
A.
*Gas formation in the cyst.
B.
Calcification of the cyst wall.
C.
Heterogeneous attenuation of cyst contents.
D.
Enhancement of the cyst wall after IV contrast injection.
E.
Extrapancreatic extension.
989. Which feature of pancreatic pseudocyst on CT suggests secondary infection?
A.
Calcification of the cyst wall
B.
*Gas formation in the cyst
C.
Heterogeneous attenuation of cyst contents
D.
Enhancement of the cyst wall after IV contrast injection
E.
Extrapancreatic extension
990. Which imaging technique or procedure is most specific for differentiating a mucinous cystic tumor
from a microcystic adenoma?
A.
Endoscopic retrograde cholangiopancreatography (ERCP).
B.
*Percutaneous cyst aspiration
C.
MR cholangiopancreatography
D.
CT
E.
Endoscopic sonography
991. Which is the common cause of chronic pancreatitis?
A.
Gall stones
B.
Autoimmune diseases
C.
*Alcohol
D.
None of above
E.
Tumour of stomach
992. Which of the following is not true for ectopic pancreas
A.
stomach and duodenum are the most common site
B.
Ectopic pancreas appears as a submucosal irregular nodule in the wall
C.
D.
E.
993.
A.
B.
C.
D.
E.
994.
A.
B.
C.
D.
E.
995.
A.
B.
C.
D.
E.
996.
A.
B.
C.
D.
E.
997.
A.
B.
C.
D.
E.
998.
A.
B.
C.
D.
E.
999.
A.
B.
C.
D.
E.
1000.
A.
B.
C.
D.
E.
1001.
A.
*Islet tissue is present in all the organs where ectopic pancreas is present
Ulceration, bleeding and obstruction are the most common symptoms
None of above
Which of the listed organs breaks down or modifies toxic substances?
*Liver
Gall bladder
Pancreas
Spleen
Duodenum
Which of the listed organs excretes digestive substances but does not produce them?
*Gall bladder
Pancreas
Liver
Spleen
Duodenum
Which organ also functions as an endocrine gland
*Pancreas
Gall bladder
Liver
Spleen
Duodenum
Which organ excretes a buffer that neutralizes acid from the stomach?
*Pancreas
Gall bladder
Liver
Spleen
Duodenum
Which organ functions in detoxification of the blood
Pancreas
Gall bladder
*Liver
Spleen
Duodenum
Which organ produces bile salts
Pancreas
Gall bladder
*Liver
Spleen
Duodenum
Which organ releases a buffer that neutralizes acid from the stomach
*Pancreas
Gall bladder
Liver
Spleen
Duodenum
Which organ releases digestive substances but does not produce them
Pancreas
*Gall bladder
Liver
Spleen
Duodenum
Which organ releases the digestive hormone secretin
Pancreas
B.
Gall bladder
C.
Liver
D.
Spleen
E.
*Duodenum
1002. Zollinger-Ellison syndrome is characterized by all of the following except
A.
increased levels of the hormone gastrin
B.
non-beta cell islet tumour of pancreas
C.
severe peptic ulceration
D.
gastric acid hypersecretion
E.
*Serum amylase is increased
1003. Elevated serum ferritin, serum iron and percent transferrin saturation are most consistent with the
diagnosis of:
A.
*Hemochromatosis
B.
Lead poisoning
C.
Iron deficiency anemia
D.
Anemia of chronic disease
E.
Porphyria
1004. Which is not seen in Iron deficiency anaemia:
A.
*Hyper-segmented neutrophils
B.
MCHC<35 g/dL
C.
Microcytosis
D.
hypochromia
E.
MCV< 80 fL
1005. Most sensitive and specific test for diagnosis of iron deficiency is:
A.
Serum iron level
B.
*Serum ferritin level
C.
Serum transferrin receptor population
D.
Transferrin saturation
E.
Hb level
1006. The earliest sign of iron deficiency anaemia is:
A.
Decrease in transferrin level
B.
Decrease in transferrin level, increase in iron binding capacity
C.
*Decrease in serum ferritin level
D.
Increase in iron binding capacity
E.
Increase in serum folic acid level
1007. Iron overload occurs in all, except:
A.
Anemia due to chronic infection
B.
Myelodysplastic syndrome
C.
*Polycythemia vera
D.
Sideroblastic anaemia
E.
Thalassemia
1008. All are true regarding Anaemia of Chronic Diseases, except:
A.
*Decreased serum ferritin level
B.
Normocytic anemia
C.
Low Total Fe Binding Capacity
D.
Increased Bone Marrow Fe
E.
Decreased serum Fe
1009. Anemia of chronic disease is characterized by all, except
A.
*Increased total iron binding capacity
B.
Increased serum ferritin
C.
Increased macrophage iron in bone marrow
D.
Decreased serum iron
E.
Normocytic anemia
1010. Not present in Sideroblastic anaemia:
A.
*Decreased serum iron
B.
Decreased transferrin saturation
C.
Sideroblast cells in blood smear film
D.
Ineffective erythropoiesis
E.
Microcytic anaemia
1011. Which is not seen in a chronic case of Sickle cell anemia :
A.
Pulmonary hypertension
B.
Cardiomegaly
C.
*Hepatomegaly
D.
Splenomegaly
E.
structural orthopedic abnormalities
1012. All are true for sickle cell anemia, except
A.
*Leukopenia
B.
Pulmonary arterial hypertension
C.
Fish vertebra
D.
Increased size of heart
E.
Gallstones
1013. The mother has sickle cell disease; Father is normal; Chances of children having sickle cell disease
and sickle cell trait respectively are :
A.
*0 and 100%
B.
25 and 25%
C.
50 and 50%
D.
10 and 50%
E.
25 and 25%
1014. In Beta thalassemia, there is:
A.
*Decrease in beta chain, increase in alpha chain
B.
Sick shaped RBC
C.
Decrease in beta chain, decrease in alpha chain
D.
Increase in beta chain, increase in alpha chain
E.
Increase in beta chain, decrease in alpha chain
1015. The peripheral blood smear of a patient shows features of thalassemia, also presented with anaemia.
Family history is also +ve. The investigation done to establish the diagnosis is:
A.
Biochemical blood test
B.
Blood spherocyte estimation
C.
*Hb-electrophoresis
D.
Bone marrow aspiration
E.
ESR estimation
1016. Diagnosis of beta Thalassemia is established by:
A.
Blood spherocyte estimation
B.
Target cells in peripheral smear
C.
*Hb electrophoresis
D.
Osmotic fragility test
E.
Hb A C estimation
1017. All are seen is extravascular Hemolytic anemia except:
A.
Hemosiderinuria
B.
Reticulocytosis
C.
*Increased haptoglobin
D.
Spherocytosis
E.
elevated indirect bilirubin level
1018. Reticulocytosis is NOT a feature of
A.
Paroxysmal nocturnal hemoglobinuria
B.
Following acute bleeding
C.
D.
E.
1019.
A.
B.
C.
D.
E.
1020.
A.
B.
C.
D.
E.
1021.
A.
B.
C.
D.
E.
1022.
A.
B.
C.
D.
E.
1023.
A.
B.
C.
D.
1024.
A.
B.
C.
D.
E.
1025.
A.
B.
C.
D.
E.
1026.
A.
B.
C.
D.
E.
1027.
A.
B.
*Aplastic anemia
Hereditary spherocytosis
Acquired hemolytic anemia
Coomb's +ve Hemolytic Anaemia may be seen in all of the following except:
*Alcoholic cirrhosis
Chronic active hepatitis
Primary biliary cirrhosis
Primary sclerosing cholangitis
Systemic lupus erythematosus
Which of the following conditions may be associated with Coomb's positive hemolytic anaemia:
*Systemic lupus erythematosus.
Thrombotic thrombocytopenic purpura.
Progressive systemic sclerosis.
HUS
Alcoholic cirrhosis
Most iron is stored in combination with:
Sulphate
Folic acid
Transferrin
*Ferritin
Ascorbic acid
Low serum iron and elevated TIBC are seen in :
*Iron deficiency anemia
Sideroblastic anemia
Thalassemia major
Sickle cell anemia
Megaloblastic anemia
Microcytic hypochromic anemia is seen in
Iron deficiency anemia
Thalassemia
Vitamin C deficiency
*Iron deficiency anemia, Thalassemia
All of them
Sickle cell anemia
Coombs positive hemolytic anemia associated with:
*SLE
TTP
Sickle cell anemia
Macrocytes in peripheral blood cells are seen in all except:
*Aplastic anemia
Chronic gastritis A
Gastrectomy
Bl2 deficiency
Folate deficiency
Hypersegmented neutrophils are seen in:
CML
*Addisonian anemia
Iron deficiency anemia
Myelodysplasias
CLL
Common features of haemolytic anemia are:
*Decreased RBC life span
Increased Haptoglobin
C.
Unconjugated hyperbilirubinemia
D.
Bile salt and bile pigments in urine
E.
None of them
1028. Increased osmotic fragility is seen in:
A.
Non-Hodgkin's lymphoma
B.
Sickle cell anemia
C.
£-Thalassemia
D.
*Hereditary spherocytosis
E.
CLL
1029. Ferritin is seen in following except:
A.
*Brain
B.
Liver
C.
skeletal muscles
D.
bone marrow
E.
Spleen
1030. The total daily loss of iron amounts to about:
A.
2 mg
B.
0.5 mg
C.
*1 mg
D.
10 mg
E.
0.05 mg
1031. Microcytic anemia is seen in cases of :
A.
Sprue
B.
Pernicious anemia
C.
Latun infestation
D.
*Iron deficiency anemia
E.
Aplastic anemia
1032. Elevated serum ferritin with normal body iron stores is a feature of:
A.
*Liver damage
B.
Thalassemia major
C.
Sideroblastic anemia
D.
Sickle cell anemia
E.
Iron deficiency
1033. Macrocytic anaemia is seen in all except
A.
Tropical Sprue
B.
Pernicious anaemia
C.
Latum infestation
D.
*Iron deficiency
E.
Folic deficiency
1034. Raised platelet count may be found in
A.
*Acute haemorrhage, CML
B.
ALL
C.
Acute haemorrhage
D.
Aplastic anemia
E.
Hemolytic anemia
1035. Disease usually without splenomegaly for which Splenectomy is effective
A.
*Thalassemia
B.
Sickle cell disease
C.
Idiopathic thrombocytopenic purpura
D.
Aplastic anemia
E.
Iron deficiency anemia
1036. Consider the following: Serum iron (1), MCV (2), serum total iron binding capacity (3), serum
ferritin (4). Which of these are decreased in iron deficiency anaemia?
A.
B.
C.
D.
E.
1037.
A.
B.
C.
D.
E.
1038.
A.
B.
C.
D.
E.
1039.
A.
B.
C.
D.
E.
1040.
A.
B.
C.
D.
E.
1041.
A.
B.
C.
D.
E.
1042.
A.
B.
C.
D.
E.
1043.
A.
B.
C.
D.
E.
1044.
A.
B.
C.
D.
E.
1 and 2
1, 3 and 4
*1, 2 and 4
1, 2, 3 and 4
None of them
Low serum iron and increased total iron binding capacity are seen in:
*Iron deficiency anaemia
Anemia of chronic infection
Megaloblastic anemia
Sideroblastic anaemia
Folic acid deficiency anemia
The following are the features of thalassemia except
*Increased osmotic fragility
Bone marrow hyperplasia
hypochromasia and microcytosis
slight icteric discoloration of the sclerae
Serum iron level is elevated
All of the following are features of sideroblastic anemia except
Microcytosis with hypochromia
Increased iron stores in bone marrow
Response to pyridoxine therapy
*Response to folic acid therapy
None of them
Intrinsic factor for absorption of vitamin Bl2 is secreted by the :
*Parietal cells
Peptic cells
Chief cells
Goblet cells
Endothelial cells
Hypersegmented Neutrophils are seen in:
Aplastic anaemia
Iron deficiency anaemia
Leukemia
*Megaloblastic anaemia
Anemia in chronic diseases
Aplastic anemia can be caused by all except:
*ampicillini
chloramphenicol
cquinine
phenytoin
carbamazepine
"M" spike on serum electrophoresis is seen in:
Acute lymphoblastic leukaemia
Alpha-chain disease
AIDS
*Waldenstrom's macroglobulinaemia
Chronic myelogenous leukemia
The single most powerful predictor of survival in multiple myeloma is
*Serum beta 2-microglobulin level
"M" component production
Bone marrow plasmocytosis
Serum calcium level
Waldenstrom's macroglobulinaemia
1045.
A.
B.
C.
D.
E.
1046.
A.
B.
C.
D.
E.
1047.
A.
B.
C.
D.
E.
1048.
A.
B.
C.
D.
E.
1049.
A.
B.
C.
D.
E.
1050.
A.
B.
C.
D.
E.
1051.
A.
B.
C.
D.
E.
1052.
A.
B.
C.
D.
E.
1053.
A.
B.
C.
D.
Which of the following is true of iron deficiency anemia:
Iron binding capacity is decreased
Increased number of sideroblasts
*Serum iron concentration is decreased
Hypersegmented Neutrophils are seen
Normal serum ferritin
Anemia of chronic diseases is characterised by all except:
*Decreased serum ferritin
Increased macrophage bone iron
Reduced TIBC
Reduced serum iron
mild normocytic anemia
A deficiency of folic acid usually occurs in association with all of the following except:
*Latum infestation
alcoholism
Late pregnancy
methotrexate
Intestinal malabsorption
Features of Vitamin B12 deficiency megaloblastic anemia include:
Smooth and atrophic tongue
Paresthesia
Macrocytosis
*All of the above
Hypersegmentation of neutrophils
Folic acid deficiency in megaloblastic anemia is due to:
Intrinsic factor deficiency
Inhibition of purine synthesis
*Folate trap mechanism
Methylcobalamine co-factor deficiency
None of them
Haemolytic anemias are characterised by all except:
Reticulocytosis
Hypercellular marrow
*Increased haptoglobulin
Methemalbuminemia
Decreased of RBC
The most common cause of aplastic anemia is:
*Idiopathic
Chloramphenicol
Phenylbutazone
Petroleum products
Prednisolone
Treatment of choice for aplastic anemia is:
Methotrexat
ampicillini
chloramphenicol
*Bone marrow transplantation
plasmaphoresis
Most sensitive and specific test for diagnosis of iron deficiency is:
*Serum ferritin levels
Serum iron levels
Serum transferrin receptor population
Transferrin saturation
E.
1054.
A.
B.
C.
D.
E.
1055.
A.
B.
C.
D.
E.
1056.
A.
B.
C.
D.
E.
1057.
A.
B.
C.
D.
E.
1058.
A.
B.
C.
D.
E.
1059.
A.
B.
C.
D.
E.
1060.
A.
B.
C.
D.
E.
1061.
A.
B.
C.
D.
E.
1062.
A.
B.
C.
Hb, Ht
Laboratory finding in anemia of chronic disease is
*TIBC decreased
Serum iron decreased
Ferritin decreased
Hypoalbulinemia
Macrocytic anemia
Reduced serum iron and iron binding capacity is seen in:
Sideroblastic anemia
Thalassemia
Iron deficiency anemia
*anemia of chronic disease
Hemolytic anemia
Following features may be present in patients of paroxysmal nocturnal haemoglobinuria except
Raised lactate dehydrogenase
Reticulocytjsis
*High leucocyte alkaline phosphatase
Hemosiderinuria
Decreased RBC
Mucosal transfer of iron in GIT by
*Transferrin
Apoferritin
Apotransferrin
Ferritin
Haptoglobin
Pernicious anemia is due to:
Iron deficiency
Chronic liver disease
Bleeding
*Atrophic gastritis
Hemolysis
Spherocytosis is best diagnosed by:
Splenic puncture
BM aspiration
Plasma
*Peripheral blood smear
Phenotyping
Coomb's positive hemolytic anaemia is seen in:
*Systemic lupus erythematosus
Thrombotic thrombocytopenic purpura
Scleroderma
Polyarteritis nodosa
Idiopathic thrombocytopenic purpura
Paroxysmal nocturnal haemoglobinuria is due to:
Antibodies
Congenital membrane defect
Inherited autoimmune disorder
*Complement mediated RBC lysis
Lysis of RBC in spleen
The following statements about Fanconi's anaemia are true except:
*Autosomal dominant inheritance
Progressive pancytopenia
Increased predisposition to malignancy
D.
Increased chromosomal fragility
E.
Many patients eventually develop acute myelogenous leukemia
1063. Increased RBC level is seen in:
A.
Glomerulonephritis
B.
Vitamin D excess
C.
*Cor pulmonale
D.
Acute heart failure
E.
Pneumonia
1064. Which is not true of polycythemia vera:
A.
Low erythropoietin
B.
*Raised ESR
C.
Raised LAP scores
D.
Raised total blood volume
E.
Gouty arthritis may be present in up to 20% of patients
1065. All are true regarding paroxysmal nocturnal haemoglobinuria, except:
A.
Direct antiglobulin test (direct Coombs' test) is negative
B.
Red urine
C.
*Presence of antibodies
D.
Reticulocytosis
E.
Raised bilirubin
1066. Which of the following is NOT seen in Paroxysmal Nocturnal Hemoglobinuria:
A.
Thrombosis
B.
Hemosiderinuria
C.
*Increased haptoglobin
D.
Thrombocytopenia
E.
Anemia
1067. A stem cell disorder affecting all the three cell lines platelets, RBCs and leucocytes is:
A.
*Paroxysmal nocturnal haemoglobinuna
B.
Hemolytic anaemia
C.
Paroxysmal cold haemoglobinuria
D.
Blackfan Diamond syndrome
E.
Folic acid deficiency anemia
1068. A 20 yrs adult presents with severe hypoplastic anemia. What is most effective treatment:
A.
a-interferon
B.
IL-2
C.
ATG therapy
D.
*Bone marrow transplantation
E.
Chloramphenicol
1069. Data of auscultation of a patient with anemia are the following:
A.
Tripple rhythm
B.
Bradycardia
C.
Accentuation of the II sound above the pulmonary artery
D.
*Systolic murmur at the apex
E.
Accentuation of the II sound above the aorta
1070. What is the dosage of Vincristine during the treatment of Ph-negative ALL for patients in second or
greater relapse or whose disease has progressed following 2 or more antileukemia therapies?
A.
0,01 mg/ m² IV infusion over 1 hr q7days for 4 weeks,
B.
0,1 mg/ m² IV infusion once a week for 4 weeks,
C.
*2.25 mg/m² IV infusion over 1 hr q7days for 4 weeks,
D.
0,5 mg/kg intravenously once a week for 4 weeks,
E.
1 mg/kg intravenously once a week for 4 weeks,
1071. What is the reason thought to be the cause of complications in patient with chronic myelocytic
leukemia?
A.
*Cerebral ishemia
B.
Gum bleeding
C.
High white cells count
D.
Hypertension
E.
Leukemic infiltration of organs
1072. What genetic sign indicates on chronic myelogenous leukemia
A.
No sings
B.
XXX or XXY combination
C.
*Philadelphia chromosome in abnormal cells
D.
chromosomes in the 19-st pare
E.
Lack of chromosome 13 pare
1073. What are the main laboratory findings in patient with chronic lymphatic leukemia?
A.
Thrombocytosis
B.
Thrombocytosis and lymphocytosis
C.
Lymphopenia
D.
*Anemia and thrombocytopenia
E.
Philadelphia chromosome in abnormal cells
1074. Which drug is mostly used for the treatment of chronic lymphatic leukemia?
A.
Prednisone
B.
*Chlorambucil
C.
Methotrexate
D.
Myleran
E.
Ampicilin
1075. What is the Acute Lymphatic Leukemia?
A.
*Disorder of the blood-forming tissue (white cells)
B.
Disorder of the blood-forming tissue (red cells)
C.
Leukocytosis
D.
Increasing of blood pressure
E.
Decreasing of blood pressure
1076. What is a possible level of platelets in the patient with Acute Lymphoblastic Leukemia?
A.
*100.000 – 150.000/L
B.
Over 200.000/L
C.
150.000– 200.000/L
D.
Below 100.000/L
E.
Absence of Pl
1077. What are the chief laboratory findings in patient with chronic lymphatic leukemia?
A.
Thrombocytosis
B.
Thrombocytosis and lymphocytosis
C.
*Lymphocytosis
D.
No specific findings
E.
Philadelphia chromosome in abnormal cells
1078. In which patients with chronic myelogenous leukemia the intracerebral hemorrhage occurs more
frequently?
A.
*With high white cells counts (over 300 x 1012/L)
B.
With low white cells counts
C.
With no changes
D.
Increased level of erythrocytes
E.
Anemia
1079. Bone marrow transplant is the treatment of choice in all, except:
A.
Aplastic anemia
B.
AML in 1st remission
C.
ALL in second remission
D.
*ITP
E.
1080.
A.
B.
C.
D.
E.
1081.
A.
B.
C.
D.
E.
1082.
A.
B.
C.
D.
E.
1083.
A.
B.
C.
D.
E.
1084.
A.
B.
C.
D.
E.
1085.
A.
B.
C.
D.
E.
1086.
A.
B.
C.
D.
E.
1087.
A.
B.
C.
D.
E.
1088.
A.
B.
C.
Lympoma
Findings in hemophilia A include:
*Partial thromboplastin time increased
Increased clotting time
Increased prothrombin time
Bleeding time increased
Thrombocytopenia
Christmas disease is treated by
F.F. blood
*F.F. plasma
Cryoprecipitate
Steroids
Vincristine
Most important aspect of treatment of DIC :
Heparin
FFP and cryoprecipitate
*Treat the primary cause
Fluids
F.F. plasma
Fragmented RBC's are seen in
ITP
Spherocytosis
*DIC
Patient on cardiac valves
CLL
Oral anticoagulant therapy is monitored with indexes
*INR
PTT
TT
Clot lysis time
Bleeding time
Patients with coagulation abnormality due to liver diseases are likely to have :
Prolonged bleeding time
Thrombocytosis
*Prolonged prothrombin time
Decreased partial thromboplastin time
Bleeding time prolonged
Patient on heparin therapy should be monitored with:
*Clotting time
Bleeding time
Prothrombin time
Fibrinogen
Bleeding time
Heparin's mode of action is
Not easily controlled
Inhibits factor VII synthesis
*Inhibits synthesis of thrombin
Activates plasmin
Blocks fibrinogen
In von Willebrand's disease, which is true :
*Normal prothrombin time
Decreased platelet
Normal partial thromboplastin time
D.
E.
1089.
A.
B.
C.
D.
E.
1090.
A.
B.
C.
D.
E.
1091.
A.
B.
C.
D.
E.
1092.
A.
B.
C.
D.
E.
1093.
A.
B.
C.
D.
E.
1094.
A.
B.
C.
D.
E.
1095.
A.
B.
C.
D.
E.
1096.
A.
B.
C.
D.
E.
1097.
A.
B.
Decreased platelets
None of above
Haemophilia A is characterised by the presence of following features, except
Bleeding into soft tissues, muscles & joints
Loss factor VIII levels
*Prolonged prothrombin time
Prolonged partial thromboplastin time
All is true
Factor deficiency in Christmas disease is
*IX
II
VII
VIII
XI
Palpable purpura may be seen in the following except
Acute meningococcemia
Vasculitis
Essential mixed cryoglobulinemia
*Scurvy
Henoch–Schönlein purpura
In blood transfusion, all may be seen except:
Tetany
Thrombocytopenia
*Hypokalemia
Haemosiderosis
Dizziness
All is are true of idiopathic thrombocytopenic purpura except:
Treatment with steroids
Autoimmune nature
*Prolonged clotting time
Splenectomy indicated
Petechia
True about idiopathic thrombocytopenic purpura is:
*Increase megakaryocytes in bone marrow
Frequent joint haemorrhage
Most common in children
Males commonly affected
VIII factor deficiency
True about acute form of idiopathic thrombocytopenic purpura is:
It is diagnosed by poor clot retraction
Splenomegaly is present in a majority
*Remits of its own without any treatment in 80% cases
Decrease level of megakaryocytes in bone marrow
Frequent joint haemorrhage
All are features of idiopathic thrombocytopenic purpura except:
*Increased clotting time
Magakaryocytic thrombocytopenia
Good response to steroids
Bleeding into skin and mucosa
Petechia
Treatment of neonatal idiopathic thrombocytopenia is
Pulse methylprednisolone
Dexamethasone
C.
*Immunoglobulins
D.
Platelets transfusion
E.
Splenectomy
1098. What is true of haemophilia:
A.
*Increased PTT
B.
Platelets level decreased
C.
Increased factor VIII
D.
Increased factor IX
E.
BT decreased
1099. Small capillaries bleeds in:
A.
*Thrombocytopenia
B.
Coagulation failure deficiency
C.
Anemia
D.
Agranulocytosis
E.
Hemophilia
1100. Following are seen in haemolytic uraemic syndrome except:
A.
Uraemia
B.
Thrombocytopenia
C.
*Positive Coomb's test
D.
Hypofibringonenaemia
E.
Renal failure
1101. DIC in pregnancy is seen in the following condition except :
A.
Amniotic fluid embolism
B.
Abruptio placentae
C.
*Fat embolism
D.
Intrauterine death
E.
Sepsis
1102. Disseminated intravascular coagulation can occur in all of the following excep :
A.
Snake bite
B.
Placenta praevia
C.
*Haemophilia
D.
Falciparum malaria
E.
Intrauterine death
1103. True about DIC are all except:
A.
Increase in fibrin degradation products
B.
Increase in PT and PIT
C.
*Thrombocytosis
D.
Hypofibrinogenaemia
E.
Anemia
1104. Thrombotic thrombocytopenic purpura is characterised by the presence of the following features,
except:
A.
Normocytic normochromic blood picture
B.
Renal failure
C.
*Coomb's positive haemolytic anemia
D.
Neurological abnormalities
E.
Trombocytopenia
1105. Which is not seen in congenital spherocytosis:
A.
*Positive direct Coomb's test
B.
Improvement on splenectomy
C.
Increased osmotic fragility
D.
Leg ulcers
E.
Pigmented gallstones
1106. Bleeding time is prolonged in:
A.
*Von Willebrand's disease
B.
Christmas disease
C.
Haemophilia A
D.
Polycythemia
E.
Haemophilia B
1107. In DIC which is true:
A.
Increased clotting time
B.
Increase in level of fibrin degradation products
C.
*All are true
D.
Bleeding from multiple mucosal sites
E.
None is true
1108. Consider the following statements : Disseminated intravascular coagulopathy results in bleeding and
is treated by:
A.
Replacement of coagulation factors
B.
Platelet concentrates
C.
Heparin
D.
*Replacement of coagulation factors, Platelet concentrates and Heparin are correct
E.
Replacement of coagulation factors and Platelet concentrates are correct
1109. Thrombocytopenia occurs in all except:
A.
Thrombotic thrombocytopenic purpura
B.
DIC
C.
*Henoch Schonlein purpura
D.
Wiskott Aldrich syndrome
E.
Hemolytic uremic syndrom
1110. Platelet count is decreased in all of the following condition except:
A.
Idiopathic thrombocytopenic purpura
B.
Thrombotic thrombocytopenic purpura
C.
Systemic lupus erythematosus
D.
*Henoch Schonlein purpura
E.
Hemolytic uremic syndrom
1111. Autoimmune destruction of platelet is seen in:
A.
Rheumatoid arthritis
B.
Reiter disease
C.
Still’s disease
D.
*Systemic lupus erythematosus
E.
Rheumatoid arthritis, Still’s disease
1112. Mechanism of Idiopathic thrombocytopenic purpura is:
A.
Vasculitis
B.
Antibodies to vascular epithelium
C.
*Antibodies to platelets
D.
Antibody to clotting factors
E.
Fungal infektious
1113. The following laboratory determinants is abnormally prolonged in idiopathic thrombocytopenic
purpura:
A.
Prothrombin time
B.
Clotting time
C.
No changed
D.
*Bleeding time
E.
Prothrombin time and Clotting time
1114. Platelet transfusion is not indicated in:
A.
DIC
B.
*SLE
C.
Dilutional Thrombocytopenia
D.
E.
1115.
A.
B.
C.
D.
E.
1116.
A.
B.
C.
D.
E.
1117.
A.
B.
C.
D.
E.
1118.
A.
B.
C.
D.
E.
1119.
A.
B.
C.
D.
E.
1120.
A.
B.
C.
D.
E.
1121.
A.
B.
C.
D.
E.
1122.
A.
B.
C.
D.
E.
1123.
A.
B.
Aplastic Anemia
Active bleeding and platelet count less than 50,000/uL
All of the following feature may be seen in thrombotic thrombocytopenic purpura, except:
Fever
Haemolysis
Low platelets count
*Hyperglusemia
Fever and thrombocytopenia
False statement regarding disseminated intravascular coagulopathy is:
*Decreased PTT
Thrombocytopenia
Decreased fibrinogen
Increased PT
Prolonged BT
The commonest mode of inheritance of Von Willebrand' s disease:
Codominant
Autosomal recessive
X-linked recessive
*Autosomal dominant X-linked recessive
Y-linked recessive
True about Haemophilia A are all except:
PTT increased
Clotting time is increased.
*PT increased
Serum levels of factor VIII are decreased.
Hematoma
All are true about hemophilia, except
*Increased Bleeding time
Decreased factor VIII
Decreased factor IX
Increased partial thromboplastin time
Decreased factor XI
All of the following conditions predispose to thrombosis except:
Paroxysmal nocturnal hemoglobinuria
Horhocystinurea
*Hypomagnesemia
Behcets syndrome
Cilliary arythmia
Palpable purpura could occur in the following conditions, except:
Acute meningococcemia
Small-vessel vasculitis.
Disseminated gonococcal infection.
*Thrombocytopenia
Essential mixed cryoglobulinemia
Melphalan is used in:
Wilm's tumor
Retinoblastoma
*Multiple myeloma
Nephroblastoma
ALL
Bence Jones proteins are excreted in the urine in
Chronic lymphocytic leukemia
Waldenstroms macroglobulinemia
C.
D.
E.
1124.
A.
B.
C.
D.
E.
1125.
A.
B.
C.
D.
E.
1126.
A.
B.
C.
D.
E.
1127.
A.
B.
C.
D.
E.
1128.
A.
B.
C.
D.
E.
1129.
A.
B.
C.
D.
E.
1130.
A.
B.
C.
D.
E.
1131.
A.
B.
C.
D.
E.
1132.
A.
Rheumatic fever
*Multiple myeloma
AML
The main diagnostic criteria of multiple myeloma include
Lytic bone marrow lesions
Decreased B2 microglobulin
Bence Jones proteinuria
*Plasmacytosis > 30%
Rouleaux formation on peripheral smear
Which is not a feature of multiple myeloma
*Cutaneous nodules
Bony lesions
Renal failure
'M' Spike
Hypercalciemia
Disseminated intravascular coagulation is associated with :
Myelomonocytic leukemia
Chronic myeloid leukemia
Chronic lymphocytic leukemia
*Acute promyelocytic leukemia
None of above
Acute leukaemic phase is not seen in :
*CLL
AML
Blast crisis CML
Hodgkin's lymphoma
AML, Blast crisis CML
The translocation in Burkitt's lymphoma is between chromosomes:
9 and 22
11 and 13
*8 and 14
8 and 12
11 and 22
Following are seen in multiple myeloma except :
Osteolysis
M-Spike
*Retroperitoneal nodes
Plasmacytosis
Blasts
All of the following are seen in multiple myeloma except :
Renal failure
M-spike
*Gum hypertrophy
Good response to melphalan
Bony lesions
In multiple myeloma best indicator of prognosis is
Serum level of Ca"
*Serum beta-2 microglobulins
Number of plasma cells in marrow
Serum alkaline phosphatase
Creatinin level
What is seen in multiple myeloma?
Increased Alkaline phosphatase
B.
Decreased IgA
C.
*Hypercalcemia
D.
Hypouricemia
E.
Fat bone marrow
1133. The most common cause of transfusion hepatitis is:
A.
*Hepatitis C
B.
Hepatitis A
C.
Hepatitis E
D.
Hepatitis D
E.
Hepatitis B
1134. All the following are true about multiple myeloma except :
A.
Hypercalcemia
B.
Hyperphosphatemia
C.
*Hyperglucemia
D.
Hypergammaglobulinemia
E.
Hyperproteinemia
1135. Most common type of Hodgkin's lymphoma is:
A.
Lymphocyte predominance
B.
Mixed cellularity
C.
*Nodular sclerosis
D.
None of above
E.
Lymphocyte depletion
1136. Which of the followings combinations of cytogenetic abnormality and associated
leukemia/lymphoma is incorrect?
A.
*t (9:20): ALL
B.
t (8:14): Burkitts lymphoma
C.
t (15:17): AML-M3
D.
t (9:22): CML
E.
t (15:17): AML-M3 and t (9:22): CML
1137. 'Hairy cell leukemia' is a Neoplastic proliferation of:
A.
T. cells
B.
Myeloid cells
C.
*B. cells
D.
Macrophages
E.
Lymphocytes
1138. Which of the following is not used in treatment of hairy cell leukemia:
A.
Steroid
B.
Pentostatin
C.
Splenectomy
D.
Alpha-interferon
E.
*Alendronic acid
1139. Which of these is true regarding CML?
A.
*Size of splenomegaly indicates prognosis
B.
Phagocytic activity of WBC is reduced
C.
Sudan black stain is specific for myeloblast
D.
Myeloblast, granuloblast and lymphoblast become PH chromosome +ve. –ve following remission
E.
Leukocytosis
1140. Which one of the following is not a criterion for making a diagnosis of chronic myeloid leukemia in
accelerated phaseA.
Blasts 10-19% of WBC's in peripheral blood
B.
Basophils 10-19% Basophils 10-19% of WBC'S in peripheral blood
C.
*Persistent thrombocytosis (>1000 x 109 /L) unresponsive to therapy
D.
Increasing spleen size unresponsive to therapy
E.
1141.
A.
B.
C.
D.
E.
1142.
A.
B.
C.
D.
E.
1143.
A.
B.
C.
D.
E.
1144.
A.
B.
C.
D.
E.
1145.
A.
B.
C.
D.
E.
1146.
A.
B.
C.
D.
E.
1147.
A.
B.
C.
D.
E.
1148.
A.
B.
C.
D.
E.
1149.
A.
B.
C.
Blasts 10-19% and of WBC's
Haemoglobin F is raised in:
*Juvenile chronic myelomonocytic leukemia
Hereditary spherocytosis
Congenital red cell aplasia
Myasthenia gravis
Erythremia vera
Leukocyte alkaline phosphate is increased in all, except
Polycythemia vera
Myelofibrosis
*CML
Leukemoid reaction
Essential thrombocytosis
All the following are seen in Multiple myeloma except:
Visual Disturbance
Bleeding tendency
*Dystrophic calcification
Proteinuria
Bone lesion
All the following are true about multiple myeloma except:
*t(8-14) translocation
Osteolytic bone disease
Light chain proliferation
Bence-Jones proteins in urine
Hypercalcemia
True regarding multiple myeloma is all. except:
*Increased alkaline phosphatase
Bone pain
Lytic lesions
Tarterate resistance acid phosphatase positive
Hypercalcemi
Which is not seen in multiple myeloma :
Anemia
Hypercalcemia
*Increased alkaline phosphatase
Raised ESR
Hyperproteinemia
Significant elevation ESR is seen in:
Polycythemia vera
CHF
*Multiple myeloma
Sickle cell anemia
None of above
Commonest site of lytic lesion, in multiple myeloma is:
*Spine
Scapula
Clavicle
Radial bone
Ulnar bone
All seen in multiple myeloma, except:
Lytic bone lesion
Hypercalcemia
Paraproteins
D.
*Plasmacytosis of bone marrow < 2%
E.
Hyperproteinemia
1150. A patient of multiple myeloma presents with bony lesions. What is the best marker for prognosis of
the disease:
A.
Serum calcium level
B.
Beta 2 microglobulin
C.
Beta 1 microglobulin
D.
*Bone marrow plasma cell
E.
Bens Jons protein
1151. Which of the following is not commonly seen in Polycythemia Vera?
A.
Thrombosis
B.
Hyperuricemia
C.
*Microcytosis
D.
Prone for acute leukemia
E.
Increased of ESR
1152. In Polycythemia vera, all the following are seen except:
A.
*Thrombocytopenia
B.
Increased GI bleed
C.
Thrombosis
D.
Transient visual loss
E.
Thrombocytosis
1153. All of the following are seen in polycythemia vera except:
A.
Increased Vit B12 binding capacity (>9000 micromols/dL)
B.
Leukocytosis
C.
*Thrombocytopenia
D.
Increased platelets
E.
Thrombosis
1154. Which is not seen in polycythemia vera :
A.
Increased Vit B12 binding capacity
B.
Increase RBC count
C.
*Increase erythropoietin level
D.
Ocular congestion
E.
Thrombosis
1155. What are the main laboratory findings in patient with chronic lymphatic leukemia?
A.
*Anemia and thrombocytopenia
B.
Thrombocytosis
C.
Thrombocytosis and lymphocytosis
D.
None of above
E.
Philadelphia chromosome in abnormal cells
1156. All of the following are poor prognostic factors for acute myeloid leukemias, except:
A.
*Presence of t(8:21)
B.
Age more than 60 years
C.
Leucocytes count more than l00 x 109/L
D.
Secondary leukemias
E.
None of above
1157. Which one of the following is not a criterion for making a diagnosis of chronic myeloid leukemia in
accelerated phaseA.
Basophils 10-19% of WBC'S in peripheral blood
B.
Increasing spleen size unresponsive to therapy
C.
*Blasts 30 % of WBC's in peripheral blood
D.
Persistent thrombocytosis (>1000 x 109 /L)
E.
None of above
1158. Thrombocytopenia usually not seen in:
A.
DIC
B.
Leukemia
C.
*Henoch–Schönlein purpura
D.
Metastasis
E.
None of above
1159. Thrombocytopenia occurs in all except:
A.
*Henoch–Schönlein purpura
B.
TTP
C.
DIC
D.
Wiskott Aldrich syndrome
E.
ITP
1160. Platelet count may be decreased in all of the following condition except:
A.
Idiopathic thrombocytopenic purpura
B.
Thrombotic thrombocytopenic purpura
C.
Systemic lupus erythematosus
D.
DIC
E.
*Henoch–Schönlein purpura
1161. Autoimmune destruction of platelet is seen in :
A.
Rheumatoid arthritis
B.
Reiter disease
C.
*SLE
D.
Polyarteritis nodosa
E.
ALL of above
1162. Cause of ITP is :
A.
Vasculitis
B.
Antibody to vascular epithelium
C.
*Antibody to platelets
D.
Antibody to clotting factors
E.
Complement defect
1163. The following laboratory indexes is abnormally changed in ITP :
A.
PTT
B.
Prothrombin time
C.
*Bleeding time
D.
Clotting time
E.
WBC count
1164. Disseminated intravascular coagulation differs from thrombotic thrombocytopenic purpura. In this
reference the DIC is most likely characterized by the following :
A.
*Decreased coagulation factor levels
B.
Significant numbers of schistocytes
C.
A brisk reticulocytosis
D.
Significant thrombocytopenia
E.
None of above
1165. The Von Willebrand' s disease :
A.
*Autosomal dominant
B.
Codominant
C.
Autosomal recessive
D.
X-linked recessive
E.
None of above
1166. True about Von Willebrand's disease is all except:
A.
*Defect in VWF is detected
B.
Increased bleeding time
C.
Factor VIII c levels are decreased in circulation.
D.
APTT is increased
E.
None of above
1167. In a patient 15 years old of Hemophilia to be taken for dental extraction true is all, except
A.
*Extraction should be done under general anaesthesia and skilled anaesthetic care
B.
All patients should be screened for HIV
C.
Factor VIII may be needed
D.
Dose of Lignocaine required for anaesthesia is same as that for normal individuals
E.
Cryoprecipitate may be needed
1168. Palpable purpura can occur in
A.
Idiopathic thrombocytopenic purpura
B.
Scurvy
C.
*Henoch–Schönlein purpura
D.
Monoclonal cryoglobulinemia
E.
DIC
1169. Bone marrow transplantion is the treatment of choice in all, except:
A.
Aplastic anemia
B.
AML in 1st remission
C.
*ITP
D.
Relapsed acute myelogenous leukemia
E.
Multiple myeloma
1170. Criteria of diagnosis of hemophilia A include :
A.
*Partial thromboplastin time increased
B.
Increased clotting time
C.
Increased prothrombin time
D.
Bleeding time increased
E.
Platelets count decreased
1171. Christmas disease is treated by the following drugs
A.
F.F. blood
B.
Cryoprecipitate
C.
*F.F. plasma
D.
Steroids
E.
Chemotherapy
1172. Fragmented RBC's can be in
A.
ITP
B.
Spherocytosis
C.
*DIC
D.
Patient with artificial cardiac valves
E.
Aplastic anemia
1173. Patient 45 years old, сomplaints of weakness, dizziness, dyspnea, pearching sensations in his tongue.
At inspection signs of vitamin B12 deficiency anaemia are revealed. Which changes of a tongue are typical
for this disorder?
A.
*Smooth tongue with loss of papillae
B.
Normal
C.
Coated
D.
Clean
E.
Swollen
1174. A blood test for determination of serum iron is indicated for the patient with iron deficiency anaemia.
Indicate normal concentration of serum iron in norm in Men
A.
35 to 56 μg/dL
B.
15 to 47 μg/dL
C.
165 to 276 μg/dL
D.
*65 to 176 μg/dL
E.
95 to 376 μg/dL
1175. Pernicious anaemia can be caused by following except:
A.
B.
C.
D.
E.
1176.
A.
B.
C.
D.
E.
1177.
A.
B.
C.
D.
E.
1178.
A.
B.
C.
D.
E.
1179.
A.
B.
C.
D.
E.
1180.
A.
B.
C.
D.
E.
1181.
A.
B.
C.
D.
E.
1182.
A.
B.
C.
D.
E.
1183.
A.
B.
C.
D.
E.
Bacterial overgrowth in intestines
Vegan diet
Colchicines
*Blood loss
Celiac sprue
Peculiarity of a tongue in pernicious anaemia is the following:
Like a “geographical map”
Covered with white coating
*Tongue appears smooth, shiny, bright red and clean
Swollen
Clean
Which from the following symptoms is most characteristic for anaemia?
Petechia
Cyanosys
*Pallor of skin and mucous membranes
Edema
Fever
Haemorrhagic syndrome can occur in all following diseases except of:
*Decreased amount of erythrocytes and haemoglobin
Decreased amount of thrombocytes
Functional incompetence of thrombocytes
A deficit of factors of blood coagulation in blood serum
Vascular wall damage caused by immune reactions
Complete blood test in patients with vitamin B12 deficiency anaemia shows all changes except :
Decreased amount of erythrocytes and haemoglobin
Macrocytosis
Increased colour index more than 1,1
*Decreased colour index below 0.8
Cabot rings
Which concentration of serum iron can be in chronic iron deficiency anaemia?
*6-12 mkmol/l
20-30 mkmol/l
12-20 mkmol/l
2-6 mkmol/l
None of above
Symptoms of iron deficiency are the following:
*All mentioned symptoms
Hair early falles down
Brittleness of nails
Deranged taste
Koilonychia
Which among the following diseases may be causative agents of B12 deficiency anaemia?
Chronic gastritis of type B
Peptic gastric ulcer
*Chronic gastritis of type A
Chronic pancreatitis
Chronic cholecystitis
All of the following cause Microcytic Hypochromic anemia except:
Thalassemia
Lactation
Pregnancy
Hemoglobinuria
*Acute blood loss
1184. All of the following cause Microcytic Hypochromic anemia except:
A.
*Hemolytic anemia
B.
Gastrointestinal bleeding
C.
Lactation
D.
Pregnancy
E.
Hemoglobinuria
1185. All of the following cause microcytic hypochromic anaemia, except:
A.
*Vitamin B12 deficiency
B.
Thalassemia
C.
Iron deficiency
D.
Gastrointestinal bleeding
E.
Lactation
1186. Elevated serum ferritin, serum iron and percent transferrin saturation are most consistent with the
diagnosis of:
A.
Iron deficiency anemia
B.
Anemia of chronic disease
C.
*Hemochromatosis
D.
Lead poisoning
E.
Thalassemia
1187. Elevated serum ferritin, serum iron and percent transferrin saturation are most consistent with the
diagnosis of:
A.
Gastrointestinal bleeding
B.
Anemia of chronic disease
C.
*Hemochromatosis
D.
Hemolytic anemia
E.
Thalassemia
1188. Which is not seen in Iron deficiency anaemia:
A.
*Hyper-segmented neutrophils
B.
Microcytosis preceeds hypochromia
C.
MCHC<50%
D.
Commonest cause of anaemia
E.
Low level of serum ferritin
1189. Which is seen in vitamin B12 deficiency aanaemia:
A.
*Hyper-segmented neutrophils
B.
Microcytosis preceeds hypochromia
C.
MCHC<50%
D.
Commonest cause of anaemia
E.
Low level of serum ferritin
1190. Most sensitive and specific test for diagnosis of iron deficiency is:
A.
Serum iron levels
B.
*Serum ferritin levels
C.
Serum transferrin receptor population
D.
Transferrin saturation
E.
MCHC<50%
1191. The sign of iron deficiency anaemia are all of the following except:
A.
Increase in iron binding capacity
B.
Decrease in serum ferritin level
C.
Decrease in serum iron level
D.
Decreased MCV
E.
*Hyper-segmented neutrophils
1192. The earliest sign of iron deficiency anaemia:
A.
Increase in iron binding capacity
B.
*Decrease in serum ferritin level
C.
Decrease in serum iron level
D.
Decreased MCV
E.
Transferrin saturation
1193. Iron overload occurs in all, except:
A.
Thalassemia and multiple frequent blood transfusions
B.
Myelodysplastic syndrome
C.
*Polycythemia vera
D.
Sideroblastic anaemia
E.
Severe chronic haemolysis
1194. In Anaemia of Chronic Diseases there are all except:
A.
Decreased serum Fe
B.
*Decreased Ferritin
C.
Decreased Total Fe Binding Capacity
D.
Increased Bone Marrow Fe
E.
Increased MCV
1195. In anaemia of chronic disease there are all except:
A.
Decreased TIBC
B.
Increased macrophage iron in marrow
C.
*Decreased serum ferritin level
D.
Decreased serum iron level
E.
Increased Bone Marrow Fe
1196. Anemia of chronic disease is characterized by all, except
A.
Decreased serum iron
B.
*Increased total iron binding capacity (TIBC)
C.
Increased serum ferritin
D.
Increased macrophage iron in bone marrow
E.
Normal serum Fe
1197. Not present in Sideroblastic anaemia is:
A.
Dimorphic population of RBC
B.
*Decreased transferrin saturation
C.
Sideroblast cells in blood smear film
D.
Ineffective erythropoiesis
E.
Marked erythroid hyperplasia in bone marrow
1198. What genetic change is characteristic of chronic myelogenous leukemia?
A.
*Philadelphia chromosome in abnormal cells
B.
No sings
C.
3 chromosomes in the 21-st pare
D.
Lack of chromosome 13 pare
E.
Translocations of bcl-2 from chromosome 8 to chromosome 14
1199. What are the laboratory findings in patient with chronic lymphocytic leukemia?
A.
Thrombocytosis and anemia
B.
*Anemia and thrombocytopenia
C.
Thrombocytosis and lymphocytosis
D.
Erythremia and leukocytosis
E.
Thrombocytopenia and leukopenia
1200. What are laboratory findings in patient with chronic lymphocytic leukemia?
A.
Thrombocytosis
B.
*Anemia and thrombocytopenia
C.
Thrombocytosis and lymphocytosis
D.
Erythremia and leukocytosis
E.
Philadelphia chromosome in abnormal cells
1201. Which of the following medications is mostly used in the treatment of chronic lymphocytic
leukemia?
A.
B.
C.
D.
E.
1202.
A.
B.
C.
D.
E.
1203.
A.
B.
C.
D.
E.
1204.
A.
B.
C.
D.
E.
1205.
A.
B.
C.
D.
E.
1206.
A.
B.
C.
D.
E.
1207.
A.
B.
C.
D.
E.
1208.
A.
B.
C.
D.
E.
1209.
A.
B.
C.
D.
E.
*Chlorambucil
Prednisone
Cyclophosphamide
Myleran
Penicillin
Description of the Acute Lymphatic Leukemia
*Disorder of the blood-forming tissue (white cells)
Disorder of the blood-forming tissue (red cells)
Cardiac pain
Increasing of blood pressure
Decreasing of blood pressure
All of the following statements are true about sickle cell disease except
Vaso-occlusive crisis
Aplastic Crisis
*Hypertensive crisis
Sequestration Crisis
Hyper hemolytic crisis
Sickle cell trait patient do not have manifestations as that of Sickle cell disease, because:
*50% HbS is required for occurrence of sickling
HbA prevents sickling
HbS is less than 50% & HbA has low affinity for HbS
HbA prevents polymerization ofHbs
Acute infection is the most common cause of mortality before 3 years of age.
Which is not seen in a chronic case of Sickle cell anemia?
Hepatomegaly
Pulmonary hypertension
Cardiomegaly
Splenomegaly
*Peripheral neuropathy
All are true for sickle cell anemia, except
Pulmonary arterial hypertension
Fish vertebra
*Leukopenia
Increased size of heart
Patient may require frequent blood transfusions
Commonest acute presentation of sickle cell anaemia is:
Priapism
*Bone pain
Fever
Splenomegaly
Pulmonary hypertension
All of the following are the signs of anemia except
Malaise
Dyspnea on exertion
Increasing cardiac output
Palpitations
*Dyspepsia
All of the following are the common signs of anemia except
Malaise
Pallor
Increasing cardiac output
Palpitations
*Dysphagia
1210.
A.
B.
C.
D.
E.
1211.
A.
B.
C.
D.
E.
1212.
A.
B.
C.
D.
E.
1213.
A.
B.
C.
D.
E.
1214.
A.
B.
C.
D.
E.
1215.
A.
B.
C.
D.
E.
1216.
A.
B.
C.
D.
E.
1217.
A.
B.
C.
D.
E.
1218.
A.
B.
C.
D.
For diagnosis of anemia clinicians request all of the following except
Complete blood counts
Red blood cells
*Leukocyturea
Hemoglobin level
MCV
When the cause of anemia is not obvious, clinicians use all of the following tests except
ESR
Ferritin
Serum iron
*CRP
Transferrin
When the cause of anemia is not obvious, clinicians use all of the following tests except
Ferritin
Serum iron
*CRP
Transferrin
RBC folate level
When the cause of anemia is not obvious, clinicians use all of the following tests except
Ferritin
Serum iron
*Fibrinogen
Serum vitamin B12
RBC folate level
When the cause of anemia is not obvious, clinicians use all of the following tests except
Ferritin
Serum iron
*Fibrinogen
Serum vitamin B12
Bone marrow examination
When the cause of anemia is not obvious, clinicians use all of the following tests except
Ferritin
Serum iron
*Fibrinogen
Bone marrow examination
Serum creatinine
When the cause of anemia is not obvious, clinicians use all of the following tests except
Ferritin
Bilirubin
*Proteinemia
Serum vitamin B12
Serum creatinine
When the cause of anemia is not obvious, clinicians use all of the following tests except
Ferritin
Bilirubin
*Cholesterol level
Bone marrow examination
Serum creatinine
Microcytic anemia are all of the following except
Iron deficiency anemia
Anemia of chronic disease
Alpha- thalassemia
*Aplastic anemia
E.
Beta-thalassemia
1219. Microcytic anemia are all of the following except
A.
Iron deficiency anemia
B.
Anemia of chronic disease
C.
Alpha- thalassemia
D.
*Pernicious anemia
E.
Beta-thalassemia
1220. Macrocytic anemia are all of the following except
A.
*Anemia of chronic disease
B.
Vitamin B12 deficiency
C.
Folic acid deficiency
D.
Methotrexate-induced anemia
E.
Pernicious anemia
1221. Normocytic anemia are all of the following except
A.
Acute blood loss
B.
Acquired hemolytic anemia
C.
Aplastic anemia (bone marrow failure)
D.
Congenital hemolytic anemia
E.
*Vitamin B12 deficiency
1222. Lymph node biopsy shows "owl-eyed" appearance of abundant lymphocytes. What is the name of the
finding the pathologist is describing?
A.
*Reed-Sternberg cells.
B.
Auer cells.
C.
Smudge cells.
D.
Lymphocytic clue cells.
E.
Jon-Fredrickson phenomenon.
1223. Lymph node biopsy shows Reed-Sternberg cells. What is the name of disease?
A.
Acute leukaemia
B.
Chronic leukaemia
C.
*Lymphoma
D.
Multiple myeloma
E.
Myelofibrosis
1224. The average blood volume for an adult is about
A.
45 liters
B.
450 ml
C.
5 gallons
D.
*5 liters
E.
3 liters
1225. Low blood osmolarity
A.
causes the blood to absorb excess tissue fluid as it passes through the capillaries
B.
could lead to increased blood volume and increased blood pressure
C.
*allows too much fluid to remain in the tissues and leads to edema
D.
could be caused by increased plasma protein levels
E.
could be caused by increased plasma sodium levels
1226. Each body function is correctly matched with the corresponding function of the blood on the right
EXCEPT
A.
respiration - transports oxygen and carbon dioxide
B.
*immune defense - platelet factors initiate clotting
C.
acid-base balance - buffers acids and bases
D.
thermoregulation - allows heat to escape from the body at the skin
E.
Blood viscosity is due to the presence of the plasma proteins and erythrocytes.
1227. Which statement concerning blood viscosity is correct?
A.
The viscosity of blood is 4.5 to 5.5 higher than the viscosity of water.
B.
*Blood viscosity is due to the presence of the plasma proteins and erythrocytes.
C.
The higher the blood viscosity the harder the heart has to work to move blood through the vessels
D.
Anemia increases blood viscosity.
E.
Hypocoaculation increases blood viscosity.
1228. Other than water, the most common component of plasma is
A.
Chloride
B.
Urea
C.
*Protein
D.
Sodium
E.
Calcium
1229. Which problem could NOT be caused by a deficiency of plasma proteins?
A.
Edema
B.
*reduced ability to transport oxygen
C.
reduced ability to clot
D.
reduced ability to transport molecules such as lipids
E.
Edema and reduced ability to clot
1230. The antibodies that circulate in the plasma are made by
A.
the liver, which makes all of the plasma proteins
B.
the kidney, which monitors plasma composition
C.
*B lymphocytes, which are part of the immune system
D.
the spleen, which uses them to tag red blood cells for destruction
E.
the heart, which monitors blood composition
1231. If you have some blood from which the formed elements have been removed, how can you tell if it is
plasma or serum?
A.
plasma contains hemoglobin; serum does not
B.
*serum is yellow; plasma has no color
C.
serum contains antibodies; plasma does not
D.
plasma contains clotting proteins; serum does not
E.
serum contains complement; plasma does not
1232. Descriptions of leukaemia are following except
A.
Myelocytic leukemia involves myeloblasts.
B.
Lymphocytic leukemia involves lymphocytes.
C.
Acute leukemia involves blast-type cells and primarily affects children.
D.
*Platelets stick together and form a plug that temporarily seals the break in the vessel wall
E.
Chronic leukemia is more prevalent in older people.
1233. In which choice are the major groups of plasma proteins listed in order from highest to lowest
percent?
A.
fibrinogen, globulins, albumins
B.
*globulins, fibrinogen, albumins
C.
albumins, globulins, fibrinogen
D.
albumins, fibrinogen, globulins
E.
globulins, albumins, fibrinogen
1234. Decrease in numbers of red blood cells or hemoglobin within red blood cells:
A.
*Anemia
B.
Erythrocytosis
C.
Thrombocytosis
D.
Leukemia
E.
Leukocytosis
1235. Increase in numbers of malignant white blood cells:
A.
Leukocytosis
B.
Erythremia
C.
Thrombocytosis
D.
Erythrocytosis
E.
*Leukemia
1236. CBC revealed a WBC of 72,000 and a differential that was 83% lymphocytes. What is the MOST
likely diagnosis?
A.
Acute lymphocytic leukemia
B.
*Chronic lymphocytic leukemia
C.
Acute prolymphocytic leukemia
D.
Chronic prolymphocytic leukaemia
E.
Lymphoma
1237. Patient undergoes genetic testing, which reveals a t(9;22) translocation. Which of the following
would a complete blood count most likely show?
A.
Increased lymphocyte count
B.
Increased plasmacyte count
C.
Numerous lymphoblasts
D.
*Increased granulocytes
E.
Pancytopenia
1238. Thrombocytopenia that is caused by increased platelet destruction is most closely associated with
which of the following conditions?
A.
Aplastic anemia
B.
Combination chemotherapy
C.
Acute leukemia
D.
*Systemic lupus erythematosus (SLE)
E.
Excessive ethanol intake
1239. Increased levels of hemoglobin A2 (Hb 8) are characteristic of which of the following disorders?
A.
Sickle cell trait
B.
*beta -Thalassemia trait
C.
Glucose-6-phosphate dehydrogenase (G6PD) deficiency
D.
Unstable hemoglobin disease
E.
a-Thalassemia trait
1240. Which of the following conditions results in prolongation of the partial thromboplastin time (PTT),
but not the prothrombin time (PT)?
A.
Variceal hemorrhage as a result of cirrhosis
B.
*Menorrhagia resulting from von Willebrand's disease (vWD)
C.
Therapy with broad-spectrum antibiotics
D.
Therapy with coumarin for phlebitis
E.
(Celiac (sprue) disease
1241. Which of the following conditions causes delayed, deep tissue-type bleeding?
A.
Uremia
B.
*Hemophilia A
C.
Therapy with aspirin
D.
von Willebrand's disease (vWD)
E.
Idiopathic thrombocytopenic purpura (ITP)
1242. Which of the following statements regarding patients with idiopathic thrombocytopenic purpura
(ITP) is true?
A.
Bone marrow megakaryocytes are generally decreased
B.
Platelet-associated immunoglobulin G (IgG) is diagnostic
C.
Splenomegaly and other cytopenias are usually present
D.
The platelet life span is prolonged
E.
*Splenectomy can be effective therapy
1243. Disseminated intravascular coagulation (DIC) initiated by activation of intrinsic pathway is most
likely in which of the following clinical settings?
A.
*Gram-negative bacteremia
B.
Amniotic fluid embolism
C.
Multiple trauma
D.
Promyelocytic leukemia
E.
Trousseau's syndrome
1244. Which of the following maneuvers is least effective in acute myelogenous leukemia (AML), with the
intention of initiating induction phase therapy?
A.
Empiric, aggressive use of broad-spectrum antibiotics
B.
Maintenance of platelet counts in excess of 15,000-20,000/mm3
C.
Stimulation of marrow recovery by growth factors
D.
*Vaccination with pneumococcal vaccine and use of immunoglobulins
E.
Isolation techniques of varying intensity
1245. Which of the following congenital conditions is associated with hypercoagulability due to an
abnormal substrate molecule, rendering it insensitive to its agonist?
A.
Antithrombin III deficiency
B.
Protein С deficiency
C.
*Factor V (Leiden) deficiency
D.
Hemophilia A
E.
Trousseau's syndrome
1246. Acute lymphocytic leukemia (ALL)
A.
*A 4-year-old patient with pancytopenia and circulating blasts
B.
A 60-year-old patient with pancytopenia and circulating blasts
C.
A 20%-30% long-term survival
D.
A patient with bleeding and infection
E.
A patient with gum and skin infiltration
1247. In which of the following clinical situations associated with bleeding does laboratory testing reveal a
normal bleeding time?
A.
A 58-year-old man with renal failure who requires hemodialysis
B.
A 50-year-old woman with rheumatoid arthritis on chronic aspirin therapy
C.
A 30-year-old man with endocarditis who requires high doses of penicillin
D.
*A 5-year-old boy with hemophilia and active hemarthrosis
E.
An 18-year-old woman with von Willebrand's disease (vWD) and menorrhagia
1248. Acute myelogenous leukemia (AML)
A.
A 4-year-old patient with pancytopenia and circulating blasts
B.
*A 60-year-old patient with pancytopenia and circulating blasts
C.
A 70%-80% long-term survival
D.
A patient with bleeding and infection
E.
A patient with gum and skin infiltration
1249. Acute lymphocytic leukemia (ALL)
A.
A 4-year-old patient with pancytopenia and circulating blasts
B.
A 60-year-old patient with pancytopenia and circulating blasts
C.
*A 70%-80% long-term survival
D.
A patient with infection
E.
A patient with gum and skin infiltration
1250. Both ALL and AML
A.
A 4-year-old patient with pancytopenia and circulating blasts
B.
A 60-year-old patient with pancytopenia and circulating blasts
C.
A 70%-80% long-term survival
D.
*A patient with bleeding and infection
E.
A patient with gum and skin infiltration
1251. Acute myelogenous leukemia (AML)
A.
A 4-year-old patient with pancytopenia and circulating blasts
B.
A 60-year-old patient with pancytopenia and circulating blasts
C.
A 70%-80% long-term survival
D.
A patient with bleeding and infection
E.
*A patient with gum and skin infiltration
1252. A 67-year-old woman with mitral stenosis is started on warfarin by her cardiologist. On the third day,
painful red areas appear on her thigh and breast.
A.
*Protein С deficiency
B.
Paroxysmal nocturnal hemoglobinuria (PNH)
C.
antiphospholipid syndrome associated with systemic lupus erythematosus (SLE)
D.
Trousseau's syndrome
E.
Idiopathic thrombopenic purpura
1253. What is the initial therapy that is most appropriate for 37-year-old man with Philadelphia
chromosome-positive chronic myelogenous leukemia (CML)?
A.
Aggressive, marrow ablative chemotherapy
B.
Interferon (IFN)
C.
*Bone marrow transplantation (BMT)
D.
Supportive therapy such as transfusions о cellular components and erythropoietin
E.
Antibiotics
1254. What is the initial therapy that is most appropriate for 9-year-old girl with pancytopenia and marrow
findings consistent with standard risk acute lymphocytic leukemia (ALL)?
A.
*Aggressive, marrow ablative chemotherapy
B.
Interferon (IFN)
C.
Bone marrow transplantation (BMT)
D.
Supportive therapy such as transfusions о cellular components and erythropoietin
E.
Antibiotics
1255. What is the initial therapy that is most appropriate for 23-year-old man with severe aplastic anemia?
A.
Aggressive, marrow ablative chemotherapy
B.
Interferon (IFN)
C.
*Bone marrow transplantation (BMT)
D.
Supportive therapy such as transfusions о cellular components and erythropoietin
E.
Antibiotics
1256. All the following disorders impair the release of oxygen to body tissues EXCEPT
A.
Methemoglobinemia
B.
Carbon monoxide poisoning
C.
Hyperventilation
D.
Hypothermia
E.
*Acidosis
1257. Coumarin-induced skin necrosis is occasionally associated with the institution of oral anticoagulants
in patients with
A.
antithrombin III deficiency
B.
factor VIII deficiency
C.
plasminogen deficiency
D.
hemophilia B
E.
*protein C deficiency
1258. A 62-year-old chronic alcoholic and heavy smoker present with a 3-cm, firm, right midcervical neck
mass. An excisional biopsy reveals squamous cell carcinoma. Which of the following is the most appropriate
approach at this time?
A.
*Bronchoscopy, esophagoscopy, and laryngoscopy
B.
CT of the neck
C.
CT of the brain
D.
Neck dissection
E.
Radiation therapy
1259. In persons who have chronic myelogenous leukemia, the translocation that accounts for the
Philadelphia
A.
chromosome most commonly is found in
B.
all cells of the body
C.
*all three hematopoietic cell lines but not in nonhematopoietic cells
D.
all cells of the granulocytic cell line but not in nongranulocytic cells
E.
all bone marrow stem cells but not in mature cells
F.
all bone marrow stem cells and certain mature granulocytes
1260. All the following statements regarding toxic effects of chemotherapy are correct EXCEPT
A.
*of all the antineoplastic agents, anthracyclines suppress bone marrow stem cells to the greatest
degree
B.
vincristine is a relatively weak myelosuppressive agent and can be administered during periods of
low blood counts
C.
cisplatin-induced nausea and vomiting can usually be controlled by metoclopramide or
dexamethasone or both
D.
the use of melphalan (phenylalanine mustard) has been associated with secondary leukemia
E.
cisplatin can produce hypocalcemia by inducing renal electrolyte wasting
1261. All of the following findings would help to distinguish £-thalassemia trait from iron deficiency
EXCEPT
A.
*microcytic red blood cells
B.
presence of anemia
C.
elevated hemoglobin A2 level
D.
normal transferrin saturation
E.
normal serum ferritin concentration
1262. True statements regarding both hemophilia A (factor VIII deficiency) and hemophilia B (factor IX
deficiency) include all of the following EXCEPT
A.
the defective gene is located on the X chromosome
B.
*the affected factors require vitamin K for biologic activity
C.
the partial thromboplastin time is elevated, but the prothrombin time is normal
D.
joint bleeding is common
E.
the optimal therapy is replacement with recombinant factors
1263. Stable-phase chronic myelogenous leukemia (CML) is associated with all of the following EXCEPT
A.
splenomegaly
B.
basophilia
C.
low leukocyte alkaline phosphatase
D.
*diagnostic bone marrow findings
E.
favorable response to interferon-a
1264. All of the following represent clinicopathologic associations EXCEPT
A.
acute monoblastic leukemia and gum infiltration
B.
acute promyelocytic leukemia and bleeding
C.
acute lymphoblastic leukemia (T-cell type) and mediastinal mass
D.
L3 acute lymphoblastic leukemia and CNS involvement
E.
*acute megakaryoblastic leukemia and polyneuropathy
1265. Which of the following statements concerning the diagnosis of pernicious anemia is true?
A.
The presence of antiparietal-cell antibodies is diagnostic of pernicious anemia
B.
Hematologic response to folate therapy alone rules out pernicious anemia as the cause of
megaloblastic anemia
C.
Hyperkalemia may be a consequence of vitamin B12 therapy
D.
Bone marrow examination would be expected to reveal marked depletion of erythrocyte precursors in
persons with untreated pernicious anemia
E.
*Serum gastrin levels usually are elevated in persons with pernicious anemia
1266. A 45-year-old woman with long-standing rheumatoid arthritis is diagnosed as having "anemia of
chronic disease." The predominant mechanism causing this type of anemia in persons with chronic
inflammatory disorders is
A.
defective porphyrin synthesis
B.
impaired incorporation of iron into porphyrin
C.
intravascular hemolysis
D.
depressed erythroid maturation due to decreased erythropoietin production
E.
*impaired transfer of reticuloendothelial storage iron to marrow erythroid precursors
1267. Which of the following statements best characterizes the hemolysis associated with glucose-6phosphate dehydrogenase (G6PD) deficiency?
A.
It is more severe in affected blacks than in affected persons of Mediterranean ancestry.
B.
It is more severe in females than in males.
C.
It causes the appearance of Heinz bodies on Wright staining of a peripheral smear.
D.
*It most often is precipitated by infection.
E.
The best time to perform the diagnostic test is during a hemolytic crisis.
1268. Evaluation of a person who has pure red blood cell aplasia would be expected to reveal
A.
markedly hypocellular bone marrow
B.
*normochromic, normocytic red blood cells
C.
increased iron turnover on ferrokinetic studies
D.
a reticulocyte count greater than 2.0 percent
E.
decreased urinary erythropoietin content
1269. A feature of idiopathic thrombocytopenic purpura common to both children and adults is
A.
occurrence after an antecedent viral illness
B.
presence of antibodies directed against target antigens on the glycoprotein Ilb-IIIa complex
C.
*absence of splenomegaly
D.
persistence of thrombocytopenia for more than 6 months
E.
necessity of splenectomy to ameliorate thrombocytopenia
1270. Which of the following is hypochromic, microcytic anemia?
A.
anemias of chronic disease
B.
hemolytic anemias
C.
anemia of acute hemorrhage
D.
aplastic anemias
E.
*iron deficiency anemia
1271. Which of the following isn’t normochromic, normocytic anemia?
A.
anemias of chronic disease
B.
hemolytic anemias
C.
anemia of acute hemorrhage
D.
aplastic anemias
E.
*iron deficiency anemia
1272. Which of the following is hyperchromic, macrocytic anemia?
A.
anemias of chronic disease
B.
hemolytic anemias
C.
anemia of acute hemorrhage
D.
aplastic anemias
E.
*vitamin B12 deficiency
1273. Which of the following is hyperchromic, macrocytic anemia?
A.
anemias of chronic disease
B.
hemolytic anemias
C.
anemia of acute hemorrhage
D.
aplastic anemias
E.
*folate deficiency
1274. Serum unconjugated bilirubin and urine urobilinogen concentration are elevated in
A.
anemias of chronic disease
B.
*hemolytic anemias
C.
anemia of acute hemorrhage
D.
aplastic anemias
E.
iron deficiency anemia
1275. Thrombocytopenia, neutropenia are seen in
A.
anemias of chronic disease
B.
hemolytic anemias
C.
D.
E.
1276.
A.
B.
C.
D.
E.
1277.
A.
B.
C.
D.
E.
1278.
A.
B.
C.
D.
E.
1279.
A.
B.
C.
D.
E.
1280.
A.
B.
C.
D.
E.
1281.
A.
B.
C.
D.
E.
1282.
A.
B.
C.
D.
E.
1283.
A.
B.
C.
D.
E.
1284.
A.
anemia of acute hemorrhage
*aplastic anemias
iron deficiency anemia
The hyporegenerative anemias with the reticulocyte decreased production index is in
anemias of chronic disease
hemolytic anemias
anemia of acute hemorrhage
*aplastic anemias
iron deficiency anemia
Which of the following anemias most commonly occurs in patients with chronic gastritis type A?
anemias of chronic disease
hemolytic anemias
anemia of acute hemorrhage
aplastic anemias
*vitamin B12 deficiency
Which of the following anemias most commonly occurs in patients with thalassemias?
anemias of chronic disease
*hemolytic anemias
anemia of acute hemorrhage
aplastic anemias
vitamin B12 deficiency
Which of the following anemias most commonly occurs in alcoholic patient?
anemias of chronic disease
hemolytic anemias
anemia of acute hemorrhage
aplastic anemias
*megaloblastic
Coombs' test positive in
anemias of chronic disease
*autoimmune hemolytic anemias
anemia of acute hemorrhage
aplastic anemias
iron deficiency anemia
An increased number of reticulocytes is seen in
anemias of chronic disease
*hemolytic anemias
folate deficiency
aplastic anemias
iron deficiency anemia
An increased number of reticulocytes is seen in
anemias of chronic disease
folate deficiency
*anemia of acute hemorrhage
aplastic anemias
iron deficiency anemia
Mean corpuscular volume (MCV) is < 80 fL in
anemias of chronic disease
folate deficiency
anemia of acute hemorrhage
aplastic anemias
*iron deficiency anemia
Mean corpuscular volume (MCV) is < 80 fL in
anemias of chronic disease
B.
C.
D.
1285.
1286.
A.
B.
C.
D.
E.
1287.
A.
B.
C.
D.
E.
1288.
A.
B.
C.
D.
E.
1289.
A.
B.
C.
D.
E.
1290.
A.
B.
C.
D.
E.
1291.
A.
B.
C.
D.
E.
1292.
A.
B.
C.
D.
E.
1293.
A.
B.
C.
D.
E.
1294.
folate deficiency
anemia of acute hemorrhage
aplastic anemias
thalassemia
Mean corpuscular volume (MCV) is > 80 fL in
anemias of chronic disease
*folate deficiency
anemia of acute hemorrhage
aplastic anemias
thalassemia
Mean corpuscular volume (MCV) is > 80 fL in
anemias of chronic disease
*vitamin B12 deficiency
anemia of acute hemorrhage
aplastic anemias
thalassemia
Range of Mean corpuscular volume (MCV) isn’t 80 -94 fL in
anemias of chronic disease
*folate deficiency
anemia of acute hemorrhage
aplastic anemias
anemia in CLL
Range of Mean corpuscular volume (MCV) isn’t 80 -94 fL in
anemias of chronic disease
*vitamin B12 deficiency
anemia of acute hemorrhage
aplastic anemias
anemia in CLL
Range of Mean corpuscular volume (MCV) isn’t 80 -94 fL in
anemias of chronic disease
*iron deficiency anemia
anemia of acute hemorrhage
aplastic anemias
anemia in CLL
Smudged cells are seen in peripheral blood smear in
*CLL
CML
ALL
AML
ITP
Splenomegaly is the most common physical finding in patients with
CLL
*CML
ALL
AML
ITP
Philadelphia (Ph1) chromosome is seen in
CLL
*CML
ALL
AML
ITP
Which of the following is hypochromic, microcytic anemia?
A.
B.
C.
D.
E.
1295.
A.
B.
C.
D.
E.
1296.
A.
B.
C.
D.
E.
1297.
A.
B.
C.
D.
E.
1298.
A.
B.
C.
D.
E.
1299.
A.
B.
C.
D.
E.
1300.
A.
B.
C.
D.
E.
1301.
A.
B.
C.
D.
E.
1302.
A.
B.
C.
D.
E.
vitamin B12 deficiency
hemolytic anemias
folate deficiency
aplastic anemias
*iron deficiency anemia
Which of the following isn’t normochromic, normocytic anemia?
anemias of chronic disease
hemolytic anemias
anemia of acute hemorrhage
aplastic anemias
*iron deficiency anemia
Which of the following is hyperchromic, macrocytic anemia?
anemias of chronic disease
hemolytic anemias
anemia of acute hemorrhage
aplastic anemias
*vitamin B12 deficiency
Which of the following is hyperchromic, macrocytic anemia?
anemias of chronic disease
hemolytic anemias
anemia of acute hemorrhage
aplastic anemias
*folate deficiency
Serum unconjugated bilirubin and urine urobilinogen concentration are elevated in
anemias of chronic disease
*Extravascular hemolytic anemias
anemia of acute hemorrhage
aplastic anemias
iron deficiency anemia
Thrombocytopenia, neutropenia may be seen in
anemias of chronic disease
hemolytic anemias
anemia of acute hemorrhage
*CLL
iron deficiency anemia
The hyporegenerative anemias with the reticulocyte decreased production index is in
anemias of chronic disease
hemolytic anemias
anemia of acute hemorrhage
*aplastic anemias
iron deficiency anemia
Which of the following anemias most commonly occurs in patients with chronic gastritis type A?
Intravascular hemolytic anemias
Extravascular hemolytic anemias
anemia of acute hemorrhage
aplastic anemias
*vitamin B12 deficiency
Which of the following anemias most commonly occurs in patients with thalassemias?
Anemias of chronic disease
*Microcytic hemolytic anemias
Anemia of acute hemorrhage
Aplastic anemias
Vitamin B12 deficiency
1303.
A.
B.
C.
D.
E.
1304.
A.
B.
C.
D.
E.
1305.
A.
B.
C.
D.
E.
1306.
A.
B.
C.
D.
E.
1307.
A.
B.
C.
D.
E.
1308.
A.
B.
C.
D.
E.
1309.
A.
B.
C.
D.
E.
1310.
A.
B.
C.
D.
E.
1311.
A.
B.
C.
D.
Which of the following anemias most commonly occurs in patient treated by anticonvulsants?
anemias of chronic disease
hemolytic anemias
anemia of acute hemorrhage
aplastic anemias
*macrocytic
Normocytic anemia and Coombs' test positive in
anemias of chronic disease
*autoimmune hemolytic anemias
anemia of acute hemorrhage
aplastic anemias
iron deficiency anemia
Normocytic anemia and increased number of reticulocytes is seen in
anemias of chronic disease
*hemolytic anemias
folate deficiency
aplastic anemias
iron deficiency anemia
Normocytic anemia and increased number of reticulocytes is seen in
anemias of chronic disease
folate deficiency
*anemia of acute hemorrhage
aplastic anemias
iron deficiency anemia
Mean corpuscular volume (MCV) is < 80 fL and hypochromia in
anemias of chronic disease
folate deficiency
anemia of acute hemorrhage
aplastic anemias
*iron deficiency anemia
Mean corpuscular volume (MCV) is < 80 fL and hypochromia in
anemias of chronic disease
folate deficiency
anemia of acute hemorrhage
aplastic anemias
*thalassemia
Mean corpuscular volume (MCV) is > 80 fL and hyperchromia in
anemias of chronic disease
*folate deficiency
anemia of acute hemorrhage
aplastic anemias
thalassemia
Mean corpuscular volume (MCV) is > 80 fL and hyperchromia in
anemias of chronic disease
*vitamin B12 deficiency
anemia of acute hemorrhage
aplastic anemias
thalassemia
Hyperchromic anemia with range of Mean corpuscular volume (MCV) isn’t 80 -94 fL in
anemias of chronic disease
*folate deficiency
anemia of acute hemorrhage
aplastic anemias
E.
anemia in CLL
1312. Hyperchromic anemia with range Range of Mean corpuscular volume (MCV) isn’t 80 -94 fL in
A.
anemias of chronic disease
B.
*vitamin B12 deficiency
C.
anemia of acute hemorrhage
D.
aplastic anemias
E.
anemia in CLL
1313. Hypochromic anemia with range Range of Mean corpuscular volume (MCV) isn’t 80 -94 fL in
A.
anemias of chronic disease
B.
*iron deficiency anemia
C.
anemia of acute hemorrhage
D.
aplastic anemias
E.
anemia in CLL
1314. Smudged cells are seen in peripheral blood smear in
A.
*CLL
B.
CML
C.
ALL
D.
AML
E.
ITP
1315. Splenomegaly is the most common physical finding in patients with
A.
CLL
B.
*CML
C.
ALL
D.
AML
E.
ITP
1316. Philadelphia (Ph1) chromosome is seen in
A.
CLL
B.
*CML
C.
ALL
D.
AML
E.
ITP
1317. What substances are reabsorbed in renal tubules completely?
A. *Aminoacids
B. Creatinine
C. Sodium
D. Potassium
E. Chlorum
1318. What are the common findings of IgA nephropathy?
A. Heavy proteinurea
B. *Asymptomatic microscopic hematuria/proteinuria
C. White cells;
D. Isolated proteinurea
E. pyuria;
1319. What are the common findings of IgA nephropathy?
A. Pyuria
B. Heavy proteinurea
C. White cells;
D. Isolated proteinurea
E. *Gross hematuria post exercise or viral illness
1320. Acute course of glomerulonenhritis may occur on morphological case of
A. IgA nephropathy
B. minimal change disease
C. *Proliferative glomerulonephritis
D. Membranous nephropathy
E. Mesangioproliferative glomerulonephritis
1321. Chronic course of glomerulonenhritis may occur on all morphological case except
A. IgA nephropathy
B. minimal change disease
C. *Proliferative glomerulonephritis
D. Membranous nephropathy
E. Mesangioproliferative glomerulonephritis
1322. In case of IgA nephropathy the most appropriate treatment is
A. Corticosteroids
B. Cytotoxics
C. Antibiotics
D. *Memranostabilisative therapy
E. Plasmapheresis
1323. In case of minimal change disease the most appropriate treatment is
A. *Corticosteroids
B. Cytotoxics
C. Antibiotics
D. Memranostabilisotion therapy
E. Plasmapheresis
1324. What are the common findings of IgA nephropathy?
A. Heavy proteinurea
B. *Asymptomatic microscopic hematuria/proteinuria
C. White cells;
D. Isolated proteinurea
E. pyuria;
1325. What are the common findings of IgA nephropathy?
A. Heavy proteinurea
B. White cells;
C. Isolated proteinurea
D. Pyuria
E. *Gross hematuria post exercise or viral illness
1326. In case of acute poststreptococcal glomerulonephritisthe most appropriate treatment is
A. Corticosteroids
B. Cytotoxics
C. *Antibiotics, symptomatic treatment
D. Memranostabilisation therapy
E. Plasmapheresis
1327. In case of Membranous nephropathy the most appropriate treatment is
A. Corticosteroids
B. *Cytotoxics
C. Antibiotics
D. Memranostabilisation therapy
E. Plasmapheresis
1328. In case of nephrotic syndrome in diabetic nephropathy with normal renal function the most
appropriate treatment is
A. Corticosteroids
B. Cytotoxics
C. *Symptomatic treatment
D. Memranostabilisation therapy
E. Plasmapheresis
1329. In case of nephrotic syndrome in diabetic nephropathy with ESRD the most appropriate treatment is
A. Corticosteroids
B. Cytotoxics
C. symptomatic treatment
D. Memranostabilisotion therapy
E. *Renal replacement therapy
1330. In case of resistant nephrotic syndrome in mesangiocapillary Glomerulonephritis the most
appropriate treatment is
A. Corticosteroids
B. Cytotoxics
C. symptomatic treatment
D. Memranostabilisotion therapy
E. *Plasmapheresis folowwed by corticosteroids
1331. Nephrotic syndrome may occur in morphological case except
A. *IgA nephropathy
B. minimal change disease
C. Mesangiocapillary glomerulonephritis
D. Membranous nephropathe
E. Mesangioproliferative glomerulonephritis
1332. Nephrotic syndrome may occur in morphological case of
A. minimal change disease
B. Mesangiocapillary glomerulonephritis
C. Membranous nephropathe
D. Mesangioproliferative glomerulonephritis
E. *All of them
1333. Nephrotic syndrome may occur in morphological case of
A. minimal change disease
B. Proliferative glomerulonephritis
C. Membranous nephropathe
D. *All of them
E. None of them
1334. Nephrotic syndrome may occur in morphological case except
A. Proliferative glomerulonephritis
B. *Pyelonephritis
C. Dibetic nephropathy
D. Amyloidosis
E. minimal change disease
1335. The energy intake of diet in patients with nephritic syndrome is provided by increased
A. Increased content of Salt
B. Increased content of fats
C. Increased content of proteins
D. *Inceased content of carbohydrates
E. All of then
1336. What percent of adult idiopathic nephrotic syndrome is caused by minimal change disease?
A. *20%
B. 30%
C. 40%
D. 50%
E. 10%
1337. The common findings at presentation of focal segmental glomerulosclerosis are except
A. Nephrotic syndrome
B. Hypertension
C. Microscopic hematuria
D. Decreased GFR (30 ml/min)E. *Hyperfiltration
1338. The mechanisms of proteinuria are except ?
A. Overflow of elevated normal or abnormal serum proteins
B. Decreased reabsorption of normal filtered proteins
C. Increased glomerular permeability
D. Changes in renal hemodyndamics
E. *Hypercoagubility
1339. What is the most common form of idiopathic glomerulonephritis?
A. *IgA nephropathy
B. minimal change disease
C. Proliferative glomerulonephritis
D. Membranous nephropathe
E. Mesangioproliferative glomerulonephritis
1340. What are common findings a presentation of RPGN expt?
A. Oligouria
B. Hypertension
C. Edema
D. Active sediment with erythrocytes and casts
E. *Isolated mild proteinurea
1341. What is the most common cause of hematuria of glomerular origin?
A. *IgA nephropathy
B. minimal change disease
C. Proliferative glomerulonephritis
D. Membranous nephropathy
E. Mesangioproliferative glomerulonephritis
1342. In IgA nephropathy, these patients commonly present with hematuria a few days after URI or GI
infection. What are the complement levels?
A. *Normal C4 and C3 levels
B. Increasedl C4 and C3 levels
C. Decreasedl C4 and C3 levels
D. Normal C1 and C2 levels
E. Increasedl C1 and C2 levels
1343. Describe the FENA in the setting of glomerulonephritis?
A. *The FENA will be <1
B. The FENA will be <5
C. The FENA will be <3
D. The FENA will be <4
E. The FENA will be >2
1344. Which RPGN is characterized by immune complex deposits?
A. *Type II RPGN
B. Type I RPGN
C. Type III RPGN
D. Type IV RPGN
E. All of them
1345. What are common findings at presentation of membranous nephropathy?
A. Edema
B. Hypertension
C. Hypoalbuminemia
D. Normal GFR
E. *All of them
1346. Describe the laboratory evaluation of complement in IgG and IgM mediated glomerulonephritis?
A. *low levels of C3 and C4
B. low levels of C1and C2
C. high levels of C3 and C4
D. high levels of C1 and C2
E. normal levels of C3 and C4
1347. What is the most common etiology of nephrotic syndrome among Caucasians?
A. IgA nephropathy
B. minimal change disease
C. Proliferative glomerulonephritis
D. Membranous nephropathy
E. Mesangioproliferative glomerulonephritis
1348. What is the most common underlying renal pathology of nephrotic syndrome in patients with solid
tumors?
A. IgA nephropathy
B. minimal change disease
C. Proliferative glomerulonephritis
D. Membranous glomerulopathy
E. *Mesangioproliferative glomerulonephritis
1349. What are the common findings at presentation of acute glomerulonephritis?
A. Oliguria
B. Hypertension
C. Active urine sediment
D. Proteinuria which is rarely nephrotic range
E. *All of them
1350. What is the specific target of the anti-GBM antibodies in Goodpasture's syndrome?
A. *The alpha-3 chian of type IV collagen
B. The alpha-3 chian of type I collagen
C. The alpha-3 chian of type II collagen
D. The alpha-3 chian of type III collagen
E. The alpha-4 chian of type IV collagen
1351. What are causes of glomerulonephritis?
A. Post streptococcal
B. IgA nephropathy
C. Rapidly progressive glomerulonephritis
D. Crescentric glomerulonephritis
E. *All of them
1352. What amount of proteinuria per day is indicative of nephrotic syndrome?
A. *Proteinuria of more than 3.5 grams/day
B. Proteinuria of more than 2.5 grams/day
C. Proteinuria of more than 1.5 grams/day
D. Proteinuria of more than 3.0 grams/day
E. Proteinuria of more than 2.0 grams/day
1353. Describe important findings in post streptococcal glomerulonephritis following pharyngitis?
A. -20 day latent period
B. ASO >250 u/ml
C. Low C3
D. Following cutaneous strep infections there is a 14-21 day latent period and normal ASO titers
E. *All of them
1354. What are six common etiologies of membranous nephropathies?
A. *Idiopathic, syphilis, Hepatitis B, SLE, gold salts, and malignancy
B. Idiopathic, chronic heart failure, Hepatitis B, SLE, gold salts, and malignancy
C. Idiopathic, syphilis, Hepatitis A, SLE, gold salts, and malignancy
D. Idiopathic, syphilis, Hepatitis B, SLE, NSAIDs, and malignancy
E. Idiopathic, syphilis, Hepatitis B, SLE, gold salts, and bronchial asthma
1355. What is the most common etiology of nephrotic syndrome in African Americans?
A. Post streptococcal
B. IgA nephropathy
C. Rapidly progressive glomerulonephritis
D. Crescentric glomerulonephritis
E. *Focal segmental glomerulosclerosis
1356. Which drugs should not be used in pregnancy.
A. *Fluoroquinolones;
B. Phospomycine;
C. Aminoglucosides;
D. macrocrystalline nitrofurantoin;
E. amoxicillin
1357. The most common organism in acute bacterial pyelonephritis is
A. Klebsiella
B. Chlamydia
C. *E. Coli
D. Pseudomonas
E. Candida
1358. Symptoms of acute pyelonephritis generally develop rapidly over a few hours or a day and may
include all of following except:
A. fever;
B. vomiting;
C. chills;
D. nausea;
E. *oedema
1359. What is the most appropriate initial treatment for a patient with a nonobstructing radiolucent stone in
the right renal pelvis?
A. Hydrochlorothiazide
B. Low-sodium diet
C. Allopurinol
D. Oral sodium bicarbonate or potassium citrate
E. *Extracorporeal shock-wave lithotripsy
1360. What percentage of the population will develop nephrolithiasis?
A. *1-5 %
B. 5-10 %
C. 10 -15 %
D. 30-50 %
E. 60-70 %
1361. What are the signs and symptoms of hypercalcemia?
A. Stones
B. Bones
C. Abdominal groans
D. Psychiactric overtones
E. *All of them
1362. What is the most common type of kidney stone?
A. *2/3 are calcium stones (calcium oxalate, calcium phosphate)
B. 2/3 are urate stones
C. 1/2 are mixed stones
D. 2/3 are phosphate stones
E. 2/3 are cholesterol stones
1363. What is an autosomal dominant disease characterized by bilateral enlargement of kidneys secondary
to multiple large cysts. Patients present with hematuria, pain, hypertension, and progressive renal failure.
What is this disease?
A. *Adult polycystic kidney disease
B. Diabetic nephropathy
C. Alport’s syndrome
D. Goodpasture’s syndrome
E. Primary amuloidosis
1364. Overflow proteinuria should be suspected in patients with clinical or laboratory evidence of:
A. multiple myeloma;
B. *pyelonephritis;
C. rheumatoid arthritis;
D. diabetic nephropathy;
E. drug-induced interstitial nephritis.
1365. Nephrotic syndrome in adults occurs in these cases of:
A. minimal change disease (MCD),
B. focal and segmental glomerulosclerosis (FSGS),
C. membranous glomerulopathy,
D. *all of them;
E. one of the above.
1366. The pathognomonic morphologic lesion in FSGS is sclerosis with hyalinosis involving portions
(segmental) of fewer than:
A. 10% (focal) of glomeruli on a tissue section;
B. 20% (focal) of glomeruli on a tissue section;
C. 30% (focal) of glomeruli on a tissue section;
D. *40% (focal) of glomeruli on a tissue section;
E. 50% (focal) of glomeruli on a tissue section.
1367. Evaluation of the routine urine analysis includes everything except:
A. amount of the urine;
B. urine sediment;
C. transparence;
D. *the common diuresis;
E. reaction.
1368. Idiopathic FSGS (Focal and Segmental Glomerulosclerosis) typically presents as nephrotic syndrome
(~66 %) or subnephrotic proteinuria (~33%) in association with
A. hypertension,
B. mild renal insufficiency,
C. abnormal urine sediment
D. *all of them;
E. none of the above.
1369. Causes of Membranoproliferative (Mesangiocapillary) Glomerulonephritis (MPGN) associated with
chronic infections are all the following except:
A. Hepatitis B and C,
B. HIV,
C. bacterial endocarditis,
D. Leukemias,
E. *visceral abscess.
1370. Causes of Membranoproliferative (Mesangiocapillary) Glomerulonephritis (MPGN) associated with
systemic immune-complex disease are:
A. systemic lupus erythematosus;
B. mixed cryoglobulinemia;
C. Sjogren's syndrome;
D. *all of them;
E. none of the above.
1371. Most patients with type I MPGN present with all of the following except:
A. heavy proteinuria;
B. nephrotic syndrome;
C. *light proteinuria;
D. active urinary sediment;
E. normal or mildly impaired GFR.
1372. Renal biopsy is particularly useful when clinical assessment and laboratory investigations suggest
diagnoses of all of following except:
A. glomerulonephritis,
B. hemolytic-uremic syndrome,
C. thrombotic thrombocytopenic purpura,
D. allergic interstitial nephritis.
E. *ischemic renal injury
1373. Standard approach to screen for renovascular hypertension has following noninvasive techniques:
A. the captopril-enhanced radionuclide renal scan;
B. a duplex Doppler flow study;
C. magnetic resonance (MRI) angiography
D. *all of them;
E. none of the above.
1374. The dipstick examination should includes everything except:
A. protein;
B. *creatinine;
C. glucose;
D. ketones;
E. bilirubin.
1375. Microscopic examination should check for all of following except:
A. crystals;
B. cells;
C. casts;
D. infecting organisms;
E. *ketones.
1376. Urinary examination according to Zimnitsky is needed for evaluation of all of following except:
A. the common diuresis;
B. daily urine;
C. *renal clearance;
D. night urine;
E. fluctuation of the specific gravity of urine in every portion.
1377. Urinary examination according to Necheporenco determines:
A. amount of erythrocytes in 1 ml (or 1 L) of urine;
B. amount of leucocytes in 1 ml (or 1 L) of urine;
C. amount of casts in 1 ml (or 1 L) of urine
D. *all of them;
E. none of the above.
1378. Which of the following antimicrobial therapies would be inappropriate and potentially harmful in a
patient receiving cyclosporine?
A. Amoxicillin-clavulanate
B. *Erythromycin
C. Cefuroxime axetil
D. Ciprofloxacin
E. Penicillin
1379. Which is the most appropriate antihypertensive therapy for patient with glomerulonephritis?
A. Intensive lifestyle modification
B. Diuretic
C. Nondihydropyridine calcium channel blocker
D. *Angiotensin-converting enzyme inhibitor
E. Beta-blockers
1380. What is the most appropriate method to evaluate this patients renal function?
A. Measure the serum creatinine
B. Perform 24-hour urine collection to assess creatinine and volume
C. Perform technetium-99m-diethylenetriam me pentaacetic acid renal flow scanning
D. Measure the fractional excretion of sodium
E. *Estimate by using a creatinine-based formula
1381. What is the best test to evaluate the patients risk for diabetic nephropathy and cardiovascular disease?
A. *Microalbumin-to-creatinine ratio
B. Serum protein electrophoresis
C. Urine amino acid levels
D. 24-hour urine total protein
E. Annual urine dipstick analysis
1382. The indications for dialysis in renal failure are except?
A. Fluid overload
B. Refractory hyperkalemia
C. BUN > 130
D. Pericardial friction rub
E. *Hemorragi stroke
1383. Hyperkalemia can be due to decreased GFR only if GFR is below level^
A. *GFR < 20 ml/min
B. GFR < 70 ml/min
C. GFR < 50 ml/min
D. GFR < 30 ml/min
E. GFR < 40 ml/min
1384. What is the normal daily water losses in sweat?
A. *200-400 ml
B. 100-200 ml
C. 50-100 ml
D. 150-200 ml
E. 250-300 ml
1385. Which analgesics are associated with analgesic nephropathy?
A. *Phenacitin and acetaminophen
B. Aspirin
C. Ibuprofen
D. Diclofenac
E. All of themd
1386. What are the effects of hyperkalemia on the EKG?
A. Tent shaped T-waves
B. Decreased or absent p waves
C. Short QT interval
D. Widening QRS complex
E. *All of them
1387. To delay onset of ESRD in patients with diabetic nephropathy, what are four clinical tactics?
A. Decrease BP to below 130/85
B. Use ACE inhibitors
C. Dietary protein restriction to 0.6-0.8 g/kg
D. Maintain strict glycemic control
E. *All of them
1388. What is the most common cause of hypocalcemia?
A. *Chronic renal failure
B. Acute renal failure
C. Dietary restriction of calcium
D. Renal tubular acidosis
E. Nephrocalcinosis
1389. What is the BUN to Creatinine ratio in patients with post-renal renal failure?
A. *Approximately >10 to 1
B. Approximately <10 to 1
C. Approximately >20 to 1
D. Approximately <2 to 1
E. Approximately 1 to 1
1390. Findings that may be found in patients with chronic renal failure include all of following except:
A. anemia;
B. *hyperproteinemia;
C. hypercreatininemia;
D. radiologic evidence of renal osteodystrophy;
E. radiologic evidence of small scarred kidneys.
1391. Characteristic ECG feature of hyperkalemia:
A. U waves
B. Narrow QRS complex
C. *Tall T waves
D. Sort PR interval
E. Pathologic Q wave
1392. Absolute contraindication for recombinant erythropoietin prescription:
A. *Noncontrolled blood hypertension
B. uremic neuropathy
C. hypoparathyroidism
D. aseptic bone necrosis
E. carbohydrate disturbunces
1393. What is the definition of oliguria?
A. Oliguria is <600 ml of urine per 24
B. *Oliguria is <400 ml of urine per 24
C. Oliguria is <800 ml of urine per 24 hours
D. Oliguria is is less than 100 ml urine per 24 hours
E. Oliguria is <500 ml of urine per 24
1394. What is the definition of anuria?
A. anuria is less than 500 ml urine per 24 hours
B. anuria is less than 400 ml urine per 24 hours
C. anuria is less than 300 ml urine per 24 hours
D. anuria is less than 200 ml urine per 24 hours
E. *anuria is less than 100 ml urine per 24 hours
1395. What is the definition of oliguria and anuria?
A. Oliguria is <600 ml of urine per 24 hours and anuria is less than 300 ml urine per 24 hours
B. Oliguria is <500 ml of urine per 24 hours and anuria is less than 200 ml urine per 24 hours
C. *Oliguria is <400 ml of urine per 24 hours and anuria is less than 100 ml urine per 24 hours
D. Oliguria is <650 ml of urine per 24 hours and anuria is less than 150 ml urine per 24 hours
E. Oliguria is <1000 ml of urine per 24 hours and anuria is less than 100 ml urine per 24 hours
1396. What is the most common cause of intra-renal renal failure?
A. *Acute tubular necrosis
B. Hypovolemia;
C. Low cardiac output;
D. Renovascular obstruction;
E. Renal ischemia
1397. What are the symptoms of hypernatremia?
A. Seizures
B. Confusion
C. Pulmonary or peripheral edema
D. Respiratory paralysis
E. *All of them
1398. The symptoms of hypernatremia are all of the following except?
A. Seizures
B. Confusion
C. Pulmonary or peripheral edema
D. Respiratory paralysis
E. *Hypovolemia
1399. Is there a difference in the effects of furosemide and thiazide diuretics on the calcium excretion by
the kidney?
A. *Yes, furosemide increase calcium excretion by the kidney and thiazide diuretics decrease calcium
excretion by the kidney
B. Yes, furosemide decrease calcium excretion by the kidney and thiazide diuretics increase calcium
excretion by the kidney
C. Yes, furosemide increase calcium excretion by the kidney and thiazide diuretics are neutral
D. Yes, thiazide diuretics decrease calcium excretion by the kidney
E. No, the difference is absent
1400. What diuretics can cause worsening of hyperkaliemia in chronic renal failure patients
A. loop diuretics
B. thiazide diuretics
C. *aldoserone antagonists
D. all of them
E. none of them
1401. In patients with pre-renal azotemia, would you expect the urine sodium to be increased or decreased?
A. *The urine sodium in pre-renal azotemia is generally decreased <20
B. The urine sodium in pre-renal azotemia is generally increased >20
C. The urine sodium in pre-renal azotemia is normal
D. The urine sodium in pre-renal azotemia is generally decreased <40
E. The urine sodium in pre-renal azotemia is generally increased >40
1402. What is the primary cause of volume overload in end stage renal disease?
A. *because of inablility of kidney to excrete sodium.
B. because of inablility of kidney to excrete potassium
C. because of inablility of kidney to excrete Magnesium
D. because of inablility of secondary hyperaldosteronism
E. none of them
1403. Can cephalosporins affect the serum creatinine measurement?
A. No, Cephalosporins don”t affect the serum creatinine measurement?
B. *Cephalosporins can result in a false increase in the serum creatinine
C. Cephalosporins can result in a false decrease in the serum creatinine
D. Cephalosporins can result in a true increase in the serum creatinine
E. Cephalosporins can result in a true decrease in the serum creatinine
1404. How quickly will the creatinine rise in the setting of complete renal failure?
A. *The creatinine will rise approximately 1-2 mg/dL per day
B. The creatinine will rise approximately 2-3 mg/dL per day
C. The creatinine will rise approximately 3-5 mg/dL per day
D. The creatinine will rise approximately 3-4 mg/dL per day
E. The creatinine wil not change
1405. What are the classic urinary findings of acute tubular necrosis?
A. Hyaline casts
B. *Muddy brown granular casts
C. Fatty casts
D. Leucocyte casts
E. Erythrocte casts
1406. What is the most likely diagnosis in a patient with positive Chvostek's and/or Trousseau's sign and
leg cramps?
A. *Hypocalcemia
B. Hypopotassemia
C. Hypophosphatemia
D. Hypermagnesiemia
E. Hypercalcemia
1407. What are the common EKG changes seen in hypomagnesemia?
A. Prolonged PR or QT intervals,
B. T-wave flattening or inversion
C. ST straightening
D. *All of them
E. None of them
1408. When metabolic alkalosis occurs because of volume contraction, how does the serum chloride
typically respond?
A. *The urinary chloride is typically low (<10 meq/L)
B. The urinary chloride is typically low (<20 meq/L)
C. The urinary chloride is typically high (>10 meq/L)
D. The urinary chloride is typically high (>20 meq/L)
E. The urinary chloride is typically normal
1409. Ccommon findings in autosomal dominant polycystic kidney disease are except?
A. Flank or back pain
B. gross hematuria
C. abdominal mass
D. frequent UTI's
E. *nephrotic synrome
1410. Common findings in autosomal dominant polycystic kidney disease are ?
A. Flank or back pain
B. gross hematuria
C. abdominal mass
D. frequent UTI's
E. *All of them
1411. What are 6 common findings in autosomal dominant polycystic kidney disease?
A. *Flank or back pain, gross hematuria, abdominal mass, frequent UTI's, hypertension, and
nephrolithiasis
B. Flank or back pain, gross hematuria, small kidneys, frequent UTI's, hypertension, and nephrolithiasis
C. Flank or back pain, gross hematuria, abdominal mass, nephritic syndrome, hypertension, and
nephrolithiasis
D. Flank or back pain, gross hematuria, abdominal mass, frequent UTI's, hypotension, and nephrolithiasis
E. None of them
1412. What are common findings in autosomal dominant polycystic kidney disease?
A. gross hematuria,
B. frequent UTI's,
C. arterial hypertension,
D. nephrolithiasis
E. *all of them
1413. In a patient with hypercalciuric nephrolithiasis, which of the following is associated with increased
risk for stone formation?
A. Low-sodium diet
B. *Low-calcium diet
C. Low-oxalate diet
D. Low-protein diet
E. Low-purine diet
1414. Can dobutamine affect the serum creatinine measurement?
A. *Dobutamine can falsely decrease the serum creatinine
B. Dobutamine can falsely increase the serum creatinine
C. Dobutamine don’t interfere the serum creatinine
D. Dobutamine can true decrease the serum creatinine
E. None of them
1415. What is the normal daily water losses in urine?
A. 2.0-2.5 liters
B. *1.2-1.5 liters
C. 0.5-1.0 liters
D. 2.2-3.5 liters
E. 0.8-1.2 liters
1416. Complications of long-term dialysis include all the following EXCEPT
A. *Increased risk of gastrointestinal malignancy
B. myocardial infarction
C. carpal tunnel syndrome
D. protein-calorie malnutrition
E. high-output congestive heart failure
1417. In patients with chronic renal failure, all the following are important contributors to bone disease
EXCEPT
A. impaired renal production of 1,25-dihydroxyvitamin D3
B. hyperphosphatemia
C. aluminum-containing antacids
D. *loss of vitamin D and calcium via dialysis
E. metabolic acidosis
1418. Why is hypomagnesemia associated with hypocalcemia?
A. Hypomagnesemia causes a shift of calcium into bone
B. *Hypomagnesemia inhibits the secretion and action of parathyroid hormone
C. Hypomagnesemia causes renal calcium wasting
D. Hypomagnesemia impairs the peripheral actions of vitamin
E. None of them
1419. What is the most common cause of death in patients with acute tubular necrosis?
A. Hypophosphatemia
B. Hypermagnesiemia
C. Hypercalcemia
D. *Hyperkalemia
E. Hypocalcemia
1420. What is the effect of lithium on the kidney?
A. *Lithium can result in a nephrogenic diabetes insipidus
B. Lithium can result in a diabetes mellitus
C. Lithium can result in a a acute tubular necrosis
D. Lithium can result in a toxic ebcephalopathy
E. Lithium can result in anemia
1421. What are the findings of methanol ingestion?
A. Systemic acidosis
B. Direct neurotoxicity (secondary to the metabolites formic acid and formaldehyde)
C. Direct hepatotoxicity
D. *All of them
E. None of them
1422. What are the steps to treating myoglobinuria?
A. Volume repletion with IVF
B. Mannitol diuresis
C. Alkalinize urine with IV bicarbonate
D. *All of them
E. None of them
1423. List three signs on physical exam of hyperkalemia?
A. Decreased DTR's
B. Weakness
C. Respiratory failure
D. *All of them
E. None of them
1424. What conditions result in a the fractional excretion of sodium (FENA) of <1%?
A. Prerenal azotemia,
B. acute glomerulonephritis,
C. hepatorenal syndrome,
D. renal transplant rejection
E. *All of them
1425. What is the effect of hypercalcemia on the heart?
A. Short or absent ST segment
B. Decreased QT interval
C. Tachicardia
D. *All of them
E. None of them
1426. What is stage I hypertension?
A. *Stage I hypertension is characterized by a SBP of 140-159 or a DBP of 90-99 averaged from 2 or
more readings on two or more occassions
B. Stage I hypertension is characterized by a SBP of 120-139 or a DBP of 80-85 averaged from 2 or more
readings on two or more occassions
C. Stage I hypertension is characterized by a SBP of 150-159 or a DBP of 90-99 averaged from 2 or more
readings on two or more occassions
D. Stage I hypertension is characterized by a SBP of 160-179 or a DBP of 100-110 averaged from 2 or
more readings on two or more occassions
E. Stage I hypertension is characterized by a SBP of 130-159 or a DBP of 90-95 averaged from 2 or more
readings on two or more occassions
1427. What are common findings associated with analgesic nephropathy?
A. Nocturia,
B. polyuria,
C. sterile pyuria,
D. predisposition to volume depletion,
E. *all of them
1428. What are common findings associated with analgesic nephropathy?
A. renal colic,
B. hematuria,
C. hypertension
D. polyuria
E. *all of them
1429. What are the most common causes of acute tubular necrosis?
A. *ischemia or toxic injury
B. Acute glomerulonephritis
C. Acute pyelonephritis
D. All of them
E. None of them
1430. Which type of kidney stone is associated with urinary tract infections, particularly with urease
producing bacteria such as proteus, pseudomonas, and staphylococcus)?
A. *Struvite stones (staghorn calculi)
B. Urate stones
C. Oxalate stones
D. Phoaphate stones
E. Cholesterol stones
1431. The common findings at presentation of focal segmental glomerulosclerosis are except
A. Nephrotic syndrome
B. Hypertension
C. Microscopic hematuria
D. Decreased GFR (30 ml/min)E. *All of them
1432. What is the most common cause of hypokalemia?
A. *Diuretics
B. Chronic renal failure
C. Acute renal failure
D. Interstitial nephritis
E. Pyelonephritis
1433. Name one drug commonly associated with distal renal tubule acidosis?
A. *Amphotericin B
B. Erythromycine
C. Sulphanilamides
D. NSAID’s
E. Diuretics
1434. What underlying diagnosis is suggested in a patient with acute renal failure and white blood cells or
white blood cell casts?
A. *Interstitial nephritis
B. Glomerulonephritis
C. Pyelonephritis
D. Renal stones
E. Amyloidosis
1435. What are the findings of nephritic syndrome on urinanalysis?
A. *RBC casts and hematuria
B. WBS casts and pyuria
C. Isolated proteinuria
D. Hematuria and pyuria
E. No pathological findings
1436. What underlying diagnosis is suggested in a patient with acute renal failure and red blood cell casts in
the urine?
A. Interstitial nephritis
B. *Glomerulonephritis
C. Pyelonephritis
D. Renal stones
E. Amyloidosis
1437. Methods of renal replacement therapy are:
A. Hemodialysis
B. Peritoneal dialysis
C. Kidney transplantation
D. *All of them
E. None of them
1438. Methods of renal replacement therapy are except:
A. Hemodialysis
B. Peritoneal dialysis
C. Kidney transplantation
D. *Hemosorbtion
E. All of them
1439. Methods of renal replacement therapy are except:
A. *Hemofiltration
B. Hemodialysis
C. Peritoneal dialysis
D. Kidney transplantation
E. All of them
1440. Methods of renal replacement therapy are:
A. Hemofiltration
B. Hemosorbtion
C. Plasmapheresis
D. All of them
E. *None of them
1441. Methods of renal replacement therapy are:
A. Hemofiltration
B. Hemosorbtion
C. Plasmapheresis
D. *Hemodialysis
E. All of them
1442. How many nephrons are there in each kidney?
A. 100 – 200 thousands
B. 200 – 300 thousands
C. 500 – 700 thousands
D. *1,2 – 1,5 millions
E. 2 – 3 millions
1443. What does “kidney hyperplasia” mean?
A. *Congenital decrease of kidney sizes
B. Congenital increase of kidney sizes
C. Congenital cystic transformation of kidneys
D. Congenital tubular disorders of kidneys
E. Congenital glomerular disorders of kidneys
1444. Describe the characteristics of iuxtaglomerulonephrones:
A. Presence of secondary capillary network
B. Greater diameter of efferent arterioles compared to afferent
C. Rennin production
D. *All of them
E. None of them
1445. The characteristics of cortex nephrones include:
A. Presence of secondary capillary network
B. Greater diameter of efferent arterioles compared to afferent
C. Rennin production
D. All of them
E. *None of them
1446. The kidney functions are:
A. Maintains of fluid load in body
B. Maintains of constant osmotic pressure plasma
C. Maintains of constant electrolyte balance
D. Maintains of blood acidity
E. *All of them
1447. The kidney functions are except:
A. Hormone production
B. Maintains of constant osmotic pressure plasma
C. Maintains of constant electrolyte balance
D. Maintains of blood acidity
E. *Synthesis of urea and creatinine
1448. The indices used to evaluate renal function are
A. Glomerular filtration rate
B. Plasma Creatinine level
C. Plasma Urea level
D. All of them
E. None of them
1449. The indices used to evaluate renal function are except
A. Glomerular filtration rate
B. Plasma Creatinine level
C. Plasma Urea level
D. Zimnitsky test
E. *Nechiporenko test
1450. Describe the characteristics of cortex nephrones:
A. Absence of secondary capillary network
B. Greater diameter of afferent arterioles compared to efferent
C. Cortex localization
D. *All of them
E. None of them
1451. Normal levels of Glomerular filtration rate (GFR) are
A. 140-190 ml/min
B. *90-120 ml/min
C. 60-90 ml/min
D. 30-60 ml/min
E. 90-190 ml/min
1452. Processes of ultrafiltration in kidneys are stopped when systemic arterial blood pressure is ;
A. *<60 mm Hg
B. 140 -160 mm Hg
C. 120 -140 mm Hg
D. 90 -100 mm Hg
E. 70 -80 mm Hg
1453. Normal levels of tubular water reabsorbing are
A. *97-99 %
B. 95-96 %
C. 93-94 %
D. 90-92 %
E. 87-90 %
1454. Normal sizes of kidneys in adults are
A. *10-12 сm х 5-6 сm х 3-4 сm
B. 15-16 сm х 7-8 сm х 5-6 сm
C. 7-8 сm х 3-4 сm х 2-3 сm
D. 5-7 сm х 2-4 сm х 1-3 сm
E. 18-22 сm х 8-10 сm х 6-7 сm
1455. What kidney cells have endocrine function?
A. Gumptreht’s cells
B. Juxtaglomerular cells
C. Interstitial cells
D. None of them
E. *All of them
1456. Kidney cells have endocrine function except
A. Gumptreht’s cells
B. Juxtaglomerular cells
C. Interstitial cells
D. *Epithelial cells of distal tubuls
E. All of them
1457. Anomalies of kidneys topic and shape are
A. Kidneys dystopia
B. L-like kinnney
C. C-like kidney
D. *All of them
E. None of them
1458. Anomalies of kidneys topic and shape are except:
A. Kidneys dystopia
B. L-like kidney
C. C-like kidney
D. All of them
E. *Polycystic kidney
1459. Anomalies of kidneys structure are:
A. Kidneys dystopia
B. L-like kinnney
C. C-like kidney
D. All of them
E. *None of them
1460. Anomalies of kidneys structure are:
A. Polycystic kidney
B. Multicystic kidney
C. Megacalix
D. *All of them
E. None of them
1461. What kidney structures are visible on longitudinal incision
A. Cortex
B. Pelvis
C. Cerebral part
D. Calices
E. *All of them
1462. What kidney structures are visible on longitudinal incision?
A. *Cortex, pelvis, cerebral part, calices
B. Cortex, cerebral part, calices
C. Cortex, pelvis, calices
D. Cortex, pelvis, cerebral part
E. Pelvis, cerebral part, calices
1463. The structure of nephron includes:
A. Glomerulus
B. Glomerulus’s capsule
C. Renal tubules
D. *All of them
E. None of them
1464. The most common organism in acute bacterial pyelonephritis is
A. Klebsiella
B. Chlamydia
C. *E coli
D. Pseudomonas
E. Candida
1465. What is the definition of hematuria?
A. *Greater than 3-5 red cells per high power field
B. Greater than 0-1 red cells per high power field
C. Greater than 1-2 red cells per high power field
D. Greater than 2-3 red cells per high power field
E. Greater than 1-4 red cells per high power field
1466. The risk factors for poor neurological outcome include
A. diuretic-induced hyponatremia,
B. malnutrition,
C. liver disease,alcoholism,
D. female gender
E. *All of them
1467. What are the mechanisms of proteinuria?
A. Overflow of elevated normal or abnormal serum proteins
B. Decreased reabsorption of normal filtered proteins
C. Increased glomerular permeability
D. Changes in renal hemodyndamics
E. *All of them
1468. How many nephrons are there in each kidney?
A. 100 – 200 thousands
B. 200 – 300 thousands
C. 500 – 700 thousands
D. *1,2 – 1,5 millions
E. 2 – 3 millions
1469. What does “kidney hyperplasia” mean?
A. *Congenital decrease of kidney sizes
B. Congenital increase of kidney sizes
C. Congenital cystic transformation of kidneys
D. Congenital tubular disorders of kidneys
E. Congenital glomerular disorders of kidneys
1470. Describe the characteristics of cortex nephrones:
A. Absence of secondary capillary network
B. Greater diameter of afferent arterioles compared to efferent
C. Cortex localization
D. *All of them
E. None of them
1471. Describe the characteristics of iuxtaglomerulonephrones:
A. Presence of secondary capillary network
B. Greater diameter of efferent arterioles compared to afferent
C. Rennin production
D. *All of them
E. None of them
1472. The characteristics of cortex nephrones include:
A. Presence of secondary capillary network
B. Greater diameter of efferent arterioles compared to afferent
C. Rennin production
D. All of them
E. *None of them
1473. The kidney functions are:
A. Maintains of fluid load in body
B. Maintains of constant osmotic pressure plasma
C. Maintains of constant electrolyte balance
D. Maintains of blood acidity
E. *All of them
1474. The kidney functions are except:
A. Hormone production
B. Maintains of constant osmotic pressure plasma
C. Maintains of constant electrolyte balance
D. Maintains of blood acidity
E. *Synthesis of urea and creatinine
1475. The indices used to evaluate renal function are
A. *Glomerular filtration rate
B. Plasma Creatinine level
C. Plasma Urea level
D. All of them
E. None of them
1476. The indices used to evaluate renal function are except
A. Glomerular filtration rate
B. Plasma Creatinine level
C. Plasma Urea level
D. Zimnitsky test
E. *Nechiporenko test
1477. Normal levels of Glomerular filtration rate (GFR) are
A. 140-190 ml/min
B. *90-120 ml/min
C. 60-90 ml/min
D. 30-60 ml/min
E. 90-190 ml/min
1478. Processes of ultrafiltration in kidneys are stopped when systemic arterial blood pressure is ;
A. 140 -160 mm Hg
B. 120 -140 mm Hg
C. 90 -100 mm Hg
D. 70 -80 mm Hg
E. *<60 mm Hg
1479. Normal levels of tubular water reabsorbing are
A. *97-99 %
B. 95-96 %
C. 93-94 %
D. 90-92 %
E. 87-90 %
1480. Normal sizes of kidneys in adults are
A. *10-12 сm х 5-6 сm х 3-4 сm
B. 15-16 сm х 7-8 сm х 5-6 сm
C. 7-8 сm х 3-4 сm х 2-3 сm
D. 5-7 сm х 2-4 сm х 1-3 сm
E. 18-22 сm х 8-10 сm х 6-7 сm
1481. What kidney cells have endocrine function?
A. Gumptreht’s cells
B. Juxtaglomerular cells
C. Interstitial cells
D. None of them
E. *All of them
1482. Kidney cells have endocrine function except
A. Gumptreht’s cells
B. Juxtaglomerular cells
C. Interstitial cells
D. *Epithelial cells of distal tubuls
E. All of them
1483. Anomalies of kidneys topic and shape are
A. Kidneys dystopia
B. L-like kinnney
C. C-like kidney
D. *All of them
E. None of them
1484. Anomalies of kidneys topic and shape are except:
A. Kidneys dystopia
B. L-like kidney
C. C-like kidney
D. All of them
E. *Polycystic kidney
1485. Anomalies of kidneys structure are:
A. Kidneys dystopia
B. L-like kinnney
C. C-like kidney
D. All of them
E. *None of them
1486. Anomalies of kidneys structure are:
A. Polycystic kidney
B. Multicystic kidney
C. Megacalix
D. *All of them
E. None of them
1487. What kidney structures are visible on longitudinal incision
A. Cortex
B. Pelvis
C. Cerebral part
D. Calices
E. *All of them
1488. What kidney structures are visible on longitudinal incision?
A. *Cortex, pelvis, cerebral part, calices
B. Cortex, cerebral part, calices
C. Cortex, pelvis, calices
D. Cortex, pelvis, cerebral part
E. Pelvis, cerebral part, calices
1489. The structure of nephron includes:
A. Glomerulus
B. Glomerulus’s capsule
C. Renal tubules
D. *All of them
E. None of them
1490. The nephrotic syndrome is characterized by all of following except:
A. А heavy proteinuria;
B. hypoproteinaemia;
C. light proteinuria;
D. hyperproteinaemia;
E. generalized oedema.
1491. Acute nephritic syndrome is characterized by sudden onset of all of following except:
A. acute renal failure ;
B. oliguria (<400 mL of urine per day);
C. hematuria;
D. hypertension;
E. *Proteinuria more3.5 g per 24 h.
1492. Urinary tract infection has all of the following clues to diagnosis except:
A. Bacteriuria more 105 colonies per millilitre;
B. Pyuria;
C. Bladder tenderness;
D. *signs of renal osteodystrophy
E. urinary frequency.
1493. Major causes of prerenal acute renal failure are all of the following except:
A. Hypovolemia;
B. Low cardiac output;
C. Renovascular obstruction;
D. *Hyperviscosity syndrome;
E. Altered renal systemic vascular resistance ratio.
1494. Findings that suggest chronic renal failure include all of following except:
A. anemia;
B. *neuropathy;
C. rise in blood creatinine levels;
D. radiologic evidence of renal osteodystrophy;
E. radiologic evidence of small scarred kidneys.
1495. ARF in association with:
A. oliguria,
B. hypertension,
C. an "active" urine sediment (nephritic syndrome);
D. *all of them;
E. none of the above.
1496. ARF in association with fever, arthralgias, and a pruritic erythematous rash following exposure to a
new drug suggest:
A. vasculitis;
B. *allergic interstitial nephritis;
C. hemolytic-uremic syndrome;
D. thrombotic thrombocytopenic purpura;
E. glomerulonephritis.
1497. White cell casts and nonpigmented granular casts suggest;
A. *pyelonephritis;
B. vasculitis;
C. thrombotic thrombocytopenic purpura;
D. glomerulonephritis;
E. amyloidosis.
1498. Eosinophiluria (>5% of urine leukocytes) is a common finding (~90%) in:
A. *allergic interstitial nephritis induced by NSAIDs;
B. antibiotic-induced allergic interstitial nephritis;
C. thrombotic thrombocytopenic purpura;
D. amyloidosis;
E. glomerulonephritis.
1499. Heavy proteinuria is a frequent finding in patients who develop combined allergic interstitial
nephritis and minimal change glomerulopathy when treated with:
A. *NSAIDs;
B. echinaceae;
C. heparinotherapy;
D. platelet aggregation inhibitor;
E. angiotensin-converting enzyme (ACE) inhibitors.
1500. Renal biopsy is particularly useful when clinical assessment and laboratory investigations suggest
diagnoses of all of following except:
A. glomerulonephritis,
B. hemolytic-uremic syndrome,
C. thrombotic thrombocytopenic purpura,
D. allergic interstitial nephritis.
E. *ischemic renal injury
1501. Standard approach to screen for renovascular hypertension has following noninvasive techniques:
A. the captopril-enhanced radionuclide renal scan;
B. a duplex Doppler flow study;
C. magnetic resonance (MRI) angiography
D. *all of them;
E. none of the above.
1502. Evaluation of the routine urine analysis includes everything exept:
A. amount of the urine;
B. urine sediment;
C. transparence;
D. *the common diuresis;
E. reaction.
1503. The dipstick examination should includes everything exept:
A. protein;
B. *creatinine;
C. glucose;
D. ketones;
E. bilirubin.
1504. Microscopic examination should check for all of following except:
A. crystals;
B. cells;
C. casts;
D. infecting organisms;
E. *ketones.
1505. Urinary examination according to Zimnitsky is needed for evaluation of all of following except:
A. the common diuresis;
B. daily urine;
C. *renal clearance;
D. night urine;
E. fluctuation of the specific gravity of urine in every portion.
1506. Urinary examination according to Necheporenco determines:
A. amount of erythrocytes in 1 ml (or 1 L) of urine;
B. amount of leucocytes in 1 ml (or 1 L) of urine;
C. amount of casts in 1 ml (or 1 L) of urine
D. *all of them;
E. none of the above.
1507. Specific indications to renal biopsy are all of following except:
A. adult nephritic syndrome
B. *uncontrolled hypertension
C. persistent proteinuria > 1 g/24 hours
D. adult acute nephritic syndrome
E. systemic diseases with renal involvement
1508. Microscopic examination of urine in pyelonephritis reveals all of the following except:
A. numerous pus cells;
B. microorganisms;
C. *fatty casts;
D. some red cells;
E. some epithelial cells.
1509. The nephrotic syndrome is a clinical complex characterized by a number of renal and extrarenal
features, the most prominent of which are:
A. proteinuria of >3.5 g (per 24 h);
B. hypoalbuminemia;
C. edema;
D. hyperlipidemia;
E. *all of them.
1510. Low serum complement levels would be seen in patients with hematuria, proteinuria, and
hypertension resulting from all of the following EXCEPT
A. mixed essential cryoglobulinemia
B. hepatitis C-associated membranoproliferative glomerulonephritis
C. diffuse proliferative lupus nephritis
D. *Henoch-Schonlein purpura
E. poststreptococcal (or postinfectious) glomerulonephritis
1511. Nephrotic syndrome is the hallmark of all the following. primary kidney diseases EXCEPT
A. membranous glomerulopathy
B. focal segmental glomerulosclerosis
C. minimal-change disease
D. *IgA nephropathy (Berger's disease)
E. HIV-associated nephropathy with or without
1512. The presence of leucocyte casts is diagnostic of:
A. glomerular bleeding;
B. glomerulonephritis:
C. *pyelonephritis;
D. amyloidosis;
E. deep vein thrombosis
1513. Specific indications to renal biopsy are all of following except:
A. adult nephritic syndrome
B. *acute pyelonephritis
C. persistent proteinuria > 1 g/24 hours
D. adult acute nephritic syndrome
E. systemic diseases with renal involvement
1514. Which of the following studies would you choose to confirm amiloidosis?
A. Intravenous pyelography
B. Ultrasonography
C. *Renal biopsy
D. Computed tomography
E. Renal flow scanning
1515. Morphological forms of glomerulonephritis are exept:
A. minimal change disease (MCD),
B. focal and segmental glomerulosclerosis (FSGS),
C. membranous glomerulopathy,
D. proliferative glomerulonephritis;
E. masngioinflammatory glomerulonephritis
1516. What is the definition of hematuria?
A. *Greater than 3-5 red cells per high power field
B. Greater than 0-1 red cells per high power field
C. Greater than 1-2 red cells per high power field
D. Greater than 2-3 red cells per high power field
E. Greater than 1-4 red cells per high power field
1517. The risk factors for poor neurological outcome include
A. diuretic-induced hyponatremia,
B. malnutrition,
C. liver disease,alcoholism,
D. female gender
E. *All of them
1518. What are the mechanisms of proteinuria?
A. Overflow of elevated normal or abnormal serum proteins
B. Decreased reabsorption of normal filtered proteins
C. Increased glomerular permeability
D. Changes in renal hemodyndamics
E. *All of them
1519. How many nephrons are there in each kidney?
A. 100 – 200 thousands
B. 200 – 300 thousands
C. 500 – 700 thousands
D. *1,2 – 1,5 millions
E. 2 – 3 millions
1520. What does “kidney hyperplasia” mean?
A. *Congenital decrease of kidney sizes
B. Congenital increase of kidney sizes
C. Congenital cystic transformation of kidneys
D. Congenital tubular disorders of kidneys
E. Congenital glomerular disorders of kidneys
1521. Describe the characteristics of cortex nephrones:
A. Absence of secondary capillary network
B. Greater diameter of afferent arterioles compared to efferent
C. Cortex localization
D. *All of them
E. None of them
1522. Describe the characteristics of iuxtaglomerulonephrones:
A. Presence of secondary capillary network
B. Greater diameter of efferent arterioles compared to afferent
C. Rennin production
D. *All of them
E. None of them
1523. The characteristics of cortex nephrones include:
A. Presence of secondary capillary network
B. Greater diameter of efferent arterioles compared to afferent
C. Rennin production
D. All of them
E. *None of them
1524. The kidney functions are:
A. Maintains of fluid load in body
B. Maintains of constant osmotic pressure plasma
C. Maintains of constant electrolyte balance
D. Maintains of blood acidity
E. *All of them
1525. The kidney functions are except:
A. Hormone production
B. Maintains of constant osmotic pressure plasma
C. Maintains of constant electrolyte balance
D. Maintains of blood acidity
E. *Synthesis of urea and creatinine
1526. The indices used to evaluate renal function are
A. Glomerular filtration rate
B. Plasma Creatinine level
C. Plasma Urea level
D. *All of them
E. None of them
1527. The indices used to evaluate renal function are except
A. Glomerular filtration rate
B. Plasma Creatinine level
C. Plasma Urea level
D. Zimnitsky test
E. *Nechiporenko test
1528. Normal levels of Glomerular filtration rate (GFR) are
A. 140-190 ml/min
B. *90-120 ml/min
C. 60-90 ml/min
D. 30-60 ml/min
E. 90-190 ml/min
1529. Processes of ultrafiltration in kidneys are stopped when systemic arterial blood pressure is ;
A. 140 -160 mm Hg
B. 120 -140 mm Hg
C. 90 -100 mm Hg
D. 70 -80 mm Hg
E. *<60 mm Hg
1530. Normal levels of tubular water reabsorbing are
A. *97-99 %
B. 95-96 %
C. 93-94 %
D. 90-92 %
E. 87-90 %
1531. Normal sizes of kidneys in adults are
A. *10-12 сm х 5-6 сm х 3-4 сm
B. 15-16 сm х 7-8 сm х 5-6 сm
C. 7-8 сm х 3-4 сm х 2-3 сm
D. 5-7 сm х 2-4 сm х 1-3 сm
E. 18-22 сm х 8-10 сm х 6-7 сm
1532. What kidney cells have endocrine function?
A. Gumptreht’s cells
B. Juxtaglomerular cells
C. Interstitial cells
D. None of them
E. *All of them
1533. Kidney cells have endocrine function except
A. Gumptreht’s cells
B. Juxtaglomerular cells
C. Interstitial cells
D. *Epithelial cells of distal tubuls
E. All of them
1534. Anomalies of kidneys topic and shape are
A. Kidneys dystopia
B. L-like kinnney
C. C-like kidney
D. *All of them
E. None of them
1535. Anomalies of kidneys topic and shape are except:
A. Kidneys dystopia
B. L-like kidney
C. C-like kidney
D. All of them
E. *Polycystic kidney
1536. Anomalies of kidneys structure are:
A. Kidneys dystopia
B. L-like kinnney
C. C-like kidney
D. All of them
E. *None of them
1537. Anomalies of kidneys structure are:
A. Polycystic kidney
B. Multicystic kidney
C. Megacalix
D. *All of them
E. None of them
1538. What kidney structures are visible on longitudinal incision
A. Cortex
B. Pelvis
C. Cerebral part
D. Calices
E. *All of them
1539. What kidney structures are visible on longitudinal incision?
A. *Cortex, pelvis, cerebral part, calices
B. Cortex, cerebral part, calices
C. Cortex, pelvis, calices
D. Cortex, pelvis, cerebral part
E. Pelvis, cerebral part, calices
1540. The structure of nephron includes:
A. Glomerulus
B. Glomerulus’s capsule
C. Renal tubules
D. *All of them
E. None of them
1541. Complications of nephrotic syndrome that may require treatment include
A. edema,
B. hyperlipidemia,
C. thromboembolism,
D. vitamin D deficiency
E. *all of them.
1542. All of the following may result in the nephrotic syndrome except:
A. diabetes mellitus
B. Hodgkin’s lymphoma
1543. ?Which cellular elements predominate in areas of inflammation of kidney tissue during the late stages
of acute pyelonephritis?
A. *Macrophages, plasma cells, lymphocytes
B. Neutrophils, eosinophils, basophils
C. neutrophils, promyelocytes
D. lymphocytes, basophils
E. reticulocytes, neutrophils
1544. How to determine the degree of proteinuria?
A. Determination of protein in the morning urine
B. Determination of protein in the daily urine
C. Determination of protein in a three-hour portion of urine
D. *Determination of protein in the urine after water loading
E. Determination of protein in the urine after water restriction
1545. Which way of antibiotics administration is prescribed during the initial stage of acute pyelonephritis
treatment?
A. *parenteral
B. Oral
C. Rectal
D. Inhaled
E. Oral and parenteral
1546. Is significant haematuria typical for cystitis?
A. *Yes
B. No
C. There is no right answer
D. All answers are correct
E. B and C
1547. In event of which of the following diseases a positive Pasternatskyi symptom is determined?
A. Chronic glomerulonephritis
B. Renal amyloidosis
C. Hypoplasia of kidneys
D. *Infectious and inflammatory kidney disease
E. Cystitis
1548. Which clinical sign is not typical for acute pyelonephritis?
A. Raising the temperature to febrile digits with a fever
B. Backache
C. dysuric symptom
D. *The presence of hypertension
E. There is no right answer
1549. What does the presence of human renal epithelium cells in urine show?
A. glomerulonephritis
B. *pyelonephritis
C. renal amyloidosis
D. kidney tumour
E. All answers are correct
1550. What changes in the urine are typical for chronic pyelonephritis:
A. Hematuria, cylindruria
B. *Pyuria, cylindruria, proteinuria
C. Cylindruria
D. Hematuria, bakteriyuriya
E. Eritrotsituriya, cylindruria, proteinuria
1551. What are characteristics of oedemas in acute nephritis?
A. Appear in the evening
B. First appear in the lower extremities
C. First appear in the upper extremities
D. *Appear on the face in the morning
E. Anasarca appears at once
1552. What are characteristics of hypertensive form of chronic nephritis?
A. Normal blood pressure and urinary syndrome
B. High blood pressure and urinary syndrome
C. Oedema and urinary syndrome
D. *High blood pressure and swelling
E. Swelling of the whole body (anasarca) and high blood pressure
1553. What is the origin of the modified red blood cells in urine?
A. *Acute nephritis
B. Urolithiasis
C. Acute cystitis
D. Paranefrit
E. Bladder Cancer
1554. During which of diseases named pyuria can take place:
A. *Pyelonephritis
B. Paranefrit
C. Acute nephritis
D. Renal amyloidosis
E. Bladder Cancer
1555. The amount of protein in urine during the urinary syndrome:
A. *to 1 g / day
B. to 3.5 g / day
C. to 4.5 g / day
D. to 5.5 g / day
E. to 9.5 g / day
1556. The main clinical signs of tubular damage are the following, except:
A. Acidosis
B. Glycosuria
C. Hypo-, isosthenuria
D. *arterial hypertension;
E. Cylindruria
1557. Proteinuria is a urinary protein excretion in the amount more than:
A. 10 mg per day
B. 30 mg per day
C. 50 mg per day
D. *100 mg per day
E. 5 mg per day
1558. What is meant by the term "dysuric symptom"?
A. Urination at night
B. Amount of daily urine increasing
C. Amount of daily urine reduction
D. *Hard discomfort feelings and pain when urinating, urinating dysrhythmia
E. Ratio of daytime and nighttime diuresis violation
1559. What was the cause of proteinuria during the nephrotic syndrome?
A. Increased reabsorption of protein in tubules
B. Reduced reabsorption of protein in tubules
C. *Increased filtration of protein in glomerular
D. Reduced filtration of protein in glomerular
E. Increased filtration of red blood cells in glomerular
1560. That prevails in the urine sediment during acute glomerulonephritis?
A. Bacteria
B. Oxalate
C. *Erythrocytes
D. Leukocytes
E. hyaloid cylinders
1561. What is the cause of hypoproteinemia in chronic glomerulonephritis?
A. Hyperlipidemia
B. Arterial hypertension
C. *Proteinuria
D. Oedema
E. Abnormal liver function
1562. Which proteins are mainly lost in patients who suffer from nephrotic syndrome?
A. *Albumins
B. Gamaglobulina
C. Alpha-1-globulins
D. Alpha-2 globulins
E. Betaglobuliny
1563. What is important in the development of oedema during glomerulonephritis?
A. Arterial hypertension
B. Microalbuminuria
C. *Hypoproteinemia
D. Albuminosis
E. Hyperlipidemia
1564. What changes of blood electrolytes are typical for nephrotic syndrome?
A. Increasing of potassium
B. Reduction of potassium
C. *Increasing of sodium
D. Reduced sodium
E. Increased phosphorus
1565. During which kidney disease eclampsia can appear?
A. Acute pyelonephritis
B. Chronic pyelonephritis
C. Renal amyloidosis
D. *Acute glomerulonephritis
E. Myeloid nephropathy
1566. Which manifestations are untypical for renal eclampsia?
A. Intracranial hypertension
B. Attack
C. Spasm of cerebral vessels
D. *High Blood Pressure
E. Pulmonary edema
1567. Which dietary factors can provoke an attack of renal eclampsia?
A. Sweet
B. *Salt
C. Fat
D. Bitter
E. Acute
1568. Which microorganism is responsible for development of acute glomerulonephritis?
A. Colon bacillus
B. Klebsiella
C. *Beta hemolytic streptococcus
D. Aurococcus
E. Proteus vulgaris
1569. Which diseases do not precede the development of acute glomerulonephritis?
A. Scarlet fever
B. Otitis
C. Sinusitis
D. Angina
E. *Laryngitis
1570. Which triad of symptoms is typical for acute glomerulonephritis?
A. Pain, dysuria, leukocyturia
B. erythrocyturia, bacteriuria, swelling
C. Edema, hypertension, hypo- isosthenuria
D. *Hypertension, edema, proteinuria
E. Dysuria, erythrocyturia, oxaluria
1571. Which macroscopic changes of urine can take place during acute glomerulonephritis?
A. Yellow urine
B. *Urine colour "meat slops"
C. Clear urine
D. Bright red urine
E. Pale yellow, opacity
1572. What does not refere to complications of acute glomerulonephritis?
A. Acute heart failure
B. Acute renal failure
C. Hemorrhagic stroke
D. Eclampsia
E. *Acute abdomen
1573. Which disease is transient oliguria typical for?
A. Acute pyelonephritis
B. *Acute glomerulonephritis
C. Initial stage of chronic renal failure
D. Uremia
E. Chronic pyelonephritis
1574. During which desease is selective proteinuria often observed?
A. *Acute glomerulonephritis
B. Chronic glomerulonephritis
C. Renal amyloidosis
D. Diabetic nephrosclerosis
E. Tuberculosis of kidneys
1575. Reberga sample lets determine:
A. Concentrational ability of the kidney
B. *Glomerular filtration
C. Plasma blood flow
D. Transport of electrolytes
E. Secretion of glucose
1576. Which of these symptoms are more common during acute glomerulonephritis with absence of acute
kidney failure?
A. Headache
B. Asthma
C. Oliguria
D. *Increased blood pressure
E. Arthralgia
1577. Name the factors that take part in the pathogenesis of oedema syndrome during acute
glomerulonephritis:
A. *All answers are correct
B. Increased capillary permeability
C. Sodium retention in the body
D. Secondary hyperaldosteronism
E. There is no right answer
1578. The most reliable sign of rapidly progressing glomerulonephritis is:
A. Arterial hypertension
B. Nephrotic syndrome
C. *Manifestations of chronic renal failure increasing
D. Rack gross hematuria
E. Hypercholesterolemia
1579. What play a leading role in the pathogenesis of chronic glomerulonephritis?
A. Autoimmune processes
B. Immune complexes
C. Disturbances in system of renal hemodynamic
D. *All answers are correct
E. There is no right answer
1580. Which of the following symptoms are not typical for nephrotic syndrome?
A. massive proteinuria
B. Edema
C. Gross hematuria
D. Hypercholesterolemia
E. *Bacteriuria
1581. Which of the following factors causes a decrease of glomerular filtration below 80 ml / min?
A. Blood pressure increasing to 170/95 mmHg
B. Blood pressure reduction to 110/65 mmHg
C. Proteinuria higher than 100 mg / day
D. Macrohematuria
E. *Reduction of number of functioning nephrons
1582. Glomeruli and kidneys are typically increased in size in:
A. diabetic nephropathy;
B. renal amyloidosis;
C. polycystic kidney disease;
D. Renal tumor
E. *all of them;
1583. What percentage of kidney stones are radio opaque?
A. *90%
B. 80%
C. 70%
D. 50%
E. 40%
1584. What are three factors in limiting progression of chronic renal failure in Type I diabetics with chronic
renal failure,?
A. ACE inhibitors
B. Dietary protein restriction
C. Careful control of serum glucose level
D. *All of them
E. None of the
1585. What is the most common renal complication of autosomal dominant polycystic kidney disease?
A. gross hematuria,
B. frequent UTI's,
C. *arterial hypertension,
D. nephrolithiasis
E. all of them
1586. Аindings in Hepatorenal syndrome are?
A. Azotemia
B. Hyponatremia
C. Progressive oligouria
D. Hypotension
E. *All of them
1587. What are indications for dialysis in the setting of renal failure?
A. Fluid overload
B. Refractory hyperkalemia
C. BUN > 130
D. Pericardial friction rub
E. *All of them
1588. The metabolic complications of nephrotic syndrome include all of the following except:
A. protein malnutrition;
B. *iron-resistant microcytic hypochromic anemia;
C. Hypocalcemia;
D. hypolipidemia;
E. secondary hyperparathyroidism.
1589. What percentage of patients with polycystic kidney disease develop adenocarcinoma of the kidney?
A. 5 %
B. 10 %
C. 15 %
D. 30%
E. *1.5 %
1590. What percentage of adults by the age of 50 y/o will develop simple or solitary renal cysts?
A. *25-30%
B. 1-5 %
C. 5-10 %
D. 30-50 %
E. 10 -15 %
1591. Is nephrolithiasis more common in men?
A. *Yes, more common 2:1
B. No, more common in female 2:1
C. No, equal in both
D. Yes, more common 3:1
E. None of them
1592. The nephrotic syndrome is a clinical complex characterized by a number of renal and extrarenal
features, the most prominent of which are:
A. proteinuria of >3.5 g (per 24 h);
B. hypoalbuminemia;
C. edema;
D. hyperlipidemia;
E. *all of them.
1593. Select the drug which is given in usual dosages, may be effective in treating urinary tract infections in
patients with acute renal failure. Why do you choose this drug?
A. Gentamicin;
B. Nitrofuratoin;
C. *Cephalexin;
D. Carbenicillin;
E. Chloramphenicol
1594. The Diet of patient with nephrotic syndrome has to exclude
A. Increased content of Salt
B. Increased content of fats
C. Increased content of proteins
D. *All of then
E. None of them
1595. What substances are reabsorbed in renal tubules completely?
A. *Glucose
B. Creatinine
C. Sodium
D. Potassium
E. Chlorum
1596. Endocrine function of kidneys include synthesis of all except
A. Renin
B. Erythropoetin
C. Prostaglandins
D. *Aldosterone
E. Calcitriol
1597. Endocrine function of kidneys include synthesis of all except
A. Renin
B. Erythropoetin
C. Prostaglandins
D. *Insulin
E. Calcitriol
1598. Endocrine function of kidneys include synthesis of all except
A. Renin
B. Erythropoetin
C. Prostaglandins
D. *Cortizol
E. Calcitriol
1599. What substance indicates the disturbed excretory function of kidneys best of all:
A. *Creatinine
B. Urea
C. Uric acid
D. Potassium
E. Sodium
1600. Decreased GFR (Glomerular filtration rate) will be in a case of^
A. Acute pyelonephritis
B. Acute glomerulonephritis
C. Renal amiloidosis
D. *Acute renal failure
E. Nephrolithiasis
1601. Decreased GFR (Glomerular filtration rate) will be in a case of^
A. Acute pyelonephritis
B. Acute glomerulonephritis
C. Renal amiloidosis
D. *Chronic renal failure
E. Nephrolithiasis
1602. The best method to evaluate renal function is to measure
A. *Glomerular filtration rate
B. Creatinine
C. Urea
D. Uric acid
E. Potassium
1603. Glomerular filtration rate (GFR) can be calculated by means of^
A. Cockroft-Gault formula
B. MDRD formula
C. Inulin test
D. *All of them
E. None of them
1604. Glomerular filtration rate (GFR) can be calculated by means of:
A. Cockroft-Gault formula
B. MDRD formula
C. Clearance of endogenic creatinine test
D. *All of them
E. None of the
1605. Indication to renal biopsy are:
A. *Differential diagnosis of kidney disease
B. Suspicion for renal tumor
C. Suspicion for renal tuberculosis
D. Oedema
E. Suspicion for renal abscess
1606. Indication to renal biopsy are:
A. *Nephrotic syndrome
B. Suspicion for renal tumor
C. Suspicion for renal tuberculosis
D. Oedema
E. Suspicion for renal abscess
1607. Indication to renal biopsy are:
A. *Changes in urine analysis of unknown etiology
B. Suspicion for renal tumor
C. Suspicion for renal tuberculosis
D. Oedema
E. Suspicion for renal abscess
1608. The purpose of renal biopsy is to evaluate
A. Renal function
B. Stage of renal failure
C. *Diagnosis and optimal treatment
D. All of them
E. None of them
1609. Absolute contraindication to renal biopsy are:
A. Anemia
B. Arterial hypertension
C. Hematuria
D. Oedema
E. *Single kidney
1610. Absolute contraindication to renal biopsy are:
A. Anemia
B. Arterial hypertension
C. Hematuria
D. Edema
E. *Polycystic kidney
1611. Absolute contraindication to renal biopsy are:
A. Anemia
B. Arterial hypertension
C. Hematuria
D. Edema
E. *Hydronephrosis
1612. Complications of renal biopsy may be
A. Hepatitis B
B. Hepatitis C
C. HIV
D. Arterial hypertension
E. *Pain syndrome
1613. Complications of renal biopsy may be
A. Hepatitis B
B. Hepatitis C
C. HIV
D. Arterial hypertension
E. *Arterio-venous fistula
1614. Complications of renal biopsy may be
A. Hepatitis B
B. Hepatitis C
C. HIV
D. Arterial hypertension
E. *Hypertermic sundrome
1615. Complications of renal biopsy may be
A. Hepatitis B
B. Hepatitis C
C. HIV
D. Arterial hypertension
E. *Collapse
1616. Complications of renal biopsy may be
A. Hepatitis B
B. Hepatitis C
C. HIV
D. Arterial hypertension
E. *Temporary anuria
1617. Complications of renal biopsy are except:
A. Collapse
B. Hypertermic syndrome
C. Temporary anuria
D. Pain syndrome
E. *Hepatitis C
1618. Complications of renal biopsy are except:
A. Collapse
B. Hypertermic syndrome
C. Temporary anuria
D. Pain syndrome
E. *Hepatitis B
1619. Complications of renal biopsy are except:
A. Collapse
B. Hypertermic syndrome
C. Temporary anuria
D. Pain syndrome
E. *HIV
1620. Complications of renal biopsy are except:
A. Collapse
B. Hypertermic syndrome
C. Temporary anuria
D. Pain syndrome
E. *Arterial hypertension
1621. Risk factors of pyelonephritis are except
A. Diabetes malliyus
B. Pregnancy
C. Diturbancres of urodynamics
D. Refluxes
E. *Arterial hypertension
1622. Risk factors of pyelonephritis are except
A. Diabetes malliyus
B. Pregnancy
C. Diturbancres of urodynamics
D. Refluxes
E. *Heart failure
1623. Risk factors of pyelonephritis are
A. Diabetes malliyus
B. Pregnancy
C. Diturbancres of urodynamics
D. Refluxes
E. *All of them
1624. The symptoms of acute pyelonephritis are
A. Intoxication
B. Low back pain
C. Dysuria
D. Leucocyturia
E. *All of them
1625. The symptoms of acute pyelonephritis are except
A. Intoxication
B. Low back pain
C. Dysuria
D. Leucocyturia
E. *Arterial hypertension
1626. The symptoms of acute pyelonephritis are except
A. Intoxication
B. Low back pain
C. Dysuria
D. Leukocyturia
E. *Renal osteodystrophy
1627. Complications od acute pyelonephritis are except
A. Perinephritis
B. Renal abscess
C. Papillary necrosis
D. Urosepsis
E. *Xanthogranulomatosis pyelonephritis
1628. Drug of choice for patient with rheumatoid artritis
A. *Methotrexate
B. NSAIDs
C. Hydroxychloroquine sulphate
D. Sulphasalazine
E. D-penicillamine
1629. The best drug to treat SLE is :
A. Asprin
B. Indomethacin
C. Prostoglandines
D. Procainamide
E. *Steroids
1630. Drug-induced systemic lupus erythematosus (SLE) can be characterized by all of the following
statements EXCEPT?
A. Twenty percent of patients receiving procainamide develop drug-induced lupus
B. *Most patients on hydralazine develop a positive antinuclear antibody (ANA) test; however, only 10
percent suffer from lupuslike symptoms
C. If patients with drug-induced lupus fail to respond within several weeks of discontinuing the offending
agent, a trial of corticosteroids is indicated
D. If a patient with drug-induced lupus has persistent symptoms for longer than 6 months, an anti-ds
antibody
E. None of the above
1631. For the Systemic lupus erythematosus choose the most likely diagnostic antibody from the below list
of options.
A. Antimicrosomal antibody
B. C-ANCA
C. *Anti-dsDNA
D. Antiparietal cell antibody
E. Antiacetylcholine receptor antibody
1632. In which of the following diseases test for anticentromere antibody is positive?
A. *Scleroderma
B. Mixed connective tissue disorder
C. SLE
D. Rheumatoid arthritis
E. Polymyalgia rheumatica
1633. Supporting evidence of a recent group A streptococcal infection may be all of the following except:
A. *recent malar rash
B. elevated or increasing streptococcal antibody test - antistreptolysin- O (ASO)
C. positive throat culture
D. recent scarlatine
E. all mentioned above
1634. A 39-year-old male developed acute, severe pain and swelling of the left elbow. Within next few
hours, pain worsened and he was unable to move the elbow joint, which was tender, erythematous, and
swollen on examination. Choose the most likely diagnosis from the below list of options.
A. Ankylosing spondylitis
B. Erythema nodosum
C. Osteoarthritis + synoviitis
D. *Gouty arthritis
E. Psoriatic arthritis
1635. For the Systemic lupus erythematosus choose the most likely diagnostic antibody from the below list
of options.
A. Antimicrosomal antibody
B. C-ANCA
C. *Antinuclea antibody
D. Antiparietal cell antibody
E. Antiacetylcholine receptor antibody
1636. A 27-yr-old woman presents with a 3-week history of fever, pleuritic chest pain, stiffness and
swelling in the wrists, MCP joints and PIP joints. On examination, there is bilateral pretibial oedema.
Choose the most likely treatment from the below list of options.
A. Asprin
B. Indomethacin
C. Prostoglandines
D. Procainamide
E. *Steroids
1637. A 52-yr-old woman complains of a 8-month history of Raynaud's phenomenon, progressive skin
tightness, thickening of fingers and hands, dyspnoea on exertion and dysphagia. Choose the most likely
diagnosis from the below list of options.
A. Rheumatoid arthritis
B. Polymyalgia rheumatica
C. SLE
D. Polyarteritis nodosa
E. *Scleroderma
1638. A 21-year-old woman is referred to you for evaluation of new-onset Raynaud’s phenomenon. She has
no other symptoms. Her physical examination is unremarkable except for dilated nail-fold capillaries and a
few scattered telangiectasias. Tests for specific ANA reveal the presence of antibodies to topoisomerase I.
The most likely eventual diagnosis for this patient is which of the following?
A. Isolated Raynaud’s disease
B. *Diffuse systemic sclerosis
C. Limited systemic sclerosis
D. Mixed connective tissue disease (MCTD)
E. SLE
1639. A 25-year-old woman is referred to you for evaluation of new-onset Raynaud’s phenomenon. She has
no other symptoms. Her physical examination is unremarkable except for dilated nail-fold capillaries and a
few scattered telangiectasias. Tests for specific ANA reveal the presence of antibodies to topoisomerase
I.Choose the most likely treatment from the below list of options.
A. *D-penicillamine
B. Methotrexate
C. Indomethacin
D. Aspirin
E. Nifidipine
1640. The bad prognostic indicator for scleroderma is:
A. *Renal involvement
B. Goose skin appearance
C. Oedema legs
D. Widening of pulse pressure
E. Calcification
1641. Which of the following is a typical immune complex disease?
A. Osler-Weber-Rendu syndrome
B. Urticaria pigmentosa
C. *Lupus erythematosus
D. Gastric ulcer
E. Pneumonia
1642. Which of the following statements regarding the use of cyclophosphamide in the treatment of
rheumatic disorders such as lupus nephritis and vasculitis is correct?
A. The dose of cyclophosphamide should be increased until substantial leukopenia is observed.
B. The absorption of oral cyclophosphamide is poor, so the intravenous route is preferred.
C. Oral cyclophosphamide should be administered at night and fluids restricted in order to decrease the
formation of urine.
D. *Regular monitoring of the patient’s complete blood cell count and urinalysis is required.
E. None of the above
1643. A 42-yr-old woman complains of a 4-month history of Raynaud's phenomenon, progressive skin
tightness, thickening of fingers and hands, dyspnoea on exertion and dysphagia. Choose the most likely
treatment from the below list of options.
A. *D-penicillamine
B. Methotrexate
C. Indomethacin
D. Aspirin
E. Nifidipine
1644. Woman of 35 years with Raynauds phenomenon, polyarthritis, dysphagia of 5 years and mild
sclerodactyly, blood showing Anticentromere antibody positive, the likely cause is:
A. *Scleroderma
B. Mixed connective tissue disorder
C. SLE
D. Rheumatoid arthritis
E. Polymyalgia rheumatica
1645. A 29-yr-old woman presented with DVT of the left calf. She had a history of recurrent abortions and
arthritis. Choose the most likely diagnosis from the below list of options.
A. Protein C deficiency
B. Thrombotic thrombocytopenia
C. *Antiphospholipid syndrome
D. SLE
E. Polycythemia rubra vera
1646. A 25-yr-old woman presents with deep venous thrombosis in the right leg. Her past history includes
three miscarriages. Her blood tests show mild thrombocytopenia and a positive serology test for syphilis.
Choose the most likely diagnosis from the below list of options.
A. Takayasu's arteritis
B. Polymyalgia rheumatica
C. Mixed connective tissue disorder
D. *Antiphospholipid syndrome
E. Polyarteritis nodosa
1647. Which of the following is an initiation for institution of therapy with hydroxychloroquine in a patient
with systemic lupus erythematosus (SLE)?
A. Thrombocytopenia
B. *Subacute cutaneous lupus
C. Central nervous system vasculitis
D. Diffuse proliferative glomerulonephritis (DPGN)
E. Gangrene of digits
1648. Which of the following statements regarding cyclophosphamide therapy for life-threatening lupus,
including severe lupus nephritis, is true?
A. Cyclophosphamide is approved for use by the Food and Drug Administration (FDA) as therapy for
severe lupus nephritis.
B. The standard dose for cyclophosphamide intermittent intravenous therapy (0.5-1.0 g/m2 of body
surface area) has been shown to be more efficacious than lower doses.
C. *The standard protocol for intravenous use (once a month times 6, then quarterly for 2 more years) has
been shown to reduce posttreatment disease flares compared to once a month times six doses.
D. Hematopoietic ablation with very high doses of cyclophosphamide is superior to the standard regimen
in efficacy.
E. Hemorrhagic cystitis does not occur with intravenous dosage.
1649. Which of the following APhS symptoms predictably respond(s) to treatment with heparin?
A. Livedo reticularis
B. Cardiac valve disease
C. *Recurrent fetal loss
D. Cognitive dysfunction
E. Renal microangiopathy
1650. A 23-year-old woman who is 27 weeks pregnant is admitted to hospital with proteinuria, a “purple
cold toe,” and painful swollen metacarpophalangeal (MCP) joints, and wrists. She was diagnosed with
systemic lupus erythematosus (SLE) 1 year previously after miscarrying her first pregnancy. Which of the
following statements is/are correct?
A. She should stop all her medications because of the risks to her fetus.
B. Corticosteroid therapy should not be given because it will have teratogenic effects on the fetus.
C. *If antiphospholipid antibodies are positive, she should be placed on anticoagulants.
D. Her infant should be delivered immediately.
E. None of the above
1651. Which of the following is one of the earliest predictors of whether a patient with Raynaud’s
phenomenon will go on to develop true scleroderma?
A. Constipation
B. Decreased vital capacity
C. Thickening of the forearm skin
D. *Dilated and dropout capillaries at the nail base of fingers
E. None of the above
1652. Which of the following autoantibodies is more likely to be found in patients with systemic sclerosis?
A. anti-Smith (anti-Sm)
B. Anti-ribonucleoprotein (RNP)
C. *Anti-DNA topoisomerase 1 (anti-Scl70)
D. Anti-citrullinated peptide
E. Anticentromere
1653. Vegetations on undersurface of A.V. valves are found in:
A. Acute Rheumatic corditis
B. *Limban Sack's endocarditis
C. Non thrombotic bacterial endocarditis
D. Chronic rheumatic carditis
E. MVP
1654. Drug-induced systemic lupus erythematosus (SLE) can be characterized by which of the following
statements?
A. Twenty percent of patients receiving procainamide develop drug-induced lupus
B. *Nephritis is a frequent consequence of hydralazine-induced lupus
C. If patients with drug-induced lupus fail to respond within several weeks of discontinuing the offending
agent, a trial of corticosteroids is indicated
D. If a patient with drug-induced lupus has persistent symptoms for longer than 6 months, an anti-ds
antibody and CH50 levels should be drawn
E. None of the above
1655. Anti-double stranded DNA is highly specific for
A. Systemic sclerosis
B. *SLE.
C. Polymyositis
D. Dermatopolymyositis
E. Henoch schonlein purpura
1656. A 35 year old lady complains dysphagia, Raynaud's phenomenon, sclerodactyly. Investigations show
anticentromere antinuclear antibody. The likely diagnosis is:
A. Systemic lupus erythematosis
B. *Systemic sclerosis
C. Mixed connective tissue disorder
D. Rheumatoid arthritis
E. Dermatopolymyositis
1657. 33-year-old patient, presents with complaints of tightness of fingers. There is also history of
dysphagia. Choose the most likely results of the investigations from the below list of options.
A. *Anticentromere antibody
B. Anti dsDNA antibodies
C. ANCA
D. Antimitochondrial antibody
E. Smooth muscle antibody
1658. A 41-year-old woman complains of weakness, fatigue, fever up to 38°C, rash on the face skin, pain in
the wrists and the elbows. On physical exam, erythematous rash on the cheeks with “butterfly” appearance,
the wrists and elbow joints are involved symmetrically, swollen, tender on motions, on urinalysis:
proteinuria, leukocyturia, casts. What is the main mechanism of disease development?
A. Production of antibodies to myocytes.
B. *Production of antibodies to double-stranded DNA.
C. Production of antibodies to endothelial cells.
D. Production of antibodies to myosin.
E. Production of antimitochondrial antibodies.
1659. A 39-year-old woman presents with induration and atrophy of the fingertips and is diagnosed as
having systemic sclerosis (scleroderma). Visceral involvement may lead to all of the following EXCEPT
A. Esophageal hypomotility
B. *obstructive lung disease
C. right-sided heart failure
D. malabsorbtion
E. renovascular hypertension
1660. A 30-year-old woman was admitted to the rheumatological department complaining of malaise,
general weakness, fever, painfull joints (ankles, knees), edema on legs, dyspnea on excursion, palpitation,
rash across the face, which occurred after sunbathe, hair losing. Physical examination revels malar rash,
enlargement of heart, tachycardia, edema on legs. The most likely diagnosis is
A. Dermatomyositis
B. Progressive muscule dystrophy
C. *Systemic lupus erythematosus
D. Myasthenia,
E. Polymyalgia rheumatica,
1661. A 39-year-old woman presents with induration and atrophy of the fingertips and is diagnosed as
having systemic sclerosis (scleroderma). Which of the following drugs you will prescribe?
A. *D-penicillamine
B. Ampicillin
C. Allopurinol
D. Nitrates
E. digoxin
1662. A 50-year-old woman has had Raynaud’s phenomenon of the hands for 15 years. The condition has
become worse during the last year, and she has developed arthralgia and arthritis involving the hands and
wrists as well as mild sclerodactyly and difficulty swallowing solid foods. Anticentromere antibodies are
present in high titers: antiribonucleoprotein antibodies are not detectable. The most likely diagnosis of this
woman’s disorder is
A. *systemic sclerosis
B. mixed connective-tissue disease
C. overlap syndrome
D. dermatomyositis
E. Systemic lupus erythematosus
1663. A 32-year-old woman was admitted to the rheumatological department complaining of malaise,
general weakness, fever, painfull joints (ankles, knees), edema on legs, dyspnea on excursion, palpitation,
rash across the face, which occurred after sunbathe, hair losing. Physical examination revels malar rash,
enlargement of heart, tachycardia, edema on legs. All of the following drugs you can prescribe (may be
simultaneously) EXCEPT
A. Glucocorticoids
B. Methotrexate
C. *Procainamide
D. Diclofenac
E. Azathioprine
1664. A 18-year-old woman was admitted to the rheumatological department complaining of malaise,
general weakness, fever, dyspnea on excursion. Physical examination revels malar rash, oral ulcer, signs of
pneumonitis, laboratory test revels severe anemia, leucopenia thrombocytopenia, ESR -53 mm/h, urine
analysis – protein 0,33, L- 5-8 in the field of vision, Er. – 8- 10 in the field of vision. Which of the following
drugs you prescribe?
A. *Glucocorticoids
B. Methotrexate
C. amoxacicllin
D. cyclophosphanum
E. chlorambucil
1665. A 18-year-old woman was admitted to the rheumatological department complaining of malaise,
general weakness, fever, dyspnea on excursion. Physical examination revels malar rash, oral ulcer, signs of
pneumonitis, laboratory test revels anemia, leucopenia thrombocytopenia. All of the following criteria are
useful for diagnosis EXCEPT
A. Antinuclear antibodies
B. anti-phospholipid antibodies
C. Proteinuria > 0.5 g/d
D. lymphocytopenia
E. *The heliotrope rash
1666. A 39-year-old woman presents with induration and atrophy of the fingertips and is diagnosed as
having systemic sclerosis (scleroderma). All of the following criteria are true for diagnosis EXCEPT
A. Digital pitting scars
B. Bibasilar pulmonary fibrosis
C. *Heart failure
D. Proximal scleroderma
E. Sclerodactyly
1667. A 35-year-old woman was admitted to the rheumatological department complaining of malaise,
general weakness, fever, arthritis, edema on legs, dyspnea on excursion, palpitation, rash across the face,
which occurred after sunbathe, hair losing. Physical examination revels malar rash, enlargement of heart,
tachycardia, edema on legs. All of the following drugs you can prescribe (may be simultaneously) EXCEPT
A. Glucocorticoids
B. Methotrexate
C. *Gold
D. Diclofenac
E. Azathioprine
1668. A 22-year-old woman was admitted to the rheumatological department complaining of malaise,
general weakness, fever, dyspnea on excursion. Physical examination revels malar rash, oral ulcers, signs of
pneumonitis, laboratory test revels anemia, leucopenia thrombocytopenia, ESR -35 mm/h, urine analysis –
protein 0,33, L- 5-8 in the field of vision, Er. – 8- 10 in the field of vision. All of the following criteria are
useful for diagnosis EXCEPT
A. Antinuclear antibodies
B. anti-phospholipid antibodies
C. Proteinuria > 0.5 g/d
D. *lymphocytosis
E. anaemia
1669. A 27-year-old woman was admitted to the rheumatological department complaining of malaise,
general weakness, fever, dyspnea on excursion. Physical examination revels malar rash, oral ulcers, signs of
pneumonitis, laboratory test revels anemia, leucopenia thrombocytopenia, ESR -35 mm/h, urine analysis –
protein 0,33, L- 5-8 in the field of vision, Er. – 8- 10 in the field of vision. All of the following criteria are
useful for diagnosis
A. Antimicrosomal antibody
B. Antireticulin antibody
C. *Anti-dsDNA
D. P-ANCA antibody
E. Rheumatoid factor
1670. Serologic tests should be performed in patients with specific musculoskeletal complaints for
assessing all the following EXCEPT:
A. rheumatoid factor
B. antinuclear antibodies (ANA)
C. uric acid level
D. *GGT
E. antistreptolysin O (ASO) titer
1671. 37-year-old patient, presents with induration and atrophy of the fingertips, dysphagia, Raynaud's
phenomenon. Choose the most likely results of the investigations from the below list of options.
A. *Anticentromere antibody
B. Anti dsDNA antibodies
C. ANCA
D. Antimitochondrial antibody
E. Smooth muscle antibody
1672. Serological findings in patients with systemic lupus erythematosus (SLE) will NOT include which
one of the following:
A. A raised serum IgG level
B. Antibodies to double-stranded DNA
C. Decreased C3 and C4 levels
D. A positive ANA
E. *Antibodies to cyclic citrullinated peptide (anti-CCP antibodies)
1673. Which one of the following statements about autoimmune diseases is true
A. Affect about 1 person in every 1000
B. *Are more common in women
C. Tend to begin in early childhood
D. Are an inevitable consequence of autoimmune responses
E. Are usually mediated by type I hypersensitivity
1674. A 32-year-old woman is evaluated because of a 5-year history of Raynaud’s phenomenon and
thickened skin over the fingers, small patches of calcinosis on the distal fingers and dorsum of one hand. She
reports mild dyspnea when she climbs stairs. What changes reveals chest X-ray?
A. Bronchiolitis with organizing pneumonia
B. *Bibasal pneumofibrosis
C. Lung cancer
D. Emphysema
E. Pulmonary tromboembolism
1675. Which of the following clinical features is NOT commonly found in limited systemic sclerosis?
A. Raynaud's phenomenon
B. Oesophageal dysfunction
C. *Eczema
D. Sclerodactyly
E. Telangiectasia
1676. Which one of the following statements about systemic connective tissue diseases is not true?
A. Can cluster within families
B. Are often associated with particular HLA alleles
C. *Are usually mediated by type I hypersensitivity
D. May occur in humans with inherited defects in apoptosis
E. Are more common in women
1677. Classification of Systemic Sclerosis includes all of the following EXCEPT
A. Limited cutaneous scleroderma
B. Diffuse cutaneous scleroderma
C. Scleroderma sine scleroderma
D. *Eczema
E. Overlap syndromes
1678. Which of the following occurs rarely in Limited cutaneous scleroderma
A. Skin involvement
B. Raynaud phenomenon
C. Dilated nailfold capillary loops
D. *Renal disease
E. Anticentromere antibody
1679. Which of the following doesn’t occur in Diffuse cutaneous scleroderma
A. *Vegetations on undersurface of A.V. valves
B. Raynaud phenomenon
C. Truncal and acral skin involvement
D. Nailfold capillary dilation
E. interstitial lung
1680. Cardiac involvement in SSC are all of the following EXCEPT
A. Pericardial effusion
B. Cor pulmonale
C. Conduction abnormalities
D. *Vegetations on undersurface of A.V. valves
E. arrhythmias
1681. 52 year old male with chronic joint pain. There is asymmetric soft tissue swelling, predominantly in
the DIP of the right first digit, with calcifications. Further questioning revealed a history of repeated attacks
of acute joint pain. There is bony erosion at multiple DIP's of the right hand that is periarticular, well
corticated and has a thin overhanging edge. There is no associated osteopenia. Choose the most likely
diagnosis from the below list of options.
A. Ankylosing spondylitis
B. Erythema nodosum
C. Osteoarthritis
D. *Chronic gouty arthritis
E. Psoriatic arthritis
1682. 52 yo male with chronic joint pain. There is asymmetric soft tissue swelling, predominantly in the
DIP of the right first digit, with calcifications. Further questioning revealed a history of repeated attacks of
acute joint pain. Which of the following medications is indicated for the treatment of chronic arthritis?
A. colchicine
B. *allopurinol
C. corticosteroids
D. indomethacin
E. ibuprofen
1683. 55 years old, came to a musculoskeletal specialist seeking advice for a 3-year history of progressively
worsening pain in both knees. Her knees were stiff for about 20 minutes when she arose in the morning and
for a few minutes after getting up from a chair during the day Her symptoms were worse on humid or cold
days, and she occasionally felt as if one of her knees would give out. Choose the most likely diagnosis from
the below list of options.
A. Ankylosing spondylitis
B. Erythema nodosum
C. *Osteoarthritis
D. Gout
E. Psoriatic arthritis
1684. 55 yo male with swelling and pain of his fingers. Patient also with similar complaints regarding his
great toe of the left foot. Soft tissue swelling with subtle soft tissue nodules primarily of the 2nd and 3rd
digits. Choose the most likely diagnosis from the below list of options.
A. Ankylosing spondylitis
B. *Chronic Gouty arthritis
C. Rheumatoid arthritis
D. Osteoarthritis
E. Psoriatic arthritis
1685. 57 yo male with swelling and pain of his fingers. Patient also with similar complaints regarding his
great toe of the left foot. Soft tissue swelling with subtle soft tissue nodules primarily of the 2nd and 3rd
digits. Erosions with overhanging edges at the distal aspects of the proximal phalanges of the 2nd and 3rd
digits and metatarsophalangeal joints. Which of the following medications is indicated for the treatment of
chronic arthritis?
A. colchicine
B. corticosteroids
C. indomethacin
D. *allopurinol
E. ibuprofen
1686. 63 yo male with swelling and pain of his fingers. Patient also with similar complaints regarding his
great toe of the left foot. Soft tissue swelling with subtle soft tissue nodules primarily of the 2nd and 3rd
digits. Erosions with overhanging edges at the distal aspects of the proximal phalanges of the 2nd and 3rd
digits and metatarsophalangeal joints. Which of the following medications is contraindicated for the
treatment of exacerbation of arthritis
A. corticosteroids
B. colchicine
C. indomethacin
D. ibuprofen
E. *allopurinol
1687. 55 years old, came to a musculoskeletal specialist seeking advice for a 3-year history of progressively
worsening pain in both knees. Her knees were stiff for about 20 minutes when she arose in the morning and
for a few minutes after getting up from a chair during the day. She had difficulty walking > 30 minutes
because of pain, and her symptoms were exacerbated by kneeling, squatting, or descending stairs. Although
sitting, resting, and reclining relieved her pain, she became stiff if she stayed in one position for too long.
Which of the following changes will reveal X-ray examination?
A. Erosions
B. Subluxations
C. *Osteophytes
D. Osteoporosis
E. Ankylosis
1688. 59 yo female with swelling and pain of his fingers. Patient also with similar complaints regarding his
great toe of the left foot. Soft tissue swelling with subtle soft tissue nodules primarily of the 2nd and 3rd
digits. Erosions with overhanging edges at the distal aspects of the proximal phalanges of the 2nd and 3rd
digits and metatarsophalangeal joints. Which of the following medications is indicated for the treatment of
exacerbation of arthritis?
A. allopurinol
B. corticosteroids
C. *colchicine
D. indomethacin
E. ibuprofen
1689. 70-year-old, married, teacher. She has been under treatment for hypertension and is finally stable on
medication. Initial Chief complaint: painful right knee, arthritis in right hip. She had had arthritic pain in her
right hip confirmed by x-ray about 10 years ago. The knee pain is a newer problem and she suspects it is also
arthritis. There is slight crepitus with motion of the right knee. Which of the following treatment do use?
A. Methotrexat
B. Colchicines
C. *Chondroitin sulfate
D. Tetracycline
E. Allopurinol
1690. 70-year-old teacher. She has been under treatment for hypertension and is finally stable on
medication. Initial Chief complaint: painful right knee, arthritis in right hip. She had had arthritic pain in her
right hip confirmed by x-ray about 10 years ago. The knee pain is a newer problem and she suspects it is also
arthritis. There is slight crepitus with motion of the right knee. Which of the following changes will reveal
X-ray examination?
A. Erosions
B. Subluxations
C. *Osteophytes
D. Osteoporosis
E. Ankylosis
1691. A 38-year-old man with a 13-yr history of back pain, presented with acute pain and swelling of one
knee. On examination, the joint was tender and restricted in movement. The knee effusion was aspirated; the
fluid contained a polymorphonuclear leucocytosis but no organisms or rheumatoid factor. Fifteen months
later he developed an iritis in his left eye, low back pain and stiffness. He has peripheral joints arthritis but
pain could be elicited in both sacroiliac joints. X-rays of his spine is shown in the picture. Choose the
medications for patient with shown changes from the below list of options.
A. *Methotrexate
B. Actovegin
C. Mydocalm
D. Warfarin
E. Pentoxifylline
1692. A 38-year-old man with a 13-yr history of back pain, presented with acute pain and swelling of one
knee. On examination, the joint was tender and restricted in movement. X-ray of the knee showed
periarticular osteoporosis. Fifteen months later he developed an iritis in his left eye, low back pain and
stiffness. His peripheral joints were normal but pain could be elicited in both sacroiliac joints. Choose the
most likely diagnosis from the below list of options for patient with shown changes
A. *Ankylosing spondylitis
B. Reactive arthritis
C. Osteoarthritis
D. Gout
E. Psoriatic arthritis
1693. A 50-year-old male is evaluated because of pain in the right inguinal area, lower back, and hands.
The inguinal pain worsens as he walks, and all pain increases as the day progresses. On physical
examination, internal rotation of the right hip elicits the groin pain. The second, third, and fourth
metacarpophalangeal joints are swollen, warm, and tender to pressure. Radiographs show severe
osteoarthritis in the right hip. Which of the following is the best test to confirm the diagnosis?
A. Rheumatoid factor
B. HLA-B27
C. *Serum iron and iron-binding capacity
D. Fasting plasma glucose
E. Serum uric acid
1694. A 55-year-old man presented to the clinic with a nodule on his elbow. He reported that the nodule
had been bothering him for 2 years and had steadily increased in size recently. Further questioning revealed
a history of repeated attacks of acute joint pain. The examination was also notable for a subcutaneous nodule
at the left metatarsal-phalangeal joint and left metacarpal-phalangeal joint, as well as evidence of arthritis
involving both hands. Which of the following is characteristic of the arthritis?
A. *Depositions of monosodium urate monohydrate crystals within the periarticular soft tissues
B. Overgrowth of bones within joints
C. Autoimmune synovitis
D. Osteoporosis
E. Rheumatoid nodules over joints
1695. A 64-year-old woman presents with a 1-year history of pain in her thumbs. She also has nontender
bony overgrowth at the distal interphalangeal joints. The patient says that her mother had the same fingers
and she worries that she will become crippled. Choose the most likely diagnosis from the below list of
options.
A. Ankylosing spondylitis
B. Rheumatoid arthritis
C. *Osteoarthritis
D. Gout
E. Psoriatic arthritis
1696. A 68-year-old man is evaluated because of a 3-year history of bilateral knee pain and low back pain.
He has some stiffness for approximately 15 minutes when she awakens in the morning, and during the
afternoon her pain is worse. On physical examination, he has slight swelling and tenderness to pressure of
the distal interphalangeal joints 2-5 on both hands. There is slight crepitus with motion of the right knee.
Which of the following treatment do use?
A. Methotrexat
B. *Glucosamine
C. Colchicines
D. Tetracycline
E. Allopurinol
1697. A 38-year-old man with a 13-yr history of back pain, presented with acute pain and swelling of one
knee. On examination, the joint was tender and restricted in movement. Later he developed an iritis in his
left eye, low back pain and stiffness. His peripheral joints were normal but pain could be elicited in both
sacroiliac joints. X-rays of his spine is shown in the picture. Which of the following medications isn’t
indicated for the treatment of the disease?
A. *Pentoxifylline
B. Cyclosporine
C. Methotrexate
D. Sulfasalazine
E. Corticosteroids
1698. A 57-year-old man presented to the clinic with a nodule on his elbow. He reported that the nodule
had been bothering him for 2 years and had steadily increased in size recently. Further questioning revealed
a history of repeated attacks of acute joint pain. The examination was also notable for a subcutaneous nodule
at the left metatarsal-phalangeal joint and left metacarpal-phalangeal joint, as well as evidence of arthritis
involving both hands. Which of the following changes will reveal X-ray examination?
A. Erosions
B. Subluxations
C. Osteophytes
D. Osteoporosis
E. *"punched-out" lytic lesion
1699. A 58-year-old man presented to the clinic with a nodule on his elbow. He reported that the nodule
had been bothering him for 4 years and had steadily increased in size recently. Further questioning revealed
a history of repeated attacks of acute joint pain. The examination was also notable for a subcutaneous nodule
at the left metatarsal-phalangeal joint and left metacarpal-phalangeal joint, as well as evidence of arthritis
involving both hands. Which of the following medications is indicated for the treatment of chronic arthritis?
A. *allopurinol
B. colchicine
C. corticosteroids
D. indomethacin
E. ibuprofen
1700. A 60-year-old woman is evaluated because of a 1-year history of bilateral knee pain and low back
pain. She has some stiffness for approximately 15 minutes when she awakens in the morning, and during the
afternoon her pain is worse. On physical examination, she has slight swelling and tenderness to pressure of
the distal interphalangeal joints 2-5 on both hands. All the joints are brought through full range of motion
without pain. There is slight crepitus with motion of the right knee. Which of the following treatment do
use?
A. methotrexat
B. *intra-articular injection of sodium hyaluronate
C. probenecid
D. colchicine
E. allopurinol
1701. A 64-year-old woman presents with a 1-year history of pain in her thumbs. A focused examination
reveals squaring at the base of both first digits, worse on the right, and pain on pressure over the first carpal
metacarpal joints. She also has nontender bony overgrowth at the distal interphalangeal joints. On physical
examination, there is crepitus on passive motion of the left knee, which has a slight varus deformity. Her
feet turn out slightly, and the heels are in valgus. Which of the following changes will reveal X-ray
examination?
A. Erosions
B. Subluxations
C. *Osteophytes
D. Osteoporosis
E. Ankylosis
1702. A 68-year-old woman has had four or five episodes of joint pain and swelling, lasting 3 to 8 days,
involving the right knee and left elbow. Her most recent episode was 4 months ago. On physical
examination, none of her joints is swollen or tender, but there is marked crepitus on extension of the knee.
She also has a positive bulge sign over the left knee and pain on full extension of the left elbow. Which one
of the following tests would confirm the diagnosis?
A. *Arthrocentesis of the knee and laboratory analysis of the synovial fluid
B. Measurement of serum uric acid
C. Measurement of serum rheumatoid factor
D. Radiograph of the knee
E. MRI of the knee with gadolinium contrast
1703. A 72-year-old retired lawyer is evaluated because of swelling of his right knee. He has mild
osteoarthritis and stays active swimming and playing tennis and golf. On physical examination, he has an
effusion in the right knee. At a point in the mid-thigh (measured from the superior border of the patella), the
right thigh is 4 cm smaller in diameter than the left. There is crepitus with right knee flexion. The knee is not
warm. Which of the following is the best next step in the management of this patient?
A. Order radiographs of both knees and treat the patient with ibuprofen.
B. Order radiographs of the right hip and right knee.
C. *Aspirate the joint fluid in the right knee and refer the patient for physical therapy.
D. Aspirate the joint fluid in the right knee and obtain blood cultures.
E. Treat the patient with celecoxib, and advise him to stay off his feet for 48 hours.
1704. On the third day of hospitalization, he developed acute, severe pain and swelling of the left elbow.
Within next few hours, pain worsened and he was unable to move the elbow joint, which was tender,
erythematous, and swollen on examination. Never investigated in the past, we also noted a firm 4 - 6 cm
mass on each elbow, and another one surrounding the proximal inter-phalangeal joint of right middle finger.
When asked particularly, he noticed similar episodes of arthritis of great toe in the past. Choose the most
likely diagnosis from the below list of options.
A. Ankylosing spondylitis
B. Erythema nodosum
C. Osteoarthritis + synovitis
D. *Exacerbation of chronic gouty arthritis
E. Psoriatic arthritis
1705. On the third day of hospitalization, he developed acute, severe pain and swelling of the left elbow.
Within next few hours, pain worsened and he was unable to move the elbow joint, which was tender,
erythematous, and swollen on examination. Never investigated in the past, we also noted a firm 4 - 6 cm
mass on each elbow, and another one surrounding the proximal inter-phalangeal joint of right middle finger.
When asked particularly, he noticed similar episodes of arthritis of great toe in the past. Which of the
following medications is indicated for the treatment of the arthritis?{
A. allopurinol
B. corticosteroids
C. *colchicine
D. tetracycline
E. mydocalmi
1706. A 65-year-old woman is evaluated because of a 1-year history of bilateral knee pain and low back
pain. She has some stiffness for approximately 15 minutes when she awakens in the morning, and during the
afternoon her pain is worse. On physical examination, she has slight swelling and tenderness to pressure of
the distal interphalangeal joints 2-5 on both hands. There is slight crepitus with motion of the right knee.
Which of the following treatment do use?
A. methotrexat
B. allopurinol.
C. probenecid
D. colchicine
E. *Intra-articular Hyaluronic Acid Injections
1707. A 65-year-old woman is evaluated because of a 1-year history of bilateral knee pain and low back
pain. She has some stiffness for approximately 15 minutes when she awakens in the morning, and during the
afternoon her pain is worse. On physical examination, she has slight swelling and tenderness to pressure of
the distal interphalangeal joints 2-5 on both hands. There is slight crepitus with motion of the right knee. All
of the following are X-ray signs of arthritis except
A. Joint space narrowing
B. Marginal osteophytes
C. Subchondral cysts
D. Bony sclerosis
E. *Marginal erosions
1708. A 30-yr-old woman presents with fits and arthralgia. She is noted to have thinning of her hair and
oral ulceration. Choose the single most likely investigation from the list of options below.
A. Serum calcium
B. Serum glucose
C. Serum electrolytes
D. Serum urea and creatinine
E. *Anti-dsDNA
1709. A 20-yr-old woman from Trinidad presents with pleuritic chest pain, breathlessness, arthralgia,
myalgia and facial rash. She has a small left pleural effusion. ESR is 80 mm/hr. C-reactive protein is 3 mg/l.
Choose the most likely diagnosis from the below list of options.
A. Tuberculosis
B. Sarcoidosis
C. *SLE
D. Uraemia
E. Wadenstorm's macroglobulinaemia
1710. For Positive fluorescent antinuclear antibody choose the single most likely diagnosis from the below
list of options.
A. Wegener's granulomatosis
B. Goodpasture's syndrome
C. Multiple myeloma
D. *Lupus nephritis
E. Diabetic neprtropathy
1711. A 35-yr-old woman presented with arthralgia, myalgia, facial rash and blood pressure of 190/110 and
impaired renal function. Urine microscopy showed scanty red cells and granular casts. Renal biopsy showed
linear IgG on glomerular basement membrane. Choose the most likely diagnosis from the below list of
options.
A. Tuberculosis
B. Renal tubular acidosis
C. *Lupus nephritis
D. Uraemia
E. Renal artery stenosis
1712. For a hypertensive 48-yr-old lady who has been on hydralazine therapy choose the single most likely
diagnosis from the below list of options.
A. *SLE
B. Nephroblastoma
C. Haemolytic uraemic syndrome
D. Juvenile chronic arthritis
E. Henoch-Schonlein purpura
1713. A 16-yr-old boy has had a pyrexia, rash, raised ESR, 1 week after faringitis. Subcutaneous nodules
and a generalised rash is noted on the trunk. Choose the single most likely diagnosis from the list of options
below.
A. *Erythema marginatum
B. Maculopapular rash
C. Erythema multiforme
D. Lichen planus
E. Erythema nodosum
1714. A 17-year-old boy is brought by his father for evaluation of fever. All of the following would suggest
the diagnosis of acute rheumatic fever EXCEPT:
A. *shortened P-Q interval on ECG.
B. rapid, involuntary, purposeless movements
C. migratory polyarthritis
D. subcutaneous nodules
E. a rash on the trunk and proximal extremities
1715. A 18-yr-old boy presents with polyarthritis and abdominal pain. He had a sore throat a week ago. On
examination he is noted to have an early blowing diastolic murmur at the left sternal edge. Choose the single
most likely diagnosis from the list of options below.
A. *Acute rheumatic fever
B. Congestive heart failure
C. Aortic stenosis
D. SLE
E. Mitral stenosis
1716. A 27-yr-old woman with a heart condition has a rash on her arms. It is pink or red and ring like. It
moves up and down her arms and she can not get rid of it. Choose the single most likely diagnosis from the
list of options below.
A. *Erythema marginatum
B. Maculopapular rash
C. Erythema multiforme
D. Lichen planus
E. Erythema nodosum
1717. A 43-yr-old drug addict is noted to have a PSM at the bottom of the sternum. Giant 'cv' waves are
present in the JVP. Choose the single most likely diagnosis from the list of options below.
A. *TR, infective endocarditis
B. Rheumatic MR
C. Congenital AS
D. Atrial myxoma
E. Congenital PS
1718. A 44-yr-old woman presents with malaise and low-grade fever for 3 weeks. She recently had
treatment for a dental abscess. On examination she has splinter haemorrhages and an early diastolic murmur.
Choose the single most likely treatment from the list of options below.
A. *2 or more antibiotics together
B. Amoxycillin
C. Ciprofloxacin
D. Cefotaxime
E. Co-amoxiclav
1719. A young man complains of red and sore eyes, painful urination and low back pain. Three months ago
he had UTI and was successfully treated for it. Choose the most likely diagnosis from the below list of
options.
A. *Reiter's syndrome
B. Psoriatic arthropathy
C. Osteoarthritis
D. Haemochromatosis
E. Gonococcal arthritis
1720. Accurate statement about rheumatoid factors include all of the following EXCEPT?
A. They are antibodies to the Fc fragment of immunoglobulin G
B. They are associated with several conditions in which there is chronic antigenic stimulation
C. Their presence in the serum of persons with rheumatoid arthritis correlates with a worse prognosis than
that for persons with seronegative disease
D. Rheumatoid factor may be seen in association with many other diseases
E. *They are associated with Still's syndrome
1721. A 20-yr-old man presents with urethitis and a painful swollen knee. Choose the most likely diagnosis
from the below list of options.
A. Rheumatoid arthritis
B. *Reactive arthritis
C. Osteoarthritis
D. Haemochromatosis
E. Gonococcal arthritis
1722. A 22-yr-old male soldier presents with a 2-week history of a swollen right knee, conjunctivitis and
arthritis. Choose the most likely diagnosis from the below list of options.
A. Ankylosing spondylitis
B. *Reactive arthritis
C. Osteoarthritis
D. Gout
E. Psoriatic arthritis
1723. This 43 year-old female had a history- old female had a history of shortness of breath and palpitations
for 7 years with a progressive increase one month before death. She was known to have both systolic and
diastolic murmurs, dilated chambers. Is the disease pericardial, myocardial or endocardial
A. Endocardial
B. Pericardial
C. *Myocardial
D. Pericardial, myocardial and endocardial
E. Pericardial and myocardial
1724. This 43 year-old female had a history: old female had a history of shortness of breath and palpitations
for 7 years with a progressive increase one month before death. She was known to have both systolic and
diastolic murmus, dilated chambers. Choose the most likely diagnosis from the below list of options.
A. *Dilated cardiomyopathy
B. Infective endocarditis
C. Degenerative valvular disease
D. Connective tissue disease
E. Congenital valvular disease
1725. Vegetations on undersurface of A.V. valves are found in:
A. *Limban Sack's endocarditis
B. Acute Rheumatic corditis
C. Non thrombotic bacterial endocarditis
D. Chronic rheumatic carditis
E. MVP
1726. What group of drugs has a significant role in the treatment of the patients with severe carditis
accompanied by congestive heart failure:
A. *steroids
B. salicylates
C. cefazolin
D. erythromycin
E. heparin
1727. What investigation can detect vegetations in approximately 90% of patients with proven endocarditis:
A. *Transoesophageal echocardiogram
B. Echocardiogram
C. ECG
D. Chest X-ray
E. CT
1728. What is duration of the antibiotic treatment of the patients with acute rheumatic fever?
A. *not less then two weeks
B. not less then 20 days
C. not less then a week
D. not less then 5 days
E. not less then a month
1729. Which cardiac valve is most frequently affected as a result of rheumatic fever?
A. *Mitral
B. Aortic
C. Tricuspid
D. Pulmonic
E. Mitral and Pulmonic
1730. What is specific for rheumatic fever?
A. *Subcutaneous nodule
B. Erythema marginatum
C. Osler's nodes
D. Erythema nodosum
E. Janeways lesions
1731. Which of the following are immune complex lesions in infective endocarditis
A. *Oslers nodes
B. mycotic aneurysms
C. anemia
D. vegetations
E. meningitis
1732. Which of the following is the most common late manifestation of acute rheumatic fever?
A. *Chorea
B. Pharyngitis
C. Erythema marginatum
D. Iritis
E. Enthesopathy
1733. Which of the following is not one of the major Jones criteria for the diagnosis of rheumatic fever?
A. *Urethritis
B. Carditis
C. Arthritis
D. Chorea
E. Erythema marginatum
1734. Which of the following statements best characterized tricuspid regurgitation?
A. *Infective endocarditis is a common cause
B. The murmur heard in tricuspid regurgitation decreases with inspiration
C. Large a waves in jugular venous pulse are characteristic
D. Correction of coexistent left ventricular failure rarely improves the condition
E. Cardiac catheterization is required for the diagnosis
1735. 79 y/o female with foot pain and swelling for many years. There are "punched out" articular erosions
with overhanging cortex and associated soft tissue masses. Serum uric acid was markedly elevated. Choose
the most likely diagnosis from the below list of options.
A. Ankylosing spondylitis
B. Rheumatoid arthritis
C. Osteoarthritis
D. *Chronic Gouty arthritis
E. Psoriatic arthritis
1736. A 25-yr-old man presents with a 10-yr history of back pain, peripheral arthritis, worse in the morning
and one episode of iritis. Choose the medications from the below list of options.
A. *Methotrexate
B. Actovegin
C. Mydocalm
D. Warfarin
E. Pentoxifylline
1737. Echocardiographic findings for infective endocarditis are all of the following EXCEPT:
A. *New valvar stenosis
B. Mobile echo dense masses attached to valve leaflets or endocardium
C. Periannular abscesses
D. New dehiscence of a prosthetic valve
E. New valvar regurgitation
1738. Feature that distinguish rheumatic fever from myocarditis:
A. *migratory arthritis
B. ST-T-wave abnormalities
C. Arrhythmias
D. heart failure
E. tachycardia.
1739. Feature that distinguishes rheumatic heart diseases from infective endocarditis is:
A. *vegetations
B. Doppler echocardiographic restriction of mitral valve leaflets, their calcification
C. pulmonary hypertension
D. cardiomegaly
E. syncope.
1740. Immunological phenomena in infective endocarditis are all of the following EXCEPT:
A. *Erythema marginatum
B. glomerular nephritis
C. rheumatoid factor
D. Osler's nodes
E. Roth spots
1741. In which disease we expect to see vegetations?
A. *Infective endocarditis
B. Viral myocarditis
C. Acute pericarditis
D. Rheumatic heart disease
E. Hypertrophied cardiomyopathy
1742. Low serum complement levels would be seen in patients with hematuria, proteinuria resulting from
all of the following EXCEPT
A. *Henoch-Schonlein purpura
B. diffuse proliferative lupus nephritis
C. hepatitis C-associated membranoproliferative glomerulonephritis
D. streptococcal infective endocarditis glomerulonephritis
E. rheumatism
1743. Major criteria of rheumatic fever are the following EXCEPT:
A. *Recent group A streptococcal infection
B. Carditis
C. Migratory polyarthritis
D. Sydenham's chorea
E. Erythema marginatum
1744. Migrating polyarthritis is characteristically seen in
A. *rheumatic arthritis
B. syphilitic arthritis
C. rheumatoid arthritis
D. psoriatic arthritis
E. gouty arthritis
1745. Migratory polyarthritis in rheumatic fever most often affecting following joints except:
A. *small joints of hands
B. the ankles
C. wrists
D. knees
E. elbows
1746. Minor criteria for infective endocarditis are all of the following EXCEPT:
A. *Migratory arthritis
B. Fever >38В°C
C. Embolic/vascular phenomena
D. Immunological phenomena
E. Blood cultures compatible but not typical for endocarditis
1747. Minor Criterion of rheumatic fever is:
A. *Prolonged P-Q interval
B. sinus tachycardia
C. sinus bradycardia
D. inversion of T waves
E. ectopic beats
1748. Noncardiac Manifestations of infective endocarditis are the following except:
A. *a rash on the trunk and proximal extremities
B. splenomegaly
C. petechiae
D. arterial emboli
E. Osler's nodes
1749. Patients with acute rheumatic fever in initial attack have evidence of carditis, which may be
characterized by all are the following EXCEPT:
A. sinus tachycardia
B. the murmur of mitral regurgitation
C. an S3 gallop
D. *the first heart sound is accentuated
E. cardiomegaly
1750. Patients with acute rheumatic fever in initial attack have evidence of carditis, which may be
characterized by all are the following EXCEPT:
A. *the first heart sound is accentuated
B. sinus tachycardia
C. the murmur of mitral regurgitation
D. an S3 gallop
E. cardiomegaly
1751. Rheumatic fever caused by:
A. *beta-hemolytic streptococci.
B. Staphylococcus lugdunensis
C. Enterococci
D. The HACEK group
E. H. influenza
1752. 79 y/o female with foot pain and swelling for many years. There are "punched out" articular erosions
with overhanging cortex and associated soft tissue masses. Serum uric acid was markedly elevated. Which of
the following medications is indicated for the treatment of chronic arthritis?
A. *allopurinol
B. colchicine
C. corticosteroids
D. indomethacin
E. ibuprofen
1753. 86 year old male with chronic gout presents with bilateral foot pain. Physician desired radiograph for
evaluation of extent of pathologic changes. Prominent erosive destruction of bilateral first metatarsals and
second distal phalanx with well-preserved joint spaces throughout. Choose the most likely changes from the
below list of options.
A. High ESR
B. Oseoporosis on X-ray
C. Positive blood culture
D. *Urate crystals on joint aspirate
E. Positive rheumatoid factor
1754. A 27-yr-old man presents with a 10-yr history of back pain, peripheral arthritis worse in the morning
and one episode of iritis. Choose the medications for patient with shown changes from the below list of
options.
A. *Sulfasalazine
B. Actovegin
C. Mydocalm
D. Warfarin
E. Pentoxifylline
1755. A 50-yr-old woman complains of stiffness in her fingers worse at the end of the day. The DIP joints
and the first metacarpophalangeal joints are affected. Choose the most likely diagnosis from the below list of
options.
A. *Osteoarthritis
B. Rheumatoid arthritis
C. Psoriatic arthropathy
D. Haemochromatosis
E. Gonococcal arthritis
1756. A 50-yr-old woman complains of stiffness in her fingers worse at the end of the day. The DIP joints
and the first metacarpophalangeal joints are affected. Choose the most likely diagnosis from the below list of
options.
A. Rheumatoid arthritis
B. Psoriatic arthropathy
C. *Osteoarthritis
D. Haemochromatosis
E. Gonococcal arthritis
1757. A 53-yr-old woman complains of redness, swelling and stiffness in the distal interphalangeal joints of
her hands, but has no other joint complaints. Choose the most likely diagnosis from the below list of options.
A. Gout
B. Still's disease
C. Pseudogout
D. Rheumatoid arthritis
E. *Osteoarthritis
1758. A 57-year-old woman has had five episodes of joint pain and swelling, lasting 3 to 8 days, involving
the right knee and left elbow. Her most recent episode was 4 months ago. Classical picture of exacerbation
of the arthritis includes all of the following except
A. Excruciating and sudden pain
B. Low-grade fever may also be present
C. *Overgrowth at the distal interphalangeal joints
D. Warmness
E. Redness
1759. A 60-yr-old alcoholic man presents with a hot swollen first metatarsophalangeal joint and a lesion on
the rim of the left pinna. Choose the investigation for diagnosis from the below list of options.
A. HLA-B27
B. *Synovial fluid analysis
C. X-ray
D. Anti-dsDNA antibody
E. Rheumatoid factor
1760. A 63-year-old man is evaluated because of a 2-year history of bilateral knee pain and low back pain.
He has some stiffness for approximately 15 minutes when she awakens in the morning, and during the
afternoon her pain is worse. On physical examination, he has slight swelling and tenderness to pressure of
the distal interphalangeal joints 2-5 on both hands. There is slight crepitus with motion of the right knee.
Which of the following treatment do use?
A. Methotrexat
B. Colchicines
C. *Chondroitin sulfate
D. Tetracycline
E. Allopurinol
1761. A 65-year-old man comes to your office complaining of knee pain that began insidiously about a year
ago. He has no other rheumatic symptoms. Which of the following signs is not sign of the arthritis?
A. Pain is related to use
B. Pain gets worse during the day
C. Minimal morning stiffness (<20 min) and after inactivity (gelling)
D. *Warmth of skin over effected joints
E. Range of motion decreases
1762. A 65-year-old man comes to your office complaining of knee pain that began insidiously about a year
ago. He has no other rheumatic symptoms. Which of the following signs isn’t sign of the arthritis? Which of
the following signs isn’t sign of the arthritis?
A. Joint instability
B. Bony enlargement
C. Restricted movement
D. Crepitus
E. *Tophi
1763. A 65-yr-old man has started chemotherapy for lymphoma. He has developed a painful swollen hot
right knee. He is apyrexial. Choose the most likely changes from the below list of options.
A. *Urate crystals on joint aspirate
B. High ESR
C. Erosions on X-ray
D. Positive blood culture
E. Positive rheumatoid factor
1764. A 55-yr-old man has started chemotherapy for CML. He has developed a painful swollen hot right
knee. He is apyrexial. Choose the most likely diagnosis from the below list of options.
A. *Urate crystals on joint aspirate
B. High ESR
C. Erosions on X-ray
D. Positive blood culture
E. Positive rheumatoid factor
1765. A 68-year-old man presents with an acutely red and swollen right great toe without history of trauma.
Which of the following findings is most useful for making a diagnosis of gout in this patient?
A. Persistent elevation of serum uric acid
B. Good response to colchicines trial
C. *Radiograph showing marginal joint erosion in the first metatarso-phalangeal (MTP) joint
D. An associated right ankle effusion
E. A painless elbow nodule
1766. A 70-yr-old woman complains of arthritis in the fingers and big toe. On examination she has bony
swellings of the first carpometacarpal joint and the distal interphalangeal joints and has an affected
metatarsophalangeal joint. Choose the most likely diagnosis from the below list of options.
A. Rheumatoid arthritis
B. Hyperparathyroidism
C. Psoriatic arthropathy
D. *Osteoarthritis
E. Haemochromatosis
1767. 73-yr-old fit farmer presents with pain on weight bearing and restricted movements of the right hip.
Choose the most likely diagnosis from the below list of options.
A. Ankylosing spondylitis
B. Erythema nodosum
C. *Osteoarthritis
D. Gout
E. Psoriatic arthritis
1768. All of the following are X-ray signs of Osteoarthritis except
A. Joint space narrowing
B. Marginal osteophytes
C. Subchondral cysts
D. Bony sclerosis
E. *Marginal erosions
1769. A 73-yr-old fit farmer presents with pain on weight bearing and restricted movements of the right hip.
Choose the most likely X-ray changes from the below List of options.
A. Cystic changes and lytic lesions
B. *Joint spaces are narrowed with reactive subchondral sclerosis, osteophytes
C. Periarticular osteopenia, marginal erosions
D. acroosteolysis
E. Fluffy bone periostitis and erosions at the interphalangeal joints
1770. All of the following physical findings may be seen in osteoarthritis except
A. Heberden nodes
B. Bouchar nodes
C. Bony crepitus on joint movement
D. Bounniere deformity
E. *Acute arthritis of metatarsophalangeal joints.
1771. An elderly man presents with a red, warm swollen metatarsophalangeal joint following a right total
hip replacement operation. Choose the most likely diagnosis from the below list of options.
A. Ankylosing spondylitis
B. Reactive arthritis
C. Osteoarthritis
D. *Acute Gouty arthritis
E. Psoriatic arthritis
1772. An elderly man started frusemide 2-weeks ago and now presents with a red, hot, swollen first
metatarsal phalangeal joint. Choose the most likely diagnosis from the below list of options.
A. Ankylosing spondylitis
B. Joint sepsis
C. Osteoarthritis
D. *Gout
E. Psoriatic arthritis
1773. Classical picture of acute gouty arthritis includes all of the following except
A. Excruciating and sudden pain
B. Stiffness in the joint
C. Low-grade fever may also be present
D. Warmness
E. *Overgrowth at the distal interphalangeal joints
1774. Each of the following agents may be useful in the treatment of acute gouty arthritis except
A. indomethacin
B. *allopurinol
C.
colchicines
D. intraarticular glucocorticoids
E. aspirin
1775. Each of the following may cause hyperuricemia and thereby may provoke an attack of gouty arthritis
except
A. thiazide diuretics
B. exercise
C. cyclosporine
D. methotrexate
E. *ascorbic acid
1776. Gouty arthritis is characterized by:
A. chiefly involvement of the lumbosacral portion of spine;
B. gradual onset;
C. frequently associated with renal disease;
D. absence of renal disease;
E. *involvement of the first metatarsophalanges joint.
1777. Hyperuricemia is defined as a serum uric acid concentration above
A. *7 mg per dL (420 Вµmol per L).
B. 4 mg per dL (240 Вµmol per L).
C. 5 mg per dL (300 Вµmol per L).
D. 8 mg per dL (480 Вµmol per L).
E. 9 mg per dL (540 Вµmol per L).
1778. Overproduction of urate may be in all of the following except
A. Primary idiopathic hyperuricemia
B. *Hypertension
C. Hemolytic processes
D. Lymphoproliferative disease
E. Myeloproliferative disease
1779. Overproduction of urate may be in all of the following except
A. Polycythemia vera
B. Psoriasis (severe)
C. *Peptic ulcer
D. Exercise
E. Alcohol
1780. Patients with gouty arthritis characterized by any of the following clinical characteristics except:
A. tophaceous deposits
B. *rheumatoid factor positive
C. "punched out" erosive changes on radiographs
D. acute attack frequency of greater than or equal to 2 attacks per year
E. hyperuricemia
1781. Risk factors and possible causes of osteoarthritis. Risk factors and possible causes of osteoarthritis
are all of the following except
A. Age
B. Obesity
C. *Stress
D. Prior injury
E. Acromegaly
1782. Secondary osteoarthritis may be caused by all of except
A. Hemaphilia
B. Hemochromatosis
C. Gouty arthritis
D. *Diabetes mellitus
E. After trauma
1783. Side effects of colchicines are all of the following except
A. Neutropenia
B. Nausea
C. Diarrhea
D. Anemia
E. *Headache
1784. Signs of osteoarthritis are all of the following except
A. Limp
B. Muscle atrophy
C. Crepitus
D. Restricted movement
E. *Warmth of skin over effected joints
1785. The client has just been diagnosed with chronic gout. Which tablets should be prepared to teach about
side effects and precautions to this client?
A. *Allopurinol (Zyloprim)
B. Furosemide (Lasix)
C. Cortisol (prednisone)
D. Diphenhydramine (Benadryl)
E. Indometacin (Indocin)
1786. The client is being admitted for a total hip replacement. She says that she is glad she will be having
anesthesia shortly because she has a painful abscessed tooth and is taking an antibiotic for it. What is the best
action?
A. Take the client's temperature
B. *Notify the surgeon immediately
C. Document the report as the only action
D. Arrange for the client to receive the antibiotic intravenously
E. Sent patient to dentist
1787. The client with arthritis and all the following allergies is prescribed to take celecoxib (Celebrex)
daily. Which allergy is most important for the nurse to report to the physician?
A. Latex
B. Peanut
C. Shellfish
D. *Sulfa drugs
E. Aspirin
1788. Which activity should be suggested to the client with osteoarthritis as being most likely to slow the
development of disabilities associated with osteoarthritis?
A. Eating less red meat and starchy vegetables
B. Avoiding any form of physical exercise
C. *Swimming 3 times per week
D. Taking an aspirin daily
E. Taking an ascorbic acid daily
1789. Which client is most likely to develop secondary gout?
A. *60-year-old man with renal insufficiency
B. 60-year-old woman with type 2 diabetes mellitus
C. 40-year-old woman who has been taking oral contraceptives for 15 years
D. 40-year-old male athlete who has been taking NSAIDs for a sprained ankle
E. 40-year-old drug abuser
1790. Which client is most likely to develop secondary gout?
A. *56-year-old man with Polycythemia vera
B. 56-year-old woman with type 2 diabetes mellitus
C. 33-year-old woman who has been suffering from SLE last 3 years
D. 40-year-old male athlete who has been taking NSAIDs for a sprained ankle
E. 40-year-old drug abuser
1791. Which condition or action represents a modifiable risk factor for osteoarthritis?
A. *Obesity
B. Hypertension
C. Cigarette smoking
D. Walking as exercise
E. Fatty foods
1792. Which condition or health problem demonstrates inflammation without invasion?
A. Allergic rhinitis
B. Viral hepatitis
C. *Osteoarthritis
D. Cellulitis
E. Septic arthritis
1793. Which joints are most frequently affected by osteoarthritis?
A. Jaw and ankles
B. Neck and wrists
C. *Hips and knees
D. Elbows and shoulders
E. Thoracic spine and shoulders
1794. Which of the following is a sign for Gouty arthritis?
A. Simultaneous involvement of the same joint areas on both sides of the body.
B. *Acute arthritis of first metatarsophalangeal joint
C. Symmetric sacroilitis
D. Asymmetric sacroilitis
E. Nonerosive arthritis
1795. Which of the following is characteristic of Osteoarthritis?
A. Severe night pain
B. Morning stiffness
C. *Short starting stiffness
D. Weight loss
E. Asymmetric joint involvement
1796. Which of the following joint findings is most suggestive of an inflammatory, rather than an
osteoarthritic, cause of joint pain?
A. Painful range of motion
B. Crepitus
C. Bony articular enlargement
D. *Swelling and warmth
E. Instability
1797. Which of the following medications is contraindicated for the treatment of an acute attack of gout?
A. *allopurinol
B. colchicine
C. corticosteroids
D. indomethacin
E. ibuprofen
1798. Which physical change in the client with osteoarthritis indicates disease advancement?
A. The hands are involved
B. *The hip shows subluxation
C. Hip involvement is bilateral.
D. The client is older than 65 years
E. Shoulders involvement
1799. Which statement made by the client with gout indicates a need for further teaching regarding the
management of this disorder?
A. have been drinking at least 3 liters of water each day
B. *When the pain is very bad, I have a drink of my favorite wine.
C. Although my favorite food is shrimp, I no longer have it on a regular basis.
D. My wife and I are substituting chicken and other poultry for red meat these days.
E. Although my favorite food is mushrooms, I no longer have it on a regular basis.
1800. Which X-ray changes are specific for osteoartrosis?
A. Syndesmophytes
B. *Osteophytes
C. Fractures
D. Erosions
E. Osteoporosis
1801. Male with chronic joint pain. There is asymmetric soft tissue swelling, predominantly in the DIP of
the right first toe, with calcifications. Further questioning revealed a history of repeated attacks of acute joint
pain. Choose the most likely diagnosis from the below list of options.
A. Ankylosing spondylitis
B. Erythema nodosum
C. Osteoarthritis
D. *Chronic gouty arthritis
E. Psoriatic arthritis
1802. A 22-yr-old male soldier presents with a 2-week history of a swollen right knee, conjunctivitis and
arthritis. Choose the most likely diagnosis from the below list of options.
A. Ankylosing spondylitis
B. Psoriatic arthritis
C. Joint sepsis
D. *Reactive arthritis
E. Rheumatoid arthritis
1803. Disease-modifying' antirheumatic drugs for rheumatoid arthritis are all of the following except
A. *NSAIDs
B. Hydroxychloroquine sulphate
C. Sulphasalazine
D. Methotrexate
E. D-penicillamine
1804. A 15-yr-old boy complains of pain in the temperomandibular joint for 3 months. On examination, the
SHO in A&E notices micrognathia, loss of neck extension and unequal lengths of the boy's lower limbs.
Tests for rheumatoid factor were negative. Choose the most likely diagnosis from the below list of options.
A. Gout
B. Still's disease
C. Pseudogout
D. *Rheumatoid arthritis
E. Osteoarthritis
1805. A 25-yr-old man presents with urethritis, conjunctivitis and a swollen left knee. Choose the most
likely diagnosis from the below list of options.
A. *Reiter's syndrome
B. Psoriatic arthropathy
C. Osteoarthritis
D. Haemochromatosis
E. Gonococcal arthritis
1806. A 30-yr-old flight attendant presents with gritty eyes and painful knees especially during standing. He
has just returned from Thailand Choose the most likely diagnosis from the below list of options.
A. *Reactive arthritis
B. Ankylosing spondylitis
C. Psoriatic arthritis
D. Joint sepsis
E. Rheumatoid arthritis
1807. A 31-yr-old travel agent presents with painful knees, red eyes and dysuria. Choose the most likely
diagnosis from the below list of options.
A. Polyarteritis nodosa
B. Giant cell arteritis
C. Rheumatoid arthritis
D. Ankylosing spondylitis
E. *Reiter's syndrome
1808. A 41-yr-old woman presents with painful knees, red eyes and dysuria. She has just returned from a
trip. Choose the most likely diagnosis from the below list of options.
A. Ankylosing spondylitis
B. Psoriatic arthritis
C. Joint sepsis
D. *Reiter's syndrome
E. Polyarteritis nodosa
1809. A 32-yr-old man presents with rash on his penis, pain in the left heel and right-sided stiffness in the
lower back upon rising in the morning. Choose the most likely diagnosis from the below list of options.
A. Ankylosing spondylitis
B. Psoriatic arthritis
C. Joint sepsis
D. *Reactive arthritis
E. Polyarteritis nodosa
1810. A 44-yr-old woman presents with stiff, sausage-shaped fingers and MCP joint swelling worse in the
morning. Choose the most likely diagnostic antibody for the disease from the below list of options.
A. Antimicrosomal antibody
B. Antireticulin antibody
C. Anti-dsDNA
D. P-ANCA antibody
E. *Rheumatoid factor, anti-CCP
1811. A 35-yr-old woman has pain and stiffness in her hands, wrists, elbows, knees and ankles. There is
swelling of her MCP joints bilaterally. There is a nodule on her left elbow. Choose the most likely diagnosis
from the below list of options
A. High ESR
B. High serum urate
C. Positive blood culture
D. *Positive rheumatoid factor
E. High WBC count
1812. A 36-yr-old lady presented with swollen tender knee joints. She says they feel stiff especially in the
mornings. On examination she was found to have splenomegaly, a temperature of 38В°C and ulcerated
lower limbs with Hb9 g/dl. WBC and platelets were decreased. Albumin was 20 g/l. Choose the most likely
diagnosis from the below list of options.
A. Gout
B. Still's disease
C. Felty's syndrome
D. *Rheumatoid arthritis
E. Osteoarthritis
1813. A 43-year-old woman has experienced morning stiffness, swelling of the small joints of hands,
wrists, knees, ankles, elbows. The proximal interphalangeal joints are fusiform swelling. X-ray examination
of which joints you need for confirming diagnosis?
A. *Hands
B. Ankles
C. Knees
D. Elbows
E. Spine
1814. A 52 -yr-old woman presents with stiff, sausage-shaped fingers and MCP joint swelling worse in the
morning (within 2 hours). Choose the most likely diagnostic antibody for the disease from the below list of
options.
A. *Rheumatoid factor
B. Antimicrosomal antibody
C. Antireticulin antibody
D. Anti-dsDNA
E. P-ANCA antibody
1815. A 48-year-old man complains of joint pain and stiffness for the past 3 months. All of the following
are signs of rheumatoid arthritis (RA) except
A. early morning stiffness lasting 1 hour
B. *numbness and blanching of fingers on exposure to cold
C. symmetrical involvement of the metacarpophalangeal joints
D. a non-tender subcutaneous mass over the Achilles tendon
E. synovial fluid with high viscosity and 30,000 lymphocytes/mm3
1816. 71-yr-old woman with longstanding polyarthritis on immunosuppressive drugs presents with
systemic malaise and fever and has severe pain, redness, heat and swelling of the wrist. Choose the most
likely diagnosis from the below list of options.
A. Ankylosing spondylitis
B. *Joint sepsis
C. Osteoarthritis
D. Gout
E. Psoriatic arthritis
1817. A woman who has rheumatoid arthritis suddenly develops pain and swelling in the right calf. The
most likely diagnosis is
A. ruptured plantaris tendon
B. pes anserinus bursitis
C. *ruptured popliteal cyst
D. thrombophlebitis
E. Achilles tendonitis
1818. A young man complains of red and sore eyes, painful urination and low back pain. Nine months ago
he had UTI and was successfully treated for it. Choose the treatment from the below list of options
A. Cyclophosphanum
B. Penicillin
C. Prednisone
D. Ceftriaxone
E. *Tetracycline
1819. An 18-year-old woman comes to the emergency department complaining of severe right knee, right
wrist, and left ankle pain, morning stiffness. Physical examination also reveals tenderness and swelling of
involved joints. While awaiting results of laboratory testing, this patient should receive which of the
following treatments?
A. Corticosteroids
B. *Nonsteroidal anti-inflammatory drugs (NSAIDs)
C. Antibiotics
D. Local care of skin lesions
E. Splinting of painful joints
1820. An 28-year-old woman comes to the emergency department complaining of severe knees, wrists, and
ankles pain, morning stiffness. Physical examination also reveals tenderness and swelling of involved joints.
Choose the most likely diagnosis for patient with shown changes from the below list of options.
A. Ankylosing spondylitis
B. *Rheumatoid arthritis
C. Osteoarthritis
D. Gout
E. Psoriatic arthritis
1821. Arthritis associated with rheumatoid arthritis can be manifest in several different ways. Which of the
following is characteristic of rheumatoid arthritis
A. asymmetric oligoarticular arthritis
B. *rheumatoid factor-positive symmetric polyarthritis
C. arthritis of distal interphalangeal joints
D. severe destructive polyarthritis (arthritis mutilans)
E. spondilitis and sacroiliitis with or without peripheral arthritis
1822. Association of arthritis and urethritis is typical for:
A. Reynolds’s phenomen;
B. RA;
C. Psoriatic arthritis;
D. *Reiters syndrome;
E. ankylosing spondilitis;
1823. Which disease has ulnar deviation more often?
A. *Rheumatoid arthritis
B. Gout
C. Still's disease
D. Pseudogout
E. Osteoarthritis
1824. Choose the most likely diagnostic antibody for Rheumatoid arthritis from the below list of options.
A. Antimicrosomal antibody
B. Antireticulin antibody
C. Anti-dsDNA
D. P-ANCA antibody
E. *Rheumatoid factor
1825. Each of the following agents has been demonstrated to alter the course of rheumatoid arthritis except
A. gold
B. *omega-3 fatty acid
C. methotrexate
D. D-penicillamine
E. Hydroxychloroquine
1826. Felty’s syndrome occurs in patients with seropositive rheumatoid arthritis with all of the following
except
A. Neutropenia
B. Thrombocytopenia
C. Anemia
D. *lymphocytopenia
E. splenomegaly
1827. For arthritis in Rheumatoid arthritis more typical is:
A. starting pain;
B. night pain;
C. *morning stiffness;
D. weight loss;
E. asymmetric joint involvement;
1828. For rheumatoid arthritis more typical is
A. *morning stiffness;
B. starting pain;
C. night pain;
D. weight loss;
E. asymmetric joint involvement;
1829. How often patients with Rheumatoid arthritis have Rheumatoid factor positive?
A. 60 %
B. 65 %
C. 75 %
D. *85 %
E. 90 %
1830. How often patients with Rheumatoid arthritis have anti-cyclic citrullinated peptide?
A. *98 %
B. 92 %
C. 67 %
D. 78 %
E. 85 %
1831. Lung disorders in rheumatoid arthritis may include all of the following except
A. pleuritis (with effusions characterized by low glucose)
B. *pulmonary thromboembolism
C. interstitial fibrosis
D. nodules
E. Caplans syndrome
1832. Reactive arthritis may be caused by all of the following enteric infection except
A. Salmonella typhimurium
B. Shigella flexneri
C. Yersinea enterocoliticas
D. *Chlamydia trachomatis
E. Campylobacter jejuni
1833. Reiters syndrome is more frequently a response to
A. infection with shigella;
B. infection with staphylococci;
C. infection with salmonella;
D. infection with streptococci;
E. *infection with Chlamydia.
1834. Side-effects of NSAIDs include all of the following except
A. fluid retention
B. upper gastrointestinal haemorrhage
C. *hypotension
D. interstitial nephritis
E. agranulocytosis
1835. The diagnosis of rheumatoid arthritis is based entirely on clinical grounds. Choose correct clinical
characteristics associated with rheumatoid arthritis
A. *prolonged morning stiffness
B. migratory polyarthritis
C. arthritis involving the distal interphalangeal joints
D. arthritis involving the sacroiliac joints
E. night painfull arthritis
1836. The client is being admitted for a total hip replacement. She says that she is glad she will be having
anesthesia shortly because she has a painful abscessed tooth and is taking an antibiotic for it. What is the best
action?
A. Take the client's temperature
B. *Notify the surgeon immediately
C. Document the report as the only action
D. Arrange for the client to receive the antibiotic intravenously
E. Sent patient to dentist
1837. The client on long-term corticosteroid therapy for rheumatoid arthritis has developed abdominal
striae. Side effect of which drug may cause pitting edema of both legs?
A. *Prednisone
B. Methotrexat
C. Folic acid
D. NSAIDs
E. None of the above
1838. Patient with arthritis and all the following allergies is prescribed to take celecoxib (Celebrex) daily.
Which allergy is most important for the nurse to report to the physician?
A. Latex
B. Nut
C. Shellfish
D. *Sulfasalasini
E. Diclofenacy
1839. The older adult client taking prednisone, methotrexate, folic acid, NSAIDs for rheumatoid arthritis
now has pitting edema of both legs. Side effect of which drug may cause pitting edema of both legs?
A. Prednisone
B. Methotrexat
C. folic acid
D. *NSAIDs
E. None of the above
1840. This patient will likely develop bilateral inflammation of the proximal interphalangeal and
metacarpophalangeal joints and the wrists.
A. This patient is positive for antinuclear antibody.
B. This patient has a history of antecedent bloody diarrhea.
C. Knee arthrocentesis will reveal intracellular monosodium urate crystals.
D. This patient has a history of antecedent UTI
E. *This patient is positive for rheumatoid factor.
1841. What precaution is most important to teach the client with rheumatoid arthritis who will be taking 20
mg of prednisone daily?
A. Take this drug at bedtime.
B. *Avoid crowds and anyone who is ill
C. Drink at least 3 liters of fluid per day.
D. Wash your face 3 times per day with an antibacterial soap.
E. Avoid sunlight.
1842. What factor or condition in the client who has joint stiffness and pain is more associated with
progressive systemic sclerosis than with rheumatoid arthritis?
A. The client is female
B. The pain is worse in the morning
C. The client has never been pregnant
D. *The fingers are sausage-like, with nonpitting edema
E. The client has limited motion
1843. What precaution is most important to teach the client with rheumatoid arthritis who will be taking
methotrexate?
A. *Avoid sunlight.
B. Take this drug at bedtime.
C. Avoid crowds and anyone who is ill.
D. Drink at least 3 liters of fluid per day.
E. Wash your face 3 times per day with an antibacterial soap.
1844. Which change is characteristic of Reiters syndrome?
A. *Dactylitis
B. Bamboo Spine
C. Erosive arthritis of wrists
D. Acute arthritis of metatarsophalangeal joint
E. Subcutaneous nodules
1845. Which clinical manifestation in a male client with arthritis indicates or suggests the possibility of
Reiter's syndrome?
A. Stooped posture
B. Unplanned weight loss
C. *Pain/burning on urination
D. Ringing in the ears at night
E. Dry eyes
1846. Which clinical manifestation would be an unexpected finding in a patient with rheumatoid arthritis?
A. Anemia
B. Warm, tender joints
C. Symmetric and bilateral joint involvement
D. Elevated ESR
E. *Crepitus of involved joints
1847. Which following drug doesn’t alter the course of rheumatoid arthritis?
A. *NSAIDs
B. Hydroxychloroquine sulphate
C. Sulphasalazine
D. Methotrexate
E. D-penicillamine
1848. Which joints of the body may be involved in rheumatoid arthritis mainly?
A. hips;
B. *small joints of hands
C. ankles;
D. shoulders;
E. sacroiliac joints
1849. Which of the following clinical predictors is not associated with severe and progressive rheumatoid
arthritis?
A. Persistent swelling of the proximal interphalangeal joints
B. High C-reactive protein level or elevated erythrocyte sedimentation rate
C. *Swelling of the distal interphalangeal joints
D. Large number of swollen joints
E. High titers of rheumatoid factor
1850. Which of the following conditions is LEAST likely to occur in late extraarticular seropositive
rheumatoid arthritis?
A. Neutropenia
B. Dry eyes
C. Leg ulcers
D. Sensorimotor polyneuropathy
E. *Hepatitis
1851. A 70-yr-old fit farmer presents with pain on weight bearing and restricted movements of the right hip.
Choose the most likely diagnosis from the below list of options.
A. Ankylosing spondylitis
B. Erythema nodosum
C. *Osteoarthritis
D. Gout
E. Psoriatic arthritis
1852. An elderly man started frusemide 4-weeks ago and now presents with a red, hot, swollen first
metatarsal phalangeal joint. Choose the most likely diagnosis from the below list of options.
A. Ankylosing spondylitis
B. Joint sepsis
C. Osteoarthritis
D. *Gout
E. Psoriatic arthritis
1853. A 50-yr-old woman complains of redness, swelling and stiffness in the distal interphalangeal joints of
her hands, but has no other joint complaints. Choose the most likely diagnosis from the below list of options.
A. Gout
B. Still's disease
C. Pseudogout
D. Rheumatoid arthritis
E. *Osteoarthritis
1854. A 55-yr-old woman complains of stiffness in her fingers worse at the end of the day. The DIP joints
and the first metacarpophalangeal joints are affected. Choose the most likely diagnosis from the below list of
options.
A. *Osteoarthritis
B. Rheumatoid arthritis
C. Psoriatic arthropathy
D. Haemochromatosis
E. Gonococcal arthritis
1855. A 68-yr-old woman complains of arthritis in the fingers and big toe. On examination she has bony
swellings of the first carpometacarpal joint and the distal interphalangeal joints and has an affected
metatarsophalangeal joint. Choose the most likely diagnosis from the below list of options.
A. Rheumatoid arthritis
B. Hyperparathyroidism
C. Psoriatic arthropathy
D. *Osteoarthritis
E. Haemochromatosis
1856. An 67-yr-old man presents with a red, warm swollen metatarsophalangeal joint following a surgical
treatment. Choose the most likely diagnosis from the below list of options.
A. Ankylosing spondylitis
B. Reactive arthritis
C. Osteoarthritis
D. *Acute Gouty arthritis
E. Psoriatic arthritis
1857. A 58-yr-old alcoholic man presents with a hot swollen first metatarsophalangeal joint and a lesion on
the rim of the right pinna. Choose the investigation for diagnosis from the below list of options.
A. HLA-B27
B. *Synovial fluid analysis
C. X-ray
D. Anti-dsDNA antibody
E. Rheumatoid factor
1858. A 60-yr-old man has started chemotherapy for CML. He has developed a painful swollen hot right
knee. Choose the most likely changes from the below list of options.
A. *Hyperurecemia
B. Erosions on X-ray
C. Positive blood culture
D. Positive rheumatoid factor
E. High ESR
1859. On physical examination, internal rotation of the left hip elicits the groin pain. The second, third, and
fourth metacarpophalangeal joints are swollen, warm, and tender to pressure. Radiographs show severe
osteoarthritis in the left hip. Which of the following is the best test to confirm the diagnosis?
A. Rheumatoid factor
B. HLA-B27
C. *Serum iron and iron-binding capacity
D. Fasting plasma glucose
E. Serum uric acid
1860. Which of the following is a sign for Gouty arthritis?
A. Simultaneous involvement of the same joint areas on both sides of the body.
B. *Acute arthritis of first metatarsophalangeal joint
C. Symmetric sacroilitis
D. Asymmetric sacroilitis
E. Positive RF
1861. A 70-year-old woman has had four or five episodes of joint pain and swelling, lasting 2 to 7 days,
involving the right knee and left elbow. On physical examination, none of her joints is swollen or tender, but
there is marked crepitus on extension of the knee. She also has a positive bulge sign over the left knee and
pain on full extension of the left elbow. Which one of the following tests would confirm the diagnosis?
A. *Arthrocentesis of the knee and laboratory analysis of the synovial fluid
B. Measurement of serum uric acid
C. Measurement of serum rheumatoid factor
D. Radiograph of the knee
E. MRI of the knee with gadolinium contrast
1862. A 73-year-old retired patient is evaluated because of swelling of his right knee. On physical
examination, he has an effusion in the right knee. At a point in the mid-thigh (measured from the superior
border of the patella), the right thigh is 5 cm smaller in diameter than the left. There is crepitus with right
knee flexion. The knee is not warm. Which of the following is the best next step in the management of this
patient?
A. Order radiographs of both knees and treat the patient with ibuprofen.
B. Order radiographs of the right hip and right knee.
C. *Aspirate the joint fluid in the right knee and refer the patient for physical therapy.
D. Aspirate the joint fluid in the right knee and obtain blood cultures.
E. Treat the patient with celecoxib, and advise him to stay off his feet for 48 hours.
1863. Which of the following joint findings is most suggestive of rheumatoid arthritis, rather than an
osteoarthritic, cause of joint pain?
A. Painful range of motion
B. Crepitus
C. Bony articular enlargement
D. *Swelling and warmth
E. Instability
1864. A 62-year-old woman has ostheoarthritis II stage. Which of the following treatment do use?
A. methotrexat
B. *intra-articular injection of sodium hyaluronate
C. probenecid
D. colchicine
E. allopurinol.
1865. All of the following physical findings may be seen in osteoarthritis except
A. Heberden’s nodes
B. Bouchar’s nodes
C. Bony crepitus on joint movement
D. Bounniere deformity
E. *Acute arthritis of metatarsophalangeal joints.
1866. Each of the following may cause hyperuricemia and thereby may provoke an attack of gouty arthritis
except
A. thiazide diuretics
B. exercise
C. cyclosporine
D. methotrexate
E. *ascorbic acid
1867. Each of the following agents may be useful in the treatment of acute gouty arthritis except
A. diclofenak
B. *allopurinol
C. colchicines
D. intraarticular injection of sodium hyaluronate
E. aspirin
1868. Which condition or health problem demonstrates osteophytes?
A. Allergic rhinitis
B. Viral hepatitis
C. *Osteoarthritis
D. Cellulitis
E. Septic arthritis
1869. Gouty arthritis is characterized by:
A. chiefly involvement of the lumbosacral portion of spine;
B. gradual onset;
C. frequently associated with renal disease;
D. absence of renal disease;
E. *Acute attack of the arthritis of first metatarsophalanges joint.
1870. Which joints are often affected by osteoarthritis?
A. Jaw and ankles
B. Neck and wrists
C. *Distal interphalangeal joints
D. Elbows and shoulders
E. Thoracic spine and shoulders
1871. Patient with which condition or action represents a modifiable risk factor for osteoarthritis?
A. *Obesity
B. Hypertension
C. Cigarette smoking
D. Walking as exercise
E. Fatty foods
1872. Which physical change in the patient with osteoarthritis indicates disease advancement?
A. The hands are involved
B. *The hip shows aseptic necrosis
C. Hip involvement is bilateral.
D. The client is older than 65 years
E. Shoulders involvement
1873. Which activity should be suggested to the client with osteoarthritis as being most likely to slow the
development of disabilities associated with osteoarthritis?
A. Eating less red meat and starchy vegetables
B. Avoiding any form of physical exercise
C. *Swimming 3 times per week
D. Taking an aspirin daily
E. Taking an ascorbic acid daily
1874. Which statement made by the client with gout indicates a need for further teaching regarding the
management of this disorder?
A. “I have been drinking at least 3 liters of water each day.”
B. * “When the pain is very bad, I have a drink of my favorite wine.”
C. “Although my favorite food is shrimp, I no longer have it on a regular basis.”
D. “My wife and I are substituting chicken and other poultry for red meat these days.”
E. “Although my favorite food is mushrooms, I no longer have it on a regular basis.”
1875. The client with arthritis and all the following allergies is prescribed to take celecoxib (Celebrex)
daily. Which allergy is most important for the nurse to report to the physician?
A. Latex
B. Peanut
C. Shellfish
D. *Sulfa drugs
E. Aspirin
1876. The client is being admitted for a total hip replacement. She says that she is glad she will be having
anesthesia shortly because she has a painful abscessed tooth and is taking an antibiotic for it. What is the best
action?
A. Take the client's temperature
B. *Notify the surgeon immediately
C. Document the report as the only action
D. Arrange for the client to receive the antibiotic intravenously
E. Sent patient to dentist
1877. The client has just been diagnosed with chronic gout. Which tablets should be prepared to teach about
side effects and precautions to this client?
A. *Allopurinol (Zyloprim)
B. Furosemide (Lasix)
C. Cortisol (prednisone)
D. Diphenhydramine (Benadryl)
E. Indometacin (Indocin)
1878. Which client is most likely to develop secondary gout?
A. *60-year-old man with renal insufficiency
B. 60-year-old woman with type 2 diabetes mellitus
C. 40-year-old woman who has been taking oral contraceptives for 15 years
D. 40-year-old male athlete who has been taking NSAIDs for a sprained ankle
E. 40-year-old drug abuser
1879. Which client is most likely to develop secondary gout?
A. *56-year-old man with Polycythemia vera
B. 56-year-old woman with type 2 diabetes mellitus
C. 33-year-old woman who has been suffering from SLE last 3 years
D. 40-year-old male athlete who has been taking NSAIDs for a sprained ankle
E. 40-year-old drug abuser
1880. Which of the following is characteristic of Osteoarthritis?
A. *Short starting stiffness
B. Severe night pain
C. Morning stiffness
D. Weight loss
E. Asymmetric joint involvement
1881. A 64-year-old woman presents with a 1-year history of pain in her thumbs. A focused examination
reveals squaring at the base of both first digits, worse on the right, and pain on pressure over the first carpal
metacarpal joints. She also has nontender bony overgrowth at the distal interphalangeal joints. The patient
says that her mother had “the same fingers” and she worries that she will become crippled. Choose the most
likely diagnosis from the below list of options.
A. *Osteoarthritis
B. Ankylosing spondylitis
C. Rheumatoid arthritis
D. Gout
E. Psoriatic arthritis
1882. A 64-year-old woman presents with a 1-year history of pain in her thumbs. A focused examination
reveals squaring at the base of both first digits, worse on the right, and pain on pressure over the first carpal
metacarpal joints. She also has nontender bony overgrowth at the distal interphalangeal joints. On physical
examination, there is crepitus on passive motion of the left knee, which has a slight varus (“bow leg”)
deformity. Her feet turn out slightly, and the heels are in valgus. Which of the following changes will reveal
X-ray examination?
A. Erosions
B. Subluxations
C. *Osteophytes
D. Osteoporosis
E. Ankylosis
1883. A 73-yr-old fit farmer presents with pain on weight bearing and restricted movements of the right hip.
Choose the most likely X-ray changes from the below List of options.
A. Cystic changes and lytic lesions
B. *Joint spaces are narrowed with reactive subchondral sclerosis, osteophytes
C. Periarticular osteopenia, marginal erosions
D. acroosteolysis
E. Fluffy bone periostitis and erosions at the interphalangeal joints
1884. A 68-year-old woman has had four or five episodes of joint pain and swelling, lasting 3 to 8 days,
involving the right knee and left elbow. She is asymptomatic between attacks, and sulindac, 200 mg twice
daily, has usually relieved the symptoms. Her most recent episode was 4 months ago. On physical
examination, none of her joints is swollen or tender, but there is marked crepitus on extension of the knee.
She also has a positive bulge sign over the left knee and pain on full extension of the left elbow. Which one
of the following tests would confirm the diagnosis?
A. Arthrocentesis of the knee and laboratory analysis of the synovial fluid
B. *Measurement of serum uric acid
C. Measurement of serum rheumatoid factor
D. Radiograph of the knee
E. MRI of the knee with gadolinium contrast
1885. A 50-yr-old woman complains of stiffness in her fingers worse at the end of the day. The DIP joints
and the first metacarpophalangeal joints are affected. Choose the most likely diagnosis from the below list of
options.
A. Rheumatoid arthritis
B. Psoriatic arthropathy
C. *Osteoarthritis
D. Haemochromatosis
E. Gonococcal arthritis
1886. A 65-yr-old man has started chemotherapy for lymphoma. He has developed a painful swollen hot
right knee. He is apyrexial. Choose the most likely diagnosis from the below list of options.
A. *Urate crystals on joint aspirate
B. High ESR
C. Erosions on X-ray
D. Positive blood culture
E. Positive rheumatoid factor
1887. Classical picture of acute gouty arthritis includes all of the following except
A. Excruciating and sudden pain
B. Stiffness in the joint
C. Low-grade fever may also be present
D. *Overgrowth at the distal interphalangeal joints
E. Warmness
1888. Which of the following medications is contraindicated for the treatment of an acute attack of gout?
A. *allopurinol
B. colchicine
C. corticosteroids
D. indomethacin
E. ibuprofen
1889. A 57-year-old woman has had five episodes of joint pain and swelling, lasting 3 to 8 days, involving
the right knee and left elbow. She is asymptomatic between attacks, and sulindac, 200 mg twice daily, has
usually relieved the symptoms. Her most recent episode was 4 months ago. Classical picture of exacerbation
of the arthritis includes all of the following except
A. Excruciating and sudden pain
B. Low-grade fever may also be present
C. Overgrowth at the distal interphalangeal joints
D. *Warmness
E. Redness
1890. A 57-year-old man presented to the clinic with a nodule on his elbow. He reported that the nodule
had been bothering him for 2 years and had steadily increased in size recently. Further questioning revealed
a history of repeated attacks of acute joint pain. The physical examination revealed a rounded, subcutaneous
nodule over the elbow, which was tender and rubbery to the touch. The examination was also notable for a
subcutaneous nodule at the left metatarsal-phalangeal joint and left metacarpal-phalangeal joint, as well as
evidence of arthritis involving both hands. Which of the following changes will reveal X-ray examination?
A. Erosions
B. Subluxations
C. Osteophytes
D. Osteoporosis
E. *"punched-out" lytic lesion
1891. A 55-year-old man presented to the clinic with a nodule on his elbow. He reported that the nodule
had been bothering him for 2 years and had steadily increased in size recently. Further questioning revealed
a history of repeated attacks of acute joint pain. The physical examination revealed a rounded, subcutaneous
nodule over the elbow, which was tender and rubbery to the touch. The examination was also notable for a
subcutaneous nodule at the left metatarsal-phalangeal joint and left metacarpal-phalangeal joint, as well as
evidence of arthritis involving both hands. Which of the following is characteristic of the arthritis?
A. *Depositions of monosodium urate monohydrate crystals within the periarticular soft tissues
B. Overgrowth of bones within joints
C. Autoimmune synovitis
D. Osteoporosis
E. Rheumatoid nodules over joints
1892. A 58-year-old man presented to the clinic with a nodule on his elbow. He reported that the nodule
had been bothering him for 4 years and had steadily increased in size recently. Further questioning revealed
a history of repeated attacks of acute joint pain. The physical examination revealed a rounded, subcutaneous
nodule over the elbow, which was tender and rubbery to the touch. The examination was also notable for a
subcutaneous nodule at the left metatarsal-phalangeal joint and left metacarpal-phalangeal joint, as well as
evidence of arthritis involving both hands. Which of the following medications is indicated for the treatment
of chronic arthritis?
A. *allopurinol
B. colchicine
C. corticosteroids
D. indomethacin
E. ibuprofen
1893. 79 y/o female with foot pain and swelling for many years. There are "punched out" articular erosions
with overhanging cortex and associated soft tissue masses. Serum uric acid was markedly elevated. Choose
the most likely diagnosis from the below list of options.
A. Ankylosing spondylitis
B. Rheumatoid arthritis
C. Osteoarthritis
D. *Chronic Gouty arthritis
E. Psoriatic arthritis
1894. Patients with gouty arthritis characterized by any of the following clinical characteristics except:
A. tophaceous deposits
B. *rheumatoid factor positive
C. "punched out" erosive changes on radiographs
D. acute attack frequency of greater than or equal to 2 attacks per year
E. hyperuricemia
1895. 73-year-old male with foot pain and swelling for many years. There are "punched out" articular
erosions with overhanging cortex and associated soft tissue masses consistent with tophaceous gout and
gouty arthritis. Choose the most likely changes from the below list of options.
A. *Urate crystals on joint aspirate
B. High ESR
C. Erosions on X-ray
D. Positive blood culture
E. Positive rheumatoid factor
1896. 79 y/o female with foot pain and swelling for many years. There are "punched out" articular erosions
with overhanging cortex and associated soft tissue masses. Serum uric acid was markedly elevated. Which of
the following medications is indicated for the treatment of chronic arthritis?
A. *allopurinol
B. colchicine
C. corticosteroids
D. indomethacin
E. ibuprofen
1897. 79 y/o female with foot pain and swelling for many years. There are "punched out" articular erosions
with overhanging cortex and associated soft tissue masses. Serum uric acid was markedly elevated. Which of
the following medications is indicated for the treatment of exacerbation of arthritis?
A. allopurinol
B. *colchicine
C. corticosteroids
D. indomethacin
E. ibuprofen
1898. 55 yo male with swelling and pain of his fingers. Patient also with similar complaints regarding his
great toe of the left foot. Soft tissue swelling with subtle soft tissue nodules primarily of the 2nd and 3rd
digits. Erosions with overhanging edges at the distal aspects of the proximal phalanges of the 2nd and 3rd
digits and metatarsophalangeal joints. Choose the most likely diagnosis from the below list of options.
A. Ankylosing spondylitis
B. *Chronic Gouty arthritis
C. Rheumatoid arthritis
D. Osteoarthritis
E. Psoriatic arthritis
1899. 57 yo male with swelling and pain of his fingers. Patient also with similar complaints regarding his
great toe of the left foot. Soft tissue swelling with subtle soft tissue nodules primarily of the 2nd and 3rd
digits. Erosions with overhanging edges at the distal aspects of the proximal phalanges of the 2nd and 3rd
digits and metatarsophalangeal joints. Which of the following medications is indicated for the treatment of
chronic arthritis?
A. colchicine
B. corticosteroids
C. indomethacin
D. *allopurinol
E. ibuprofen
1900. 59 yo female with swelling and pain of his fingers. Patient also with similar complaints regarding his
great toe of the left foot. Soft tissue swelling with subtle soft tissue nodules primarily of the 2nd and 3rd
digits. Erosions with overhanging edges at the distal aspects of the proximal phalanges of the 2nd and 3rd
digits and metatarsophalangeal joints. Which of the following medications is indicated for the treatment of
exacerbation of arthritis?
A. allopurinol
B. corticosteroids
C. *colchicine
D. indomethacin
E. ibuprofen
1901. 57 yo male with swelling and pain of his fingers. Patient also with similar complaints regarding his
great toe of the left foot. Soft tissue swelling with subtle soft tissue nodules primarily of the 2nd and 3rd
digits. Erosions with overhanging edges at the distal aspects of the proximal phalanges of the 2nd and 3rd
digits and metatarsophalangeal joints. Which of the following medications is contraindicated for the
treatment of exacerbation of arthritis?
A. corticosteroids
B. colchicine
C. indomethacin
D. ibuprofen
E. *allopurinol
1902. 52 yo male with chronic joint pain. There is asymmetric soft tissue swelling, predominantly in the
DIP of the right first digit, with calcifications. Further questioning revealed a history of repeated attacks of
acute joint pain. There is bony erosion at multiple DIP's of the right hand that is periarticular, well corticated
and has a thin overhanging edge. There is no associated osteopenia. Choose the most likely diagnosis from
the below list of options.
A. Ankylosing spondylitis
B. Erythema nodosum
C. Osteoarthritis
D. *Chronic gouty arthritis
E. Psoriatic arthritis
1903. 52 yo male with chronic joint pain. There is asymmetric soft tissue swelling, predominantly in the
DIP of the right first digit, with calcifications. Further questioning revealed a history of repeated attacks of
acute joint pain. There is bony erosion at multiple DIP's of the right hand that is periarticular, well corticated
and has a thin overhanging edge. There is no associated osteopenia. Which of the following medications is
indicated for the treatment of chronic arthritis?
A. colchicine
B. *allopurinol
C. corticosteroids
D. indomethacin
E. ibuprofen
1904. A 39-year-old male was admitted with one-day history of acute left lower quadrant pain, and was
diagnosed with acute uncomplicated diverticulitis, confirmed by computed tomography (CT) of the
abdomen. He denied smoking or illicit drug use, but admitted occasional alcohol use on every other
weekend. He did not follow any particular diet. He had an average built. He was started on intravenous
antibiotics and pain medication, which led to significant clinical improvement within two days.On the third
day of hospitalization, he developed acute, severe pain and swelling of the left elbow. Within next few hours,
pain worsened and he was unable to move the elbow joint, which was tender, erythematous, and swollen on
examination. Never investigated in the past, we also noted a firm 4 ? 6 cm mass on each elbow, and another
one surrounding the proximal inter-phalangeal joint of right middle finger. When asked particularly, he
noticed similar episodes of arthritis of great toe in the past. Which of the following medications is indicated
for the treatment of the arthritis?
A. allopurinol
B. corticosteroids
C. *colchicine
D. tetracycline
E. mydocalmi
1905. 86 year old male with chronic gout presents with bilateral foot pain. Physician desired radiograph for
evaluation of extent of pathologic changes. Prominent erosive destruction of bilateral first metatarsals and
second distal phalanx with well-preserved joint spaces throughout. Choose the most likely changes from the
below list of options.
A. High ESR
B. Oseoporosis on X-ray
C. Positive blood culture
D. *Urate crystals on joint aspirate
E. Positive rheumatoid factor
1906. Side effects of colchicines are all of the following except
A. Neutropenia
B. Nausea
C. Diarrhea
D. Anemia
E. *Headache
1907. Hyperuricemia is defined as a serum uric acid concentration above
A. *7 mg per dL (420 µmol per L).
B. 4 mg per dL (240 µmol per L).
C. 5 mg per dL (300 µmol per L).
D. 8 mg per dL (480 µmol per L).
E. 9 mg per dL (540 µmol per L).
1908. Overproduction of urate may be in all of the following except
A. Primary idiopathic hyperuricemia
B. *Essential hypertension
C. Hemolytic processes
D. Lymphoproliferative disease
E. Myeloproliferative disease
1909. Overproduction of urate may be in all of the following except
A. Polycythemia vera
B. Psoriasis (severe)
C. *Peptic ulcer
D. Exercise
E. Alcohol
1910. A 65-year-old man comes to your office complaining of knee pain that began insidiously about a year
ago. He has no other rheumatic symptoms. Which of the following signs isn’t sign of the arthritis?
A. Pain is related to use
B. Pain gets worse during the day
C. Minimal morning stiffness (<20 min) and after inactivity (gelling)
D. *Warmth of skin over effected joints
E. Range of motion decreases
1911. A 65-year-old man comes to your office complaining of knee pain that began insidiously about a year
ago. He has no other rheumatic symptoms. Which of the following signs isn’t sign of the arthritis? Which of
the following signs isn’t sign of the arthritis?
A. Joint instability
B. Bony enlargement
C. Restricted movement
D. Crepitus
E. *tophi
1912. All of the following are X-ray signs of Osteoarthritis except
A. Joint space narrowing
B. Marginal osteophytes
C. Subchondral cysts
D. Bony sclerosis
E. *Marginal erosions
1913. 55 years old, came to a musculoskeletal specialist seeking advice for a 3-year history of progressively
worsening pain in both knees. Her knees were stiff for about 20 minutes when she arose in the morning and
for a few minutes after getting up from a chair during the day. Her symptoms were worse on humid or cold
days, and she occasionally felt as if one of her knees would “give out.” Choose the most likely diagnosis
from the below list of options.
A. Ankylosing spondylitis
B. Erythema nodosum
C. *Osteoarthritis
D. Gout
E. Psoriatic arthritis
1914. 55 years old, came to a musculoskeletal specialist because she had difficulty walking > 30 minutes
because of pain, and her symptoms were exacerbated by kneeling, squatting, or descending stairs. Although
sitting, resting, and reclining relieved her pain, she became stiff if she stayed in one position for too long.
Her symptoms were worse on humid or cold days, and she occasionally felt as if one of her knees would
“give out.” Which of the following changes will reveal X-ray examination?
A. Erosions
B. Subluxations
C. *Osteophytes
D. Osteoporosis
E. Ankylosis
1915. Secondary osteoarthritis may be caused by all of the following except
A. Hemaphilia
B. Hemochromatosis
C. Gouty arthritis
D. *CLL
E. After trauma
1916. Risk factors and possible causes of osteoarthritis are all of the following except
A. Age
B. Obesity
C. *Stress
D. Prior injury
E. Acromegaly
1917. Signs of osteoarthritis are all of the following except
A. Limp
B. Muscle atrophy
C. Crepitus
D. Restricted movement
E. *Warmth of skin over effected joints
1918. A 65-year-old woman is evaluated because of a 1-year history of bilateral knee pain and low back
pain. She has some stiffness for approximately 15 minutes when she awakens in the morning, and during the
afternoon her pain is worse. There is slight crepitus with motion of the right knee. Which of the following
treatment do use?
A. methotrexat
B. *Intra-articular Hyaluronic Acid Injections
C. probenecid
D. colchicine
E. allopurinol.
1919. A 65-year-old woman is evaluated because of a 1-year history of bilateral knee pain and low back
pain. She has some stiffness for approximately 15 minutes when she awakens in the morning, and during the
afternoon her pain is worse. here is slight crepitus with motion of the right knee. All of the following are Xray signs of еру arthritis except
A. Joint space narrowing
B. Marginal osteophytes
C. Subchondral cysts
D. Bony sclerosis
E. *Marginal erosions
1920. A 68-year-old man is evaluated because of a 3-year history of bilateral knee pain and low back pain.
He has some stiffness for approximately 15 minutes when she awakens in the morning, and during the
afternoon her pain is worse. There is slight crepitus with motion of the right knee. Which of the following
treatment do use?
A. Methotrexat
B. *Glucosamine
C. Colchicines
D. Tetracycline
E. Allopurinol
1921. A 63-year-old man is evaluated because of a 2-year history of bilateral knee pain and low back pain.
He has some stiffness for approximately 15 minutes when she awakens in the morning, and during the
afternoon her pain is worse. On physical examination, he has slight swelling and tenderness to pressure of
the distal interphalangeal joints 2-5 on both hands. There is slight crepitus with motion of the right knee.
Which of the following treatment do use?
A. Methotrexat
B. Colchicines
C. *Chondroitin sulfate
D. Tetracycline
E. Allopurinol
1922. 70-year-old, married, teacher. She has been under treatment for hypertension and is finally stable on
medication. Initial Chief complaint: painful right knee, arthritis in right hip. She had had arthritic pain in her
right hip confirmed by x-ray about 10 years ago. The knee pain is a newer problem and she suspects it is also
arthritis. There is slight crepitus with motion of the right knee. Which of the following treatment do use?
A. Methotrexat
B. Colchicines
C. *Chondroitin sulfate
D. Tetracycline
E. Allopurinol
1923. 70-year-oldpatient. Chief complaint: painful right knee, arthritis in right hip. She had had arthritic
pain in her right hip confirmed by x-ray about 10 years ago. The knee pain is a newer problem and she
suspects it is also arthritis. There is slight crepitus with motion of the right knee. Which of the following
changes will reveal X-ray examination?
A. Erosions
B. Subluxations
C. *Osteophytes
D. Osteoporosis
E. Ankylosis
1924. A 68-year-old man presents with an acutely red and swollen right great toe without history of trauma.
Which of the following findings is most useful for making a diagnosis of gout in this patient?
A. Persistent elevation of serum uric acid
B. Good response to colchicines trial
C. *Radiograph showing marginal joint erosion in the first metatarso-phalangeal (MTP) joint
D. An associated right ankle effusion
E. A painless elbow nodule
1925. The best drug to treat SLE is :
A. Asprin
B. Indomethacin
C. Prostoglandines
D. Procainamide
E. *Steroids
1926. Which condition or health problem demonstrates inflammation without invasion?
A. Allergic rhinitis
B. Viral hepatitis
C. *Gouty arthritis
D. Cellulitis
E. Septic arthritis
1927. The following condition is not associated with an Anti-phopholipid syndrome:
A. Venous thrombosis
B. Recurrent fetal loss
C. *Thrombocytosis
D. Neurological manifestations
E. Recurrent thrombotic events
1928. Drug-induced systemic lupus erythematosus (SLE) can be characterized by all of the following
statements EXCEPT?
A. Twenty percent of patients receiving procainamide develop drug-induced lupus
B. *Most patients on hydralazine develop a positive antinuclear antibody (ANA) test; however, only 10
percent suffer from lupuslike symptoms
C. If patients with drug-induced lupus fail to respond within several weeks of discontinuing the offending
agent, a trial of corticosteroids is indicated
D. If a patient with drug-induced lupus has persistent symptoms for longer than 6 months, an anti-ds
antibody
E. None of the above
1929. For the Systemic lupus erythematosus choose the most likely diagnostic antibody from the below list
of options.
A. Antimicrosomal antibody
B. C-ANCA
C. *Anti-dsDNA
D. Antiparietal cell antibody
E. Antiacetylcholine receptor antibody
1930. For the Systemic lupus erythematosus choose the most likely diagnostic antibody from the below list
of options.
A. Antimicrosomal antibody
B. C-ANCA
C. *Antinuclear antibody
D. Antiparietal cell antibody
E. Antiacetylcholine receptor antibody
1931. In which of the following diseases test for anticentromere antibody is positive?
A. *Scleroderma
B. Mixed connective tissue disorder
C. SLE
D. Rheumatoid arthritis
E. Polymyalgia rheumatica
1932. A 20-yr-old woman presents with deep venous thrombosis in the left leg. Her past history includes
three miscarriages. Her blood tests show mild thrombocytopenia and a positive serology test for syphilis.
Choose the most likely diagnosis from the below list of options.
A. Takayasu's arteritis
B. Polymyalgia rheumatica
C. Mixed connective tissue disorder
D. *Antiphospholipid syndrome
E. Polyarteritis nodosa
1933. A 28-yr-old woman presents with a plethora of signs and symptoms. She complains of arthralgia,
depression, alopecia, fits, oral ulceration and facial rash. She is found to have proteinuria and normocytic
normochromic anaemia. Choose the most likely investigation from the above list of options.
A. *Antinuclear antibodies
B. ANCA
C. Antimitochondrial antibody
D. Antiplatelet antibody
E. Smooth muscle antibody
1934. A 38 –year old patient presents to clinic with melaena normal renal function, hypertension and
mononeuritis multiples. The most probable diagnosis is
A. *Classical polyarteritis nodosa
B. Microscopic polyangiitis
C. Henoch-Schonlein purpura
D. Buerger's disease
E. Wegener's granulomatosis
1935. A 28-yr-old woman presents with arthralgia, depression, alopecia, fits, oral ulceration and facial rash.
She is found to have proteinuria and normocytic normochromic anaemia. Choose the most likely results of
the investigations from the above list of options.
A. *Anti ds-DNA antibodies
B. ANCA
C. Antimitochondrial antibody
D. Antiplatelet antibody
E. Smooth muscle antibody
1936. A 35-yr-old woman presents with swellings and stiffness of her fingers. On examination she has
sausage-like fingers with flexion deformities. She is noted to have a beaked nose. X-ray of her hands reveals
deposits of calcium around her fingers and lysis of the distal phalanges. Choose the most likely diagnosis
from the below list of options.
A. Takayasu's arteritis
B. Polymyalgia rheumatica
C. SLE
D. Polyarteritis nodosa
E. *Scleroderma
1937. A 38-year-old man has had fever, weight loss, arthralgias, pleuritic chest pain, and midabdominal
pain for the last 3 months. One week ago he noticed difficulty dorsi flexing his left great toe. Blood pressure
is 160/95 mm Hg (he has always been normotensive), and laboratory studies reveal anemia of chronic
disease, high erythrocyte sedimentation rate, and polymorphonuclear leukocytosis. The chest x-ray is clear.
The most likely diagnosis is
A. giant cell arteritis
B. allergic granulomatosis
C. Wegener's granulomatosis
D. *polyarteritis nodosa
E. hypersensitivity vasculitis
1938. Low serum complement levels would be seen in patients with hematuria, proteinuria, and
hypertension resulting from all of the following EXCEPT
A. mixed essential cryoglobulinemia
B. hepatitis C-associated membranoproliferative glomerulonephritis
C. diffuse proliferative lupus nephritis
D. *Henoch-Schonlein purpura
E. poststreptococcal (or postinfectious) glomerulonephritis
1939. A 60-yr-old previously fit man presented with a 2-month history of fatigue, dyspnoea on exertion,
abdominal pain and progressive numbness in his feet. He recently developed mild polyarthritis in his hands.
On examination there was evidence of left median nerve mononeuritis. Chest radiography showed
cardiomegaly. Choose the most likely diagnosis from the below list of options.
A. Rheumatoid arthritis
B. Polymyalgia rheumatica
C. Polymyositis
D. *Polyarteritis nodosa
E. Scleroderma
1940. A 30-yr-old man presented with fever, myalgia and abdominal pain. On examination his temperature
was 38.7°C, BP was 200/120 and pulse was 130. His abdomen was tender with guarding and absent bowel
sounds. Choose the most likely diagnosis from the below list of options.
A. Kawaski's disease
B. Giant cell arteritis
C. Microscopic polyangitis
D. Antiphospholipid syndrome
E. *Polyarteritis nodosa
1941. A 15-yr-old boy was admitted with a tender swollen right knee and a tender right elbow. His past
history included recurrent sore throats and dull abdominal pain for a 5 days. On examination his temperature
was 37.9°C and there was some periumbilical tenderness. Both urine and stools were positive for blood.
Choose the most likely diagnosis from the below list of options.
A. Kawaski's disease
B. *Henoch-Schonlein purpura
C. Giant cell arteritis
D. Microscopic polyangitis
E. Antiphospholipid syndrome
1942. A 30-yr-old male presents with purpura, fever, malaise, weight loss and joint pain, chest pain. Choose
the single most likely treatment from the list of options given below.
A. Tetracycline
B. Ciprofloxacin
C. Beta 2 agonist
D. *Cyclophosphamide
E. IV aminophylline
1943. A 38-yr-old woman presents with fever, malaise, weight loss and joint pains. She has high blood
pressure and swelling feet. She complains of persistent chest pain, legs paresthesia. She has purpuric rash
and skin nodules on legs. Choose the most likely diagnosis from the below list of options.
A. Hyperparathyroidism
B. *PAN
C. Systemic sclerosis
D. Henoch-Schonlein purpura
E. Diabetes mellitus
1944. A 28-yr-old computer engineer with a long history of asthma and rhinitis presents with wheezing
cough and fever. A CXR shows patchy consolidation. Physical examination shows multiple tender
subcutaneous nodules and purpura. Choose the most likely diagnosis from the below list of options.
A. *Polyarteritis nodosa
B. Mitral stenosis
C. Pneumonia
D. SLE
E. Churg-Strauss syndrome
1945. A 13-yr-old boy presents with purpura around the buttocks and upper thighs following upper
respiratory tract infection. He also complains of arthralgia and abdominal pain. Choose the most likely
diagnosis from the below list of options.
A. Membranous glomerulonephritis
B. Idiopathic crescenteric GN
C. *Henoch-Schonlein purpura
D. Goodpasture's syndrome
E. Postinfectious glomerulonephritis
1946. Which of the following diseases occur primarily or exclusively in children?
A. Henoch-Schonlein purpura
B. Kawasaki’s disease
C. Behcet’s disease
D. Cogan’s syndrome
E. *Henoch-Schonlein purpura,
1947. A 13-year-old boy presents with a faint leg rash, bloody diarrhoea and oliguria. Further investigations
reveal heavy proteinuria and an elevated serum IgA choose the single most likely diagnosis from the below
list of options.
A. Membranous glomerulonephritis
B. Idiopathic crescenteric GN
C. *Henoch-Schonlein purpura
D. Goodpasture's syndrome
E. Postinfectious glomerulonephritis
1948. For a non-blanching maculopapular rash on flexor compartments of the lower limbs of a 13-yr-old
boy choose the single most likely diagnosis from the below list of options.
A. SLE
B. Nephroblastoma
C. Haemolytic uraemic syndrome
D. Juvenile chronic arthritis
E. *Henoch-Schonlein purpura
1949. Which of the following statements concerning Henoch-Schonlein purpura and renal disease is the
best answer?
A. Most children with Henoch-Schonlein purpura progress to chronic renal insufficiency.
B. *Adults with Henoch-Schonlein purpura need long-term follow-up to monitor for renal disease.
C. Renal disease is a result of monoclonal immunoglobulin G deposition in the glomerulus.
D. Most adults with Henoch-Schonlein purpura progress to chronic renal insufficiency.
E. None of the above
1950. A 17-yr-old boy has haematuria, hypertension, abdominal pain and a rash is noted on the flexor
surfaces of his lower limbs. Choose the single most likely diagnosis from the list of options below
A. *Henoch-Schonlein purpura
B. Kawasaki’s disease
C. Behcet’s disease
D. Cogan’s syndrome
E. None of the above
1951. Which of the following statements regarding the use of cyclophosphamide in the treatment of
rheumatic disorders such as lupus nephritis and vasculitis is correct?
A. The dose of cyclophosphamide should be increased until substantial leukopenia is observed.
B. The absorption of oral cyclophosphamide is poor, so the intravenous route is preferred.
C. Oral cyclophosphamide should be administered at night and fluids restricted in order to decrease the
formation of urine.
D. *Regular monitoring of the patient’s complete blood cell count and urinalysis is required.
E. None of the above
1952. ANCA is NOT associated with which of the following diseases:
A. Wegener's granulomatosis
B. *Henoch Schonlein purpura
C. Microscopic polyangiitis
D. Churg Strauss syndrome
E. drug-induced
1953. Which of the following isn’t characteristic of polyarteritis nodosa :
A. Weight loss greater than/equal to 4 kg.
B. Livedo reticularis.
C. *Ig-A immune deposit.
D. Muscle pain, weakness, or leg tenderness.
E. Nerve disease
1954. For Henoch-Schnlein purpura is characteristic:
A. Weight loss greater than/equal to 4 kg.
B. Livedo reticularis.
C. *Ig-A immune deposit.
D. Muscle pain, weakness, or leg tenderness.
E. Nerve disease
1955. In polyarteritis nodosa, aneurysms are seen in all, EXCEPT:
A. Kidney
B. *Spleen
C. Liver
D. Pancreas
E. Liver
1956. A patient presents with melaena normal renal function, hypertension and mononeuritis multiples. The
most probable diagnosis is
A. *Classical polyarteritis nodosa
B. Microscopic polyangiitis
C. Henoch-Schonlein purpura
D. Buerger's disease
E. Henoch-Schonlein purpura
1957. Henoch Schonlein purpura is characterized by all except:
A. *Thrombocytopenia
B. Glomerulonephritis
C. Arthralgia
D. Abdominal pain
E. Skin rash
1958. One of the following is a characteristic of Henoch - Schonlein Purpura:
A. *Blood in stool
B. Thrombocytopenia
C. Intracranial hemorrhage
D. Susceptibility to infection
E. Low ESR
1959. One of the following is a characteristic of Henoch - Schonlein Purpura:
A. *Ig-A immune deposit
B. Thrombocytopenia
C. Intracranial hemorrhage
D. Susceptibility to infection
E. Low ESR
1960. One of the following is a characteristic of Henoch - Schonlein Purpura:
A. *Skin purpura
B. Thrombocytopenia
C. Intracranial hemorrhage
D. Susceptibility to infection
E. Low ESR
1961. A 23 year old woman presents with bilateral conductive deafness, palpable purpura on the legs and
hemoptysis. Radiograph of the chest shows a thin-walled cavity in left lower zone. Investigations reveal total
leukocyte count 14000/mm3, red cell casts in the urine and serum creatinine 3 mg/dL. What is the most
probable diagnosis?
A. Henoch-Schonlein purpura
B. *Polyarteritis nodosa
C. Wegener's granulomatosis
D. Disseminated tuberculosis
E. Behcet's syndrome
1962. A 23year old woman presents with bilateral conductive deafness, palpable purpura on the legs and
hemoptysis. Radiograph of the chest shows a thin-walled cavity in left lower zone. Investigations reveal total
leukocyte count 14000/mm3, red cell casts in the urine and serum creatinine 3 mg/dL. Choose the single
most likely treatment from the list of options given below.
A. Tetracycline, prednisone
B. Ciprofloxacin, NSAIDs
C. Beta 2 agonist, heparin
D. *Cyclophosphamide, prednisone
E. IV aminophylline
1963. All of the following are primary vasculitis EXCEPT
A. Wegener’s granulomatosis
B. microscopic polyangiitis
C. Henoch-Schonlein purpura
D. *Rheumatoid vasculitis
E. polyarteritis nodosa
1964. A 25-yr-old computer engineer with malaise, fever, and weight loss. Physical examination shows
multiple tender subcutaneous nodules and livedo reticularis, peripheral neuropathy.
A. *Polyarteritis nodosa
B. leukocytoclastic vasculitis
C. Henoch-Schonlein purpura
D. Microscopic polyangiitis
E. Churg Strauss syndrome
1965. An 15-year-old man presents with abdominal pain, nausea, and vomiting. He also notes the onset of a
rash and painful joints. Physical examination is remarkable for the presence of palpable purpura distributed
over the buttocks and lower extremities as well as guaiac-positive stool. Laboratory evaluation is normal.
Other serum studies are normal. Skin biopsy would likely reveal
A. necrotizing angiitis
B. eosinophilic angiitis
C. *leukocytoclastic vasculitis
D. extravasated red blood cells without vasculitis
E. mast cell infiltration
1966. A 48-yr-old woman presents with fever, malaise, weight loss and joint pains. She has high blood
pressure and large feet. She complains of persistent chest pain. She has purpuric rash and skin nodules.
A. Acromegaly
B. *Polyarteritis nodosa
C. Systemic sclerosis
D. Herpes
E. leukocytoclastic vasculitis
1967. A 50-yr-old man presented with a 6-month history of fatigue, abdominal pain and progressive
numbness in his feet. He recently developed mild polyarthritis in his hands. On examination there was
evidence of right median nerve mononeuritis, livedo reticularis, BP 180/110 mmHg (he has always been
normotensive), and laboratory studies reveal anemia of chronic disease, high erythrocyte sedimentation rate.
Chest radiography showed cardiomegaly.
A. Rheumatoid arthritis
B. SLE
C. Antiphospholipid syndrome
D. Henoch-Schonlein purpura
E. *Polyarteritis nodosa
1968. A 38-yr-old man presented with fever, myalgia and abdominal pain and progressive numbness in his
feet. On examination his temperature was 38.9°C, BP was 200/120 and pulse was 128. His abdomen was
tender with guarding and absent bowel sounds. Choose the single most likely treatment from the list of
options given below.
A. Tetracycline, prednisone
B. Ciprofloxacin, NSAIDs
C. Beta 2 agonist, heparin
D. *Cyclophosphamide, prednisone
E. IV aminophylline
1969. Polyarteritis nodosa is vasculitis of …
A. *Medium-sized and small muscular arteries
B. Large vessels
C. Small vassels
D. Medium-sized vessel
E. Postcapillary venule
1970. Signs of kidney involvement in Polyarteritis nodosa are all of the following EXCEPT:
A. Systemic hypertension
B. Painless hematuria
C. *hydronephrosis
D. Intrarenal aneurysms
E. Multiple small cortical infarcts
1971. Signs of skin involvement in Polyarteritis nodosa are all of the following EXCEPT:
A. *dermatitis
B. livedo reticularis
C. ulcers
D. nodules
E. gangrene
1972. Signs of skin involvement in Polyarteritis nodosa are all of the following EXCEPT:
A. purpura,
B. livedo reticularis,
C. ulcers,
D. nodules
E. *papules
1973. Which isn’t characteristic for Polyarteritis nodosa?
A. Myocardial infarctions
B. *Erosive arthritis
C. Congestive heart failure
D. Systemic hypertension
E. Impaired kidney function
1974. In which of the following Hepatitis B surface antigen is +ve in 30%?
A. *polyarteritis nodosa
B. microscopic polyangiitis
C. leukocytoclastic angiitis
D. Wegener's granulomatosis
E. Henoch-Schonlein purpura
1975. Which of the following isn’t criterion for Polyarteritis nodosa?
A. Muscle pain, weakness, or leg tenderness.
B. Nerve disease (either single or multiple).
C. Diastolic blood pressure greater than 90mmHg (high blood pressure).
D. *Photosensitivity
E. Elevated kidney blood tests (BUN greater than 40 mg/dl or creatinine greater than 1.5 mg/dl).
1976. Hypersensitivity vasculitis effects mainly …
A. Medium-sized and small muscular arteries
B. Large vessels
C. Small vassels
D. Medium-sized vessel
E. *Postcapillary venule
1977. Hypersensitivity vasculitis effects mainly …
A. *vessels of skin
B. vessels of kidney
C. vessels of lung
D. coronary arteries
E. vessels of brain
1978. Small-vessel involvement with immune complex deposition are all of the following EXCEPT
A. Hypersensitivity vasculitis
B. Henoch-Schonlein purpura
C. *Polyarteritis nodosa
D. Behcet’s syndrome
E. Cryoglobulinemia
1979. IgA-dominant immune complexes is the cause of which of the following vasculitis
A. Hypersensitivity vasculitis
B. *Henoch-Schonlein purpura
C. Polyarteritis nodosa
D. Behcet’s syndrome
E. Cryoglobulinemia
1980. Henoch-Schonlein purpura is a disease that involves all of the following EXCEPT
A. purple spots on the skin
B. joint pain
C. gastrointestinal problems
D. *pneumonitis
E. glomerulonephritis
1981. Symptoms of Henoch-Schonlein purpura are all of the following EXCET
A. *Hemoptysis
B. Abdominal pain
C. Joint pain
D. Purple spots on the skin (purpura), usually over the buttocks, lower legs
E. Bloody stools
1982. Which of the following serologic studies isn’t included in laboratory studies of vasculitis?
A. antinuclear antibody
B. ANCA
C. rheumatoid factor
D. *HLA B 27
E. cryoglobulins
1983. Which of the following test isn’t included in laboratory studies of vasculitis?
A. complete blood count
B. erythrocyte sedimentation rate
C. *level of serum Ca
D. urinalysis
E. hepatitis B surface antigen
1984. A 25-year-old man had a 7 month history of progressive, painful, pruritic ulcers on both legs. He had
no evidence of systemic disease. A skin biopsy revealed a vasculitis involving medium size arteries. The
hyperpigmented patches represent the new phase of the lesions - tender dermal nodules. Which of the
following diagnoses is the most likely cause of this patient’s symptoms?
A. Lymphoma, with a paraneoplastic syndrome
B. Takayasus arteritis
C. Systemic lupus erythematosus
D. *Polyarteritis nodosa
E. Kawasaki’s disease
1985. A 25-year-old man had a 7 month history of progressive, painful, pruritic ulcers on both legs. He had
no evidence of systemic disease. A skin biopsy revealed a vasculitis involving medium size arteries. The
hyperpigmented patches represent the new phase of the lesions - tender dermal nodules. Choose the single
most likely treatment from the list of options given below.
A. *Prednisone
B. Tetracycline
C. Ciprofloxacin
D. Beta 2 agonist
E. IV aminophylline
1986. A 43 year old woman had a 25 year history of recurrent painful nodules on the medial aspect of her
ankles. Episodes lasted for 4-6 months with complete resolution of the rash during symptom free periods for
several months. His BP 160/110 mm Hg. Serological test showed Hepatitis B surface antigen is +ve. Which
of the following diagnoses is the most likely cause of this patient’s symptoms?
A. Lymphoma, with a paraneoplastic syndrome
B. Takayasus arteritis
C. Systemic lupus erythematosus
D. *Polyarteritis nodosa
E. Kawasaki’s disease
1987. A 43 year old woman had a 25 year history of recurrent painful nodules on the medial aspect of her
ankles. Episodes lasted for 4-6 months with complete resolution of the rash during symptom free periods for
several months. His BP 160/110 mm Hg. Serological test showed Hepatitis B surface antigen is +ve. Choose
the single most likely treatment from the list of options given below.
A. Tetracycline
B. *Prednisone
C. Ciprofloxacin
D. Beta 2 agonist
E. IV aminophylline
1988. A 41 year old woman has had fever, weight loss, arthralgias, his BP 160/110 mm Hg, temperature is
37.2 °C. Serological test showed Hepatitis B surface antigen is +ve. Choose the single most likely treatment
from the list of options given below.
A. Tetracycline
B. *Prednisone
C. Ciprofloxacin
D. Beta 2 agonist
E. IV aminophylline
1989. A 41 year old woman has had fever, weight loss, arthralgias, his BP 160/110 mm Hg, temperature is
37.2 °C. Serological test showed Hepatitis B surface antigen is +ve. Which of the following diagnoses is the
most likely cause of this patient’s symptoms?
A. Lymphoma, with a paraneoplastic syndrome
B. Takayasus arteritis
C. Systemic lupus erythematosus
D. *Polyarteritis nodosa
E. Kawasaki’s disease
1990. A 41 year old woman has had fever, weight loss, arthralgias, his BP 160/110 mm Hg, temperature is
37.2 °C. Serological test showed Hepatitis B surface antigen is +ve. Choose the single most likely treatment
from the list of options given below.
A. Tetracycline, Prednisone
B. Ciprofloxacin, Cyclophosphamide
C. Beta 2 agonist
D. *Prednisone, Cyclophosphamide
E. IV aminophylline
1991. A 40-year-old man developed purpuric papules on his abdomen and legs several weeks earlier. He
also complained of tea colored urine. A skin biopsy confirmed the presence of small vessel leukocytoclastic
vasculitis. Which of the following diagnoses is the most likely cause of this patient’s symptoms?
A. Systemic lupus erythematosus
B. *Henoch-Schonlein purpura
C. Polyarteritis nodosa
D. Lymphoma, with a paraneoplastic syndrome
E. Kawasaki’s disease
1992. Henoch-Schonlein purpura (HSP) is a small-vessel vasculitis characterized by all of the following
EXCEPT
A. Skin purpura
B. Arthritis
C. abdominal pain
D. hematuria
E. *epistaxis
1993. Henoch-Schonlein purpura (HSP) is a small-vessel vasculitis characterized by all of the following
EXCEPT
A. immunoglobulin A (IgA) deposition in capillaries
B. immune complex deposition in arterioles
C. *antibodies against cells of vascular wall
D. immune complex deposition in capillaries
E. immune complex deposition in venules
1994. IgA vasculitis is …
A. *Henoch-Schonlein purpura
B. Systemic lupus erythematosus
C. Polyarteritis nodosa
D. Kawasaki’s disease
E. microscopic polyangiitis
1995. Arthritis in Henoch-Schonlein purpura, most commonly involving
A. wrists
B. joints of hands
C. *knees and ankles
D. spine joints
E. hips and shoulders
1996. A leukocytoclastic vasculitis involves …
A. *predominantly skin
B. predominantly kidney
C. predominantly bowel
D. predominantly heart
E. predominantly joints
1997. Vessel damage in Vasculitis Syndromesathogenic immune complex formation and/or deposition are
all of the following EXCEPT
A. Henoch-Schonlein purpura
B. Vasculitis associated with collagen vascular diseases
C. Serum sickness and cutaneous vasculitis syndromes
D. Hepatitis B-associated polyarteritis nodosa
E. *Wegener's granulomatosis
1998. Signs of heart involvement in Polyarteritis nodosa are all of the following EXCEPT:
A. Congestive heart failure
B. myocardial infarction
C. pericarditis
D. *myocarditis
E. hypertrophy of LV
1999. Toxic Side Effects of cyclophosphamide are all of the following EXCEPT
A. Bone marrow suppression
B. *gastropathy
C. Cystitis
D. Bladder carcinoma
E. Gonadal suppression
2000. Toxic Side Effects of gluccocorticoids are all of the following EXCEPT
A. Osteoporosis
B. Cataracts
C. Glaucoma
D. *Gonadal suppression
E. Hypertension
2001. Predominantly cutaneous vasculitis (hypersensitivity vasculitis) are all of the following EXCEPT
A. *Wegener's granulomatosis
B. Drug-induced vasculitis
C. Serum sickness and serum sickness-like reactions
D. Vasculitis associated with infectious diseases
E. Vasculitis associated with connective tissue diseases
2002. In the acute stages of which disease, polymorphonuclear neutrophils infiltrate all layers of the vessel
wall and perivascular areas, which results in intimal proliferation and degeneration of the vessel wall is seen?
A. Henoch-Schonlein purpura
B. Systemic lupus erythematosus
C. *Polyarteritis nodosa
D. Kawasaki’s disease
E. microscopic polyangiitis
2003. A 37-year-old man has had fever, weight loss, arthralgias, and midabdominal pain for the last 2
months. One week ago he noticed difficulty dorsiflexing his right great toe. Blood pressure is 150/95 mm Hg
(he has always been normotensive), and laboratory studies reveal anemia of chronic disease, high
erythrocyte sedimentation rate, and polymorphonuclear leukocytosis. Which of the organs isn’t affected in
the disease?
A. Kidney
B. *Stomach
C. Liver
D. Pancreas
E. Liver
2004. A 39-year-old man has had fever, weight loss, arthralgias, and midabdominal pain for the last 3
months. One week ago he noticed difficulty dorsiflexing his right great toe. Blood pressure is 10/115 mm
Hg, and laboratory studies reveal anemia of chronic disease, high erythrocyte sedimentation rate, and
polymorphonuclear leukocytosis. He has been treated with prednisone 60 mm/d last month. Choose the
single most likely treatment from the list of options given below.
A. Tetracycline
B. *Ciprofloxacin
C. Plasmapheresis
D. Cyclophosphamide
E. IV aminophylline
2005. A 53-year-old woman was referred to hospital for recurrent palpable purpura and cutaneous ulcers
over her lower legs. Her disease started 3 years ago with erythematous papules and palpable purpuric lesions
located over her lower legs including the dorsa of her feet. In the second year of her disease, she experienced
similar lesions located on the upper back. The disease responded well to mid-potency topical steroids only to
recur shortly after stopping them. On her lower legs we found several cutaneous ulcers covered with
hemorrhagic crusts along with hyper- and hypopigmented atrophic scars and a few excoriated papules. The
rest of physical examination including accessible mucosal surfaces and peripheral pulses was normal. Which
of the following diagnoses is the most likely cause of this patient’s symptoms?
A. *Cutaneous small-vessel vasculitis
B. Henoch-Schonlein purpura
C. Systemic lupus erythematosus
D. Polyarteritis nodosa
E. Kawasaki’s disease
2006. The following condition is not associated with an Anti-phopholipid syndrome:
A. Venous thrombosis
B. Recurrent fetal loss
C. *Thrombocytosis
D. Neurological manifestations
E. Recurrent thrombotic events
2007. The best group of drugs to treat SLE is :
A. Asparcam
B. Movalis
C. Prostoglandines
D. Procainamide
E. *Steroids
2008. Drug-induced systemic lupus erythematosus (SLE) can be characterized by all of the following
statements EXCEPT?
A. Twenty percent of patients receiving procainamide develop drug-induced lupus
B. *Most patients on hydralazine develop a positive antinuclear antibody (ANA) test; however, only 10
percent suffer from lupuslike symptoms
C. If patients with drug-induced lupus fail to respond within several weeks of discontinuing the offending
agent, a trial of corticosteroids is indicated
D. If a patient with drug-induced lupus has persistent symptoms for longer than 6 months, an anti-ds
antibody
E. None of the above
2009. Anticentromere antibody is positive in which of the following diseases?
A. *Scleroderma
B. Mixed connective tissue disorder
C. SLE
D. Rheumatoid arthritis
E. Polymyalgia rheumatica
2010. A 48-year-old woman comes to hospital for recurrent palpable purpura and cutaneous ulcers over her
lower legs. Biopsy was taken from a periphery of the most recent, partially ulcerated, lesion and processed
for routine histopathology and direct immunofluorescence (DIF) study. Histopathology showed partly
ulcerated epidermis and foci of collagen necrosis with leukocytoclastic vasculitis of the dermal blood
vessels. Surprisingly, DIF disclosed intercellular deposits of IgG in the epidermis along with IgG, IgM, C3
and fibrinogen deposits around dermal blood vessels in her lesional skin. Only epidermal intercellular IgG
deposits were found in the clinically uninvolved skin. Which of the following diagnoses is the most likely
cause of this patient’s symptoms?
A. Henoch-Schonlein purpura
B. Systemic lupus erythematosus
C. Polyarteritis nodosa
D. Kawasaki’s disease
E. *Cutaneous small-vessel vasculitis
2011. For the Systemic lupus erythematosus choose the most likely diagnostic test from the below list of
options.
A. Antimicrosomal antibody
B. C-ANCA
C. *false positive test for syphilis
D. Antiparietal cell antibody
E. Antiacetylcholine receptor antibody
2012. For the Systemic lupus erythematosus choose the most likely diagnostic antibody from the below list
of options.
A. Antimicrosomal antibody
B. C-ANCA
C. *anti-Smith antibodies
D. Antiparietal cell antibody
E. Antiacetylcholine receptor antibody
2013. A 23-yr-old woman presents with deep venous thrombosis in the left leg. Her past history includes
three miscarriages. Her blood tests show mild thrombocytopenia and a positive serology test for syphilis.
Choose the most likely diagnosis from the below list of options.
A. Takayasu's arteritis
B. Polymyalgia rheumatica
C. Mixed connective tissue disorder
D. *Antiphospholipid syndrome
E. Polyarteritis nodosa
2014. A 28-yr-old woman presents with a plethora of signs and symptoms. She complains of arthralgia,
depression, alopecia, fits, oral ulceration and facial rash. She is found to have proteinuria and normocytic
normochromic anaemia. Choose the most likely investigation from the above list of options.
A. *Antinuclear antibodies
B. ANCA
C. Antimitochondrial antibody
D. Antiplatelet antibody
E. Smooth muscle antibody
2015. A 28-yr-old woman presents with a plethora of signs and symptoms. She complains of arthralgia,
depression, alopecia, fits, oral ulceration and facial rash. She is found to have proteinuria and a normocytic
normochromic anaemia. Choose the most likely results of the investigations from the above list of options.
A. *anti-Smith antibodies
B. ANCA
C. Antimitochondrial antibody
D. Antiplatelet antibody
E. Smooth muscle antibody
2016. A 43-yr-old woman presents with swellings and stiffness of her fingers. She is noted to have a beaked
nose. X-ray of her hands reveals lysis of the distal phalanges. Choose the most likely diagnosis from the
below list of options.
A. Takayasu's arteritis
B. Polymyalgia rheumatica
C. SLE
D. Polyarteritis nodosa
E. *Scleroderma
2017. Clinical findings in constrictive pericarditis may be all of the following EXCEPT:
A. Pulse pressure is normal or reduced
B. Edema
C. *Acute pulmonary edema
D. Distended cervical veins
E. Increased abdominal girth
2018. Which of the following sings is more prominent in constrictive pericarditis?
A. Edema
B. *Ascites
C. Pulmonary edema
D. Hypertension
E. Thromboembolism
2019. Which of the following investigation can’t help to distinguish constrictive pericarditis and cirrhosis of
the liver?
A. Distended neck veins
B. Chest roentgenography
C. chest CT
D. *EchoCG
E. ChestMRI
2020. Differential diagnosis of the constrictive pericarditis we should do with all of the following diseases
EXCEPT:
A. Tricuspid stenosis
B. Cor pulmonale
C. *Pericardial effusion
D. Cirrhosis of the liver
E. Restrictive cardiomyopathy
2021. Which of the following EchoCG changes is found in dilated cardiomyopathy?
A. *Left ventricular dilatation
B. Left ventricular diastolic dysfunction
C. Increased left ventricular wall thickness
D. Asymmetric septal hypertrophy
E. Systolic anterior motion (SAM) of the mitral valve
2022. What is the main cause of death in patient with dilated cardiomyopathy?
A. Pulmonary embolism
B. Stroke
C. Myocardial infarction
D. Pulmonary edema
E. *Fatal arrhythmias
2023. All of this drugs use for treatment of dilated cardiomyopathy except?
A. Diuretics
B. b-blockers
C. ACE inhibitors
D. *Warfarin
E. Aspirin
2024. Which of the following EchoCG changes is found in restrictive cardiomyopathy?
A. Left ventricular dilatation
B. Left ventricular diastolic dysfunction
C. *Increased left ventricular wall thickness
D. Asymmetric septal hypertrophy
E. Systolic anterior motion (SAM) of the mitral valve
2025. Which of the following EchoCG changes is found in hypertrophic cardiomyopathy?
A. Left ventricular dilatation
B. Left ventricular diastolic dysfunction
C. Increased left ventricular wall thickness
D. Symmetric septal hypertrophy
E. *Systolic anterior motion (SAM) of the mitral valve
2026. Standard therapy of heart failure in dilated myocardiopathy includes all of the following EXCEPT:
A. Salt restriction
B. Angiotensin-converting enzyme (ACE) inhibitors
C. *Calcium channel blockers
D. Diuretics
E. Digitalis
2027. Causes of Chronic Pericardial Effusion are all of the following EXCEPT:
A. Myxedema
B. SLE
C. Rheumatoid arthritis
D. Radiation therapy
E. *Viral
2028. In chronic constrictive pericarditis EchoCG changes are all of the following EXCEPT:
A. The ventricular end-diastolic and stroke volumes are reduced
B. The end-diastolic pressures in both ventricles are elevated
C. The mean pressure in the atria is elevated,
D. The mean pressures in pulmonic veins is elevated
E. *The left ventricle chamber is dilated
2029. All of the following may cause elevation of serum troponin EXCEPT:
A. Congestive heart failure
B. Myocarditis
C. Myocardial infarction
D. *Pneumonia
E. Pulmonary embolism
2030. All of the following conditions are associated with elevated RV diastolic pressures with the pattern
shown EXCEPT
A. *Cardiac tamponade
B. Constrictive pericarditis
C. Massive pulmonary embolism
D. Constrictive pericarditis
E. Restrictive cardiomyopathy
2031. A middle aged man presents with fever and variable heart murmur. He also complains of fever,
malaise and night sweats. On examination you find clubbing and splenomegaly. Choose the single most
likely diagnosis from the list of options below.
A. *Infective endocarditis
B. Pericardial effusion
C. SLE
D. Bacteraemia
E. Acute myocarditis
2032. A seventeen-year-old girl had a throat infection three weeks ago. Since then she has developed a
fever and an acute migratory polyarthritis involving the knees, ankles and wrists. Her parents have noticed
her legs twitching. She has a transient erythematous rash, with ring and crescent shaped patches. Which is
the SINGLE MOST likely diagnosis?
A. *Rheumatic fever
B. Allergic polyarthritis
C. Juvenile chronic arthritis
D. Lyme disease
E. Parvovirus
2033. In which disease we expect to see vegetations?
A. *Infective endocarditis
B. Viral myocarditis
C. Acute pericarditis
D. Rheumatic heart disease
E. Hypertrophied cardiomyopathy
2034. Etiology of rheumatic fever is:
A. *beta-hemolytic streptococci.
B. alfa-hemolytic streptococci
C. The HACEK group
D. H. influenza
E. Staphylococcus aureus
2035. What investigation can detect vegetations in the sizes of 2-3mm:
A. *Transoesophageal echocardiogram
B. MRI
C. ECG
D. CT
E. Chest X-ray
2036. Duration of antibiotic treatment of the patients with acute rheumatic fever?
A. *not less then two weeks
B. not less then 10 days
C. not less then a week
D. not less then 5 days
E. not less then a 3 month
2037. A 21-yr-old patient presents with a 2-week history of a swollen left knee, conjunctivitis and arthritis.
Choose the most likely diagnosis from the below list of options.
A. Ankylosing spondylitis
B. Psoriatic arthritis
C. Joint sepsis
D. *Reactive arthritis
E. Rheumatoid arthritis
2038. A 31-yr-old man presents with urethritis, conjunctivitis and a swollen left knee. Choose the most
likely diagnosis from the below list of options.
A. *Reiter's syndrome
B. Psoriatic arthropathy
C. Osteoarthritis
D. Reactive arthritis
E. Gonococci arthritis
2039. There is swelling of her MCP joints bilaterally and of wrists. There is a nodule on her left elbow.
Choose the most likely diagnosis from the below list of options
A. High ESR
B. High serum uric acid
C. Positive blood culture
D. *Positive rheumatoid factor
E. High WBC count
2040. All of the following are signs of rheumatoid arthritis (RA) except?
A. early morning stiffness lasting 2 hour
B. *numbness and blanching of fingers on exposure to cold
C. symmetrical involvement of the metacarpophalangeal joints
D. a non-tender subcutaneous mass over the Achilles tendon
E. synovial fluid with high viscosity and 30,000 lymphocytes/mm3
2041. 69-yr-old woman with longstanding polyarthritis on immunosuppressive drugs presents with
systemic malaise and fever and has severe pain, redness, heat and swelling of the wrist. Choose the most
likely diagnosis from the below list of options.
A. Ankylosing spondylitis
B. *Joint sepsis
C. Osteoarthritis
D. Gout
E. Cancer
2042. Which of the following is characteristic of rheumatoid arthritis
A. asymmetric oligoarticular arthritis
B. *symmetric polyarthritis of hand joints
C. arthritis of distal interphalangeal joints
D. severe destructive polyarthritis (arthritis mutilans)
E. sacroiliitis with or without peripheral arthritis
2043. Arthritis and urethritis is typical for:
A. Gout;
B. Rheumatoid arthritis ;
C. Psoriatic arthritis;
D. *Reiters syndrome;
E. ankylosing spondilitis;
2044. Choose the most likely diagnostic antibody for Rheumatoid arthritis from the below list of options.
A. Antimicrosomal antibody
B. C-ANCA antibody
C. Anti-dsDNA
D. P-ANCA antibody
E. *Anti -CCP antibody
2045. Drug of choice for patient with rheumatoid arhtritis
A. *Methotrexate
B. Diclofenac
C. Hydroxychloroquine sulphate
D. Leflunamide
E. D-penicillamine
2046. Felty’s syndrome occurs in patients with seropositive rheumatoid arthritis with all of the following
except
A. Neutropenia
B. Hypoproteinemia
C. Anemia
D. *lymphocytopenia
E. splenomegaly
2047. For arthritis in Rheumatoid arthritis diagnostic criterion is:
A. morning pain;
B. night pain;
C. *morning stiffness;
D. weight gain;
E. asymmetric joint involvement;
2048. Side-effects of NSAIDs include all of the following except
A. Peptic ulcer
B. upper gastrointestinal haemorrhage
C. *hypotension
D. interstitial nephritis
E. agranulocytosis
2049. Which clinical manifestation in a male with arthritis indicates or suggests the possibility of Reiter's
syndrome?
A. Stooped posture
B. Unplanned weight loss
C. *Pain/burning on urination
D. Ringing in the ears at night
E. Dry eyes
2050. Which clinical manifestation would be typical in a client with rheumatoid arthritis?
A. Anemia
B. *Warm, tender, swollen joints
C. asymmetric joint involvement
D. Elevated ESR
E. Crepitation of involved joints
2051. The best drug to treat RA is :
A. Aspirin
B. Indomethacin
C. Prostoglandines
D. NSAID’s
E. *Methotrexate
2052. Confirmation of SLE with which test?
A. Antimicrosomal antibody
B. C-ANCA
C. *Anti-dsDNA
D. P-ANCA
E. Antiacetylcholine receptor antibody
2053. For the Systemic lupus erythematosus choose the most suggestive diagnostic test from the below list
of options.
A. Antimicrosomal antibody
B. C-ANCA
C. * complement system levels
D. Antiparietal cell antibody
E. Antiacetylcholine receptor antibody
2054. Woman of 33 years with Raynauds phenomenon, polyarthritis, dysphagia of 6 years and mild
sclerodactyly, blood showing Anticentromere antibody positive, the most likely diagnosis is:
A. *Scleroderma
B. Sjogren’s syndrome
C. SLE
D. Rheumatoid arthritis
E. Polymyalgia rheumatica
2055. A 29-yr-old woman presents with alopecia and oral ulceration. Choose the single most likely
investigation to confirm diagnosis.
A. Serum calcium
B. Serum glucose
C. Serum electrolytes
D. Serum urea and creatinine
E. *Anti-dsDNA
2056. A 27-yr-old.patient with DVT of the left calf. She had a history of recurrent abortions and arthritis.
Choose the most likely diagnosis from the below list of options.
A. Protein C deficiency
B. Thrombotic thrombocytopenia
C. *Antiphospholipid syndrome
D. SLE
E. Polycythemia rubra vera
2057. A 27-yr-old.patient with fluorescent antinuclear antibody choose the single most likely diagnosis
from the below list of options.
A. Wegener's granulomatosis
B. Goodpasture's syndrome
C. Multiple myeloma
D. *Lupus nephritis
E. Diabetic neprtropathy
2058. The bad prognostic indicator for scleroderma is:
A. *Renal involvement
B. Dysphagia
C. Heart pain
D. Osteolysis
E. Arthritis
2059. Which of the following autoantibodies is more likely to be found in patients with systemic sclerosis?
A. anti-Smith (anti-Sm)
B. Anti-ribonucleoprotein (RNP)
C. *Anti SCL 70
D. Anti-CCP
E. Anticentromere
2060. Heart damage in SLE:
A. Acute Rheumatic corditis
B. *Limban Sack's endocarditis
C. Non thrombotic bacterial endocarditis
D. Chronic rheumatic carditis
E. MVP
2061. Anti-double stranded DNAand ANA ares highly specific for
A. Systemic sclerosis
B. *SLE.
C. Polymyositis
D. Dermatopolymyositis
E. Viral hepatitis
2062. Dysphagia, Raynaud's phenomenon, sclerodactyly, dyspnea, anticentromere antinuclear antibody are
characteristic for:
A. Systemic lupus erythematosis
B. *Systemic sclerosis
C. Mixed connective tissue disorder
D. Rheumatoid arthritis
E. Dermatopolymyositis
2063. Dysphagia, Raynaud's phenomenon, sclerodactyly, dyspnea. Choose the most likely results of the
investigations from the below list of options for confirming the diagnosis.
A. *Anticentromere antibody
B. Anti dsDNA antibodies
C. ANCA
D. Antimitochondrial antibody
E. Smooth muscle antibody
2064. Serologic tests should be performed in patients with specific musculoskeletal complaints for
assessing all the following EXCEPT:
A. rheumatoid factor
B. antinuclear antibodies (ANA)
C. uric acid level
D. *albumin
E. Anti-CCP
2065. Which of the following clinical features is NOT commonly found in limited systemic sclerosis?
A. Raynaud's phenomenon
B. Esophageal dysfunction
C. *Purpura
D. Sclerodactyly
E. Telangiectasia
2066. Cardiac involvement in SSC are all of the following EXCEPT
A. Pericardial effusion
B. Cor pulmonale
C. Conduction abnormalities
D. *Vegetations on undersurface of A.V. valves
E. arrhythmias,
2067. The bisphosphonate, alendronate, has clearly been established to be effective in randomised
controlled trials in which of the following conditions?
A. Osteomalacia
B. Bony metastases
C. Algodystrophy
D. *Postmenopausal osteoporosis
E. Hypercalcuria
2068. Allopurinol is used in the management of gout. Which statement below is correct?
A. Allopurinol is effective by increasing glomerular filtration of uric acid.
B. Allopurinol is effective by reducing the tubular reabsorption of uric acid.
C. *Allopurinol is effective by inhibiting the conversion of xanthine to uric acid.
D. Allopurinol is indicated in a 37 year old man with a uric acid concentration of 0.42 mg/l and a history
of one attack of gout.
E. Allopurinol is associated with aplastic anaemia in 1% of cases.
2069. All the following features favor ventricular tachycardia as the cause of broad-complex tachycardia,
EXCEPT:
A. QRS duration with right bundle branch block morphology (RBBB) is greater than 140 ms, and greater
than 160 ms with left LBBB morphology
B. LAD with RBBB morphology, extreme LAD (northwest axis) with LBBB morphology
C. AV dissociation
D. *Response to carotid sinus massage
E. Concordance of the QRS pattern in all precordial leads (i.e., all positive or all negative deflections)
2070. Which of the following will most likely respond to verapamil?
A. RVOT tachycardia
B. *Idiopathic LV tachycardia
C. Bundle branch re-entry tachycardia
D. ARVD ventricular tachycardia
E. All of the above
2071. The most common type of arrhythmia in WPW syndrome is....
A. Ventricular tachycardia
B. Ventricular fibrillation
C. Ventricular premature complexes
D. Atrial ectopics
E. *AV re-entry tachycardia
2072. Use of echocardiogram in the assessment of AF:
A. It is not necessary to perform a TTE in patients with AF
B. TEE is always needed to allow better assessment of the MV and LA
C. *TTE should be performed at baseline to diagnose associated structural/functional heart disease
D. TTE is necessary to determine stroke risk stratification for thromboprophylaxis in all patients
E. When TTE is technically difficult, MRI is needed to exclude cardiac abnormalities
2073. Which is the preferred initial treatment strategy for rate control in AF patients?
A. Digoxin
B. *Beta-blockers or rate limiting CCB
C. Combination therapy of beta-blockers and rate-limiting CCB’s
D. Amiodarone
E. Combination therapy of digoxin and beta-blockers
2074. Which option is better in regard to cardioversion?
A. *Synchronised DC shock to restore sinus rhythm
B. Amiodarone in all patients to restore sinus rhythm
C. Flecainide in patients with IHD to maintain sinus rhythm
D. Digoxin to restore sinus rhythm
E. Propaphenone to restore sinus rhythm in HCMP patients
2075. Regarding antithrombotic treatment in AF:
A. Patients with asymptomatic AF have less risk of embolism than symptomatic patients
B. Patients with atrial flutter do not need antithrombotic treatment due to the low risk of embolism of this
arrhythmia
C. *Warfarin is the best option in patients with high risk of thromboembolism
D. Aspirin has a similar effect in stroke reduction to placebo
E. Paroxysmal AF has the lowest risk of embolism
2076. Sudden cardiac death in patients >35 years old is most commonly associated with:
A. Hypertrophic cardiomyopathy
B. *Coronary artery disease
C. Congenital long QT syndrome
D. Long QT syndrome secondary to various medications
E. arrhythymogenic RV cardiomyopathy
2077. Congenital long QT syndrome can lead to
A. Complete heart block
B. *Polymorphic ventricular tachycardia
C. Acute myocardial infarction
D. Recurrent supraventricular tachycardia
E. All of the above
2078. Bundle branch re-entry ventricular tachycardia is most commonly associated with:
A. Enhanced automaticity in the right bundle
B. Enhanced automaticity in the left bundle
C. Supranormal conduction in the His bundle
D. *Abnormally slow conduction in the His bundle
E. All of the above
2079. All of the following are ECG manifestations of Wolff-Parkinson-White syndrome EXCEPT
A. *Narrow QRS
B. Initial slurred QRS upstroke
C. Short PQ interval
D. Normal QT interval
E. All of the above
2080. The initial manifestations of WPW syndrome may include which of the following?
A. Atrial fibrillation
B. AV re-entry tachycardia
C. Ventricular fibrillation
D. Wide-QRS tachycardia
E. *All of the above
2081. The presence of an r prime in V1 during narrow-QRS tachycardia is suggestive of...
A. AV re-entry
B. *AV node re-entry
C. Rate-dependent BBB
D. Atrial tachycardia
E. Atrial fibrillation
2082. Ventricular ectopic beats (VEBs) occurring in apparently normal hearts are:
A. Associated with a bad prognosis
B. Never frequent or complex
C. Less frequent with increasing age
D. *May be associated with symptoms of “missed beats”
E. Always symptomatic
2083. In patients with ventricular ectopic beats:
A. Caffeine restriction always reduces symptoms
B. *High caffeine intake should always be discouraged in symptomatic patients
C. There is good evidence that reducing caffeine intake reduces ectopics
D. Hypertension has no association with frequency of ectopic beats
E. LVH is not associated with increased frequency of ectopics
2084. Regarding ventricular ectopics and exercise:
A. *Ectopics are usually benign if they are supressed on exercise
B. Exercise-induced VEBs are not associated with adverse prognosis
C. RVOT ectopics are usually supressed on exercise
D. Ectopics induced on recovery after exercise are benign
E. All of the above
2085. An idiopathic left ventricular tachycardia (VT) shows the following...
A. Typical LBBB morphology
B. *LAD and RBBB morphology
C. Associated with a high risk of sudden cardiac death and almost always requires ICD therapy
D. Never responds to verapamil
E. All of the above
2086. Beta blockers are accepted treatment to prevent VT in:
A. Brugada syndrome, RVOT-VT, short QT syndrome and long QT syndrome
B. Brugada syndrome, RVOT-VT and long QT syndrome
C. *RVOT-VT and long QT syndrome
D. Short QT syndrome and long QT syndrome
E. Brugada syndrome and short QT syndrome
2087. Wenckebach phenomenon is defined as...
A. Progressive shortening of PR interval till a beat is dropped
B. *Progressive lengthening of PR interval till a beat is dropped
C. Irregular heart rate and PVC’s
D. Shortened QT intervals
E. Slurred QRS complex
2088. Giant a waves in JVP (“cannon” waves) are seen in
A. Sinus bradycardia
B. Atrial fibrillation
C. First-degree AV-block
D. *Complete heart block
E. Atrial ectopic
2089. All of the following findings are suggestive of LVH except:
A. (S in V1 + R in V5 or V6) > 35 mm
B. R in aVL > 11 mm
C. R in aVF > 20 mm
D. (R in I + S in III) > 25 mm
E. *R in aVR > 8 mm
2090. All of the following findings are suggestive of RVH except:
A. R in V1=7 mm (15 mm with RBBB)
B. (R in V1 + S in V5 or V6) > 10 mm
C. R < S in V6
D. R or R’ in V1
E. *S in V3 + R in aVL > 28 mm (20 mm for women)
2091. When present, each of the following heart sounds occurs shortly after S2 EXCEPT:
A. Opening snap
B. Third heart sound
C. *Ejection clic
D. Tumor plop
E. Pericardial knock
2092. Each of the following statements regarding splitting of the second heart sound is true EXCEPT:
A. Severe pulmonic valvular stenosis is associated with a softened P2
B. Delayed closure of the pulmonic valve with inspiration contributes to physiologic splitting of S2
C. Fixed splitting of S2 is the auscultatory hallmark of an ostium secundum atrial septal defect
D. Paradoxical splitting of S2 is expected in patients with a right ventricular electronically paced rhythm
E. *Right bundle branch block is associated with paradoxical splitting of S2
2093. The timing of an "innocent" murmur is usually:
A. Early systolic
B. Presystolic
C. *Midsystolic
D. Holosystolic
E. Early diastolic
2094. Which of the following pairs of medical conditions and antihypertensive medications would be
incorrect to use in a patient with essential hypertension?
A. Beta-blocker and a history of myocardial infarction
B. Alpha-blocker and prostatic hypertrophy
C. *Thiazide diuretic and gout
D. Amlodipine and heart failure
E. ACE-inhibitor and diabetes mellitus
2095. Thiazide diuretics can contribute to each of the following metabolic effects EXCEPT:
A. Hypomagnesemia
B. *Hypouricemia
C. Hypercalcemia
D. Hypercholesterolemia
E. Hyponatremia
2096. Which of the following statements regarding prevention of stroke is correct?
A. Treatment of systolic hypertension does not decrease the risk of stroke in patients older than age 60
B. Hypertension should not be a target of secondary prevention after an ischemic stroke because elevated
BP is desirable to maintain adequate cerebral perfusion
C. *Treatment with HMG-CoA reductase inhibitors reduces the risk of recurrent stroke
D. The combination of aspirin and clopidogrel is superior to aspirin alone for prevention of recurrent
stroke
E. None of the above
2097. You see a diabetic patient presents with BP readings that are 155/95 or higher. All of the following
statements about the treatment of this patient's hypertension are correct EXCEPT:
A. Pharmacologic blocade of the renin-angiotensin system reduces the risk of both microvascular and
macrovascular events
B. Aggressive BP control reduces cardiovascular events more in diabetics than in nondiabetics
C. *Calcium channel blockers show no benefit in reducing cardiovascular events
D. The goal BP for this patient is <130/80 mmHg
E. All the above statements are correct
2098. Which of the following antihypertensive agents is a known cause of autoimmune hemolytic anemia?
A. Metoprolol
B. *Methyldopa
C. Captopril
D. Losartan
E. Monoxidine
2099. All of the following characteristics are typical of hypertensive crisis EXCEPT:
A. Diastolic BP >120 mmHg
B. Retinal hemorrhages
C. *Constriction of cerebral arterioles with decreased vascular permeability
D. Proteinuria
E. Microangiopathic hemolytic anemia
2100. Which of the following pairs of medical conditions and antihypertensive medications would be
INCORRECT to use in a patient with essential hypertension?
A. Beta-blocker and a history of myocardial infarction
B. Alpha-blocker and prostatic hypertrophy
C. *Thiazide diuretic and gout
D. Amlodipine and heart failure
E. ACE-inhibitor and diabetes mellitus
2101. Which of the following statements regarding antihypertensive agents and atrial fibrillation (AF) is
TRUE?
A. *Losartan has been shown to decrease new-onset AF more effectively than atenolol
B. Valsartan has been shown to decrease new-onset AF more effectively than amlodipine
C. Atenolol has been shown to decrease new-onset AF more effectively than captopril
D. Nifedipine has been shown to decrease new-onset AF more effectively than diltiazem
E. All of the above are false
2102. Which lifestyle change has been shown to produce the biggest reduction in systolic blood pressure?
A. *10-kg weight loss
B. Dietary sodium reduction
C. Moderation of alcohol consumption
D. Change to a vegetarian diet
E. Magnesium supplement
2103. Which of these statements is true regarding renin-angiotensin system-blocking agents?
A. Less effective when combined with a diuretic than when used alone
B. More effective in patients of African descent than in white patients
C. *Preserve kidney function in addition to lowering blood pressure
D. Are the first-line antihypertensive medications in pregnancy
E. All of the above
2104. Which of the following antihypertensive agents is INCORRECTLY matched with the indication for
therapy?
A. ACE inhibitor – diabetic nephropathy
B. Beta-blocker – coronary artery disease
C. Calcium channel blocker – angina pectoris
D. * Hydrochlorthiazide – diabetes mellitus
E. Loop diuretic – heart failure
2105. Which of the following statements about microalbuminuria is TRUE?
A. To be of clinixal value, microalbuminuria must be measured in a timed 12- to 24-hour sample
B. *Microalbuminuria is a cardiovascular risk factor that is independent of traditional Framingham risk
factors
C. Microalbuminuria is a predictor of risk only in patients with diabetes
D. Microalbuminuria is present when the “spot” urine albumin-to-creatinine ration is >500 mg/g
E. All of the above statements are correct
2106. Renin-dependent hypertension includes:
A. Primary hyperaldosteronism
B. Essential hypertension
C. *Renovascular hypertension
D. Pheochromocytoma
E. Cushing syndrome
2107. Fundus picture of stage 2 of Keith-Wegener classification includes
A. *Copper wire AV nipping and focal spasm
B. Silver wire increased reflex
C. Exudate and hemorrhagic spots
D. Papilloedema
E. Retinal hemorrhages
2108. A young hypertensive patient has serum potassium 2.8 mEq/l and increased aldosterone level with
decreased plasma renin activity. The likely diagnosis is....
A. Renal artery stenosis
B. Ectopic ACTH syndrome
C. *Conn syndrome
D. Liddle syndrome
E. Cushing syndrome
2109. The proportion of essential (primary) hypertension among all hypertension causes is as high as
A. 25-30%
B. 40-45%
C. 60-65%
D. 70-75%
E. *90-95%
2110. All of the following are risk factors for hypertension, EXCEPT
A. Increased body weight
B. Family history of hypertension
C. Excessive intake of sodium
D. *Regular use of one glass of wine per day
E. Cigarette smoking
2111. All of the following statements concerning hypertension are true, EXCEPT
A. In a western adult population the prevalence of hypertension exceeds 20%
B. Hypertension is a major risk factor for cardiovascular and cerebrovascular disease
C. The pathophysiology of hypertension differs in black adults compared to South Asians and whites
D. People of African descent commonly have a high renin type hypertension*
E. An increase in potassium intake may significantly reduce blood pressure in hypertensive patients
2112. Optimal blood pressure (BP) is defined as a BP level of
A. *<120/80 mmHg
B. <130/80 mmHg
C. <140/90 mmHg
D. <160/100 mmHg
E. <125/80 mmHg
2113. Which parameter of arterial blood pressure (BP) yields the best prognostic information in patients at
risk for cardiovascular disease?
A. Systolic BP
B. Diastolic BP
C. Mean arterial BP
D. *Pulse pressure
E. None
2114. Which BP profile below identifies the patient with the highest risk for development of cardiovascular
complications?
A. 160/90 mmHg
B. *160/65 mmHg
C. 140/100 mmHg
D. 130/90 mmHg
E. 120/70 mmHg
2115. Physicians should diagnose isolated office hypertension (so-called “white-coat hypertension”)
whenever office BP is ≥140/90 mmHg at several visits, while 24-h ambulatory BP is
A. <160/100 mmHg
B. <140/90 mmHg
C. *<130/85 mmHg
D. <125/80 mmHg
E. <120/70 mmHg
2116. Target organs in hypertension include all of the following EXCEPT
A. Brain and eyes
B. Heart
C. Kidneys
D. Peripheral arteries
E. *Liver
2117. All of the following statements concerning target organ damage are true EXCEPT
A. Microalbuminuria is a sensitive marker of hypertension-induced renal damage
B. *An increase in serum creatinine when antihypertensive therapy is intensified is a sign of progressive
renal deterioration
C. Electrocardiography should be part of all routine assessment of subjects with high BP
D. Echocardiography is much more sensitive than electrocardiography in diagnosing left ventricular
hypertrophy
E. Grades 1 and 2 arteriolar retinal changes do not have a significant prognostic value
2118. The initial antihypertensive medication recommended for patients who have no compelling
indications or contraindications is
A. ACE-inhibitor
B. Calcium-channel blocker
C. Diuretics
D. Beta blocker
E. *Any of the above
2119. A 40-year-old diabetic patient presents with a blood pressure (BP) of 145/90 mmHg and proteinuria.
Which BP profile represents the best therapeutic goal for this patient?
A. <160/90
B. <140/90
C. *<130/85
D. <125/75
E. <140/85
2120. A 40-year-old diabetic patient presents with a blood pressure (BP) of 145/90 mmHg and proteinuria.
What is the best medication for the initial management of this patient’s hypertension?
A. Calcium channel blockers
B. Beta blockers
C. *ACE-inhibitors / angiotensin receptor blockers
D. Alpha blockers
E. Diuretics
2121. In patients with a history of stroke or TIA the preferred drug combination is:
A. *ACE-inhibitor and diuretic
B. Calcium-channel blocker and beta-blocker
C. Beta-blocker and diuretic
D. Angiotensin receptor blocker and beta blocker
E. Beta- and alpha blocker
2122. In the elderly with systolic hypertension antihypertensive therapy should be initiated if SBP
A. ≥120 mmHg
B. ≥130 mmHg
C. *≥140 mmHg
D. ≥160 mmHg
E. ≥170 mmHg
2123. The best medication for the treatment of isolated systolic hypertension in the elderly:
A. ACE-inhibitor and diuretic
B. *Dihydropiridine calcium-channel blocker and diuretic
C. Beta-blocker and diuretic
D. Non-dihydropiridine calcium-channel blocker and diuretic
E. Beta- and alpha blocker
2124. The following medications can be used in pregnancy EXCEPT
A. Methyldopa
B. Labetalol
C. *ACE-inhibitor / ARB
D. Hydralazine
E. Nifidipine
2125. In patients of African descent the drug of choice for initial treatment of hypertension is:
A. Verapamil
B. Atenolol
C. Perindopril
D. *Hydrochlorothiazide
E. Amlodipine
2126. Which of the following antihypertensive agents is INCORRECTLY matched with the indication for
therapy?
A. ACE inhibitor – diabetic nephropathy
B. Beta-blocker – coronary artery disease
C. Calcium channel blocker – angina pectoris
D. Diuretics – heart failure
E. *Loop diuretic – gout
2127. All the following patients should be evaluated for secondary causes of hypertension EXCEPT
A. *A 47-year-old male with strong family history hypertension who presents to your office with a BP of
150/100
B. A 26-year-old female with hematuria and a family history of early renal failure who has a BP of 160/90
C. A 73-year-old male with no past history with a BP of 165/90
D. A 58-year-old male with a history of hypertension since age 45 whose BP has become increasingly
difficult to control on four antihypertensive agents
E. A 31-year-old female with complaints of severe headaches, weight gain, and new-onset DM with a BP
of 145/90
2128. Which of the following diagnostic studies is most likely to demonstrate the cause of the headaches?
A. MRI of the head
B. MRI of the kidney
C. *MRI of the thorax
D. 24-h urinary 5-HIAA
E. 24-h urinary free cortisol
2129. A 27-year-old female has hypertension that is difficult to control. She was diagnosed at age 25. Since
that time she has been on increasing amounts of medication. Her current regimen consists of labetalol 1000
mg bid, lisinopril 40 mg qd, clonidine 0.1 mg bid, and amlodipine 5 mg qd. On physical examination she
appears to be without distress. Her BP is 170/100, and HR is 84 bpm. Cardiac exam is unremarkable,
without rubs, gallops, or murmurs. She has good peripheral pulses and has no edema. Laboratory studies
reveal a potassium of 2.8 mEq/dL and a serum bicarbonate of 32 mEq/dL. Fasting blood glucose 114 mg/dL.
What is the likely diagnosis?
A. Congenital adrenal hyperplasia
B. Fibromuscular dysplasia
C. Cushing’s syndrome
D. *Conn’s syndrome
E. Pheochromocytoma
2130. A 27-year-old female has hypertension that is difficult to control. She was diagnosed at age 25. Since
that time she has been on increasing amounts of medication. Her current regimen consists of labetalol 1000
mg bid, lisinopril 40 mg qd, clonidine 0.1 mg bid, and amlodipine 5 mg qd. On physical examination she
appears to be without distress. Her BP is 170/100, and HR is 84 bpm. Cardiac exam is unremarkable,
without rubs, gallops, or murmurs. She has good peripheral pulses and has no edema. Laboratory studies
reveal a potassium of 2.8 mEq/dL and a serum bicarbonate of 32 mEq/dL. Fasting blood glucose 114 mg/dL.
What is the best way to diagnose this disease ?
A. Renal vein renin levels
B. 24-h urine collection for metanephrines
C. MRI of the renal arteries
D. 24-h urine collection for cortisol
E. *Plasma aldosterone/renin ratio
2131. A 20-year-old female is seen in the ED with symptoms of periodic headaches, sweating, and nausea
with vomiting. She also complains of feeling light-headed with standing. Her BP on presentation is 230/135,
with a HR of 92. On standing, the patient has BP of 205/100, with a HR of 136. On ophthalmologic
examination the patient has mild blurring of the optic discs without hemorrhage. The examination is
otherwise normal. What is the best medication for the management of this patient’s hypertension?
A. *Phentolamine
B. Fenoldopam
C. Esmolol
D. Nicardipine
E. Diazoxide
2132. A 20-year-old female is seen in the ED with symptoms of periodic headaches, sweating, and nausea
with vomiting. She also complains of feeling light-headed with standing. Her BP on presentation is 230/135,
with a HR of 92. On standing, the patient has BP of 205/100, with a HR of 136. On ophthalmologic
examination the patient has mild blurring of the optic discs without hemorrhage. The examination is
otherwise normal. What test would best determine the patient’s diagnosis?
A. Plasma catecholamines
B. 24-h urine collection for 5-HIAA
C. Abdominal CT scan
D. *24-h urine collection for metanephrines and vanilylmandelic acid
E. Adrenal vein sampling for renin levels
2133. What is the correct feature of the hypertensive urgency?
A. It is always a life-threatening situation
B. Patient must be hospitalized
C. *Patients can be managed as outpatients
D. Therapy should not be given orally
E. None of the above
2134. If there is suspicion of phaeochromocytoma, the first line drug is:
A. Sodium-nitroprusside
B. Nifedipine
C. *Phentolamine
D. Verapamil
E. Enalapril
2135. If you are in your office and a patient sees you with the following symptoms: long history of
hypertension, home BP usually 160/100 mmHg, suddenly developed severe headache, blurred vision,
weakness in the right arm, blood pressure: 200/110 mmHg. What will you do:
A. Give the patient 20 mg nifedipine orally, and check the BP in 15 min. If it is still >200/100 mmHg,
give another dose of nifedipine
B. Give the patient 20 mg nifedipine, and check the BP in 30 min. If it is 160/80 mmHg, send the patient
home and give another appointment in two weeks
C. Give the patient 10 mg amlodipine, send the patient home and give another appointment in two days
D. Call the ambulance and send the patient to the nearest hospital
E. *Give the patient some pain-killer drug (e.g. acetaminophen), call the ambulance and send the patient
to the nearest hospital with stroke-unit
2136. A 28-year-old male with type 1 diabetes mellitus is seen in the clinic for routine follow-up. The
patient has hypertension. He is being maintained on an insulin pump and lisinopril 5 mg daily. He has been
on this dose of lisinopril for approximately 6 weeks. He has normal creatinine and no proteinuria. BP is
140/90 mmHg. What do you recommend ?
A. Continue the current dose of lisinopril and return for follow-up in 3 months as the patient has received
his target BP of less than 140/90 mmHg
B. Ask a patient to return in 6 weeks; if his BP remains at this level, increase the dose of lisinopril
C. *Increase lisinopril to 10 mg daily to achieve a goal BP of less than 130/80 mmHg
D. Assure the patient that he is suffering from “white-coat hypertension” and plan no intervention
E. Switch the patient from lisinopril to losartan
2137. A 49-year-old female presents to you for an initial visit to establish health care. She has no past
medical history and denies a family history of diabetes mellitus, early cardiovascular disease, or renal
disease. Her BP on presentation is 185/110 mmHg. She denies headache or visual complaints. She has no
chest pain, shortness of breath, or change in urine color. Her physical examination is normal with the
exception of an S4 gallop heard at the 4th intercostal space just to the left of the sternum. An ECG is
significant for LVH and LAD. there are no signs of cardiac ischemia. Urinalysis shows no red blood cells or
proteinuria. What is your recommendation for therapy for this patient?
A. *Initiate therapy with hydrochlorothiazide in combination with a beta-blocker
B. Initiate therapy with hydrochlorothiazide alone
C. Initiate therapy with an ACE inhibitor alone
D. Advise the patient to go to the nearest emergency department for further treatment for a hypertensive
emergency
E. Advise the patient on lifestyle modifications and plan a return visit in 3 weeks
2138. What will you do if you are on duty in a hospital and a hypertensive patient, who has been treated
with long-acting ß-blocker, long-acting dihydropyridine calcium antagonist and long-acting a-1 adrenoceptor
blocker, is admitted with the following symptoms: headache, dyspnea, ankle-swelling, BP 180/110 mmHg,
pulmonary rales, LVH on ECG, echo: LVH, left ventricular diastolic dysfunction, ejection fraction 46%.
A. Amlodipine orally
B. Nifedipine orally
C. Enalaprilate iv
D. *Enalaprilate iv + furosemide iv
E. Esmolol iv
2139. Which of the following blood pressure levels in pregnancy should be considered an emergency
requiring hospitalization?
A. ≥140/90 mmHg
B. ≥150/95 mmHg
C. ≥160/100 mmHg
D. *≥170/110 mmHg
E. A rise in SBP ≥25 mmHg or a rise in DPB ≥15 mmHg compared to pre-pregnancy values, or those in
the first trimester
2140. The following drug combinations are effective to treat hypertension EXCEPT
A. Diuretics + beta-blockers
B. Beta-blockers + dihydropiridine calcium channel blockers
C. Diuretics + ACE-inhibitors/ARB
D. ACE-inhibitors/ARB + calcium channel blockers
E. *Beta-blockers + ACE-inhibitors/ARB
2141. Each of the following statements regarding hypertension is true EXCEPT:
A. Pure "white coat hypertension is found in 20 to 30% of patients
B. *When measuring BP, an inappropriately small cuff size results in a spuriously low systolic
measurement
C. Pseudohypertension may occur in patients with sclerotic brachial arteries
D. Chronic renal disease is the second most common cause of hypertension after essential hypertension
E. Coarctation of the aorta, Cushing disease, primary aldosteronism account for approximately 1% of all
hypertensive patients
2142. With respect to renovascular disease, all of the statements are true EXCEPT:
A. Worsening renal function with ACE-inhibitor therapy suggests bilateral renovascular disease
B. Atherosclerotic disease most commonly involves the proximal third of the main renal artery
C. The most common form of fibroplastic renovascular disease in adults involves the media
D. *The incidence of renovascular hypertension falls with increased age
E. Patients with severe, accelerated hypertension have the highest prevalence of renovascular disease
2143. A newly diagnosed diabetic patient presents with BP readings that are 155/95 or higher. All of the
following statements about the treatment of this patient's hypertension are correct EXCEPT:
A. Pharmacologic blockade of the renin-angiotensin system reduces the risk of both microvascular and
macrovascular events
B. Aggressive BP control reduces cardiovascular events more in diabetics than in nondiabetics
C. *Calcium channel blockers show no benefit in reducing cardiovascular events
D. The goal BP for this patient is <130/80 mmHg
E. All the above statements are correct
2144. All of the following statements regarding the association of oral contraceptive pills and hypertension
are correct EXCEPT:
A. The likelihood of developing hypertension is increased by significant alcohol consumption
B. The incidence of hypertension is about twice as great in pill users as in non-users
C. The likelihood of developing hypertension is dependent on the age of the user
D. *Hypertension resolves in almost all cases after discontinuation of oral contraceptives
E. The mechanism for contraceptive-induced hypertension likely involves renin-aldosterone-mediated
volume expansion
2145. Each of the following statements regarding pheochromocytoma is true EXCEPT:
A. Approximately 15% of pheochromocytomas are extra-adrenal
B. Ten percent of pheochromocytomas are malignant
C. Ten percent of adrenal pheochromocytomas are bilateral
D. *Hypertension related to pheochromocytoma is nearly always episodic and only rarely persistent
E. Pheochromocytoma can be inherited
2146. All of the following are features of renovascular hypertension due to fibromuscular hyperplasia, as
opposed to atherosclerosis, EXCEPT:
A. Age <50 years
B. Female gender
C. *More likely to progress to complete renal artery occlusion
D. No family history of hypertension
E. Absence of carotid bruits
2147. Which of the following ACE-inhibitors are not prodrugs?
A. Captopril, lisinopril, ramipril
B. Lisinopril, enalapril, banazepril
C. *Captopril, lisinopril, enalaprilat
D. Moexipril, captopril, lisinopril
E. Perindopril, ramipril, enalapril
2148. Each of the following statements regarding hypertension is true EXCEPT:
A. The prevalence of hypertension rises progressively with age in both men and women
B. Systolic and diastolic hypertension are each associated with an increased crisk of coronary heart
disease
C. *The target blood pressure for hypertensive patients with cardiovascular disease or diabetes is 140/90
mmHg
D. Patients with prehypertension (SBP 120-139 mmHg or DBP 80-89 mmHg) benefit from lifestyle
modifications incl. weight reduction, regular exercise, and smoking cessation
E. Drug therapy of hypertension benefits patients greater than 80 years of age
2149. All of the following interventions have a BP-lowering effect EXCEPT:
A. A diet that reduces caloric intake by 1000 calories per day
B. Reduction of dietary sodium
C. *Daily magnesium supplements
D. Tobacco cessation
E. Reduction of ethanol consumption to less than 30 ml/day
2150. Each of the following statements regarding hypertension is true EXCEPT:
A. Pure "white coat hypertension is found in 20 to 30% of patients
B. *When measuring BP, an inappropriately small cuff size results in a spuriously low systolic
measurement
C. Pseudohypertension may occur in patients with sclerotic brachial arteries
D. Chronic renal disease is the second most common cause of hypertension after essential hypertension
E. Coarctation of the aorta, Cushing disease, primary aldosteronism account for approximately 1% of all
hypertensive patients
2151. Each of the following statements regarding hypertension is true EXCEPT:
A. Increased LV muscle mass in hypertension is an independent risk factor of cardiac mortality
B. The risk of ventricular arrhythmias in hypertension is increased 2-fold in the presence of LV
hypertrophy
C. Low birth weight is associated with an increased incidence of hypertension later in life
D. *Chronic caffeine consumption is associated with an increased risk of hypertension
E. In hypertensive adolescents, essential hypertension is the most likely etiology
2152. The following statements regarding the association of oral contraceptive pills and hypertension are
correct EXCEPT:
A. The likelihood of developing hypertension is increased by significant alcohol consumption
B. The incidence of hypertension is about twice as great in pill users as in non-users
C. The likelihood of developing hypertension is dependent on the age of the user
D. *Hypertension resolves in almost all cases after discontinuation of oral contraceptives
E. The mechanism for contraceptive-induced hypertension likely involves renin-aldosterone-mediated
volume expansion
2153. Thiazide diuretics can contribute to each of the following metabolic effects EXCEPT:
A. Hypomagnesemia
B. *Hypouricemia
C. Hypercalcemia
D. Hypercholesterolemia
E. Hyponatremia
2154. Which of the following statements regarding prevention of stroke is correct?
A. Treatment of systolic hypertension does not decrease the risk of stroke in patients older than age 60
B. Hypertension should not be a target of secondary prevention after an ischemic stroke because elevated
BP is desirable to maintain adequate cerebral perfusion
C. *Treatment with HMG-CoA reductase inhibitors reduces the risk of recurrent stroke
D. The combination of aspirin and clopidogrel is superior to aspirin alone for prevention of recurrent
stroke
E. None of the above
2155. Which of the following antihypertensive agents is a known cause of autoimmune hemolytic anemia?
A. Metoprolol
B. *Methyldopa
C. Captopril
D. Losartan
E. Monoxidine
2156. All of the following characteristics are typical of hypertensive crisis EXCEPT:
A. Diastolic BP >120 mmHg
B. Retinal hemorrhages
C. *Constriction of cerebral arterioles with decreased vascular permeability
D. Proteinuria
E. Microangiopathic hemolytic anemia
2157. Which of the following pairs of medical conditions and antihypertensive medications would be
INCORRECT to use in a patient with essential hypertension?
A. Beta-blocker and a history of myocardial infarction
B. Alpha-blocker and prostatic hypertrophy
C. *Thiazide diuretic and gout
D. Amlodipine and heart failure
E. ACE-inhibitor and diabetes mellitus
2158. Which of the following statements regarding antihypertensive agents and atrial fibrillation (AF) is
TRUE?
A. *Losartan has been shown to decrease new-onset AF more effectively than atenolol
B. Valsartan has been shown to decrease new-onset AF more effectively than amlodipine
C. Atenolol has been shown to decrease new-onset AF more effectively than captopril
D. Nifedipine has been shown to decrease new-onset AF more effectively than diltiazem
E. All of the above are false
2159. Which lifestyle change has been shown to produce the biggest reduction in systolic blood pressure?
A. *10-kg weight loss
B. Dietary sodium reduction
C. Moderation of alcohol consumption
D. Change to a vegetarian diet
E. Magnesium supplement
2160. Which of these statements is true regarding renin-angiotensin system-blocking agents?
A. Less effective when combined with a diuretic than when used alone
B. More effective in patients of African descent than in white patients
C. *Preserve kidney function in addition to lowering blood pressure
D. Are the first-line antihypertensive medications in pregnancy
E. All of the above
2161. Which of the following antihypertensive agents is INCORRECTLY matched with the indication for
therapy?
A. ACE inhibitor – diabetic nephropathy
B. Beta-blocker – coronary artery disease
C. Calcium channel blocker – angina pectoris
D. *Hydrochlorthiazide – diabetes mellitus
E. Loop diuretic – heart failure
2162. Which of the following statements about microalbuminuria is TRUE?
A. To be of clinixal value, microalbuminuria must be measured in a timed 12- to 24-hour sample
B. *Microalbuminuria is a cardiovascular risk factor that is independent of traditional Framingham risk
factors
C. Microalbuminuria is a predictor of risk only in patients with diabetes
D. Microalbuminuria is present when the “spot” urine albumin-to-creatinine ration is >500 mg/g
E. All of the above statements are correct
2163. Renin-dependent hypertension includes:
A. Primary hyperaldosteronism
B. Essential hypertension
C. *Renovascular hypertension
D. Pheochromocytoma
E. Cushing syndrome
2164. Fundus picture of stage 2 of Keith-Wegener classification includes
A. *Copper wire AV nipping and focal spasm
B. Silver wire increased reflex
C. Exudate and hemorrhagic spots
D. Papilloedema
E. Retinal hemorrhages
2165. A young hypertensive patient has serum potassium 2.8 mEq/l and increased aldosterone level with
decreased plasma renin activity. The likely diagnosis is....
A. Renal artery stenosis
B. Ectopic ACTH syndrome
C. *Conn syndrome
D. Liddle syndrome
E. Cushing syndrome
2166. The proportion of essential hypertension among all hypertension causes is as high as
A. 25-30%
B. 40-45%
C. 60-65%
D. 70-75%
E. *90-95%
2167. All of the following are risk factors for hypertension, EXCEPT
A. Increased body weight
B. Family history of hypertension
C. Excessive intake of sodium
D. *Regular use of one glass of wine per day
E. Cigarette smoking
2168. All of the following statements concerning hypertension are true, EXCEPT
A. In a western adult population the prevalence of hypertension exceeds 20%
B. Hypertension is a major risk factor for cardiovascular and cerebrovascular disease
C. The pathophysiology of hypertension differs in black adults compared to South Asians and whites
D. *People of African descent commonly have a high renin type hypertension
E. An increase in potassium intake may significantly reduce blood pressure in hypertensive patients
2169. Optimal blood pressure (BP) is defined as a BP level of
A. *<120/80 mmHg
B. <130/80 mmHg
C. <140/90 mmHg
D. <160/100 mmHg
E. <125/80 mmHg
2170. Which parameter of arterial blood pressure (BP) yields the best prognostic information in patients at
risk for cardiovascular disease?
A. Systolic BP
B. Diastolic BP
C. Mean arterial BP
D. *Pulse pressure
E. None
2171. Which BP profile below identifies the patient with the highest risk for development of cardiovascular
complications?
A. 160/90 mmHg
B. *160/65 mmHg
C. 140/100 mmHg
D. 130/90 mmHg
E. 120/70 mmHg
2172. Physicians should diagnose isolated office hypertension (so-called “white-coat hypertension”)
whenever office BP is ≥140/90 mmHg at several visits, while 24-h ambulatory BP is
A. <160/100 mmHg
B. <140/90 mmHg
C. *<130/85 mmHg
D. <125/80 mmHg
E. <120/70 mmHg
2173. Target organs in hypertension include all of the following EXCEPT
A. Brain and eyes
B. Heart
C. Kidneys
D. Peripheral arteries
E. *Liver
2174. All of the following statements concerning target organ damage are true EXCEPT
A. Microalbuminuria is a sensitive marker of hypertension-induced renal damage
B. *An increase in serum creatinine when antihypertensive therapy is intensified is a sign of progressive
renal deterioration
C. Electrocardiography should be part of all routine assessment of subjects with high BP
D. Echocardiography is much more sensitive than electrocardiography in diagnosing left ventricular
hypertrophy
E. Grades 1 and 2 arteriolar retinal changes do not have a significant prognostic value
2175. At a routine examination, an asymptomatic 46-year-old man is found to have a BP of 150/110
mmHg, but no other abnormalities are present. What do you do next?
A. Reassure the patient and repeat the physical examination in 12 months
B. Initiate antihypertensive therapy
C. *Obtain repeated BP recordings in your office and/or the patient’s home or work site
D. Hospitalize patient for renal arteriography
E. Order a 24-h ambulatory BP monitoring
2176. The initial antihypertensive medication recommended for patients who have no compelling
indications or contraindications is
A. ACE-inhibitor
B. Calcium-channel blocker
C. Diuretics
D. Beta blocker
E. *Any of the above
2177. In patients with a history of stroke or TIA the preferred drug combination is:
A. *ACE-inhibitor and diuretic
B. Calcium-channel blocker and beta-blocker
C. Beta-blocker and diuretic
D. Angiotensin receptor blocker and beta blocker
E. Beta- and alpha blocker
2178. In the elderly with systolic hypertension antihypertensive therapy should be initiated if SBP
A. ≥120 mmHg
B. ≥130 mmHg
C. ≥140 mmHg
D. *≥160 mmHg
E. ≥170 mmHg
2179. The best medication for the treatment of isolated systolic hypertension in the elderly:
A. ACE-inhibitor and diuretic
B. *Dihydropiridine calcium-channel blocker and diuretic
C. Beta-blocker and diuretic
D. Non-dihydropiridine calcium-channel blocker and diuretic
E. Beta- and alpha blocker
2180. The following medications can be used in pregnancy EXCEPT
A. Methyldopa
B. Labetalol
C. *ACE-inhibitor / ARB
D. Hydralazine
E. Nifidipine
2181. In patients of African descent the drug of choice for initial treatment of hypertension is:
A. Verapamil
B. Atenolol
C. Perindopril
D. *Hydrochlorothiazide
E. Amlodipine
2182. Which of the following antihypertensive agents is INCORRECTLY matched with the indication for
therapy?
A. ACE inhibitor – diabetic nephropathy
B. Beta-blocker – coronary artery disease
C. Calcium channel blocker – angina pectoris
D. Diuretics – heart failure
E. *Loop diuretic – gout
2183. All the following patients should be evaluated for secondary causes of hypertension EXCEPT
A. *A 47-year-old male with strong family history hypertension who presents to your office with a BP of
150/100
B. A 26-year-old female with hematuria and a family history of early renal failure who has a BP of 160/90
C. A 73-year-old male with no past history with a BP of 165/90
D. A 58-year-old male with a history of hypertension since age 45 whose BP has become increasingly
difficult to control on four antihypertensive agents
E. A 31-year-old female with complaints of severe headaches, weight gain, and new-onset DM with a BP
of 145/90
2184. Which of the following is an effective adjunct to increase the rate of smoking cessation?
A. Aldosterone
B. *Bupropion
C. Sildenafil
D. Cimetidine
E. Buspirone
2185. With which of the following is sildenafil acetate (Viagra) most likely to interact adversely?
A. Fibrinolytic therapy
B. Primary PCI
C. *Nitrates
D. Aspirin
E. Beta-blockers
2186. Bupropion hydrochloride (Zyban) is contraindicated in patients with a history of which of the
following?
A. *Seizures
B. Insulin-dependent diabetes mellitus
C. Severe chronic obstructive pulmonary disease (with reversible component)
D. Longer than 40-year history of tobacco use
E. Recent MI
2187. In which of the following has the high-sensitivity C-reactive protein (CRP) been shown to be
predictive of risk?
A. Acute MI
B. Acute coronary syndromes
C. Chronic stable angina
D. Peripheral vascular disease
E. *All of the above
2188. Which of the following has been shown to decrease the level of the high-sensitivity CRP?
A. Unopposed estrogen
B. Amlodipine besylate (Norvasc)
C. *Simvastatin
D. All of the above
E. None of the above
2189. All of the following statements regarding myocardial stunning are true EXCEPT:
A. *Stunning is a state of depressed myocardial function due to chronic hypoperfusion
B. Stunning can be global or regional
C. Stunning can follow cardiac surgery with cardiopulmonary bypass
D. Oxygen-free radicals and excess intracellular calcium likely contribute to stunning
E. Stunning affects both systolic and diastolic function
2190. What is the most common coexisting congenital anomaly in patients with coarctation of the aorta?
A. Cleft mitral valve
B. *Bicuspid aortic valve
C. Ebstein's anomaly
D. VSD
E. PDA
2191. All of the following are characteristic findings of ostium primum atrial septal defect (ASD)
EXCEPT:
A. Precordial heave
B. Fixed split S2
C. *Right axis deviation
D. Systolic ejection murmur
E. Prominent pulmonary vascular markings on CXR
2192. Choose the adult congenital disorder corresponding to the following physical examination fingings:
RV lift with a loud systolic ejection murmur along the left sternal border, with a single S2
A. Eisenmenger's syndrome
B. Coarctation of the aorta
C. PDA
D. Ebstein's anomaly
E. *Tetralogy of Fallot
2193. Choose the adult congenital disorder corresponding to the following physical examination fingings:
loud S1, holosystolic murmur in left sternal border, systolic ejection click, and hepatomegaly
A. Eisenmenger's syndrome
B. Coarctation of the aorta
C. PDA
D. *Ebstein's anomaly
E. Tetralogy of Fallot
2194. Choose the adult congenital disorder corresponding to the following physical examination fingings:
weak or delayed femoral pulses, harsh systolic murmur in the back, and a systolic ejection click in the aortic
area
A. Eisenmenger's syndrome
B. *Coarctation of the aorta
C. PDA
D. Ebstein's anomaly
E. Tetralogy of Fallot
2195. Choose the adult congenital disorder corresponding to the following physical examination fingings:
cyanosis, digital clubbing, loud P2, and a variable Graham-Steel murmur
A. *Eisenmenger's syndrome
B. Coarctation of the aorta
C. PDA
D. Ebstein's anomaly
E. Tetralogy of Fallot
2196. Choose the adult congenital disorder corresponding to the following physical examination fingings:
wide pulse pressure, prominent LV impulse, and a continuous machinery murmur enveloping S2
A. Eisenmenger's syndrome
B. Coarctation of the aorta
C. *Patent ductus arteriosus
D. Ebstein's anomaly
E. Tetralogy of Fallot
2197. Bacterial endocarditis prophylaxis is indicated in all adults who have the following congenital heart
disease EXCEPT:
A. VSD
B. Coarctation of the aorta
C. *Secundum ASD
D. Hypertrophic obstructive cardiomyopathy
E. PDA
2198. Besides pulmonary valve stenosis, which of the following is the most common associated cardiac
defect present in patients with PA stenosis?
A. *VSD
B. ASD
C. Coarctation of the aorta
D. PDA
E. Bicuspid aortic valve
2199. All of the following statements regarding patent ductus arteriosus (PDA) are true EXCEPT:
A. *The majority of cases close spontaneously after infancy
B. There is a higher incidence in mothers who acquired rubella during pregnancy
C. A decrease in the duration and intensity of the murmur has a poor prognostic implication
D. Left ventricular hypertrophy precedes RV hypertrophy
E. If it is uncorrected, approximately one third of patients die by the age of 40 years
2200. Ostium primum ASD is commonly associated with...
A. Supravalvular aortic stenosis
B. Supravalvular pulmonic stenosis
C. *Cleft mitral valve
D. Anomalous pulmonary venous drainage
E. Persistent left superior vena cava
2201. Noonan's syndrome is commonly associated with...
A. Supravalvular aortic stenosis
B. *Supravalvular pulmonic stenosis
C. Cleft mitral valve
D. Anomalous pulmonary venous drainage
E. Persistent left superior vena cava
2202. Coronary sinus ASD is commonly associated with...
A. Supravalvular aortic stenosis
B. Supravalvular pulmonic stenosis
C. Cleft mitral valve
D. Anomalous pulmonary venous drainage
E. *Persistent left superior vena cava
2203. Williams syndrome is commonly associated with...
A. Supravalvular aortic stenosis
B. *Supravalvular pulmonic stenosis
C. Cleft mitral valve
D. Anomalous pulmonary venous drainage
E. Persistent left superior vena cava
2204. Sinus venosus ASD is commonly associated with...
A. Supravalvular aortic stenosis
B. Supravalvular pulmonic stenosis
C. Cleft mitral valve
D. *Anomalous pulmonary venous drainage
E. Persistent left superior vena cava
2205. All of the following physical examination findings are usually associated with ostium secundum ASD
EXCEPT:
A. Precordial heave
B. Fixed split S2
C. *Lateral and inferior displacement of the apex beat
D. Soft systolic ejection murmur in the second left intercostal space
E. Normal S1
2206. Which of the following is an absolute contraindication to pregnancy?
A. Surgically corrected transposition of great arteries (TGA)
B. Congenitally corrected TGA
C. Ebstein's anomaly
D. *Eisenmenger's syndrome
E. Status post Fontan operation
2207. The lesions that constitute tetralogy of Fallot include all of the following EXCEPT
A. A ventricular septal defect
B. An overriding aorta
C. *An atrial septal defect
D. Right ventricular outflow obstruction
E. Right ventricular hypertrophy
2208. Match the chest radiography findings (prominent central PAs and peripheral PA prunning) with the
corresponding congenital disorder
A. *Eisenmenger's syndrome
B. Patent ductus arteriosus (PDA)
C. Ebstein's anomaly
D. Tetralogy of Fallot
E. Coarctation of the aorta
2209. Match the chest radiography findings (marked cardiomegaly, severe right atrial enlargement, and
normal lung fields) with the corresponding congenital disorder
A. Eisenmenger's syndrome
B. Patent ductus arteriosus (PDA)
C. *Ebstein's anomaly
D. Tetralogy of Fallot
E. Coarctation of the aorta
2210. Match the chest radiography findings (right aortic arch, RV enlargement, and a "boot-shaped" heart)
with the corresponding congenital disorder
A. Eisenmenger's syndrome
B. Patent ductus arteriosus (PDA)
C. Ebstein's anomaly
D. *Tetralogy of Fallot
E. Coarctation of the aorta
2211. Match the chest radiography findings (pulmonary plethora, prominent ascending aorta, proximal PA
dilatation) with the corresponding congenital disorder
A. Eisenmenger's syndrome
B. *Patent ductus arteriosus (PDA)
C. Ebstein's anomaly
D. Tetralogy of Fallot
E. Coarctation of the aorta
2212. All of the following are indications for surgical closure of an ASD EXCEPT:
A. Significant symptoms in a 65-year-old
B. RV dysfunction
C. *Pulmonary vascular resistance >15 Wood units that does not diminish with vasodilators
D. An asymptomatic 20-year-old with a Qp/Qs of 1,7 with no pulmonary hypertension
E. RV enlargement
2213. The following cardiovascular malformations are all associated with congenital rubella EXCEPT:
A. PDA
B. PA stenosis
C. *Ebstein's anomaly
D. Tetralogy of Fallot
E. Coarctation of the aorta
2214. Which of the following syndromes is associated with pumonary arterio-venous fistula?
A. Williams syndrome
B. *Weber-Osler-Rendu syndrome
C. Bland-Garland-White syndrome
D. Kartagener's syndrome
E. Crouzon's syndrome
2215. Congenital MR is commonly encountered in all of the following conditions EXCEPT:
A. *Cor triatriatum
B. Ostium primum ASD
C. Coarctation of the aorta
D. Congenitally corrected TGA
E. Subaortic stenosis
2216. In which of the following cases is surgical correction recommended?
A. Asymptomatic small VSD to decrease risk of endocarditis
B. PDA with severe pulmonary hypertension
C. *Asymptomatic subaortic stenosis with severe aortic valve insufficiency
D. Coarctation of the aorta with a transcoarctation gradient of 20 mmHg
E. Small ASD to prevent paradoxical embolization
2217. Which congenital disorder corresponds with the following chest radiography findings: prominent
central PAs (possible calcifications) and peripheral PA prunning?
A. *Eisenmenger's syndrome
B. Coarctation of the aorta
C. PDA
D. Ebstein's anomaly
E. Tetralogy of Fallot
2218. Which congenital disorder corresponds with the following chest radiography findings: right aortic
arch, RV enlargement, and a "boot-shaped" heart?
A. Eisenmenger's syndrome
B. Coarctation of the aorta
C. PDA
D. Ebstein's anomaly
E. *Tetralogy of Fallot
2219. Which congenital disorder corresponds with the following chest radiography findings: marked
cardiomegaly, severe right atrial enlargement, and normal lung fields
A. Eisenmenger's syndrome
B. Coarctation of the aorta
C. PDA
D. *Ebstein's anomaly
E. Tetralogy of Fallot
2220. Which congenital disorder corresponds with the following chest radiography findings: posterior rib
notching and a "reverse E" or "3" sign
A. Eisenmenger's syndrome
B. *Coarctation of the aorta
C. PDA
D. Ebstein's anomaly
E. Tetralogy of Fallot
2221. Which congenital disorder corresponds with the following chest radiography findings: pulmonary
plethora, prominent ascending aorta, proximal PA dilatation, and opacity at the confluence of the aortic knob
and descending aorta
A. Eisenmenger's syndrome
B. Coarctation of the aorta
C. *PDA
D. Ebstein's anomaly
E. Tetralogy of Fallot
2222. Which of the following statements regarding Ebstein's anomaly is NOT correct?
A. An ASD or PFO is present in up to 80% of patients
B. The cardinal feature is an apically displaced tricuspid valve resulting in atrialization of ventricular
tissue
C. WPW syndrome is common in these patients and multiple tracts can exist
D. *A bicuspid aortic valve is commonly present
E. A "sail sound" is a common finding on physical examination
2223. Which of the following syndromes and cardiac anomalies are INCORRECTLY matched?
A. Trisomy 21 – atrioventricular canal defects
B. Noonan syndrome – pulmonic stenosis
C. Holt-Oram syndrome – atrial septal defects
D. Marfan syndrome – mitral valve prolapse
E. *Williams syndrome – ventricular septal defects
2224. All of the following statements regarding the effects of maneuvers on the auscultation of cardiac
murmurs are true EXCEPT:
A. In patent ductus arteriosus (PDA), the diastolic phase of the murmur is intensified by isometric
handgrip
B. *The murmur of hypertrophic obstructive cardiomyopathy becomes softer with standing or during a
Valsalva strain maneuver
C. The murmur of a ventricular septal defect (VSD) increases with isometric handgrip
D. Sudden squatting or isometric handgrip increases the diastolic murmur of aortic regurgitation
E. The mid-diastolic and presystolic murmurs of mitral stenosis become louder with exercise
2225. Radiotherapy for malignant disease may affect the heart in all of the following ways EXCEPT
A. Pericarditis
B. Coronary artery disease
C. Valvular stenosis
D. Conduction abnormalities
E. *Asymmetrical septal hypertrophy
2226. Sudden cardiac death due to ventricular tachyarrhythmias is an established complication of each of
the following EXCEPT:
A. Sarcoidosis
B. Giant cell myocarditis
C. Chagas disease
D. Idiopathic dilated cardiomyopathy
E. *Hyperthyroidism
2227. Characteristics of arrhythmogenic RV dysplasia include all of the following EXCEPT
A. Fatty infiltration of the right ventricle
B. Fatty infiltration of the right atrium *
C. Monomorphic ventricular tachycardia
D. Abnormal signal-averaged ECG
E. Possible detection by cardiac MRI
2228. All of the following may cause elevation of serum troponin EXCEPT:
A. Congestive heart failure
B. Myocarditis
C. Myocardial infarction
D. *Pneumonia
E. Pulmonary embolism
2229. Each of the statements about the evolution of the ECG in acute pericarditis is correct EXCEPT:
A. Four sequential stages of abnormalities of the ST segments and T waves are typical
B. *Initial ST segment elevation is usually most prominent in ECG lead aVR
C. The ratio of the height of ST segment elevation to the height of the T wave in acute pericarditis is
typically >0.25 in lead V6
D. In acute pericarditis, the ST segment usually returns to baseline before the appearance of T wave
inversion
E. T wave inversion may persist for months after the acute presentation
2230. Which of the following statements regarding cardiac involvement in Lyme disease is CORRECT?
A. Cardiac manifestations of Lyme disease typically occur within several days of the development of
erythema chronicum migricans
B. *Ten percent of patients with Lyme disease develop cardiac manifestations
C. Supraventricular and ventricular tachyarrhythmias are the most common cardiac manifestations of
Lyme disease
D. Cardiomegaly and congestive heart failure are common among patients who develop Lyme carditis
E. Antibiotic therapy has been proven to shorten the course of active Lyme carditis
2231. Each of the following statements regarding the infectious etiologies of pericarditis is correct
EXCEPT:
A. Enteroviruses are among the most common causes of viral pericarditis
B. The most common organisms that cause bacterial pericarditis are streptococci and staphylococci
C. The prognosis of patients with bacterial pericarditis is poor
D. *Tuberculosis is the leading cause of constrictive pericarditis in Western nations
E. Antifungal therapy is generally not necessary in pericarditis caused by histolasmosis
2232. Each of the following statements regarding Chagas disease is correct EXCEPT
A. The level of parasitemia does not correspond to the severity of chronic Chagas disease
B. The disease is transmitted to humans by the reduviid bug (commonly called the kissing bug)
C. *The most common ECG abnormality in chronic Chagas disease is left bundle branch block
D. The classic echocardiographic findings are those of a dilated cardiomyopathy with an apical aneurysm
E. All of the above statements are correct
2233. Which of the following disorders is not associated with ventricular tachycardia as a cause of
syncope?
A. Hypertrophic obstructive cardiomyopathy
B. Prior myocardial infarction
C. *Atrial myxoma
D. Aortic valvular stenosis
E. Congenital long QT syndrome
2234. Each of the following statements regarding the ECG in acute pericarditis is true EXCEPT:
A. The majority of patients demonstrate ECG abnormalities
B. Global ST elevation is seen in early pericarditis
C. Sinus tachycardia is a common finding
D. PR depression is common in the majority of patients
E. *T wave inversions develop before ST elevations return to baseline
2235. Each of the following statements regarding endomyocardial fibrosis (EMF) is correct EXCEPT:
A. This condition is characteristically found in tropical and subtropical Africa
B. It is predominantly a disease of children and young adults
C. *Involvement of the mitral apparatus typically results in mitral stenosis
D. EMF involves the left ventricle in 90% of affected patients
E. Echocardiographic features include increased endocardial echoreflectivity, fibrotic obliteration of apex,
atrial enlargement, and pericardial effusion
2236. All of the following statements with regard to primary endocardial fibroelastosis (EFE) are correct
EXCEPT:
A. The condition is often familial
B. The mitral and aortic valve leaflets are usually thickened and distorted
C. *The murmur of mitral stenosis is the most common auscultatory finding
D. Symptoms of primary EFE usually develop between 2 and 12 months of age
E. Echocardiographic features include a reduced ejection fraction and increased left atrial and left
ventricular dimensions
2237. Which of the following therapies improves survival in patients with cor pulmonare secondary to
COPD?
A. Digoxin
B. *Oxygen
C. Beta-adrenergic agonists
D. Theophylline
E. Hydralazine
2238. What is NOT a cause of chronic cor pulmonale?
A. COPD
B. Recurrent pulmonary emboli
C. *Mitral stenosis
D. Kyphoscoliosis
E. Obstructive sleep apnea
2239. Which of the following conditions is associated with increased left ventricular preload?
A. Sepsis
B. Right ventricular infarction
C. *Mitral regurgitation
D. Dehydration
E. Pulmonary embolism
2240. Each of the following conditions is associated with the development of pulmponary edema EXCEPT:
A. Increased pulmonary venous pressure
B. High altitude
C. *Increased plasma oncotic pressure
D. Eclampsia
E. Heroin overdose
2241. The statements about the diagnosis of PE include all of the following EXCEPT:
A. Arterial blood gas measurement is often unhelpful in the diagnosis of acute PE
B. The most common ECG finding in patients with PE is sinus tachycardia
C. Pulmonary infarction due to PE can be visualized on a standard chest radiograph
D. *Reduced intensity of the pulmonic component of the second heart sound is typical in patients with
large PE
E. Fibrin-degradation products (e.g., D-dimer) are commonly elevated in patients with PE
2242. Heart failure is:
A. Decreasing in incidence and prevalence due to recent advances in the treatment of cardiovascular
disease
B. *One of the diagnoses with the highest readmission rate
C. A common condition across all ages
D. Can be diagnosed from a clinical response to treatment
E. All of the above statements are correct
2243. According to the ESC guidelines, the following patients have heart failure:
A. An obese, dyspnoeic lady with swollen anckles but a normal echocardiogram
B. An asymptomatic man with a previous MI and area of hypokinesis on his echocardiogram
C. *A hypertensive man with LVH, fast atrial fibrillation and dyspnea
D. An elderly man with a few basal crepitations but a normal echocardiogram
E. All of the above
2244. Which of the following sometimes occur when heart failure is absent?
A. Gallop rhythm
B. Anckle oedema
C. A past history of a MI
D. *Dyspnea
E. All of the above
2245. B-type natriuretic secretion is regulated by:
A. Fluid accumulation in the lungs
B. *Pressure and volume of the left ventricle
C. Oedema in the anckles
D. Auricular end diastolic pressure
E. All of the above
2246. Which of the following statements are true ?
A. A normal resting ECG is rare in angina pectoris
B. A normal resting ECG is common in heart failure
C. *Cardiomegaly plus pleural effusions make heart failure very likely
D. Atrial fibrillation commonly does not cause dyspnoea in patients with LVH and diastolic dysfunction
E. All of the above
2247. The value of BNP in the diagnosis of heart failure in the emergency room in patients with acute
dyspnoea:
A. *Is supported by a BNP concentration >500 pg/ml
B. Is excluded by a NT-proBNP concentration <1000 pg/ml
C. Is restricted to patients without a previous history of heart failure
D. Has not been evaluated in patients with renal dysfunction
E. All of the above statements are true
2248. Which symptoms/signs only occur when the HF is relatively advanced – that is, they are commonly
absent in mild heart failure
A. Cardiomegaly on chest X-ray
B. Orthopnea
C. Exertional dyspnea
D. *A abd B
E. B and C
2249. All patients with heart failure should be advised to:
A. Refrain from smoking
B. Restrict sodium intake
C. Avoid a flu shot
D. *A and B
E. All of the above
2250. Exercise training in chronic heart failure
A. *Has been shown to increase peak oxygen consumption by 15-20% in randomized controlled trials
B. Has been shown to increase LV ejection fraction by 15-20% in randomized controlled trials
C. Is not recommended in heart failure
D. Has never been shown to reduce mortality and morbidity
E. None of above
2251. Drugs that should be avoided in heart failure patients are:
A. Non-steroidal anti-inflammatory drugs (NSAIDS)
B. Corticosteroids
C. Beta-blockers
D. *A and B
E. All of the above
2252. The followings improve the survival figure in chronic congestive heart failure as shown by many
studies...EXCEPT
A. Bisoprolol
B. Metoprolol succinate
C. *Atenolol
D. Spironolactone
E. Carvedilol
2253. The following drugs have significantly improved survival in HF patients
A. *ACE inhibitors
B. Nitrates
C. Digoxin
D. Diuretics
E. All of the above
2254. Which of the statements about beta-blocker treatment in HF is true?
A. Randomised clinical trials show that b blockers are beneficial in patients with heart failure and
preserved left ventricular function
B. Elderly patients are the most frequently studied population in blocker trials
C. * Blockers exert a favourable effect in heart failure due to ischaemic and non-ischaemic aetiology
D. Patients with diabetes should always be excluded from blocker treatment
E. There is clear evidence that end stage heart failure patients should be treated with blockers
2255. Which of the statements about beta-blocker treatment in HF is true?
A. *Beta-blockers must be started at very low dosages
B. Treatment should be started when patients are unstable in order to stabilize them
C. When patients are treated with ACE inhibitors, blockers are contraindicated
D. Treatment should always be started during hospital stay
E. The most frequent adverse reaction to a blocker is cough
2256. Which of the statements about antiarrhythmic therapy treatment in HF is NOT true?
A. Amiodarone is an antiarrhythmic drug whose use can improve symptoms of patients with heart failure
without affecting myocardial function and survival
B. *Class I antiarrhythmic agents reduce mortality of patients with heart failure
C. In patients with documented life threatening ventricular arrhythmias, implantable cardioverterdefribrillators (ICDs) can reduce sudden death
D. Sudden death is one of the most frequent causes of death in patients with heart failure
E. The reduction of the number of premature ectopic beats with antiarrhythmic drugs can prevent sudden
mortality of heart failure patients
2257. A prophylactic ICD is useful for reducing total mortality in patients:
A. With previous myocardial infarction and low ejection fraction
B. With dilated cardiomyopathy and low ejection fraction
C. With previous myocardial infarction and low ejection fraction and broad QRS
D. A and C
E. *All of the above
2258. Biventricular pacing in severe heart failure and broad QRS:
A. Improves NYHA class
B. Improves exercise tolerance
C. Reduces hospitalisation for heart failure
D. Reduces heart rate
E. *All of the above
2259. Which of the following treatments most consistently improves EF in patients who have systolic heart
failure?
A. Diuretics
B. Beta-blockers
C. *ACE-inhibitors
D. Vasodilators
E. All of the above
2260. BNP has which of the following properties?
A. Urine volume increases
B. Sodium excretion is enhanced
C. More BNP is secreted
D. A decrease in plasma aldosterone concentration occurs
E. *All of the above occur
2261. All of the following are indications for heart transplantation EXCEPT
A. Dilated cardiomyopathy
B. Diabetes mellitus
C. Hypertrophic cardiomyopathy
D. Age 60 to 70 years
E. *Amyloid heart disease
2262. When used chronically, all of the following drugs increase mortality EXCEPT
A. Milrinone
B. Dobutamine
C. Vesnarinone
D. Xamoterol
E. *Amlodipine
2263. Primary causes of diatolic heart failure include all of the following EXCEPT
A. Hypertrophic cardiomyopathy
B. *Dilated cardiomyopathy
C. Hypertension
D. Ischemic cardiomyopathy
E. Infiltrative cardiomyopathy
2264. Match the description with the associated form of therapy for pulmonary embolism: may be effective
in PE even 1 to 2 weeks after the onset of symptoms
A. Unfractionated heparin or low molecular weight heparin
B. Fibrinolytic therapy
C. *Both
D. Neither
E. Not applicable
2265. Match the description with the associated form of therapy for pulmonary embolism: dissolution of
recently formed thrombus is a major action
A. Unfractionated heparin or low molecular weight heparin
B. *Fibrinolytic therapy
C. Both
D. Neither
E. Not applicable
2266. Match the description with the associated form of therapy for pulmonary embolism: in general,
should be administered along with an antiplatelet agent
A. Unfractionated heparin or low molecular weight heparin
B. Fibrinolytic therapy
C. Both
D. *Neither
E. Not applicable
2267. Match the description with the associated form of therapy for pulmonary embolism: may cause
aldosterone depression
A. *Unfractionated heparin or low molecular weight heparin
B. Fibrinolytic therapy
C. Both
D. Neither
E. Not applicable
2268. Match the ECG finding that is most closely associated with chronic Chagas disease:
A. Low QRS voltage
B. AV nodal block
C. *Right bundle branch block
D. Diffuse ST segment elevation
E. Deeply inverted precordial T waves
2269. Match the ECG finding that is most closely associated with sarcoidosis:
A. Low QRS voltage
B. *AV nodal block
C. Right bundle branch block
D. Diffuse ST segment elevation
E. Deeply inverted precordial T waves
2270. Match the ECG finding that is most closely associated with apical hypertrophic cardiomyopathy
A. Low QRS voltage
B. AV nodal block
C. Right bundle branch block
D. Diffuse ST segment elevation
E. *Deeply inverted precordial T waves
2271. Match the ECG finding that is most closely associated with cardiac amyloidosis
A. *Low QRS voltage
B. AV nodal block
C. Right bundle branch block
D. Diffuse ST segment elevation
E. Deeply inverted precordial T waves
2272. Heart may be affected by radiotherapy in all of the following ways EXCEPT
A. Pericarditis
B. Coronary artery disease
C. Valvular stenosis
D. Conduction abnormalities
E. *Asymmetrical septal hypertrophy
2273. SCD (sudden cardiac death) due to ventricular tachyarrhythmias is an established complication of
each of the following EXCEPT:
A. Sarcoidosis
B. Giant cell myocarditis
C. Chagas disease
D. Idiopathic dilated cardiomyopathy
E. *Hyperthyroidism
2274. Arrhythmogenic RV dysplasia is characterized by all of the following EXCEPT
A. Fatty infiltration of the right ventricle
B. *Fatty infiltration of the right atrium
C. Monomorphic ventricular tachycardia
D. Abnormal signal-averaged ECG
E. Possible detection by cardiac MRI
2275. Each of the following may cause elevation of serum troponin EXCEPT:
A. Congestive heart failure
B. Myocarditis
C. Myocardial infarction
D. *Pneumonia
E. Pulmonary embolism
2276. All of the following statements regarding the infectious etiologies of pericarditis are correct
EXCEPT:
A. Enteroviruses are among the most common causes of viral pericarditis
B. The most common organisms that cause bacterial pericarditis are streptococci and staphylococci
C. The prognosis of patients with bacterial pericarditis is poor
D. *Tuberculosis is the leading cause of constrictive pericarditis in Western nations
E. Antifungal therapy is generally not necessary in pericarditis caused by histolasmosis
2277. All of the following statements regarding Chagas disease are correct EXCEPT
A. The level of parasitemia does not correspond to the severity of chronic Chagas disease
B. The disease is transmitted to humans by the reduviid bug (commonly called the kissing bug)
C. *The most common ECG abnormality in chronic Chagas disease is left bundle branch block
D. The classic echocardiographic findings are those of a dilated cardiomyopathy with an apical aneurysm
E. All of the above statements are correct
2278. Of the following disorders, choose one which is not associated with ventricular tachycardia as a cause
of syncope?
A. Hypertrophic obstructive cardiomyopathy
B. Prior myocardial infarction
C. *Atrial myxoma
D. Aortic valvular stenosis
E. Congenital long QT syndrome
2279. All of the following statements regarding the ECG in acute pericarditis is true EXCEPT:
A. The majority of patients demonstrate ECG abnormalities
B. Global ST elevation is seen in early pericarditis
C. Sinus tachycardia is a common finding
D. PR depression is common in the majority of patients
E. *T wave inversions develop before ST elevations return to baseline
2280. All the following features favor ventricular tachycardia as the cause of broad-complex tachycardia,
EXCEPT:
A. QRS duration with right bundle branch block morphology (RBBB) is greater than 140 ms, and greater
than 160 ms with left LBBB morphology
B. LAD with RBBB morphology, extreme LAD (northwest axis) with LBBB morphology
C. AV dissociation
D. *Response to carotid sinus massage
E. Concordance of the QRS pattern in all precordial leads (i.e., all positive or all negative deflections)
2281. Which of the following will most likely respond to verapamil?
A. RVOT tachycardia
B. *Idiopathic LV tachycardia
C. Bundle branch re-entry tachycardia
D. ARVD ventricular tachycardia
E. All of the above
2282. The most common type of arrhythmia in WPW syndrome is....
A. Ventricular tachycardia
B. Ventricular fibrillation
C. Ventricular premature complexes
D. Atrial ectopics
E. *AV re-entry tachycardia
2283. Use of echocardiogram in the assessment of AF:
A. It is not necessary to perform a TTE in patients with AF
B. TEE is always needed to allow better assessment of the MV and LA
C. *TTE should be performed at baseline to diagnose associated structural/functional heart disease
D. TTE is necessary to determine stroke risk stratification for thromboprophylaxis in all patients
E. When TTE is technically difficult, MRI is needed to exclude cardiac abnormalities
2284. Which is the preferred initial treatment strategy for rate control in AF patients?
A. Digoxin
B. *Beta-blockers or rate limiting CCB
C. Combination therapy of beta-blockers and rate-limiting CCB’s
D. Amiodarone
E. Combination therapy of digoxin and beta-blockers
2285. Which option is better in regard to cardioversion?
A. *Synchronised DC shock to restore sinus rhythm
B. Amiodarone in all patients to restore sinus rhythm
C. Flecainide in patients with IHD to maintain sinus rhythm
D. Digoxin to restore sinus rhythm
E. Propaphenone to restore sinus rhythm in HCMP patients
2286. Regarding antithrombotic treatment in AF:
A. Patients with asymptomatic AF have less risk of embolism than symptomatic patients
B. Patients with atrial flutter do not need antithrombotic treatment due to the low risk of embolism of this
arrhythmia
C. *Warfarin is the best option in patients with high risk of thromboembolism
D. Aspirin has a similar effect in stroke reduction to placebo
E. Paroxysmal AF has the lowest risk of embolism
2287. Sudden cardiac death in patients >35 years old is most commonly associated with:
A. Hypertrophic cardiomyopathy
B. *Coronary artery disease
C. Congenital long QT syndrome
D. Long QT syndrome secondary to various medications
E. arrhythymogenic RV cardiomyopathy
2288. Congenital long QT syndrome can lead to
A. Complete heart block
B. *Polymorphic ventricular tachycardia
C. Acute myocardial infarction
D. Recurrent supraventricular tachycardia
E. All of the above
2289. Bundle branch re-entry ventricular tachycardia is most commonly associated with:
A. Enhanced automaticity in the right bundle
B. Enhanced automaticity in the left bundle
C. Supranormal conduction in the His bundle
D. *Abnormally slow conduction in the His bundle
E. All of the above
2290. All of the following are ECG manifestations of Wolff-Parkinson-White syndrome EXCEPT
A. *Narrow QRS
B. Initial slurred QRS upstroke
C. Short PQ interval
D. Normal QT interval
E. All of the above
2291. The initial manifestations of WPW syndrome may include which of the following?
A. Atrial fibrillation
B. AV re-entry tachycardia
C. Ventricular fibrillation
D. Wide-QRS tachycardia
E. *All of the above
2292. The presence of an r prime in V1 during narrow-QRS tachycardia is suggestive of...
A. AV re-entry
B. *AV node re-entry
C. Rate-dependent BBB
D. Atrial tachycardia
E. Atrial fibrillation
2293. Ventricular ectopic beats (VEBs) occurring in apparently normal hearts are:
A. Associated with a bad prognosis
B. Never frequent or complex
C. Less frequent with increasing age
D. *May be associated with symptoms of “missed beats”
E. Always symptomatic
2294. In patients with ventricular ectopic beats:
A. Caffeine restriction always reduces symptoms
B. *High caffeine intake should always be discouraged in symptomatic patients
C. There is good evidence that reducing caffeine intake reduces ectopics
D. Hypertension has no association with frequency of ectopic beats
E. LVH is not associated with increased frequency of ectopics
2295. Regarding ventricular ectopics and exercise:
A. *Ectopics are usually benign if they are supressed on exercise
B. Exercise-induced VEBs are not associated with adverse prognosis
C. RVOT ectopics are usually supressed on exercise
D. Ectopics induced on recovery after exercise are benign
E. All of the above
2296. An idiopathic left ventricular tachycardia (VT) shows the following...
A. Typical LBBB morphology
B. *LAD and RBBB morphology
C. Associated with a high risk of sudden cardiac death and almost always requires ICD therapy
D. Never responds to verapamil
E. All of the above
2297. Beta blockers are accepted treatment to prevent VT in:
A. Brugada syndrome, RVOT-VT, short QT syndrome and long QT syndrome
B. Brugada syndrome, RVOT-VT and long QT syndrome
C. *RVOT-VT and long QT syndrome
D. Short QT syndrome and long QT syndrome
E. Brugada syndrome and short QT syndrome
2298. Wenckebach phenomenon is defined as...
A. Progressive shortening of PR interval till a beat is dropped
B. *Progressive lengthening of PR interval till a beat is dropped
C. Irregular heart rate and PVC’s
D. Shortened QT intervals
E. Slurred QRS complex
2299. Giant a waves in JVP (“cannon” waves) are seen in
A. Sinus bradycardia
B. Atrial fibrillation
C. First-degree AV-block
D. *Complete heart block
E. Atrial ectopic
2300. All of the following findings are suggestive of LVH except:
A. (S in V1 + R in V5 or V6) > 35 mm
B. R in aVL > 11 mm
C. R in aVF > 20 mm
D. (R in I + S in III) > 25 mm
E. *R in aVR > 8 mm
2301. All of the following findings are suggestive of RVH except:
A. R in V1=7 mm (15 mm with RBBB)
B. (R in V1 + S in V5 or V6) > 10 mm
C. R < S in V6
D. R or R’ in V1
E. *S in V3 + R in aVL > 28 mm (20 mm for women)
2302. When present, each of the following heart sounds occurs shortly after S2 EXCEPT:
A. Opening snap
B. Third heart sound
C. *Ejection clic
D. Tumor plop
E. Pericardial knock
2303. Each of the following statements regarding splitting of the second heart sound is true EXCEPT:
A. Severe pulmonic valvular stenosis is associated with a softened P2
B. Delayed closure of the pulmonic valve with inspiration contributes to physiologic splitting of S2
C. Fixed splitting of S2 is the auscultatory hallmark of an ostium secundum atrial septal defect
D. Paradoxical splitting of S2 is expected in patients with a right ventricular electronically paced rhythm
E. *Right bundle branch block is associated with paradoxical splitting of S2
2304. The timing of an "innocent" murmur is usually:
A. Early systolic
B. Presystolic
C. *Midsystolic
D. Holosystolic
E. Early diastolic
2305. Which of the following pairs of medical conditions and antihypertensive medications would be
incorrect to use in a patient with essential hypertension?
A. Beta-blocker and a history of myocardial infarction
B. Alpha-blocker and prostatic hypertrophy
C. *Thiazide diuretic and gout
D. Amlodipine and heart failure
E. ACE-inhibitor and diabetes mellitus
2306. Thiazide diuretics can contribute to each of the following metabolic effects EXCEPT:
A. Hypomagnesemia
B. *Hypouricemia
C. Hypercalcemia
D. Hypercholesterolemia
E. Hyponatremia
2307. Which of the following statements regarding prevention of stroke is correct?
A. Treatment of systolic hypertension does not decrease the risk of stroke in patients older than age 60
B. Hypertension should not be a target of secondary prevention after an ischemic stroke because elevated
BP is desirable to maintain adequate cerebral perfusion
C. *Treatment with HMG-CoA reductase inhibitors reduces the risk of recurrent stroke
D. The combination of aspirin and clopidogrel is superior to aspirin alone for prevention of recurrent
stroke
E. None of the above
2308. You see a diabetic patient presents with BP readings that are 155/95 or higher. All of the following
statements about the treatment of this patient's hypertension are correct EXCEPT:
A. Pharmacologic blocade of the renin-angiotensin system reduces the risk of both microvascular and
macrovascular events
B. Aggressive BP control reduces cardiovascular events more in diabetics than in nondiabetics
C. *Calcium channel blockers show no benefit in reducing cardiovascular events
D. The goal BP for this patient is <130/80 mmHg
E. All the above statements are correct
2309. Which of the following antihypertensive agents is a known cause of autoimmune hemolytic anemia?
A. Metoprolol
B. *Methyldopa
C. Captopril
D. Losartan
E. Monoxidine
2310. All of the following characteristics are typical of hypertensive crisis EXCEPT:
A. Diastolic BP >120 mmHg
B. Retinal hemorrhages
C. *Constriction of cerebral arterioles with decreased vascular permeability
D. Proteinuria
E. Microangiopathic hemolytic anemia
2311. Which of the following pairs of medical conditions and antihypertensive medications would be
INCORRECT to use in a patient with essential hypertension?
A. Beta-blocker and a history of myocardial infarction
B. Alpha-blocker and prostatic hypertrophy
C. *Thiazide diuretic and gout
D. Amlodipine and heart failure
E. ACE-inhibitor and diabetes mellitus
2312. Which of the following statements regarding antihypertensive agents and atrial fibrillation (AF) is
TRUE?
A. *Losartan has been shown to decrease new-onset AF more effectively than atenolol
B. Valsartan has been shown to decrease new-onset AF more effectively than amlodipine
C. Atenolol has been shown to decrease new-onset AF more effectively than captopril
D. Nifedipine has been shown to decrease new-onset AF more effectively than diltiazem
E. All of the above are false
2313. Which lifestyle change has been shown to produce the biggest reduction in systolic blood pressure?
A. *10-kg weight loss
B. Dietary sodium restriction
C. Moderation of alcohol consumption
D. Vegetarian diet
E. Calcium supplement
2314. Which of these statements is true regarding renin-angiotensin system-blocking agents?
A. Less effective when combined with a diuretic than when used alone
B. More effective in patients of African descent than in white patients
C. *Preserve kidney function in addition to lowering blood pressure
D. Are the first-line antihypertensive medications in pregnancy
E. All of the above
2315. Which of the following antihypertensive agents is INCORRECTLY matched with the indication for
therapy?
A. ACE inhibitor – diabetic nephropathy
B. Beta-blocker – coronary artery disease
C. Calcium channel blocker – angina pectoris
D. *Hydrochlorthiazide – diabetes mellitus
E. Loop diuretic – heart failure
2316. Which of the following statements about microalbuminuria is TRUE?
A. To be of clinixal value, microalbuminuria must be measured in a timed 12- to 24-hour sample
B. *Microalbuminuria is a cardiovascular risk factor that is independent of traditional Framingham risk
factors
C. Microalbuminuria is a predictor of risk only in patients with diabetes
D. Microalbuminuria is present when the “spot” urine albumin-to-creatinine ration is >500 mg/g
E. All of the above statements are correct
2317. Renin-dependent hypertension includes:
A. Primary hyperaldosteronism
B. Essential hypertension
C. *Renovascular hypertension
D. Pheochromocytoma
E. Cushing syndrome
2318. Fundus picture of stage 2 of Keith-Wegener classification includes
A. *Copper wire AV nipping and focal spasm
B. Silver wire increased reflex
C. Exudate and hemorrhagic spots
D. Papilloedema
E. Retinal hemorrhages
2319. A young hypertensive patient has serum potassium 2.8 mEq/l and increased aldosterone level with
decreased plasma renin activity. The likely diagnosis is....
A. Renal artery stenosis
B. Ectopic ACTH syndrome
C. *Conn syndrome
D. Liddle syndrome
E. Cushing syndrome
2320. The proportion of essential (primary) hypertension among all hypertension causes is as high as
A. 25-30%
B. 40-45%
C. 60-65%
D. 70-75%
E. *90-95%
2321. All of the following are risk factors for hypertension, EXCEPT
A. Increased body weight
B. Family history of hypertension
C. Excessive intake of sodium
D. *Regular use of one glass of wine per day
E. Cigarette smoking
2322. All of the following statements concerning hypertension are true, EXCEPT
A. In a western adult population the prevalence of hypertension exceeds 20%
B. Hypertension is a major risk factor for cardiovascular and cerebrovascular disease
C. The pathophysiology of hypertension differs in black adults compared to South Asians and whites
D. *People of African descent commonly have a high renin type hypertension
E. An increase in potassium intake may significantly reduce blood pressure in hypertensive patients
2323. Optimal blood pressure (BP) is defined as a BP level of
A. *<120/80 mmHg
B. <130/80 mmHg
C. <140/90 mmHg
D. <160/100 mmHg
E. <125/80 mmHg
2324. Which parameter of arterial blood pressure (BP) yields the best prognostic information in patients at
risk for cardiovascular disease?
A. Systolic BP
B. Diastolic BP
C. Mean arterial BP
D. *Pulse pressure
E. None
2325. Which BP profile below identifies the patient with the highest risk for development of cardiovascular
complications?
A. 160/90 mmHg
B. *160/65 mmHg
C. 140/100 mmHg
D. 130/90 mmHg
E. 120/70 mmHg
2326. Physicians should diagnose isolated office hypertension (so-called “white-coat hypertension”)
whenever office BP is ≥140/90 mmHg at several visits, while 24-h ambulatory BP is
A. <160/100 mmHg
B. <140/90 mmHg
C. *<130/85 mmHg
D. <125/80 mmHg
E. <120/70 mmHg
2327. Target organs in hypertension include all of the following EXCEPT
A. Brain and eyes
B. Heart
C. Kidneys
D. Peripheral arteries
E. *Liver
2328. All of the following statements concerning target organ damage are true EXCEPT
A. Microalbuminuria is a sensitive marker of hypertension-induced renal damage
B. *An increase in serum creatinine when antihypertensive therapy is intensified is a sign of progressive
renal deterioration
C. Electrocardiography should be part of all routine assessment of subjects with high BP
D. Echocardiography is much more sensitive than electrocardiography in diagnosing left ventricular
hypertrophy
E. Grades 1 and 2 arteriolar retinal changes do not have a significant prognostic value
2329. The initial antihypertensive medication recommended for patients who have no compelling
indications or contraindications is
A. ACE-inhibitor
B. Calcium-channel blocker
C. Diuretics
D. Beta blocker
E. *Any of the above
2330. A 40-year-old diabetic patient presents with a blood pressure (BP) of 145/90 mmHg and proteinuria.
Which BP profile represents the best therapeutic goal for this patient?
A. <160/90
B. <140/90
C. *<130/85
D. <125/75
E. <140/85
2331. A 40-year-old diabetic patient presents with a blood pressure (BP) of 145/90 mmHg and proteinuria.
What is the best medication for the initial management of this patient’s hypertension?
A. Calcium channel blockers
B. Beta blockers
C. *ACE-inhibitors / angiotensin receptor blockers
D. Alpha blockers
E. Diuretics
2332. In patients with a history of stroke or TIA the preferred drug combination is:
A. *ACE-inhibitor and diuretic
B. Calcium-channel blocker and beta-blocker
C. Beta-blocker and diuretic
D. Angiotensin receptor blocker and beta blocker
E. Beta- and alpha blocker
2333. In the elderly with systolic hypertension antihypertensive therapy should be initiated if SBP
A. ≥120 mmHg
B. ≥130 mmHg
C. *≥140 mmHg
D. ≥160 mmHg
E. ≥170 mmHg
2334. The best medication for the treatment of isolated systolic hypertension in the elderly:
A. ACE-inhibitor and diuretic
B. *Dihydropiridine calcium-channel blocker and diuretic
C. Beta-blocker and diuretic
D. Non-dihydropiridine calcium-channel blocker and diuretic
E. Beta- and alpha blocker
2335. The following medications can be used in pregnancy EXCEPT
A. Methyldopa
B. Labetalol
C. *ACE-inhibitor / ARB
D. Hydralazine
E. Nifidipine
2336. In patients of African descent the drug of choice for initial treatment of hypertension is:
A. Verapamil
B. Atenolol
C. Perindopril
D. *Hydrochlorothiazide
E. Amlodipine
2337. Which of the following antihypertensive agents is INCORRECTLY matched with the indication for
therapy?
A. ACE inhibitor – diabetic nephropathy
B. Beta-blocker – coronary artery disease
C. Calcium channel blocker – angina pectoris
D. Diuretics – heart failure
E. *Loop diuretic – gout
2338. All the following patients should be evaluated for secondary causes of hypertension EXCEPT
A. *A 47-year-old male with strong family history hypertension who presents to your office with a BP of
150/100
B. A 26-year-old female with hematuria and a family history of early renal failure who has a BP of 160/90
C. A 73-year-old male with no past history with a BP of 165/90
D. A 58-year-old male with a history of hypertension since age 45 whose BP has become increasingly
difficult to control on four antihypertensive agents
E. A 31-year-old female with complaints of severe headaches, weight gain, and new-onset DM with a BP
of 145/90
2339. Which of the following diagnostic studies is most likely to demonstrate the cause of the headaches?
A. MRI of the head
B. MRI of the kidney
C. *MRI of the thorax
D. 24-h urinary 5-HIAA
E. 24-h urinary free cortisol
2340. A 27-year-old female has hypertension that is difficult to control. She was diagnosed at age 25. Since
that time she has been on increasing amounts of medication. Her current regimen consists of labetalol 1000
mg bid, lisinopril 40 mg qd, clonidine 0.1 mg bid, and amlodipine 5 mg qd. On physical examination she
appears to be without distress. Her BP is 170/100, and HR is 84 bpm. Cardiac exam is unremarkable,
without rubs, gallops, or murmurs. She has good peripheral pulses and has no edema. Laboratory studies
reveal a potassium of 2.8 mEq/dL and a serum bicarbonate of 32 mEq/dL. Fasting blood glucose 114 mg/dL.
What is the likely diagnosis?
A. Congenital adrenal hyperplasia
B. Fibromuscular dysplasia
C. Cushing’s syndrome
D. *Conn’s syndrome
E. Pheochromocytoma
2341. A 27-year-old female has hypertension that is difficult to control. She was diagnosed at age 25. Since
that time she has been on increasing amounts of medication. Her current regimen consists of labetalol 1000
mg bid, lisinopril 40 mg qd, clonidine 0.1 mg bid, and amlodipine 5 mg qd. On physical examination she
appears to be without distress. Her BP is 170/100, and HR is 84 bpm. Cardiac exam is unremarkable,
without rubs, gallops, or murmurs. She has good peripheral pulses and has no edema. Laboratory studies
reveal a potassium of 2.8 mEq/dL and a serum bicarbonate of 32 mEq/dL. Fasting blood glucose 114 mg/dL.
What is the best way to diagnose this disease ?
A. Renal vein renin levels
B. 24-h urine collection for metanephrines
C. MRI of the renal arteries
D. 24-h urine collection for cortisol
E. *Plasma aldosterone/renin ratio
2342. A 21-year-old female is seen in the ED with symptoms of periodic headaches, sweating, and nausea
with vomiting. She also complains of feeling light-headed with standing. Her BP on presentation is 230/135,
with a HR of 92. On standing, the patient has BP of 205/100, with a HR of 136. On ophthalmologic
examination the patient has mild blurring of the optic discs without hemorrhage. The examination is
otherwise normal. What is the best medication for the management of this patient’s hypertension?
A. *Phentolamine
B. Fenoldopam
C. Esmolol
D. Nicardipine
E. Diazoxide
2343. A 20-year-old female is seen in the ED with symptoms of periodic headaches, sweating, and nausea
with vomiting. She also complains of feeling light-headed with standing. Her BP on presentation is 230/135,
with a HR of 92. On standing, the patient has BP of 205/100, with a HR of 136. On ophthalmologic
examination the patient has mild blurring of the optic discs without hemorrhage. The examination is
otherwise normal. What test would best determine the patient’s diagnosis?
A. Plasma catecholamines
B. 24-h urine collection for 5-HIAA
C. Abdominal CT scan
D. *24-h urine collection for metanephrines and vanilylmandelic acid
E. Adrenal vein sampling for renin levels
2344. What is the correct feature of the hypertensive urgency?
A. It is always a life-threatening situation
B. Patient must be hospitalized
C. *Patients can be managed as outpatients
D. Therapy should not be given orally
E. None of the above
2345. If there is suspicion of phaeochromocytoma, the first line drug is:
A. Sodium-nitroprusside
B. Nifedipine
C. *Phentolamine
D. Verapamil
E. Enalapril
2346. If you are in your office and a patient sees you with the following symptoms: long history of
hypertension, home BP usually 160/100 mmHg, suddenly developed severe headache
blurred vision, weakness in the right arm, blood pressure: 200/110 mmHg, What will you do:
A. Give the patient 20 mg nifedipine orally, and check the BP in 15 min. If it is still >200/100 mmHg,
give another dose of nifedipine
B. Give the patient 20 mg nifedipine, and check the BP in 30 min. If it is 160/80 mmHg, send the patient
home and give another appointment in two weeks
C. Give the patient 10 mg amlodipine, send the patient home and give another appointment in two days
D. Call the ambulance and send the patient to the nearest hospital
E. *Give the patient some pain-killer drug (e.g. acetaminophen), call the ambulance and send the patient
to the nearest hospital with stroke-unit
2347. A 28-year-old male with type 1 diabetes mellitus is seen in the clinic for routine follow-up. The
patient has hypertension. He is being maintained on an insulin pump and lisinopril 5 mg daily. He has been
on this dose of lisinopril for approximately 6 weeks. He has normal creatinine and no proteinuria. BP is
140/90 mmHg. What do you recommend ?
A. Continue the current dose of lisinopril and return for follow-up in 3 months as the patient has received
his target BP of less than 140/90 mmHg
B. Ask a patient to return in 6 weeks; if his BP remains at this level, increase the dose of lisinopril
C. *Increase lisinopril to 10 mg daily to achieve a goal BP of less than 130/80 mmHg
D. Assure the patient that he is suffering from “white-coat hypertension” and plan no intervention
E. Switch the patient from lisinopril to losartan
2348. A 49-year-old female presents to you for an initial visit to establish health care. She has no past
medical history and denies a family history of diabetes mellitus, early cardiovascular disease, or renal
disease. Her BP on presentation is 185/110 mmHg. She denies headache or visual complaints. She has no
chest pain, shortness of breath, or change in urine color. Her physical examination is normal with the
exception of an S4 gallop heard at the 4th intercostal space just to the left of the sternum. An ECG is
significant for LVH and LAD. there are no signs of cardiac ischemia. Urinalysis shows no red blood cells or
proteinuria. What is your recommendation for therapy for this patient?
A. *Initiate therapy with hydrochlorothiazide in combination with a beta-blocker
B. Initiate therapy with hydrochlorothiazide alone
C. Initiate therapy with an ACE inhibitor alone
D. Advise the patient to go to the nearest emergency department for further treatment for a hypertensive
emergency
E. Advise the patient on lifestyle modifications and plan a return visit in 3 weeks
2349. What will you do if you are on duty in a hospital and a hypertensive patient, who has been treated
with long-acting ß-blocker, long-acting dihydropyridine calcium antagonist and long-acting a-1 adrenoceptor
blocker, is admitted with the following symptoms: headache, dyspnea, ankle-swelling, BP 180/110 mmHg,
pulmonary rales, LVH on ECG, echo: LVH, left ventricular diastolic dysfunction, ejection fraction 46%.
A. Amlodipine orally
B. Nifedipine orally
C. Enalaprilate iv
D. *Enalaprilate iv + furosemide iv
E. Esmolol iv
2350. Which of the following blood pressure levels in pregnancy should be considered an emergency
requiring hospitalization?
A. ≥140/90 mmHg
B. ≥150/95 mmHg
C. ≥160/100 mmHg
D. *≥170/110 mmHg
E. A rise in SBP ≥25 mmHg or a rise in DPB ≥15 mmHg compared to pre-pregnancy values, or those in
the first trimester
2351. The following drug combinations are effective to treat hypertension EXCEPT
A. Diuretics + beta-blockers
B. Beta-blockers + dihydropiridine calcium channel blockers
C. Diuretics + ACE-inhibitors/ARB
D. ACE-inhibitors/ARB + calcium channel blockers
E. *Beta-blockers + ACE-inhibitors/ARB
2352. Each of the following statements regarding hypertension is true EXCEPT:
A. Pure "white coat hypertension is found in 20 to 30% of patients
B. *When measuring BP, an inappropriately small cuff size results in a spuriously low systolic
measurement
C. Pseudohypertension may occur in patients with sclerotic brachial arteries
D. Chronic renal disease is the second most common cause of hypertension after essential hypertension
E. Coarctation of the aorta, Cushing disease, primary aldosteronism account for approximately 1% of all
hypertensive patients
2353. With respect to renovascular disease, all of the statements are true EXCEPT:
A. Worsening renal function with ACE-inhibitor therapy suggests bilateral renovascular disease
B. Atherosclerotic disease most commonly involves the proximal third of the main renal artery
C. The most common form of fibroplastic renovascular disease in adults involves the media
D. *The incidence of renovascular hypertension falls with increased age
E. Patients with severe, accelerated hypertension have the highest prevalence of renovascular disease
2354. A newly diagnosed diabetic patient presents with BP readings that are 155/95 or higher. All of the
following statements about the treatment of this patient's hypertension are correct EXCEPT:
A. Pharmacologic blockade of the renin-angiotensin system reduces the risk of both microvascular and
macrovascular events
B. Aggressive BP control reduces cardiovascular events more in diabetics than in nondiabetics
C. *Calcium channel blockers show no benefit in reducing cardiovascular events
D. The goal BP for this patient is <130/80 mmHg
E. All the above statements are correct
2355. All of the following statements regarding the association of oral contraceptive pills and hypertension
are correct EXCEPT:
A. The likelihood of developing hypertension is increased by significant alcohol consumption
B. The incidence of hypertension is about twice as great in pill users as in non-users
C. The likelihood of developing hypertension is dependent on the age of the user
D. *Hypertension resolves in almost all cases after discontinuation of oral contraceptives
E. The mechanism for contraceptive-induced hypertension likely involves renin-aldosterone-mediated
volume expansion
2356. Each of the following statements regarding pheochromocytoma is true EXCEPT:
A. Approximately 15% of pheochromocytomas are extra-adrenal
B. Ten percent of pheochromocytomas are malignant
C. Ten percent of adrenal pheochromocytomas are bilateral
D. *Hypertension related to pheochromocytoma is nearly always episodic and only rarely persistent
E. Pheochromocytoma can be inherited
2357. All of the following are features of renovascular hypertension due to fibromuscular hyperplasia, as
opposed to atherosclerosis, EXCEPT:
A. Age <50 years
B. Female gender
C. *More likely to progress to complete renal artery occlusion
D. No family history of hypertension
E. Absence of carotid bruits
2358. Which of the following ACE-inhibitors are not prodrugs?
A. Captopril, lisinopril, ramipril
B. Lisinopril, enalapril, banazepril
C. *Captopril, lisinopril, enalaprilat
D. Moexipril, captopril, lisinopril
E. Perindopril, ramipril, enalapril
2359. Each of the following statements regarding hypertension is true EXCEPT:
A. The prevalence of hypertension rises progressively with age in both men and women
B. Systolic and diastolic hypertension are each associated with an increased crisk of coronary heart
disease
C. *The target blood pressure for hypertensive patients with cardiovascular disease or diabetes is 140/90
mmHg
D. Patients with prehypertension (SBP 120-139 mmHg or DBP 80-89 mmHg) benefit from lifestyle
modifications incl. weight reduction, regular exercise, and smoking cessation
E. Drug therapy of hypertension benefits patients greater than 80 years of age
2360. All of the following interventions have a BP-lowering effect EXCEPT:
A. A diet that reduces caloric intake by 1000 calories per day
B. Reduction of dietary sodium
C. *Daily magnesium supplements
D. Tobacco cessation
E. Reduction of ethanol consumption to less than 30 ml/day
2361. Each of the following statements regarding hypertension is true EXCEPT:
A. Pure "white coat hypertension is found in 20 to 30% of patients
B. *When measuring BP, an inappropriately small cuff size results in a spuriously low systolic
measurement
C. Pseudohypertension may occur in patients with sclerotic brachial arteries
D. Chronic renal disease is the second most common cause of hypertension after essential hypertension
E. Coarctation of the aorta, Cushing disease, primary aldosteronism account for approximately 1% of all
hypertensive patients
2362. Each of the following statements regarding hypertension is true EXCEPT:
A. Increased LV muscle mass in hypertension is an independent risk factor of cardiac mortality
B. The risk of ventricular arrhythmias in hypertension is increased 2-fold in the presence of LV
hypertrophy
C. Low birth weight is associated with an increased incidence of hypertension later in life
D. *Chronic caffeine consumption is associated with an increased risk of hypertension
E. In hypertensive adolescents, essential hypertension is the most likely etiology
2363. The following statements regarding the association of oral contraceptive pills and hypertension are
correct EXCEPT:
A. The likelihood of developing hypertension is increased by significant alcohol consumption
B. The incidence of hypertension is about twice as great in pill users as in non-users
C. The likelihood of developing hypertension is dependent on the age of the user
D. *Hypertension resolves in almost all cases after discontinuation of oral contraceptives
E. The mechanism for contraceptive-induced hypertension likely involves renin-aldosterone-mediated
volume expansion
2364. Thiazide diuretics can contribute to each of the following metabolic effects EXCEPT:
A. Hypomagnesemia
B. *Hypouricemia
C. Hypercalcemia
D. Hypercholesterolemia
E. Hyponatremia
2365. Which of the following statements regarding prevention of stroke is correct?
A. Treatment of systolic hypertension does not decrease the risk of stroke in patients older than age 60
B. Hypertension should not be a target of secondary prevention after an ischemic stroke because elevated
BP is desirable to maintain adequate cerebral perfusion
C. *Treatment with HMG-CoA reductase inhibitors reduces the risk of recurrent stroke
D. The combination of aspirin and clopidogrel is superior to aspirin alone for prevention of recurrent
stroke
E. None of the above
2366. Which of the following antihypertensive agents is a known cause of autoimmune hemolytic anemia?
A. Metoprolol
B. *Methyldopa
C. Captopril
D. Losartan
E. Monoxidine
2367. All of the following characteristics are typical of hypertensive crisis EXCEPT:
A. Diastolic BP >120 mmHg
B. Retinal hemorrhages
C. *Constriction of cerebral arterioles with decreased vascular permeability
D. Proteinuria
E. Microangiopathic hemolytic anemia
2368. Which of the following pairs of medical conditions and antihypertensive medications would be
INCORRECT to use in a patient with essential hypertension?
A. Beta-blocker and a history of myocardial infarction
B. Alpha-blocker and prostatic hypertrophy
C. *Thiazide diuretic and gout
D. Amlodipine and heart failure
E. ACE-inhibitor and diabetes mellitus
2369. Which of the following statements regarding antihypertensive agents and atrial fibrillation (AF) is
TRUE?
A. *Losartan has been shown to decrease new-onset AF more effectively than atenolol
B. Valsartan has been shown to decrease new-onset AF more effectively than amlodipine
C. Atenolol has been shown to decrease new-onset AF more effectively than captopril
D. Nifedipine has been shown to decrease new-onset AF more effectively than diltiazem
E. All of the above are false
2370. Which lifestyle change has been shown to produce the biggest reduction in systolic blood pressure?
A. *10-kg weight loss
B. Dietary sodium reduction
C. Moderation of alcohol consumption
D. Change to a vegetarian diet
E. Magnesium supplement
2371. Which of these statements is true regarding renin-angiotensin system-blocking agents?
A. Less effective when combined with a diuretic than when used alone
B. More effective in patients of African descent than in white patients
C. *Preserve kidney function in addition to lowering blood pressure
D. Are the first-line antihypertensive medications in pregnancy
E. All of the above
2372. Which of the following antihypertensive agents is INCORRECTLY matched with the indication for
therapy?
A. ACE inhibitor – diabetic nephropathy
B. Beta-blocker – coronary artery disease
C. Calcium channel blocker – angina pectoris
D. * Hydrochlorthiazide – diabetes mellitus
E. Loop diuretic – heart failure
2373. Which of the following statements about microalbuminuria is TRUE?
A. To be of clinixal value, microalbuminuria must be measured in a timed 12- to 24-hour sample
B. *Microalbuminuria is a cardiovascular risk factor that is independent of traditional Framingham risk
factors
C. Microalbuminuria is a predictor of risk only in patients with diabetes
D. Microalbuminuria is present when the “spot” urine albumin-to-creatinine ration is >500 mg/g
E. All of the above statements are correct
2374. Renin-dependent hypertension includes:
A. Primary hyperaldosteronism
B. Essential hypertension
C. *Renovascular hypertension
D. Pheochromocytoma
E. Cushing syndrome
2375. Fundus picture of stage 2 of Keith-Wegener classification includes
A. *Copper wire AV nipping and focal spasm
B. Silver wire increased reflex
C. Exudate and hemorrhagic spots
D. Papilloedema
E. Retinal hemorrhages
2376. A young hypertensive patient has serum potassium 2.8 mEq/l and increased aldosterone level with
decreased plasma renin activity. The likely diagnosis is....
A. Renal artery stenosis
B. Ectopic ACTH syndrome
C. *Conn syndrome
D. Liddle syndrome
E. Cushing syndrome
2377. The proportion of essential (primary) hypertension among all hypertension causes is as high as
A. 25-30%
B. 40-45%
C. 60-65%
D. 70-75%
E. *90-95%
2378. All of the following are risk factors for hypertension, EXCEPT
A. Increased body weight
B. Family history of hypertension
C. Excessive intake of sodium
D. *Regular use of one glass of wine per day
E. Cigarette smoking
2379. All of the following statements concerning hypertension are true, EXCEPT
A. In a western adult population the prevalence of hypertension exceeds 20%
B. Hypertension is a major risk factor for cardiovascular and cerebrovascular disease
C. The pathophysiology of hypertension differs in black adults compared to South Asians and whites
D. *People of African descent commonly have a high renin type hypertension
E. An increase in potassium intake may significantly reduce blood pressure in hypertensive patients
2380. Optimal blood pressure (BP) is defined as a BP level of
A. *<120/80 mmHg
B. <130/80 mmHg
C. <140/90 mmHg
D. <160/100 mmHg
E. <125/80 mmHg
2381. Which parameter of arterial blood pressure (BP) yields the best prognostic information in patients at
risk for cardiovascular disease?
A. Systolic BP
B. Diastolic BP
C. Mean arterial BP
D. *Pulse pressure
E. None
2382. Which BP profile below identifies the patient with the highest risk for development of cardiovascular
complications?
A. 160/90 mmHg
B. *160/65 mmHg
C. 140/100 mmHg
D. 130/90 mmHg
E. 120/70 mmHg
2383. Physicians should diagnose isolated office hypertension (so-called “white-coat hypertension”)
whenever office BP is ≥140/90 mmHg at several visits, while 24-h ambulatory BP is
A. <160/100 mmHg
B. <140/90 mmHg
C. *<130/85 mmHg
D. <125/80 mmHg
E. <120/70 mmHg
2384. Target organs in hypertension include all of the following EXCEPT
A. Brain and eyes
B. Heart
C. Kidneys
D. Peripheral arteries
E. *Liver
2385. All of the following statements concerning target organ damage are true EXCEPT
A. Microalbuminuria is a sensitive marker of hypertension-induced renal damage
B. *An increase in serum creatinine when antihypertensive therapy is intensified is a sign of progressive
renal deterioration
C. Electrocardiography should be part of all routine assessment of subjects with high BP
D. Echocardiography is much more sensitive than electrocardiography in diagnosing left ventricular
hypertrophy
E. Grades 1 and 2 arteriolar retinal changes do not have a significant prognostic value
2386. At a routine examination, an asymptomatic 46-year-old man is found to have a BP of 150/110
mmHg, but no other abnormalities are present. What do you do next?
A. Reassure the patient and repeat the physical examination in 12 months
B. Initiate antihypertensive therapy
C. *Obtain repeated BP recordings in your office and/or the patient’s home or work site
D. Hospitalize patient for renal arteriography
E. Order a 24-h ambulatory BP monitoring
2387. The initial antihypertensive medication recommended for patients who have no compelling
indications or contraindications is
A. ACE-inhibitor
B. Calcium-channel blocker
C. Diuretics
D. Beta blocker
E. *Any of the above
2388. In patients with a history of stroke or TIA the preferred drug combination is:
A. *ACE-inhibitor and diuretic
B. Calcium-channel blocker and beta-blocker
C. Beta-blocker and diuretic
D. Angiotensin receptor blocker and beta blocker
E. Beta- and alpha blocker
2389. In the elderly with systolic hypertension antihypertensive therapy should be initiated if SBP
A. ≥120 mmHg
B. ≥130 mmHg
C. *≥140 mmHg
D. ≥160 mmHg
E. ≥170 mmHg
2390. The best medication for the treatment of isolated systolic hypertension in the elderly:
A. ACE-inhibitor and diuretic
B. *Dihydropiridine calcium-channel blocker and diuretic
C. Beta-blocker and diuretic
D. Non-dihydropiridine calcium-channel blocker and diuretic
E. Beta- and alpha blocker
2391. The following medications can be used in pregnancy EXCEPT
A. Methyldopa
B. Labetalol
C. *ACE-inhibitor / ARB
D. Hydralazine
E. Nifidipine
2392. In patients of African descent the drug of choice for initial treatment of hypertension is:
A. Verapamil
B. Atenolol
C. Perindopril
D. *Hydrochlorothiazide
E. Amlodipine
2393. Which of the following antihypertensive agents is INCORRECTLY matched with the indication for
therapy?
A. ACE inhibitor – diabetic nephropathy
B. Beta-blocker – coronary artery disease
C. Calcium channel blocker – angina pectoris
D. Diuretics – heart failure
E. *Loop diuretic – gout
2394. All the following patients should be evaluated for secondary causes of hypertension EXCEPT
A. *A 47-year-old male with strong family history hypertension who presents to your office with a BP of
150/100
B. A 26-year-old female with hematuria and a family history of early renal failure who has a BP of 160/90
C. A 73-year-old male with no past history with a BP of 165/90
D. A 58-year-old male with a history of hypertension since age 45 whose BP has become increasingly
difficult to control on four antihypertensive agents
E. A 31-year-old female with complaints of severe headaches, weight gain, and new-onset DM with a BP
of 145/90
2395. Which of the following is an effective adjunct to increase the rate of smoking cessation?
A. Aldosterone
B. *Bupropion
C. Sildenafil
D. Cimetidine
E. Buspirone
2396. With which of the following is sildenafil acetate (Viagra) most likely to interact adversely?
A. Fibrinolytic therapy
B. Primary PCI
C. *Nitrates
D. Aspirin
E. Beta-blockers
2397. Bupropion hydrochloride (Zyban) is contraindicated in patients with a history of which of the
following?
A. *Seizures
B. Insulin-dependent diabetes mellitus
C. Severe chronic obstructive pulmonary disease (with reversible component)
D. Longer than 40-year history of tobacco use
E. Recent MI
2398. In which of the following has the high-sensitivity C-reactive protein (CRP) been shown to be
predictive of risk?
A. Acute MI
B. Acute coronary syndromes
C. Chronic stable angina
D. Peripheral vascular disease
E. *All of the above
2399. Which of the following has been shown to decrease the level of the high-sensitivity CRP?
A. Unopposed estrogen
B. Amlodipine besylate (Norvasc)
C. *Simvastatin
D. All of the above
E. None of the above
2400. All of the following statements regarding myocardial stunning are true EXCEPT:
A. *Stunning is a state of depressed myocardial function due to chronic hypoperfusion
B. Stunning can be global or regional
C. Stunning can follow cardiac surgery with cardiopulmonary bypass
D. Oxygen-free radicals and excess intracellular calcium likely contribute to stunning
E. Stunning affects both systolic and diastolic function
2401. What is the most common coexisting congenital anomaly in patients with coarctation of the aorta?
A. Cleft mitral valve
B. *Bicuspid aortic valve
C. Ebstein's anomaly
D. VSD
E. PDA
2402. All of the following are characteristic findings of ostium primum atrial septal defect (ASD)
EXCEPT:
A. Precordial heave
B. Fixed split S2
C. *Right axis deviation
D. Systolic ejection murmur
E. Prominent pulmonary vascular markings on CXR
2403. Choose the adult congenital disorder corresponding to the following physical examination fingings:
RV lift with a loud systolic ejection murmur along the left sternal border, with a single S2
A. Eisenmenger's syndrome
B. Coarctation of the aorta
C. PDA
D. Ebstein's anomaly
E. *Tetralogy of Fallot
2404. Choose the adult congenital disorder corresponding to the following physical examination fingings:
loud S1, holosystolic murmur in left sternal border, systolic ejection click, and hepatomegaly
A. Eisenmenger's syndrome
B. Coarctation of the aorta
C. PDA
D. *Ebstein's anomaly
E. Tetralogy of Fallot
2405. Choose the adult congenital disorder corresponding to the following physical examination fingings:
weak or delayed femoral pulses, harsh systolic murmur in the back, and a systolic ejection click in the aortic
area
A. Eisenmenger's syndrome
B. *Coarctation of the aorta
C. PDA
D. Ebstein's anomaly
E. Tetralogy of Fallot
2406. Choose the adult congenital disorder corresponding to the following physical examination fingings:
cyanosis, digital clubbing, loud P2, and a variable Graham-Steel murmur
A. *Eisenmenger's syndrome
B. Coarctation of the aorta
C. PDA
D. Ebstein's anomaly
E. Tetralogy of Fallot
2407. Choose the adult congenital disorder corresponding to the following physical examination fingings:
wide pulse pressure, prominent LV impulse, and a continuous machinery murmur enveloping S2
A. Eisenmenger's syndrome
B. Coarctation of the aorta
C. *Patent ductus arteriosus
D. Ebstein's anomaly
E. Tetralogy of Fallot
2408. Bacterial endocarditis prophylaxis is indicated in all adults who have the following congenital heart
disease EXCEPT:
A. VSD
B. Coarctation of the aorta
C. *Secundum ASD
D. Hypertrophic obstructive cardiomyopathy
E. PDA
2409. Besides pulmonary valve stenosis, which of the following is the most common associated cardiac
defect present in patients with PA stenosis?
A. *VSD
B. ASD
C. Coarctation of the aorta
D. PDA
E. Bicuspid aortic valve
2410. All of the following statements regarding patent ductus arteriosus (PDA) are true EXCEPT:
A. *The majority of cases close spontaneously after infancy
B. There is a higher incidence in mothers who acquired rubella during pregnancy
C. A decrease in the duration and intensity of the murmur has a poor prognostic implication
D. Left ventricular hypertrophy precedes RV hypertrophy
E. If it is uncorrected, approximately one third of patients die by the age of 40 years
2411. Ostium primum ASD is commonly associated with...
A. Supravalvular aortic stenosis
B. Supravalvular pulmonic stenosis
C. *Cleft mitral valve
D. Anomalous pulmonary venous drainage
E. Persistent left superior vena cava
2412. Noonan's syndrome is commonly associated with...
A. Supravalvular aortic stenosis
B. *Supravalvular pulmonic stenosis
C. Cleft mitral valve
D. Anomalous pulmonary venous drainage
E. Persistent left superior vena cava
2413. Coronary sinus ASD is commonly associated with...
A. Supravalvular aortic stenosis
B. Supravalvular pulmonic stenosis
C. Cleft mitral valve
D. Anomalous pulmonary venous drainage
E. *Persistent left superior vena cava
2414. Williams syndrome is commonly associated with...
A. Supravalvular aortic stenosis
B. *Supravalvular pulmonic stenosis
C. Cleft mitral valve
D. Anomalous pulmonary venous drainage
E. Persistent left superior vena cava
2415. Sinus venosus ASD is commonly associated with...
A. Supravalvular aortic stenosis
B. Supravalvular pulmonic stenosis
C. Cleft mitral valve
D. *Anomalous pulmonary venous drainage
E. Persistent left superior vena cava
2416. All of the following physical examination findings are usually associated with ostium secundum ASD
EXCEPT:
A. Precordial heave
B. Fixed split S2
C. *Lateral and inferior displacement of the apex beat
D. Soft systolic ejection murmur in the second left intercostal space
E. Normal S1
2417. Which of the following is an absolute contraindication to pregnancy?
A. Surgically corrected transposition of great arteries (TGA)
B. Congenitally corrected TGA
C. Ebstein's anomaly
D. *Eisenmenger's syndrome
E. Status post Fontan operation
2418. The lesions that constitute tetralogy of Fallot include all of the following EXCEPT
A. A ventricular septal defect
B. An overriding aorta
C. *An atrial septal defect
D. Right ventricular outflow obstruction
E. Right ventricular hypertrophy
2419. Match the chest radiography findings (Fig. 1.11) with the corresponding congenital disorder
A. *Eisenmenger's syndrome
B. Patent ductus arteriosus (PDA)
C. Ebstein's anomaly
D. Tetralogy of Fallot
E. Coarctation of the aorta
2420. Match the chest radiography findings (Fig. 1.12) with the corresponding congenital disorder
A. Eisenmenger's syndrome
B. Patent ductus arteriosus (PDA)
C. *Ebstein's anomaly
D. Tetralogy of Fallot
E. Coarctation of the aorta
2421. Match the chest radiography findings (Fig. 1.13) with the corresponding congenital disorder
A. Eisenmenger's syndrome
B. Patent ductus arteriosus (PDA)
C. Ebstein's anomaly
D. *Tetralogy of Fallot
E. Coarctation of the aorta
2422. Match the chest radiography findings (Fig. 1.14) with the corresponding congenital disorder
A. Eisenmenger's syndrome
B. *Patent ductus arteriosus (PDA)
C. Ebstein's anomaly
D. Tetralogy of Fallot
E. Coarctation of the aorta
2423. All of the following are indications for surgical closure of an ASD EXCEPT:
A. Significant symptoms in a 65-year-old
B. RV dysfunction
C. *Pulmonary vascular resistance >15 Wood units that does not diminish with vasodilators
D. An asymptomatic 20-year-old with a Qp/Qs of 1,7 with no pulmonary hypertension
E. RV enlargement
2424. The following cardiovascular malformations are all associated with congenital rubella EXCEPT:
A. PDA
B. PA stenosis
C. *Ebstein's anomaly
D. Tetralogy of Fallot
E. Coarctation of the aorta
2425. Which of the following syndromes is associated with pumonary arterio-venous fistula?
A. Williams syndrome
B. *Weber-Osler-Rendu syndrome
C. Bland-Garland-White syndrome
D. Kartagener's syndrome
E. Crouzon's syndrome
2426. Congenital MR is commonly encountered in all of the following conditions EXCEPT:
A. *Cor triatriatum
B. Ostium primum ASD
C. Coarctation of the aorta
D. Congenitally corrected TGA
E. Subaortic stenosis
2427. In which of the following cases is surgical correction recommended?
A. Asymptomatic small VSD to decrease risk of endocarditis
B. PDA with severe pulmonary hypertension
C. *Asymptomatic subaortic stenosis with severe aortic valve insufficiency
D. Coarctation of the aorta with a transcoarctation gradient of 20 mmHg
E. Small ASD to prevent paradoxical embolization
2428. Which congenital disorder corresponds with the following chest radiography findings: prominent
central PAs (possible calcifications) and peripheral PA prunning?
A. *Eisenmenger's syndrome
B. Coarctation of the aorta
C. PDA
D. Ebstein's anomaly
E. Tetralogy of Fallot
2429. Which congenital disorder corresponds with the following chest radiography findings: right aortic
arch, RV enlargement, and a "boot-shaped" heart?
A. Eisenmenger's syndrome
B. Coarctation of the aorta
C. PDA
D. Ebstein's anomaly
E. *Tetralogy of Fallot
2430. Which congenital disorder corresponds with the following chest radiography findings: marked
cardiomegaly, severe right atrial enlargement, and normal lung fields
A. Eisenmenger's syndrome
B. Coarctation of the aorta
C. PDA
D. *Ebstein's anomaly
E. Tetralogy of Fallot
2431. Which congenital disorder corresponds with the following chest radiography findings: posterior rib
notching and a "reverse E" or "3" sign
A. Eisenmenger's syndrome
B. *Coarctation of the aorta
C. PDA
D. Ebstein's anomaly
E. Tetralogy of Fallot
2432. Which congenital disorder corresponds with the following chest radiography findings: pulmonary
plethora, prominent ascending aorta, proximal PA dilatation, and opacity at the confluence of the aortic knob
and descending aorta
A. Eisenmenger's syndrome
B. Coarctation of the aorta
C. *PDA
D. Ebstein's anomaly
E. Tetralogy of Fallot
2433. Which of the following statements regarding Ebstein's anomaly is NOT correct?
A. An ASD or PFO is present in up to 80% of patients
B. The cardinal feature is an apically displaced tricuspid valve resulting in atrialization of ventricular
tissue
C. WPW syndrome is common in these patients and multiple tracts can exist
D. *A bicuspid aortic valve is commonly present
E. A "sail sound" is a common finding on physical examination
2434. Which of the following syndromes and cardiac anomalies are INCORRECTLY matched?
A. Trisomy 21 – atrioventricular canal defects
B. Noonan syndrome – pulmonic stenosis
C. Holt-Oram syndrome – atrial septal defects
D. Marfan syndrome – mitral valve prolapse
E. *Williams syndrome – ventricular septal defects
2435. All of the following statements regarding the effects of maneuvers on the auscultation of cardiac
murmurs are true EXCEPT:
A. In patent ductus arteriosus (PDA), the diastolic phase of the murmur is intensified by isometric
handgrip
B. *The murmur of hypertrophic obstructive cardiomyopathy becomes softer with standing or during a
Valsalva strain maneuver
C. The murmur of a ventricular septal defect (VSD) increases with isometric handgrip
D. Sudden squatting or isometric handgrip increases the diastolic murmur of aortic regurgitation
E. The mid-diastolic and presystolic murmurs of mitral stenosis become louder with exercise
2436. Radiotherapy for malignant disease may affect the heart in all of the following ways EXCEPT
A. Pericarditis
B. Coronary artery disease
C. Valvular stenosis
D. Conduction abnormalities
E. *Asymmetrical septal hypertrophy
2437. Sudden cardiac death due to ventricular tachyarrhythmias is an established complication of each of
the following EXCEPT:
A. Sarcoidosis
B. Giant cell myocarditis
C. Chagas disease
D. Idiopathic dilated cardiomyopathy
E. *Hyperthyroidism
2438. Characteristics of arrhythmogenic RV dysplasia include all of the following EXCEPT
A. Fatty infiltration of the right ventricle
B. *Fatty infiltration of the right atrium
C. Monomorphic ventricular tachycardia
D. Abnormal signal-averaged ECG
E. Possible detection by cardiac MRI
2439. All of the following may cause elevation of serum troponin EXCEPT:
A. Congestive heart failure
B. Myocarditis
C. Myocardial infarction
D. *Pneumonia
E. Pulmonary embolism
2440. Each of the statements about the evolution of the ECG in acute pericarditis is correct EXCEPT:
A. Four sequential stages of abnormalities of the ST segments and T waves are typical
B. *Initial ST segment elevation is usually most prominent in ECG lead aVR
C. The ratio of the height of ST segment elevation to the height of the T wave in acute pericarditis is
typically >0.25 in lead V6
D. In acute pericarditis, the ST segment usually returns to baseline before the appearance of T wave
inversion
E. T wave inversion may persist for months after the acute presentation
2441. Which of the following statements regarding cardiac involvement in Lyme disease is CORRECT?
A. Cardiac manifestations of Lyme disease typically occur within several days of the development of
erythema chronicum migricans
B. *Ten percent of patients with Lyme disease develop cardiac manifestations
C. Supraventricular and ventricular tachyarrhythmias are the most common cardiac manifestations of
Lyme disease
D. Cardiomegaly and congestive heart failure are common among patients who develop Lyme carditis
E. Antibiotic therapy has been proven to shorten the course of active Lyme carditis
2442. Each of the following statements regarding the infectious etiologies of pericarditis is correct
EXCEPT:
A. Enteroviruses are among the most common causes of viral pericarditis
B. The most common organisms that cause bacterial pericarditis are streptococci and staphylococci
C. The prognosis of patients with bacterial pericarditis is poor
D. *Tuberculosis is the leading cause of constrictive pericarditis in Western nations
E. Antifungal therapy is generally not necessary in pericarditis caused by histolasmosis
2443. Each of the following statements regarding Chagas disease is correct EXCEPT
A. The level of parasitemia does not correspond to the severity of chronic Chagas disease
B. The disease is transmitted to humans by the reduviid bug (commonly called the kissing bug)
C. *The most common ECG abnormality in chronic Chagas disease is left bundle branch block
D. The classic echocardiographic findings are those of a dilated cardiomyopathy with an apical aneurysm
E. All of the above statements are correct
2444. Which of the following disorders is not associated with ventricular tachycardia as a cause of
syncope?
A. Hypertrophic obstructive cardiomyopathy
B. Prior myocardial infarction
C. *Atrial myxoma
D. Aortic valvular stenosis
E. Congenital long QT syndrome
2445. Each of the following statements regarding the ECG in acute pericarditis is true EXCEPT:
A. The majority of patients demonstrate ECG abnormalities
B. Global ST elevation is seen in early pericarditis
C. Sinus tachycardia is a common finding
D. PR depression is common in the majority of patients
E. *T wave inversions develop before ST elevations return to baseline
2446. Each of the following statements regarding endomyocardial fibrosis (EMF) is correct EXCEPT:
A. This condition is characteristically found in tropical and subtropical Africa
B. It is predominantly a disease of children and young adults
C. *Involvement of the mitral apparatus typically results in mitral stenosis
D. EMF involves the left ventricle in 90% of affected patients
E. Echocardiographic features include increased endocardial echoreflectivity, fibrotic obliteration of apex,
atrial enlargement, and pericardial effusion
2447. All of the following statements with regard to primary endocardial fibroelastosis (EFE) are correct
EXCEPT:
A. The condition is often familial
B. The mitral and aortic valve leaflets are usually thickened and distorted
C. *The murmur of mitral stenosis is the most common auscultatory finding
D. Symptoms of primary EFE usually develop between 2 and 12 months of age
E. Echocardiographic features include a reduced ejection fraction and increased left atrial and left
ventricular dimensions
2448. Which of the following therapies improves survival in patients with cor pulmonare secondary to
COPD?
A. Digoxin
B. *Oxygen
C. Beta-adrenergic agonists
D. Theophylline
E. Hydralazine
2449. What is NOT a cause of chronic cor pulmonale?
A. COPD
B. Recurrent pulmonary emboli
C. *Mitral stenosis
D. Kyphoscoliosis
E. Obstructive sleep apnea
2450. Which of the following conditions is associated with increased left ventricular preload?
A. Sepsis
B. Right ventricular infarction
C. *Mitral regurgitation
D. Dehydration
E. Pulmonary embolism
2451. Each of the following conditions is associated with the development of pulmponary edema EXCEPT:
A. Increased pulmonary venous pressure
B. High altitude
C. *Increased plasma oncotic pressure
D. Eclampsia
E. Heroin overdose
2452. The statements about the diagnosis of PE include all of the following EXCEPT:
A. Arterial blood gas measurement is often unhelpful in the diagnosis of acute PE
B. The most common ECG finding in patients with PE is sinus tachycardia
C. Pulmonary infarction due to PE can be visualized on a standard chest radiograph
D. *Reduced intensity of the pulmonic component of the second heart sound is typical in patients with
large PE
E. Fibrin-degradation products (e.g., D-dimer) are commonly elevated in patients with PE
2453. Heart failure is:
A. Decreasing in incidence and prevalence due to recent advances in the treatment of cardiovascular
disease
B. *One of the diagnoses with the highest readmission rate
C. A common condition across all ages
D. Can be diagnosed from a clinical response to treatment
E. All of the above statements are correct
2454. According to the ESC guidelines, the following patients have heart failure:
A. An obese, dyspnoeic lady with swollen anckles but a normal echocardiogram
B. An asymptomatic man with a previous MI and area of hypokinesis on his echocardiogram
C. *A hypertensive man with LVH, fast atrial fibrillation and dyspnea
D. An elderly man with a few basal crepitations but a normal echocardiogram
E. All of the above
2455. Which of the following sometimes occur when heart failure is absent?
A. Gallop rhythm
B. Anckle oedema
C. A past history of a MI
D. *B and C
E. A, B, and C
2456. B-type natriuretic secretion is regulated by:
A. Fluid accumulation in the lungs
B. *Pressure and volume of the left ventricle
C. Oedema in the anckles
D. Auricular end diastolic pressure
E. All of the above
2457. Which of the following statements are true ?
A. A normal resting ECG is rare in angina pectoris
B. A normal resting ECG is common in heart failure
C. *Cardiomegaly plus pleural effusions make heart failure very likely
D. Atrial fibrillation commonly does not cause dyspnoea in patients with LVH and diastolic dysfunction
E. All of the above
2458. The value of BNP in the diagnosis of heart failure in the emergency room in patients with acute
dyspnoea:
A. *Is supported by a BNP concentration >500 pg/ml
B. Is excluded by a NT-proBNP concentration <1000 pg/ml
C. Is restricted to patients without a previous history of heart failure
D. Has not been evaluated in patients with renal dysfunction
E. All of the above statements are true
2459. Which symptoms/signs occur when the HF is relatively advanced ?
A. Cardiomegaly on chest X-ray
B. Orthopnea
C. Exertional dyspnea
D. A abd B
E. *All of the above
2460. All patients with heart failure should be advised to:
A. Refrain from smoking
B. Restrict sodium intake
C. Receive a flu shot
D. A and B
E. *All of the above
2461. Exercise training in chronic heart failure
A. *Has been shown to increase peak oxygen consumption by 15-20% in randomized controlled trials
B. Has been shown to increase LV ejection fraction by 15-20% in randomized controlled trials
C. Is not recommended in heart failure
D. Has never been shown to reduce mortality and morbidity
E. Non of above
2462. Drugs that should be avoided in heart failure patients are:
A. Non-steroidal anti-inflammatory drugs (NSAIDS)
B. Corticosteroids
C. Beta-blockers
D. *A and B
E. All of the above
2463. The followings improve the survival figure in chronic congestive heart failure as shown by many
studies...EXCEPT
A. Bisoprolol
B. Metoprolol succinate
C. *Atenolol
D. Spironolactone
E. Carvedilol
2464. The following drugs have significantly improved survival in HF patients
A. *ACE inhibitors
B. Nitrates
C. Digoxin
D. Diuretics
E. All of the above
2465. Which of the statements about beta-blocker treatment in HF is true?
A. Randomised clinical trials show that b blockers are beneficial in patients with heart failure and
preserved left ventricular function
B. Elderly patients are the most frequently studied population in blocker trials
C. * Blockers exert a favourable effect in heart failure due to ischaemic and non-ischaemic aetiology
D. Patients with diabetes should always be excluded from blocker treatment
E. There is clear evidence that end stage heart failure patients should be treated with blockers
2466. Which of the statements about beta-blocker treatment in HF is true?
A. *Beta-blockers must be started at very low dosages
B. Treatment should be started when patients are unstable in order to stabilize them
C. When patients are treated with ACE inhibitors, blockers are contraindicated
D. Treatment should always be started during hospital stay
E. The most frequent adverse reaction to a blocker is cough
2467. Which of the statements about antiarrhythmic therapy treatment in HF is NOT true?
A. Amiodarone is an antiarrhythmic drug whose use can improve symptoms of patients with heart failure
without affecting myocardial function and survival
B. *Class I antiarrhythmic agents reduce mortality of patients with heart failure
C. In patients with documented life threatening ventricular arrhythmias, implantable cardioverterdefribrillators (ICDs) can reduce sudden death
D. Sudden death is one of the most frequent causes of death in patients with heart failure
E. The reduction of the number of premature ectopic beats with antiarrhythmic drugs can prevent sudden
mortality of heart failure patients
2468. A prophylactic ICD is useful for reducing total mortality in patients:
A. With previous myocardial infarction and low ejection fraction
B. With dilated cardiomyopathy and low ejection fraction
C. With previous myocardial infarction and low ejection fraction and broad QRS
D. A and C
E. *All of the above
2469. Biventricular pacing in severe heart failure and broad QRS:
A. Improves NYHA class
B. Improves exercise tolerance
C. Reduces hospitalisation for heart failure
D. Reduces heart rate
E. *All of the above
2470. Which of the following treatments most consistently improves EF in patients who have systolic heart
failure?
A. Diuretics
B. Beta-blockers
C. *ACE-inhibitors
D. Vasodilators
E. All of the above
2471. BNP has which of the following properties?
A. Urine volume increases
B. Sodium excretion is enhanced
C. More BNP is secreted
D. A decrease in plasma aldosterone concentration occurs
E. *All of the above occur
2472. All of the following are indications for heart transplantation EXCEPT
A. Dilated cardiomyopathy
B. Diabetes mellitus
C. Hypertrophic cardiomyopathy
D. Age 60 to 70 years
E. *Amyloid heart disease
2473. When used chronically, all of the following drugs increase mortality EXCEPT
A. Milrinone
B. Dobutamine
C. Vesnarinone
D. Xamoterol
E. *Amlodipine
2474. Primary causes of diatolic heart failure include all of the following EXCEPT
A. Hypertrophic cardiomyopathy
B. *Dilated cardiomyopathy
C. Hypertension
D. Ischemic cardiomyopathy
E. Infiltrative cardiomyopathy
2475. Match the description with the associated form of therapy for pulmonary embolism: may be effective
in PE even 1 to 2 weeks after the onset of symptoms
A. Unfractionated heparin or low molecular weight heparin
B. Fibrinolytic therapy
C. *Both
D. Neither
E. Not applicable
2476. Match the description with the associated form of therapy for pulmonary embolism: dissolution of
recently formed thrombus is a major action
A. Unfractionated heparin or low molecular weight heparin
B. *Fibrinolytic therapy
C. Both
D. Neither
E. Not applicable
2477. Match the description with the associated form of therapy for pulmonary embolism: in general,
should be administered along with an antiplatelet agent
A. Unfractionated heparin or low molecular weight heparin
B. Fibrinolytic therapy
C. Both
D. *Neither
E. Not applicable
2478. Match the description with the associated form of therapy for pulmonary embolism: may cause
aldosterone depression
A. *Unfractionated heparin or low molecular weight heparin
B. Fibrinolytic therapy
C. Both
D. Neither
E. Not applicable
2479. Match the ECG finding that is most closely associated with chronic Chagas disease:
A. Low QRS voltage
B. AV nodal block
C. *Right bundle branch block
D. Diffuse ST segment elevation
E. Deeply inverted precordial T waves
2480. Match the ECG finding that is most closely associated with sarcoidosis:
A. Low QRS voltage
B. *AV nodal block
C. Right bundle branch block
D. Diffuse ST segment elevation
E. Deeply inverted precordial T waves
2481. Match the ECG finding that is most closely associated with apical hypertrophic cardiomyopathy
A. Low QRS voltage
B. AV nodal block
C. Right bundle branch block
D. Diffuse ST segment elevation
E. *Deeply inverted precordial T waves
2482. Match the ECG finding that is most closely associated with cardiac amyloidosis
A. *Low QRS voltage
B. AV nodal block
C. Right bundle branch block
D. Diffuse ST segment elevation
E. Deeply inverted precordial T waves
2483. Heart may be affected by radiotherapy in all of the following ways EXCEPT
A. Pericarditis
B. Coronary artery disease
C. Valvular stenosis
D. Conduction abnormalities
E. *Asymmetrical septal hypertrophy
2484. SCD (sudden cardiac death) due to ventricular tachyarrhythmias is an established complication of
each of the following EXCEPT:
A. Sarcoidosis
B. Giant cell myocarditis
C. Chagas disease
D. Idiopathic dilated cardiomyopathy
E. *Hyperthyroidism
2485. Arrhythmogenic RV dysplasia is characterized by all of the following EXCEPT
A. Fatty infiltration of the right ventricle
B. *Fatty infiltration of the right atrium
C. Monomorphic ventricular tachycardia
D. Abnormal signal-averaged ECG
E. Possible detection by cardiac MRI
2486. Each of the following may cause elevation of serum troponin EXCEPT:
A. Congestive heart failure
B. Myocarditis
C. Myocardial infarction
D. *Pneumonia
E. Pulmonary embolism
2487. All of the following statements regarding the infectious etiologies of pericarditis are correct
EXCEPT:
A. Enteroviruses are among the most common causes of viral pericarditis
B. The most common organisms that cause bacterial pericarditis are streptococci and staphylococci
C. The prognosis of patients with bacterial pericarditis is poor
D. *Tuberculosis is the leading cause of constrictive pericarditis in Western nations
E. Antifungal therapy is generally not necessary in pericarditis caused by histolasmosis
2488. All of the following statements regarding Chagas disease are correct EXCEPT
A. The level of parasitemia does not correspond to the severity of chronic Chagas disease
B. The disease is transmitted to humans by the reduviid bug (commonly called the kissing bug)
C. *The most common ECG abnormality in chronic Chagas disease is left bundle branch block
D. The classic echocardiographic findings are those of a dilated cardiomyopathy with an apical aneurysm
E. All of the above statements are correct
2489. Of the following disorders, choose one which is not associated with ventricular tachycardia as a cause
of syncope?
A. Hypertrophic obstructive cardiomyopathy
B. Prior myocardial infarction
C. *Atrial myxoma
D. Aortic valvular stenosis
E. Congenital long QT syndrome
2490. All of the following statements regarding the ECG in acute pericarditis is true EXCEPT:
A. The majority of patients demonstrate ECG abnormalities
B. Global ST elevation is seen in early pericarditis
C. Sinus tachycardia is a common finding
D. PR depression is common in the majority of patients
E. *T wave inversions develop before ST elevations return to baseline
2491. All the following features favor ventricular tachycardia as the cause of broad-complex tachycardia,
EXCEPT:
A. QRS duration with right bundle branch block morphology (RBBB) is greater than 140 ms, and greater
than 160 ms with left LBBB morphology
B. LAD with RBBB morphology, extreme LAD (northwest axis) with LBBB morphology
C. AV dissociation
D. *Response to carotid sinus massage
E. Concordance of the QRS pattern in all precordial leads (i.e., all positive or all negative deflections)
2492. Which of the following will most likely respond to verapamil?
A. RVOT tachycardia
B. *Idiopathic LV tachycardia
C. Bundle branch re-entry tachycardia
D. ARVD ventricular tachycardia
E. All of the above
2493. The most common type of arrhythmia in WPW syndrome is....
A. Ventricular tachycardia
B. Ventricular fibrillation
C. Ventricular premature complexes
D. Atrial ectopics
E. *AV re-entry tachycardia
2494. Use of echocardiogram in the assessment of AF:
A. It is not necessary to perform a TTE in patients with AF
B. TEE is always needed to allow better assessment of the MV and LA
C. *TTE should be performed at baseline to diagnose associated structural/functional heart disease
D. TTE is necessary to determine stroke risk stratification for thromboprophylaxis in all patients
E. When TTE is technically difficult, MRI is needed to exclude cardiac abnormalities
2495. Which is the preferred initial treatment strategy for rate control in AF patients?
A. Digoxin
B. *Beta-blockers or rate limiting CCB
C. Combination therapy of beta-blockers and rate-limiting CCB’s
D. Amiodarone
E. Combination therapy of digoxin and beta-blockers
2496. Which option is better in regard to cardioversion?
A. *Synchronised DC shock to restore sinus rhythm
B. Amiodarone in all patients to restore sinus rhythm
C. Flecainide in patients with IHD to maintain sinus rhythm
D. Digoxin to restore sinus rhythm
E. Propaphenone to restore sinus rhythm in HCMP patients
2497. Regarding antithrombotic treatment in AF:
A. Patients with asymptomatic AF have less risk of embolism than symptomatic patients
B. Patients with atrial flutter do not need antithrombotic treatment due to the low risk of embolism of this
arrhythmia
C. *Warfarin is the best option in patients with high risk of thromboembolism
D. Aspirin has a similar effect in stroke reduction to placebo
E. Paroxysmal AF has the lowest risk of embolism
2498. Sudden cardiac death in patients >35 years old is most commonly associated with:
A. Hypertrophic cardiomyopathy
B. *Coronary artery disease
C. Congenital long QT syndrome
D. Long QT syndrome secondary to various medications
E. arrhythymogenic RV cardiomyopathy
2499. Congenital long QT syndrome can lead to
A. Complete heart block
B. *Polymorphic ventricular tachycardia
C. Acute myocardial infarction
D. Recurrent supraventricular tachycardia
E. All of the above
2500. Bundle branch re-entry ventricular tachycardia is most commonly associated with:
A. Enhanced automaticity in the right bundle
B. Enhanced automaticity in the left bundle
C. Supranormal conduction in the His bundle
D. *Abnormally slow conduction in the His bundle
E. All of the above
2501. All of the following are ECG manifestations of Wolff-Parkinson-White syndrome EXCEPT
A. *Narrow QRS
B. Initial slurred QRS upstroke
C. Short PQ interval
D. Normal QT interval
E. All of the above
2502. The initial manifestations of WPW syndrome may include which of the following?
A. Atrial fibrillation
B. AV re-entry tachycardia
C. Ventricular fibrillation
D. Wide-QRS tachycardia
E. *All of the above
2503. The presence of an r prime in V1 during narrow-QRS tachycardia is suggestive of...
A. AV re-entry
B. *AV node re-entry
C. Rate-dependent BBB
D. Atrial tachycardia
E. Atrial fibrillation
2504. Ventricular ectopic beats (VEBs) occurring in apparently normal hearts are:
A. Associated with a bad prognosis
B. Never frequent or complex
C. Less frequent with increasing age
D. *May be associated with symptoms of missed beats
E. Always symptomatic
2505. In patients with ventricular ectopic beats:
A. Caffeine restriction always reduces symptoms
B. *High caffeine intake should be discouraged in symptomatic patients
C. There is good evidence that reducing caffeine intake reduces ectopics
D. Hypertension has no association with frequency of ectopic beats
E. LVH is not associated with increased frequency of ectopics
2506. Regarding ventricular ectopics and exercise:
A. *Ectopics are usually benign if they are supressed on exercise
B. Exercise-induced VEBs are not associated with adverse prognosis
C. RVOT ectopics are usually supressed on exercise
D. Ectopics induced on recovery after exercise are benign
E. All of the above
2507. An idiopathic left ventricular tachycardia (VT) shows the following...
A. Typical LBBB morphology
B. *LAD and RBBB morphology
C. Associated with a high risk of sudden cardiac death and almost always requires ICD therapy
D. Never responds to verapamil
E. All of the above
2508. Beta blockers are accepted treatment to prevent VT in:
A. Brugada syndrome, RVOT-VT, short QT syndrome and long QT syndrome
B. Brugada syndrome, RVOT-VT and long QT syndrome
C. *RVOT-VT and long QT syndrome
D. Short QT syndrome and long QT syndrome
E. Brugada syndrome and short QT syndrome
2509. Wenckebach phenomenon is defined as...
A. Progressive shortening of PR interval till a beat is dropped
B. *Progressive lengthening of PR interval till a beat is dropped
C. Irregular heart rate and PVC’s
D. Shortened QT intervals
E. Slurred QRS complex
2510. Giant a waves in JVP (“cannon” waves) are seen in
A. Sinus bradycardia
B. Atrial fibrillation
C. First-degree AV-block
D. *Complete heart block
E. Atrial ectopic
2511. All of the following findings are suggestive of LVH except:
A. (S in V1 + R in V5 or V6) > 35 mm
B. R in aVL > 11 mm
C. R in aVF > 20 mm
D. (R in I + S in III) > 25 mm
E. *R in aVR > 8 mm
2512. All of the following findings are suggestive of RVH except:
A. R in V1=7 mm (15 mm with RBBB)
B. (R in V1 + S in V5 or V6) > 10 mm
C. R < S in V6
D. R or R’ in V1
E. *S in V3 + R in aVL > 28 mm (20 mm for women)
2513. When present, each of the following heart sounds occurs shortly after S2 EXCEPT:
A. Opening snap
B. Third heart sound
C. *Ejection clic
D. Tumor plop
E. Pericardial knock
2514. Each of the following statements regarding splitting of the second heart sound is true EXCEPT:
A. Severe pulmonic valvular stenosis is associated with a softened P2
B. Delayed closure of the pulmonic valve with inspiration contributes to physiologic splitting of S2
C. Fixed splitting of S2 is the auscultatory hallmark of an ostium secundum atrial septal defect
D. Paradoxical splitting of S2 is expected in patients with a right ventricular electronically paced rhythm
E. *Right bundle branch block is associated with paradoxical splitting of S2
2515. The timing of an "innocent" murmur is usually:
A. Early systolic
B. Presystolic
C. *Midsystolic
D. Holosystolic
E. Early diastolic
2516. Which of the following pairs of medical conditions and antihypertensive medications would be
incorrect to use in a patient with essential hypertension?
A. Beta-blocker and a history of myocardial infarction
B. Alpha-blocker and prostatic hypertrophy
C. *Thiazide diuretic and gout
D. Amlodipine and heart failure
E. ACE-inhibitor and diabetes mellitus
2517. Thiazide diuretics can contribute to each of the following metabolic effects EXCEPT:
A. Hypomagnesemia
B. *Hypouricemia
C. Hypercalcemia
D. Hypercholesterolemia
E. Hyponatremia
2518. Which of the following statements regarding prevention of stroke is correct?
A. Treatment of systolic hypertension does not decrease the risk of stroke in patients older than age 60
B. Hypertension should not be a target of secondary prevention after an ischemic stroke because elevated
BP is desirable to maintain adequate cerebral perfusion
C. *Treatment with HMG-CoA reductase inhibitors reduces the risk of recurrent stroke
D. The combination of aspirin and clopidogrel is superior to aspirin alone for prevention of recurrent
stroke
E. None of the above
2519. You see a diabetic patient presents with BP readings that are 155/95 or higher. All of the following
statements about the treatment of this patient's hypertension are correct EXCEPT:
A. Pharmacologic blocade of the renin-angiotensin system reduces the risk of both microvascular and
macrovascular events
B. Aggressive BP control reduces cardiovascular events more in diabetics than in nondiabetics
C. *Calcium channel blockers show no benefit in reducing cardiovascular events
D. The goal BP for this patient is <130/80 mmHg
E. All the above statements are correct
2520. Which of the following antihypertensive agents is a known cause of autoimmune hemolytic anemia?
A. Metoprolol
B. *Methyldopa
C. Captopril
D. Losartan
E. Monoxidine
2521. All of the following characteristics are typical of hypertensive crisis EXCEPT:
A. Diastolic BP >120 mmHg
B. Retinal hemorrhages
C. *Constriction of cerebral arterioles with decreased vascular permeability
D. Proteinuria
E. Microangiopathic hemolytic anemia
2522. Which of the following pairs of medical conditions and antihypertensive medications would be
INCORRECT to use in a patient with essential hypertension?
A. Beta-blocker and a history of myocardial infarction
B. Alpha-blocker and prostatic hypertrophy
C. *Thiazide diuretic and gout
D. Amlodipine and heart failure
E. ACE-inhibitor and diabetes mellitus
2523. Which of the following statements regarding antihypertensive agents and atrial fibrillation (AF) is
TRUE?
A. *Losartan has been shown to decrease new-onset AF more effectively than atenolol
B. Valsartan has been shown to decrease new-onset AF more effectively than amlodipine
C. Atenolol has been shown to decrease new-onset AF more effectively than captopril
D. Nifedipine has been shown to decrease new-onset AF more effectively than diltiazem
E. All of the above are false
2524. Which lifestyle change has been shown to produce the biggest reduction in systolic blood pressure?
A. *10-kg weight loss
B. Dietary sodium reduction
C. Moderation of alcohol consumption
D. Change to a vegetarian diet
E. Magnesium supplement
2525. Which of these statements is true regarding renin-angiotensin system-blocking agents?
A. Less effective when combined with a diuretic than when used alone
B. More effective in patients of African descent than in white patients
C. *Preserve kidney function in addition to lowering blood pressure
D. Are the first-line antihypertensive medications in pregnancy
E. All of the above
2526. Which of the following antihypertensive agents is INCORRECTLY matched with the indication for
therapy?
A. ACE inhibitor – diabetic nephropathy
B. Beta-blocker – coronary artery disease
C. Calcium channel blocker – angina pectoris
D. *Hydrochlorthiazide – diabetes mellitus
E. Loop diuretic – heart failure
2527. Which of the following statements about microalbuminuria is TRUE?
A. To be of clinixal value, microalbuminuria must be measured in a timed 12- to 24-hour sample
B. *Microalbuminuria is a cardiovascular risk factor that is independent of traditional Framingham risk
factors
C. Microalbuminuria is a predictor of risk only in patients with diabetes
D. Microalbuminuria is present when the “spot” urine albumin-to-creatinine ration is >500 mg/g
E. All of the above statements are correct
2528. Renin-dependent hypertension includes:
A. Primary hyperaldosteronism
B. Essential hypertension
C. *Renovascular hypertension
D. Pheochromocytoma
E. Cushing syndrome
2529. Fundus picture of stage 2 of Keith-Wegener classification includes
A. *Copper wire AV nipping and focal spasm
B. Silver wire increased reflex
C. Exudate and hemorrhagic spots
D. Papilloedema
E. Retinal hemorrhages
2530. A young hypertensive patient has serum potassium 2.8 mEq/l and increased aldosterone level with
decreased plasma renin activity. The likely diagnosis is....
A. Renal artery stenosis
B. Ectopic ACTH syndrome
C. *Conn syndrome
D. Liddle syndrome
E. Cushing syndrome
2531. The proportion of essential (primary) hypertension among all hypertension causes is as high as
A. 25-30%
B. 40-45%
C. 60-65%
D. 70-75%
E. *90-95%
2532. All of the following are risk factors for hypertension, EXCEPT
A. Increased body weight
B. Family history of hypertension
C. Excessive intake of sodium
D. *Regular use of one glass of wine per day
E. Cigarette smoking
2533. All of the following statements concerning hypertension are true, EXCEPT
A. In a western adult population the prevalence of hypertension exceeds 20%
B. Hypertension is a major risk factor for cardiovascular and cerebrovascular disease
C. The pathophysiology of hypertension differs in black adults compared to South Asians and whites
D. *People of African descent commonly have a high renin type hypertension
E. An increase in potassium intake may significantly reduce blood pressure in hypertensive patients
2534. Optimal blood pressure (BP) is defined as a BP level of
A. *<120/80 mmHg
B. <130/80 mmHg
C. <140/90 mmHg
D. <160/100 mmHg
E. <125/80 mmHg
2535. Which parameter of arterial blood pressure (BP) yields the best prognostic information in patients at
risk for cardiovascular disease?
A. Systolic BP
B. Diastolic BP
C. Mean arterial BP
D. *Pulse pressure
E. None
2536. Which BP profile below identifies the patient with the highest risk for development of cardiovascular
complications?
A. 160/90 mmHg
B. *160/65 mmHg
C. 140/100 mmHg
D. 130/90 mmHg
E. 120/70 mmHg
2537. Physicians should diagnose isolated office hypertension (so-called “white-coat hypertension”)
whenever office BP is ≥140/90 mmHg at several visits, while 24-h ambulatory BP is
A. <160/100 mmHg
B. <140/90 mmHg
C. *<130/85 mmHg
D. <125/80 mmHg
E. <120/70 mmHg
2538. Target organs in hypertension include all of the following EXCEPT
A. Brain and eyes
B. Heart
C. Kidneys
D. Peripheral arteries
E. *Liver
2539. All of the following statements concerning target organ damage are true EXCEPT
A. Microalbuminuria is a sensitive marker of hypertension-induced renal damage
B. *An increase in serum creatinine when antihypertensive therapy is intensified is a sign of progressive
renal deterioration
C. Electrocardiography should be part of all routine assessment of subjects with high BP
D. Echocardiography is much more sensitive than electrocardiography in diagnosing left ventricular
hypertrophy
E. Grades 1 and 2 arteriolar retinal changes do not have a significant prognostic value
2540. The initial antihypertensive medication recommended for patients who have no compelling
indications or contraindications is
A. ACE-inhibitor
B. Calcium-channel blocker
C. Diuretics
D. Beta blocker
E. *Any of the above
2541. A 40-year-old diabetic patient presents with a blood pressure (BP) of 145/90 mmHg and proteinuria.
Which BP profile represents the best therapeutic goal for this patient?
A. <160/90
B. <140/90
C. *<130/85
D. <125/75
E. <140/85
2542. A 40-year-old diabetic patient presents with a blood pressure (BP) of 145/90 mmHg and proteinuria.
What is the best medication for the initial management of this patient’s hypertension?
A. Calcium channel blockers
B. Beta blockers
C. *ACE-inhibitors / angiotensin receptor blockers
D. Alpha blockers
E. Diuretics
2543. In patients with a history of stroke or TIA the preferred drug combination is:
A. *ACE-inhibitor and diuretic
B. Calcium-channel blocker and beta-blocker
C. Beta-blocker and diuretic
D. Angiotensin receptor blocker and beta blocker
E. Beta- and alpha blocker
2544. In the elderly with systolic hypertension antihypertensive therapy should be initiated if SBP
A. ≥120 mmHg
B. ≥130 mmHg
C. *≥140 mmHg
D. ≥160 mmHg
E. ≥170 mmHg
2545. The best medication for the treatment of isolated systolic hypertension in the elderly:
A. ACE-inhibitor and diuretic
B. *Dihydropiridine calcium-channel blocker and diuretic
C. Beta-blocker and diuretic
D. Non-dihydropiridine calcium-channel blocker and diuretic
E. Beta- and alpha blocker
2546. The following medications can be used in pregnancy EXCEPT
A. Methyldopa
B. Labetalol
C. *ACE-inhibitor / ARB
D. Hydralazine
E. Nifidipine
2547. In patients of African descent the drug of choice for initial treatment of hypertension is:
A. Verapamil
B. Atenolol
C. Perindopril
D. *Hydrochlorothiazide
E. Amlodipine
2548. Which of the following antihypertensive agents is INCORRECTLY matched with the indication for
therapy?
A. ACE inhibitor – diabetic nephropathy
B. Beta-blocker – coronary artery disease
C. Calcium channel blocker – angina pectoris
D. Diuretics – heart failure
E. *Loop diuretic – gout
2549. All the following patients should be evaluated for secondary causes of hypertension EXCEPT
A. *A 47-year-old male with strong family history hypertension who presents to your office with a BP of
150/100
B. A 26-year-old female with hematuria and a family history of early renal failure who has a BP of 160/90
C. A 73-year-old male with no past history with a BP of 165/90
D. A 58-year-old male with a history of hypertension since age 45 whose BP has become increasingly
difficult to control on four antihypertensive agents
E. A 31-year-old female with complaints of severe headaches, weight gain, and new-onset DM with a BP
of 145/90
2550. Which of the following diagnostic studies is most likely to demonstrate the cause of the headaches in
a patient with aortic coarctation?
A. MRI of the head
B. MRI of the kidney
C. *MRI of the thorax
D. 24-h urinary 5-HIAA
E. 24-h urinary free cortisol
2551. A 27-year-old female has hypertension that is difficult to control. She was diagnosed at age 25. Since
that time she has been on increasing amounts of medication. Her current regimen consists of labetalol 1000
mg bid, lisinopril 40 mg qd, clonidine 0.1 mg bid, and amlodipine 5 mg qd. On physical examination she
appears to be without distress. Her BP is 170/100, and HR is 84 bpm. Cardiac exam is unremarkable,
without rubs, gallops, or murmurs. She has good peripheral pulses and has no edema. Laboratory studies
reveal a potassium of 2.8 mEq/dL and a serum bicarbonate of 32 mEq/dL. Fasting blood glucose 114 mg/dL.
What is the likely diagnosis?
A. Congenital adrenal hyperplasia
B. Fibromuscular dysplasia
C. Cushing’s syndrome
D. *Conn’s syndrome
E. Pheochromocytoma
2552. A 27-year-old female has hypertension that is difficult to control. She was diagnosed at age 25. Since
that time she has been on increasing amounts of medication. Her current regimen consists of labetalol 1000
mg bid, lisinopril 40 mg qd, clonidine 0.1 mg bid, and amlodipine 5 mg qd. On physical examination she
appears to be without distress. Her BP is 170/100, and HR is 84 bpm. Cardiac exam is unremarkable,
without rubs, gallops, or murmurs. She has good peripheral pulses and has no edema. Laboratory studies
reveal a potassium of 2.8 mEq/dL and a serum bicarbonate of 32 mEq/dL. Fasting blood glucose 114 mg/dL.
What is the best way to diagnose this disease ?
A. Renal vein renin levels
B. 24-h urine collection for metanephrines
C. MRI of the renal arteries
D. 24-h urine collection for cortisol
E. *Plasma aldosterone/renin ratio
2553. A 20-year-old female is seen in the ED with symptoms of periodic headaches, sweating, and nausea
with vomiting. She also complains of feeling light-headed with standing. Her BP on presentation is 230/135,
with a HR of 92. On standing, the patient has BP of 205/100, with a HR of 136. On ophthalmologic
examination the patient has mild blurring of the optic discs without hemorrhage. The examination is
otherwise normal. What is the best medication for the management of this patient’s hypertension?
A. *Phentolamine
B. Fenoldopam
C. Esmolol
D. Nicardipine
E. Diazoxide
2554. A 20-year-old female is seen in the ED with symptoms of periodic headaches, sweating, and nausea
with vomiting. She also complains of feeling light-headed with standing. Her BP on presentation is 230/135,
with a HR of 92. On standing, the patient has BP of 205/100, with a HR of 136. On ophthalmologic
examination the patient has mild blurring of the optic discs without hemorrhage. The examination is
otherwise normal. What test would best determine the patient’s diagnosis?
A. Plasma catecholamines
B. 24-h urine collection for 5-HIAA
C. Abdominal CT scan
D. *24-h urine collection for metanephrines and vanilylmandelic acid
E. Adrenal vein sampling for renin levels
2555. What is the correct feature of the hypertensive urgency?
A. It is always a life-threatening situation
B. Patient must be hospitalized
C. *Patients can be managed as outpatients
D. Therapy should not be given orally
E. None of the above
2556. If there is suspicion of phaeochromocytoma, the first line drug is:
A. Sodium-nitroprusside
B. Nifedipine
C. *Phentolamine
D. Verapamil
E. Enalapril
2557. If you are in your office and a patient sees you with the following symptoms: long history of
hypertension, home BP usually 160/100 mmHg , suddenly developed severe headache, blurred vision,
weakness in the right arm, blood pressure: 200/110 mmHg. What will you do:
A. Give the patient 20 mg nifedipine orally, and check the BP in 15 min. If it is still >200/100 mmHg,
give another dose of nifedipine
B. Give the patient 20 mg nifedipine, and check the BP in 30 min. If it is 160/80 mmHg, send the patient
home and give another appointment in two weeks
C. Give the patient 10 mg amlodipine, send the patient home and give another appointment in two days
D. Call the ambulance and send the patient to the nearest hospital
E. *Give the patient some pain-killer drug (e.g. acetaminophen), call the ambulance and send the patient
to the nearest hospital with stroke-unit
2558. A 28-year-old male with type 1 diabetes mellitus is seen in the clinic for routine follow-up. The
patient has hypertension. He is being maintained on an insulin pump and lisinopril 5 mg daily. He has been
on this dose of lisinopril for approximately 6 weeks. He has normal creatinine and no proteinuria. BP is
140/90 mmHg. What do you recommend ?
A. Continue the current dose of lisinopril and return for follow-up in 3 months as the patient has received
his target BP of less than 140/90 mmHg
B. Ask a patient to return in 6 weeks; if his BP remains at this level, increase the dose of lisinopril
C. *Increase lisinopril to 10 mg daily to achieve a goal BP of less than 130/80 mmHg
D. Assure the patient that he is suffering from “white-coat hypertension” and plan no intervention
E. Switch the patient from lisinopril to losartan
2559. A 49-year-old female presents to you for an initial visit to establish health care. She has no past
medical history and denies a family history of diabetes mellitus, early cardiovascular disease, or renal
disease. Her BP on presentation is 185/110 mmHg. She denies headache or visual complaints. She has no
chest pain, shortness of breath, or change in urine color. Her physical examination is normal with the
exception of an S4 gallop heard at the 4th intercostal space just to the left of the sternum. An ECG is
significant for LVH and LAD. there are no signs of cardiac ischemia. Urinalysis shows no red blood cells or
proteinuria. What is your recommendation for therapy for this patient?
A. *Initiate therapy with hydrochlorothiazide in combination with a beta-blocker
B. Initiate therapy with hydrochlorothiazide alone
C. Initiate therapy with an ACE inhibitor alone
D. Advise the patient to go to the nearest emergency department for further treatment for a hypertensive
emergency
E. Advise the patient on lifestyle modifications and plan a return visit in 3 weeks
2560. What will you do if you are on duty in a hospital and a hypertensive patient, who has been treated
with long-acting ß-blocker, long-acting dihydropyridine calcium antagonist and long-acting a-1 adrenoceptor
blocker, is admitted with the following symptoms: headache, dyspnea, ankle-swelling, BP 180/110 mmHg,
pulmonary rales, LVH on ECG, echo: LVH, left ventricular diastolic dysfunction, ejection fraction 46%.
A. Amlodipine orally
B. Nifedipine orally
C. Enalaprilate iv
D. *Enalaprilate iv + furosemide iv
E. Esmolol iv
2561. Which of the following blood pressure levels in pregnancy should be considered an emergency
requiring hospitalization?
A. ≥140/90 mmHg
B. ≥150/95 mmHg
C. ≥160/100 mmHg
D. *≥170/110 mmHg
E. A rise in SBP ≥25 mmHg or a rise in DPB ≥15 mmHg compared to pre-pregnancy values, or those in
the first trimester
2562. The following drug combinations are effective to treat hypertension EXCEPT
A. Diuretics + beta-blockers
B. Beta-blockers + dihydropiridine calcium channel blockers
C. Diuretics + ACE-inhibitors/ARB
D. ACE-inhibitors/ARB + calcium channel blockers
E. *Beta-blockers + ACE-inhibitors/ARB
2563. Each of the following statements regarding hypertension is true EXCEPT:
A. Pure "white coat hypertension is found in 20 to 30% of patients
B. *When measuring BP, an inappropriately small cuff size results in a spuriously low systolic
measurement
C. Pseudohypertension may occur in patients with sclerotic brachial arteries
D. Chronic renal disease is the second most common cause of hypertension after essential hypertension
E. Coarctation of the aorta, Cushing disease, primary aldosteronism account for approximately 1% of all
hypertensive patients
2564. With respect to renovascular disease, all of the statements are true EXCEPT:
A. Worsening renal function with ACE-inhibitor therapy suggests bilateral renovascular disease
B. Atherosclerotic disease most commonly involves the proximal third of the main renal artery
C. The most common form of fibroplastic renovascular disease in adults involves the media
D. *The incidence of renovascular hypertension falls with increased age
E. Patients with severe, accelerated hypertension have the highest prevalence of renovascular disease
2565. A newly diagnosed diabetic patient presents with BP readings that are 155/95 or higher. All of the
following statements about the treatment of this patient's hypertension are correct EXCEPT:
A. Pharmacologic blockade of the renin-angiotensin system reduces the risk of both microvascular and
macrovascular events
B. Aggressive BP control reduces cardiovascular events more in diabetics than in nondiabetics
C. *Calcium channel blockers show no benefit in reducing cardiovascular events
D. The goal BP for this patient is <130/80 mmHg
E. All the above statements are correct
2566. All of the following statements regarding the association of oral contraceptive pills and hypertension
are correct EXCEPT:
A. The likelihood of developing hypertension is increased by significant alcohol consumption
B. The incidence of hypertension is about twice as great in pill users as in non-users
C. The likelihood of developing hypertension is dependent on the age of the user
D. *Hypertension resolves in almost all cases after discontinuation of oral contraceptives
E. The mechanism for contraceptive-induced hypertension likely involves renin-aldosterone-mediated
volume expansion
2567. Each of the following statements regarding pheochromocytoma is true EXCEPT:
A. Approximately 15% of pheochromocytomas are extra-adrenal
B. Ten percent of pheochromocytomas are malignant
C. Ten percent of adrenal pheochromocytomas are bilateral
D. *Hypertension related to pheochromocytoma is nearly always episodic and only rarely persistent
E. Pheochromocytoma can be inherited
2568. All of the following are features of renovascular hypertension due to fibromuscular hyperplasia, as
opposed to atherosclerosis, EXCEPT:
A. Age <50 years
B. Female gender
C. *More likely to progress to complete renal artery occlusion
D. No family history of hypertension
E. Absence of carotid bruits
2569. Which of the following ACE-inhibitors are not prodrugs?
A. Captopril, lisinopril, ramipril
B. Lisinopril, enalapril, banazepril
C. *Captopril, lisinopril, enalaprilat
D. Moexipril, captopril, lisinopril
E. Perindopril, ramipril, enalapril
2570. Each of the following statements regarding hypertension is true EXCEPT:
A. The prevalence of hypertension rises progressively with age in both men and women
B. Systolic and diastolic hypertension are each associated with an increased crisk of coronary heart
disease
C. *The target blood pressure for hypertensive patients with cardiovascular disease or diabetes is 140/90
mmHg
D. Patients with prehypertension (SBP 120-139 mmHg or DBP 80-89 mmHg) benefit from lifestyle
modifications incl. weight reduction, regular exercise, and smoking cessation
E. Drug therapy of hypertension benefits patients greater than 80 years of age
2571. All of the following interventions have a BP-lowering effect EXCEPT:
A. A diet that reduces caloric intake by 1000 calories per day
B. Reduction of dietary sodium
C. *Daily magnesium supplements
D. Tobacco cessation
E. Reduction of ethanol consumption to less than 30 ml/day
2572. Each of the following statements regarding hypertension is true EXCEPT:
A. Pure "white coat hypertension is found in 20 to 30% of patients
B. *When measuring BP, an inappropriately small cuff size results in a spuriously low systolic
measurement
C. Pseudohypertension may occur in patients with sclerotic brachial arteries
D. Chronic renal disease is the second most common cause of hypertension after essential hypertension
E. Coarctation of the aorta, Cushing disease, primary aldosteronism account for approximately 1% of all
hypertensive patients
2573. Each of the following statements regarding hypertension is true EXCEPT:
A. Increased LV muscle mass in hypertension is an independent risk factor of cardiac mortality
B. The risk of ventricular arrhythmias in hypertension is increased 2-fold in the presence of LV
hypertrophy
C. Low birth weight is associated with an increased incidence of hypertension later in life
D. *Chronic caffeine consumption is associated with an increased risk of hypertension
E. In hypertensive adolescents, essential hypertension is the most likely etiology
2574. The following statements regarding the association of oral contraceptive pills and hypertension are
correct EXCEPT:
A. The likelihood of developing hypertension is increased by significant alcohol consumption
B. The incidence of hypertension is about twice as great in pill users as in non-users
C. The likelihood of developing hypertension is dependent on the age of the user
D. *Hypertension resolves in almost all cases after discontinuation of oral contraceptives
E. The mechanism for contraceptive-induced hypertension likely involves renin-aldosterone-mediated
volume expansion
2575. Thiazide diuretics can contribute to each of the following metabolic effects EXCEPT:
A. Hypomagnesemia
B. *Hypouricemia
C. Hypercalcemia
D. Hypercholesterolemia
E. Hyponatremia
2576. Which of the following statements regarding prevention of stroke is correct?
A. Treatment of systolic hypertension does not decrease the risk of stroke in patients older than age 60
B. Hypertension should not be a target of secondary prevention after an ischemic stroke because elevated
BP is desirable to maintain adequate cerebral perfusion
C. *Treatment with HMG-CoA reductase inhibitors reduces the risk of recurrent stroke
D. The combination of aspirin and clopidogrel is superior to aspirin alone for prevention of recurrent
stroke
E. None of the above
2577. Which of the following antihypertensive agents is a known cause of autoimmune hemolytic anemia?
A. Metoprolol
B. *Methyldopa
C. Captopril
D. Losartan
E. Monoxidine
2578. All of the following characteristics are typical of hypertensive crisis EXCEPT:
A. Diastolic BP >120 mmHg
B. Retinal hemorrhages
C. Constriction of cerebral arterioles with decreased vascular permeability*
D. Proteinuria
E. Microangiopathic hemolytic anemia
2579. Which of the following pairs of medical conditions and antihypertensive medications would be
INCORRECT to use in a patient with essential hypertension?
A. Beta-blocker and a history of myocardial infarction
B. Alpha-blocker and prostatic hypertrophy
C. *Thiazide diuretic and gout
D. Amlodipine and heart failure
E. ACE-inhibitor and diabetes mellitus
2580. Which of the following statements regarding antihypertensive agents and atrial fibrillation (AF) is
TRUE?
A. *Losartan has been shown to decrease new-onset AF more effectively than atenolol
B. Valsartan has been shown to decrease new-onset AF more effectively than amlodipine
C. Atenolol has been shown to decrease new-onset AF more effectively than captopril
D. Nifedipine has been shown to decrease new-onset AF more effectively than diltiazem
E. All of the above are false
2581. Which lifestyle change has been shown to produce the biggest reduction in systolic blood pressure?
A. *10-kg weight loss
B. Dietary sodium reduction
C. Moderation of alcohol consumption
D. Change to a vegetarian diet
E. Magnesium supplement
2582. Which of these statements is true regarding renin-angiotensin system-blocking agents?
A. Less effective when combined with a diuretic than when used alone
B. More effective in patients of African descent than in white patients
C. *Preserve kidney function in addition to lowering blood pressure
D. Are the first-line antihypertensive medications in pregnancy
E. All of the above
2583. Which of the following antihypertensive agents is INCORRECTLY matched with the indication for
therapy?
A. ACE inhibitor – diabetic nephropathy
B. Beta-blocker – coronary artery disease
C. Calcium channel blocker – angina pectoris
D. * Hydrochlorthiazide – diabetes mellitus
E. Loop diuretic – heart failure
2584. Which of the following statements about microalbuminuria is TRUE?
A. To be of clinixal value, microalbuminuria must be measured in a timed 12- to 24-hour sample
B. *Microalbuminuria is a cardiovascular risk factor that is independent of traditional Framingham risk
factors
C. Microalbuminuria is a predictor of risk only in patients with diabetes
D. Microalbuminuria is present when the “spot” urine albumin-to-creatinine ration is >500 mg/g
E. All of the above statements are correct
2585. Renin-dependent hypertension includes:
A. Primary hyperaldosteronism
B. Essential hypertension
C. *Renovascular hypertension
D. Pheochromocytoma
E. Cushing syndrome
2586. Fundus picture of stage 2 of Keith-Wegener classification includes
A. *Copper wire AV nipping and focal spasm
B. Silver wire increased reflex
C. Exudate and hemorrhagic spots
D. Papilloedema
E. Retinal hemorrhages
2587. A young hypertensive patient has serum potassium 2.8 mEq/l and increased aldosterone level with
decreased plasma renin activity. The likely diagnosis is....
A. Renal artery stenosis
B. Ectopic ACTH syndrome
C. *Conn syndrome
D. Liddle syndrome
E. Cushing syndrome
2588. The proportion of essential (primary) hypertension among all hypertension causes is as high as
A. 25-30%
B. 40-45%
C. 60-65%
D. 70-75%
E. *90-95%
2589. All of the following are risk factors for hypertension, EXCEPT
A. Increased body weight
B. Family history of hypertension
C. Excessive intake of sodium
D. *Regular use of one glass of wine per day
E. Cigarette smoking
2590. All of the following statements concerning hypertension are true, EXCEPT
A. In a western adult population the prevalence of hypertension exceeds 20%
B. Hypertension is a major risk factor for cardiovascular and cerebrovascular disease
C. The pathophysiology of hypertension differs in black adults compared to South Asians and whites
D. People of African descent commonly have a high renin type hypertension*
E. An increase in potassium intake may significantly reduce blood pressure in hypertensive patients
2591. Optimal blood pressure (BP) is defined as a BP level of
A. *<120/80 mmHg
B. <130/80 mmHg
C. <140/90 mmHg
D. <160/100 mmHg
E. <125/80 mmHg
2592. Which parameter of arterial blood pressure (BP) yields the best prognostic information in patients at
risk for cardiovascular disease?
A. Systolic BP
B. Diastolic BP
C. Mean arterial BP
D. *Pulse pressure
E. None
2593. Which BP profile below identifies the patient with the highest risk for development of cardiovascular
complications?
A. 160/90 mmHg
B. *160/65 mmHg
C. 140/100 mmHg
D. 130/90 mmHg
E. 120/70 mmHg
2594. Physicians should diagnose isolated office hypertension (so-called “white-coat hypertension”)
whenever office BP is ≥140/90 mmHg at several visits, while 24-h ambulatory BP is
A. <160/100 mmHg
B. <140/90 mmHg
C. *<130/85 mmHg
D. <125/80 mmHg
E. <120/70 mmHg
2595. Target organs in hypertension include all of the following EXCEPT
A. Brain and eyes
B. Heart
C. Kidneys
D. Peripheral arteries
E. *Liver
2596. All of the following statements concerning target organ damage are true EXCEPT
A. Microalbuminuria is a sensitive marker of hypertension-induced renal damage
B. *An increase in serum creatinine when antihypertensive therapy is intensified is a sign of progressive
renal deterioration
C. Electrocardiography should be part of all routine assessment of subjects with high BP
D. Echocardiography is much more sensitive than electrocardiography in diagnosing left ventricular
hypertrophy
E. Grades 1 and 2 arteriolar retinal changes do not have a significant prognostic value
2597. At a routine examination, an asymptomatic 46-year-old man is found to have a BP of 150/110
mmHg, but no other abnormalities are present. What do you do next?
A. Reassure the patient and repeat the physical examination in 12 months
B. Initiate antihypertensive therapy
C. *Obtain repeated BP recordings in your office and/or the patient’s home or work site
D. Hospitalize patient for renal arteriography
E. Order a 24-h ambulatory BP monitoring
2598. The initial antihypertensive medication recommended for patients who have no compelling
indications or contraindications is
A. ACE-inhibitor
B. Calcium-channel blocker
C. Diuretics
D. Beta blocker
E. *Any of the above
2599. In patients with a history of stroke or TIA the preferred drug combination is:
A. *ACE-inhibitor and diuretic
B. Calcium-channel blocker and beta-blocker
C. Beta-blocker and diuretic
D. Angiotensin receptor blocker and beta blocker
E. Beta- and alpha blocker
2600. In the elderly with systolic hypertension antihypertensive therapy should be initiated if SBP
A. ≥120 mmHg
B. ≥130 mmHg
C. *≥140 mmHg
D. ≥160 mmHg
E. ≥170 mmHg
2601. The best medication for the treatment of isolated systolic hypertension in the elderly:
A. ACE-inhibitor and diuretic
B. *Dihydropiridine calcium-channel blocker and diuretic
C. Beta-blocker and diuretic
D. Non-dihydropiridine calcium-channel blocker and diuretic
E. Beta- and alpha blocker
2602. The following medications can be used in pregnancy EXCEPT
A. Methyldopa
B. Labetalol
C. *ACE-inhibitor / ARB
D. Hydralazine
E. Nifidipine
2603. In patients of African descent the drug of choice for initial treatment of hypertension is:
A. Verapamil
B. Atenolol
C. Perindopril
D. *Hydrochlorothiazide
E. Amlodipine
2604. Which of the following antihypertensive agents is INCORRECTLY matched with the indication for
therapy?
A. ACE inhibitor – diabetic nephropathy
B. Beta-blocker – coronary artery disease
C. Calcium channel blocker – angina pectoris
D. Diuretics – heart failure
E. *Loop diuretic – gout
2605. All the following patients should be evaluated for secondary causes of hypertension EXCEPT
A. *A 47-year-old male with strong family history hypertension who presents to your office with a BP of
150/100
B. A 26-year-old female with hematuria and a family history of early renal failure who has a BP of 160/90
C. A 73-year-old male with no past history with a BP of 165/90
D. A 58-year-old male with a history of hypertension since age 45 whose BP has become increasingly
difficult to control on four antihypertensive agents
E. A 31-year-old female with complaints of severe headaches, weight gain, and new-onset DM with a BP
of 145/90
2606. Which of the following is an effective adjunct to increase the rate of smoking cessation?
A. Aldosterone
B. *Bupropion
C. Sildenafil
D. Cimetidine
E. Buspirone
2607. With which of the following is sildenafil acetate (Viagra) most likely to interact adversely?
A. Fibrinolytic therapy
B. Primary PCI
C. *Nitrates
D. Aspirin
E. Beta-blockers
2608. Bupropion hydrochloride (Zyban) is contraindicated in patients with a history of which of the
following?
A. *Seizures
B. Insulin-dependent diabetes mellitus
C. Severe chronic obstructive pulmonary disease (with reversible component)
D. Longer than 40-year history of tobacco use
E. Recent MI
2609. In which of the following has the high-sensitivity C-reactive protein (CRP) been shown to be
predictive of risk?
A. Acute MI
B. Acute coronary syndromes
C. Chronic stable angina
D. Peripheral vascular disease
E. *All of the above
2610. Which of the following has been shown to decrease the level of the high-sensitivity CRP?
A. Unopposed estrogen
B. Amlodipine besylate (Norvasc)
C. *Simvastatin
D. All of the above
E. None of the above
2611. All of the following statements regarding myocardial stunning are true EXCEPT:
A. *Stunning is a state of depressed myocardial function due to chronic hypoperfusion
B. Stunning can be global or regional
C. Stunning can follow cardiac surgery with cardiopulmonary bypass
D. Oxygen-free radicals and excess intracellular calcium likely contribute to stunning
E. Stunning affects both systolic and diastolic function
2612. What is the most common coexisting congenital anomaly in patients with coarctation of the aorta?
A. Cleft mitral valve
B. *Bicuspid aortic valve
C. Ebstein's anomaly
D. VSD
E. PDA
2613. All of the following are characteristic findings of ostium primum atrial septal defect (ASD)
EXCEPT:
A. Precordial heave
B. Fixed split S2
C. *Right axis deviation
D. Systolic ejection murmur
E. Prominent pulmonary vascular markings on CXR
2614. Choose the adult congenital disorder corresponding to the following physical examination fingings:
RV lift with a loud systolic ejection murmur along the left sternal border, with a single S2
A. Eisenmenger's syndrome
B. Coarctation of the aorta
C. PDA
D. Ebstein's anomaly
E. *Tetralogy of Fallot
2615. Choose the adult congenital disorder corresponding to the following physical examination fingings:
loud S1, holosystolic murmur in left sternal border, systolic ejection click, and hepatomegaly
A. Eisenmenger's syndrome
B. Coarctation of the aorta
C. PDA
D. *Ebstein's anomaly
E. Tetralogy of Fallot
2616. Choose the adult congenital disorder corresponding to the following physical examination fingings:
weak or delayed femoral pulses, harsh systolic murmur in the back, and a systolic ejection click in the aortic
area
A. Eisenmenger's syndrome
B. *Coarctation of the aorta
C. PDA
D. Ebstein's anomaly
E. Tetralogy of Fallot
2617. Choose the adult congenital disorder corresponding to the following physical examination fingings:
cyanosis, digital clubbing, loud P2, and a variable Graham-Steel murmur
A. *Eisenmenger's syndrome
B. Coarctation of the aorta
C. PDA
D. Ebstein's anomaly
E. Tetralogy of Fallot
2618. Choose the adult congenital disorder corresponding to the following physical examination fingings:
wide pulse pressure, prominent LV impulse, and a continuous machinery murmur enveloping S2
A. Eisenmenger's syndrome
B. Coarctation of the aorta
C. *Patent ductus arteriosus
D. Ebstein's anomaly
E. Tetralogy of Fallot
2619. Bacterial endocarditis prophylaxis is indicated in all adults who have the following congenital heart
disease EXCEPT:
A. VSD
B. Coarctation of the aorta
C. *Secundum ASD
D. Hypertrophic obstructive cardiomyopathy
E. PDA
2620. Besides pulmonary valve stenosis, which of the following is the most common associated cardiac
defect present in patients with PA stenosis?
A. *VSD
B. ASD
C. Coarctation of the aorta
D. PDA
E. Bicuspid aortic valve
2621. All of the following statements regarding patent ductus arteriosus (PDA) are true EXCEPT:
A. *The majority of cases close spontaneously after infancy
B. There is a higher incidence in mothers who acquired rubella during pregnancy
C. A decrease in the duration and intensity of the murmur has a poor prognostic implication
D. Left ventricular hypertrophy precedes RV hypertrophy
E. If it is uncorrected, approximately one third of patients die by the age of 40 years
2622. Ostium primum ASD is commonly associated with...
A. Supravalvular aortic stenosis
B. Supravalvular pulmonic stenosis
C. *Cleft mitral valve
D. Anomalous pulmonary venous drainage
E. Persistent left superior vena cava
2623. Noonan's syndrome is commonly associated with...
A. Supravalvular aortic stenosis
B. *Supravalvular pulmonic stenosis
C. Cleft mitral valve
D. Anomalous pulmonary venous drainage
E. Persistent left superior vena cava
2624. Coronary sinus ASD is commonly associated with...
A. Supravalvular aortic stenosis
B. Supravalvular pulmonic stenosis
C. Cleft mitral valve
D. Anomalous pulmonary venous drainage
E. *Persistent left superior vena cava
2625. Williams syndrome is commonly associated with...
A. Supravalvular aortic stenosis
B. *Supravalvular pulmonic stenosis
C. Cleft mitral valve
D. Anomalous pulmonary venous drainage
E. Persistent left superior vena cava
2626. Sinus venosus ASD is commonly associated with...
A. Supravalvular aortic stenosis
B. Supravalvular pulmonic stenosis
C. Cleft mitral valve
D. *Anomalous pulmonary venous drainage
E. Persistent left superior vena cava
2627. All of the following physical examination findings are usually associated with ostium secundum ASD
EXCEPT:
A. Precordial heave
B. Fixed split S2
C. *Lateral and inferior displacement of the apex beat
D. Soft systolic ejection murmur in the second left intercostal space
E. Normal S1
2628. Which of the following is an absolute contraindication to pregnancy?
A. Surgically corrected transposition of great arteries (TGA)
B. Congenitally corrected TGA
C. Ebstein's anomaly
D. *Eisenmenger's syndrome
E. Status post Fontan operation
2629. The lesions that constitute tetralogy of Fallot include all of the following EXCEPT
A. A ventricular septal defect
B. An overriding aorta
C. *An atrial septal defect
D. Right ventricular outflow obstruction
E. Right ventricular hypertrophy
2630. Match the chest radiography findings (prominent central PAs and peripheral PA prunning) with the
corresponding congenital disorder
A. *Eisenmenger's syndrome
B. Patent ductus arteriosus (PDA)
C. Ebstein's anomaly
D. Tetralogy of Fallot
E. Coarctation of the aorta
2631. Match the chest radiography findings (marked cardiomegaly, severe right atrial enlargement, and
normal lung fields) with the corresponding congenital disorder
A. Eisenmenger's syndrome
B. Patent ductus arteriosus (PDA)
C. *Ebstein's anomaly
D. Tetralogy of Fallot
E. Coarctation of the aorta
2632. Match the chest radiography findings (RV enlargement, and a "boot-shaped" heart) with the
corresponding congenital disorder
A. Eisenmenger's syndrome
B. Patent ductus arteriosus (PDA)
C. Ebstein's anomaly
D. *Tetralogy of Fallot
E. Coarctation of the aorta
2633. Match the chest radiography findings (pulmonary plethora, prominent ascending aorta, proximal PA
dilatation) with the corresponding congenital disorder
A. Eisenmenger's syndrome
B. *Patent ductus arteriosus (PDA)
C. Ebstein's anomaly
D. Tetralogy of Fallot
E. Coarctation of the aorta
2634. All of the following are indications for surgical closure of an ASD EXCEPT:
A. Significant symptoms in a 65-year-old
B. RV dysfunction
C. *Pulmonary vascular resistance >15 Wood units that does not diminish with vasodilators
D. An asymptomatic 20-year-old with a Qp/Qs of 1,7 with no pulmonary hypertension
E. RV enlargement
2635. The following cardiovascular malformations are all associated with congenital rubella EXCEPT:
A. PDA
B. PA stenosis
C. *Ebstein's anomaly
D. Tetralogy of Fallot
E. Coarctation of the aorta
2636. Which of the following syndromes is associated with pumonary arterio-venous fistula?
A. Williams syndrome
B. *Weber-Osler-Rendu syndrome
C. Bland-Garland-White syndrome
D. Kartagener's syndrome
E. Crouzon's syndrome
2637. Congenital MR is commonly encountered in all of the following conditions EXCEPT:
A. *Cor triatriatum
B. Ostium primum ASD
C. Coarctation of the aorta
D. Congenitally corrected TGA
E. Subaortic stenosis
2638. In which of the following cases is surgical correction recommended?
A. Asymptomatic small VSD to decrease risk of endocarditis
B. PDA with severe pulmonary hypertension
C. *Asymptomatic subaortic stenosis with severe aortic valve insufficiency
D. Coarctation of the aorta with a transcoarctation gradient of 20 mmHg
E. Small ASD to prevent paradoxical embolization
2639. Which congenital disorder corresponds with the following chest radiography findings: prominent
central PAs (possible calcifications) and peripheral PA prunning?
A. *Eisenmenger's syndrome
B. Coarctation of the aorta
C. PDA
D. Ebstein's anomaly
E. Tetralogy of Fallot
2640. Which congenital disorder corresponds with the following chest radiography findings: right aortic
arch, RV enlargement, and a "boot-shaped" heart?
A. Eisenmenger's syndrome
B. Coarctation of the aorta
C. PDA
D. Ebstein's anomaly
E. *Tetralogy of Fallot
2641. Which congenital disorder corresponds with the following chest radiography findings: marked
cardiomegaly, severe right atrial enlargement, and normal lung fields
A. Eisenmenger's syndrome
B. Coarctation of the aorta
C. PDA
D. *Ebstein's anomaly
E. Tetralogy of Fallot
2642. Which congenital disorder corresponds with the following chest radiography findings: posterior rib
notching and a "reverse E" or "3" sign
A. Eisenmenger's syndrome
B. *Coarctation of the aorta
C. PDA
D. Ebstein's anomaly
E. Tetralogy of Fallot
2643. Which congenital disorder corresponds with the following chest radiography findings: pulmonary
plethora, prominent ascending aorta, proximal PA dilatation, and opacity at the confluence of the aortic knob
and descending aorta
A. Eisenmenger's syndrome
B. Coarctation of the aorta
C. *PDA
D. Ebstein's anomaly
E. Tetralogy of Fallot
2644. Which of the following statements regarding Ebstein's anomaly is NOT correct?
A. An ASD or PFO is present in up to 80% of patients
B. The cardinal feature is an apically displaced tricuspid valve resulting in atrialization of ventricular
tissue
C. WPW syndrome is common in these patients and multiple tracts can exist
D. *A bicuspid aortic valve is commonly present
E. A "sail sound" is a common finding on physical examination
2645. Which of the following syndromes and cardiac anomalies are INCORRECTLY matched?
A. Trisomy 21 – atrioventricular canal defects
B. Noonan syndrome – pulmonic stenosis
C. Holt-Oram syndrome – atrial septal defects
D. Marfan syndrome – mitral valve prolapse
E. *Williams syndrome – ventricular septal defects
2646. All of the following statements regarding the effects of maneuvers on the auscultation of cardiac
murmurs are true EXCEPT:
A. In patent ductus arteriosus (PDA), the diastolic phase of the murmur is intensified by isometric
handgrip
B. *The murmur of hypertrophic obstructive cardiomyopathy becomes softer with standing or during a
Valsalva strain maneuver
C. The murmur of a ventricular septal defect (VSD) increases with isometric handgrip
D. Sudden squatting or isometric handgrip increases the diastolic murmur of aortic regurgitation
E. The mid-diastolic and presystolic murmurs of mitral stenosis become louder with exercise
2647. Radiotherapy for malignant disease may affect the heart in all of the following ways EXCEPT
A. Pericarditis
B. Coronary artery disease
C. Valvular stenosis
D. Conduction abnormalities
E. *Asymmetrical septal hypertrophy
2648. Sudden cardiac death due to ventricular tachyarrhythmias is an established complication of each of
the following EXCEPT:
A. Sarcoidosis
B. Giant cell myocarditis
C. Chagas disease
D. Idiopathic dilated cardiomyopathy
E. *Hyperthyroidism
2649. Characteristics of arrhythmogenic RV dysplasia include all of the following EXCEPT
A. Fatty infiltration of the right ventricle
B. *Fatty infiltration of the right atrium
C. Monomorphic ventricular tachycardia
D. Abnormal signal-averaged ECG
E. Possible detection by cardiac MRI
2650. All of the following may cause elevation of serum troponin EXCEPT:
A. Congestive heart failure
B. Myocarditis
C. Myocardial infarction
D. *Pneumonia
E. Pulmonary embolism
2651. Each of the statements about the evolution of the ECG in acute pericarditis is correct EXCEPT:
A. Four sequential stages of abnormalities of the ST segments and T waves are typical
B. *Initial ST segment elevation is usually most prominent in ECG lead aVR
C. The ratio of the height of ST segment elevation to the height of the T wave in acute pericarditis is
typically >0.25 in lead V6
D. In acute pericarditis, the ST segment usually returns to baseline before the appearance of T wave
inversion
E. T wave inversion may persist for months after the acute presentation
2652. Which of the following statements regarding cardiac involvement in Lyme disease is CORRECT?
A. Cardiac manifestations of Lyme disease typically occur within several days of the development of
erythema chronicum migricans
B. *Ten percent of patients with Lyme disease develop cardiac manifestations
C. Supraventricular and ventricular tachyarrhythmias are the most common cardiac manifestations of
Lyme disease
D. Cardiomegaly and congestive heart failure are common among patients who develop Lyme carditis
E. Antibiotic therapy has been proven to shorten the course of active Lyme carditis
2653. Each of the following statements regarding the infectious etiologies of pericarditis is correct
EXCEPT:
A. Enteroviruses are among the most common causes of viral pericarditis
B. The most common organisms that cause bacterial pericarditis are streptococci and staphylococci
C. The prognosis of patients with bacterial pericarditis is poor
D. *Tuberculosis is the leading cause of constrictive pericarditis in Western nations
E. Antifungal therapy is generally not necessary in pericarditis caused by histolasmosis
2654. Each of the following statements regarding Chagas disease is correct EXCEPT
A. The level of parasitemia does not correspond to the severity of chronic Chagas disease
B. The disease is transmitted to humans by the reduviid bug (commonly called the kissing bug)
C. *The most common ECG abnormality in chronic Chagas disease is left bundle branch block
D. The classic echocardiographic findings are those of a dilated cardiomyopathy with an apical aneurysm
E. All of the above statements are correct
2655. Which of the following disorders is not associated with ventricular tachycardia as a cause of
syncope?
A. Hypertrophic obstructive cardiomyopathy
B. Prior myocardial infarction
C. *Atrial myxoma
D. Aortic valvular stenosis
E. Congenital long QT syndrome
2656. Each of the following statements regarding the ECG in acute pericarditis is true EXCEPT:
A. The majority of patients demonstrate ECG abnormalities
B. Global ST elevation is seen in early pericarditis
C. Sinus tachycardia is a common finding
D. PR depression is common in the majority of patients
E. *T wave inversions develop before ST elevations return to baseline
2657. Each of the following statements regarding endomyocardial fibrosis (EMF) is correct EXCEPT:
A. This condition is characteristically found in tropical and subtropical Africa
B. It is predominantly a disease of children and young adults
C. *Involvement of the mitral apparatus typically results in mitral stenosis
D. EMF involves the left ventricle in 90% of affected patients
E. Echocardiographic features include increased endocardial echoreflectivity, fibrotic obliteration of apex,
atrial enlargement, and pericardial effusion
2658. All of the following statements with regard to primary endocardial fibroelastosis (EFE) are correct
EXCEPT:
A. The condition is often familial
B. The mitral and aortic valve leaflets are usually thickened and distorted
C. *The murmur of mitral stenosis is the most common auscultatory finding
D. Symptoms of primary EFE usually develop between 2 and 12 months of age
E. Echocardiographic features include a reduced ejection fraction and increased left atrial and left
ventricular dimensions
2659. Which of the following therapies improves survival in patients with cor pulmonare secondary to
COPD?
A. Digoxin
B. *Oxygen
C. Beta-adrenergic agonists
D. Theophylline
E. Hydralazine
2660. What is NOT a cause of chronic cor pulmonale?
A. COPD
B. Recurrent pulmonary emboli
C. *Mitral stenosis
D. Kyphoscoliosis
E. Obstructive sleep apnea
2661. Which of the following conditions is associated with increased left ventricular preload?
A. Sepsis
B. Right ventricular infarction
C. *Mitral regurgitation
D. Dehydration
E. Pulmonary embolism
2662. Each of the following conditions is associated with the development of pulmponary edema EXCEPT:
A. Increased pulmonary venous pressure
B. High altitude
C. *Increased plasma oncotic pressure
D. Eclampsia
E. Heroin overdose
2663. The statements about the diagnosis of PE include all of the following EXCEPT:
A. Arterial blood gas measurement is often unhelpful in the diagnosis of acute PE
B. The most common ECG finding in patients with PE is sinus tachycardia
C. Pulmonary infarction due to PE can be visualized on a standard chest radiograph
D. *Reduced intensity of the pulmonic component of the second heart sound is typical in patients with
large PE
E. Fibrin-degradation products (e.g., D-dimer) are commonly elevated in patients with PE
2664. Heart failure is:
A. Decreasing in incidence and prevalence due to recent advances in the treatment of cardiovascular
disease
B. *One of the diagnoses with the highest readmission rate
C. A common condition across all ages
D. Can be diagnosed from a clinical response to treatment
E. All of the above statements are correct
2665. According to the ESC guidelines, the following patients have heart failure:
A. An obese, dyspnoeic lady with swollen anckles but a normal echocardiogram
B. An asymptomatic man with a previous MI and area of hypokinesis on his echocardiogram
C. *A hypertensive man with LVH, fast atrial fibrillation and dyspnea
D. An elderly man with a few basal crepitations but a normal echocardiogram
E. All of the above
2666. Which of the following sometimes occur when heart failure is absent?
A. Gallop rhythm
B. Anckle oedema
C. A past history of a MI
D. *B and C
E. A, B, and C
2667. B-type natriuretic secretion is regulated by:
A. Fluid accumulation in the lungs
B. *Pressure and volume of the left ventricle
C. Oedema in the anckles
D. Auricular end diastolic pressure
E. All of the above
2668. Which of the following statements are true ?
A. A normal resting ECG is rare in angina pectoris
B. A normal resting ECG is common in heart failure
C. *Cardiomegaly plus pleural effusions make heart failure very likely
D. Atrial fibrillation commonly does not cause dyspnoea in patients with LVH and diastolic dysfunction
E. All of the above
2669. The value of BNP in the diagnosis of heart failure in the emergency room in patients with acute
dyspnoea:
A. *Is supported by a BNP concentration >500 pg/ml
B. Is excluded by a NT-proBNP concentration <1000 pg/ml
C. Is restricted to patients without a previous history of heart failure
D. Has not been evaluated in patients with renal dysfunction
E. All of the above statements are true
2670. Which symptoms/signs only occur when the HF is relatively advanced – that is, they are commonly
absent in mild heart failure
A. Cardiomegaly on chest X-ray
B. Orthopnea
C. Exertional dyspnea
D. *A abd B
E. B and C
2671. All patients with heart failure should be advised to:
A. Refrain from smoking
B. Restrict sodium intake
C. Avoid a flu shot
D. *A and B
E. All of the above
2672. Exercise training in chronic heart failure
A. *Has been shown to increase peak oxygen consumption by 15-20% in randomized controlled trials
B. Has been shown to increase LV ejection fraction by 15-20% in randomized controlled trials
C. Is not recommended in heart failure
D. Has never been shown to reduce mortality and morbidity
E. Non of above
2673. Drugs that should be avoided in heart failure patients are:
A. Non-steroidal anti-inflammatory drugs (NSAIDS)
B. Corticosteroids
C. Beta-blockers
D. *A and B
E. All of the above
2674. The followings improve the survival figure in chronic congestive heart failure as shown by many
studies...EXCEPT
A. Bisoprolol
B. Metoprolol succinate
C. *Atenolol
D. Spironolactone
E. Carvedilol
2675. The following drugs have significantly improved survival in HF patients
A. *ACE inhibitors
B. Nitrates
C. Digoxin
D. Diuretics
E. All of the above
2676. Which of the statements about beta-blocker treatment in HF is true?
A. Randomised clinical trials show that b blockers are beneficial in patients with heart failure and
preserved left ventricular function
B. Elderly patients are the most frequently studied population in blocker trials
C. * Blockers exert a favourable effect in heart failure due to ischaemic and non-ischaemic aetiology
D. Patients with diabetes should always be excluded from blocker treatment
E. There is clear evidence that end stage heart failure patients should be treated with blockers
2677. Which of the statements about beta-blocker treatment in HF is true?
A. *Beta-blockers must be started at very low dosages
B. Treatment should be started when patients are unstable in order to stabilize them
C. When patients are treated with ACE inhibitors, blockers are contraindicated
D. Treatment should always be started during hospital stay
E. The most frequent adverse reaction to a blocker is cough
2678. Which of the statements about antiarrhythmic therapy treatment in HF is NOT true?
A. Amiodarone is an antiarrhythmic drug whose use can improve symptoms of patients with heart failure
without affecting myocardial function and survival
B. *Class I antiarrhythmic agents reduce mortality of patients with heart failure
C. In patients with documented life threatening ventricular arrhythmias, implantable cardioverterdefribrillators (ICDs) can reduce sudden death
D. Sudden death is one of the most frequent causes of death in patients with heart failure
E. The reduction of the number of premature ectopic beats with antiarrhythmic drugs can prevent sudden
mortality of heart failure patients
2679. A prophylactic ICD is useful for reducing total mortality in patients:
A. With previous myocardial infarction and low ejection fraction
B. With dilated cardiomyopathy and low ejection fraction
C. With previous myocardial infarction and low ejection fraction and broad QRS
D. A and C
E. *All of the above
2680. Biventricular pacing in severe heart failure and broad QRS:
A. Improves NYHA class
B. Improves exercise tolerance
C. Reduces hospitalisation for heart failure
D. Reduces heart rate
E. *All of the above
2681. Which of the following treatments most consistently improves EF in patients who have systolic heart
failure?
A. Diuretics
B. Beta-blockers
C. *ACE-inhibitors
D. Vasodilators
E. All of the above
2682. BNP has which of the following properties?
A. Urine volume increases
B. Sodium excretion is enhanced
C. More BNP is secreted
D. A decrease in plasma aldosterone concentration occurs
E. *All of the above occur
2683. All of the following are indications for heart transplantation EXCEPT
A. Dilated cardiomyopathy
B. Diabetes mellitus
C. Hypertrophic cardiomyopathy
D. Age 60 to 70 years
E. *Amyloid heart disease
2684. When used chronically, all of the following drugs increase mortality EXCEPT
A. Milrinone
B. Dobutamine
C. Vesnarinone
D. Xamoterol
E. *Amlodipine
2685. Primary causes of diatolic heart failure include all of the following EXCEPT
A. Hypertrophic cardiomyopathy
B. *Dilated cardiomyopathy
C. Hypertension
D. Ischemic cardiomyopathy
E. Infiltrative cardiomyopathy
2686. Match the description with the associated form of therapy for pulmonary embolism: may be effective
in PE even 1 to 2 weeks after the onset of symptoms
A. Unfractionated heparin or low molecular weight heparin
B. Fibrinolytic therapy
C. *Both
D. Neither
E. Not applicable
2687. Match the description with the associated form of therapy for pulmonary embolism: dissolution of
recently formed thrombus is a major action
A. Unfractionated heparin or low molecular weight heparin
B. *Fibrinolytic therapy
C. Both
D. Neither
E. Not applicable
2688. Match the description with the associated form of therapy for pulmonary embolism: in general,
should be administered along with an antiplatelet agent
A. Unfractionated heparin or low molecular weight heparin
B. Fibrinolytic therapy
C. Both
D. *Neither
E. Not applicable
2689. Match the description with the associated form of therapy for pulmonary embolism: may cause
aldosterone depression
A. *Unfractionated heparin or low molecular weight heparin
B. Fibrinolytic therapy
C. Both
D. Neither
E. Not applicable
2690. Match the ECG finding that is most closely associated with chronic Chagas disease:
A. Low QRS voltage
B. AV nodal block
C. *Right bundle branch block
D. Diffuse ST segment elevation
E. Deeply inverted precordial T waves
2691. Match the ECG finding that is most closely associated with sarcoidosis:
A. Low QRS voltage
B. *AV nodal block
C. Right bundle branch block
D. Diffuse ST segment elevation
E. Deeply inverted precordial T waves
2692. Match the ECG finding that is most closely associated with apical hypertrophic cardiomyopathy
A. Low QRS voltage
B. AV nodal block
C. Right bundle branch block
D. Diffuse ST segment elevation
E. *Deeply inverted precordial T waves
2693. Match the ECG finding that is most closely associated with cardiac amyloidosis
A. *Low QRS voltage
B. AV nodal block
C. Right bundle branch block
D. Diffuse ST segment elevation
E. Deeply inverted precordial T waves
2694. Heart may be affected by radiotherapy in all of the following ways EXCEPT
A. Pericarditis
B. Coronary artery disease
C. Valvular stenosis
D. Conduction abnormalities
E. *Asymmetrical septal hypertrophy
2695. SCD (sudden cardiac death) due to ventricular tachyarrhythmias is an established complication of
each of the following EXCEPT:
A. Sarcoidosis
B. Giant cell myocarditis
C. Chagas disease
D. Idiopathic dilated cardiomyopathy
E. *Hyperthyroidism
2696. Arrhythmogenic RV dysplasia is characterized by all of the following EXCEPT
A. Fatty infiltration of the right ventricle
B. *Fatty infiltration of the right atrium
C. Monomorphic ventricular tachycardia
D. Abnormal signal-averaged ECG
E. Possible detection by cardiac MRI
2697. Each of the following may cause elevation of serum troponin EXCEPT:
A. Congestive heart failure
B. Myocarditis
C. Myocardial infarction
D. *Pneumonia
E. Pulmonary embolism
2698. All of the following statements regarding the infectious etiologies of pericarditis are correct
EXCEPT:
A. Enteroviruses are among the most common causes of viral pericarditis
B. The most common organisms that cause bacterial pericarditis are streptococci and staphylococci
C. The prognosis of patients with bacterial pericarditis is poor
D. *Tuberculosis is the leading cause of constrictive pericarditis in Western nations
E. Antifungal therapy is generally not necessary in pericarditis caused by histolasmosis
2699. All of the following statements regarding Chagas disease are correct EXCEPT
A. The level of parasitemia does not correspond to the severity of chronic Chagas disease
B. The disease is transmitted to humans by the reduviid bug (commonly called the kissing bug)
C. *The most common ECG abnormality in chronic Chagas disease is left bundle branch block
D. The classic echocardiographic findings are those of a dilated cardiomyopathy with an apical aneurysm
E. All of the above statements are correct
2700. Of the following disorders, choose one which is not associated with ventricular tachycardia as a cause
of syncope?
A. Hypertrophic obstructive cardiomyopathy
B. Prior myocardial infarction
C. *Atrial myxoma
D. Aortic valvular stenosis
E. Congenital long QT syndrome
2701. All of the following statements regarding the ECG in acute pericarditis is true EXCEPT:
A. The majority of patients demonstrate ECG abnormalities
B. Global ST elevation is seen in early pericarditis
C. Sinus tachycardia is a common finding
D. PR depression is common in the majority of patients
E. *T wave inversions develop before ST elevations return to baseline
2702. All the following features favor ventricular tachycardia as the cause of broad-complex tachycardia,
EXCEPT:
A. QRS duration with right bundle branch block morphology (RBBB) is greater than 140 ms, and greater
than 160 ms with left LBBB morphology
B. LAD with RBBB morphology, extreme LAD (northwest axis) with LBBB morphology
C. AV dissociation
D. *Response to carotid sinus massage
E. Concordance of the QRS pattern in all precordial leads (i.e., all positive or all negative deflections)
2703. Which of the following will most likely respond to verapamil?
A. RVOT tachycardia
B. *Idiopathic LV tachycardia
C. Bundle branch re-entry tachycardia
D. ARVD ventricular tachycardia
E. All of the above
2704. The most common type of arrhythmia in WPW syndrome is....
A. Ventricular tachycardia
B. Ventricular fibrillation
C. Ventricular premature complexes
D. Atrial ectopics
E. *AV re-entry tachycardia
2705. Use of echocardiogram in the assessment of AF:
A. It is not necessary to perform a TTE in patients with AF
B. TEE is always needed to allow better assessment of the MV and LA
C. *TTE should be performed at baseline to diagnose associated structural/functional heart disease
D. TTE is necessary to determine stroke risk stratification for thromboprophylaxis in all patients
E. When TTE is technically difficult, MRI is needed to exclude cardiac abnormalities
2706. Which is the preferred initial treatment strategy for rate control in AF patients?
A. Digoxin
B. *Beta-blockers or rate limiting CCB
C. Combination therapy of beta-blockers and rate-limiting CCB’s
D. Amiodarone
E. Combination therapy of digoxin and beta-blockers
2707. Which option is better in regard to cardioversion?
A. *Synchronised DC shock to restore sinus rhythm
B. Amiodarone in all patients to restore sinus rhythm
C. Flecainide in patients with IHD to maintain sinus rhythm
D. Digoxin to restore sinus rhythm
E. Propaphenone to restore sinus rhythm in HCMP patients
2708. Regarding antithrombotic treatment in AF:
A. Patients with asymptomatic AF have less risk of embolism than symptomatic patients
B. Patients with atrial flutter do not need antithrombotic treatment due to the low risk of embolism of this
arrhythmia
C. *Warfarin is the best option in patients with high risk of thromboembolism
D. Aspirin has a similar effect in stroke reduction to placebo
E. Paroxysmal AF has the lowest risk of embolism
2709. Sudden cardiac death in patients >35 years old is most commonly associated with:
A. Hypertrophic cardiomyopathy
B. *Coronary artery disease
C. Congenital long QT syndrome
D. Long QT syndrome secondary to various medications
E. arrhythymogenic RV cardiomyopathy
2710. Congenital long QT syndrome can lead to
A. Complete heart block
B. *Polymorphic ventricular tachycardia
C. Acute myocardial infarction
D. Recurrent supraventricular tachycardia
E. All of the above
2711. Bundle branch re-entry ventricular tachycardia is most commonly associated with:
A. Enhanced automaticity in the right bundle
B. Enhanced automaticity in the left bundle
C. Supranormal conduction in the His bundle
D. *Abnormally slow conduction in the His bundle
E. All of the above
2712. All of the following are ECG manifestations of Wolff-Parkinson-White syndrome EXCEPT
A. *Narrow QRS
B. Initial slurred QRS upstroke
C. Short PQ interval
D. Normal QT interval
E. All of the above
2713. The initial manifestations of WPW syndrome may include which of the following?
A. Atrial fibrillation
B. AV re-entry tachycardia
C. Ventricular fibrillation
D. Wide-QRS tachycardia
E. *All of the above
2714. The presence of an r prime in V1 during narrow-QRS tachycardia is suggestive of...
A. AV re-entry
B. *AV node re-entry
C. Rate-dependent BBB
D. Atrial tachycardia
E. Atrial fibrillation
2715. Ventricular ectopic beats (VEBs) occurring in apparently normal hearts are:
A. Associated with a bad prognosis
B. Never frequent or complex
C. Less frequent with increasing age
D. *May be associated with symptoms of “missed beats”
E. Always symptomatic
2716. In patients with ventricular ectopic beats:
A. Caffeine restriction always reduces symptoms
B. *High caffeine intake should always be discouraged in symptomatic patients
C. There is good evidence that reducing caffeine intake reduces ectopics
D. Hypertension has no association with frequency of ectopic beats
E. LVH is not associated with increased frequency of ectopics
2717. Regarding ventricular ectopics and exercise:
A. *Ectopics are usually benign if they are supressed on exercise
B. Exercise-induced VEBs are not associated with adverse prognosis
C. RVOT ectopics are usually supressed on exercise
D. Ectopics induced on recovery after exercise are benign
E. All of the above
2718. An idiopathic left ventricular tachycardia (VT) shows the following...
A. Typical LBBB morphology
B. *LAD and RBBB morphology
C. Associated with a high risk of sudden cardiac death and almost always requires ICD therapy
D. Never responds to verapamil
E. All of the above
2719. Beta blockers are accepted treatment to prevent VT in:
A. Brugada syndrome, RVOT-VT, short QT syndrome and long QT syndrome
B. Brugada syndrome, RVOT-VT and long QT syndrome
C. *RVOT-VT and long QT syndrome
D. Short QT syndrome and long QT syndrome
E. Brugada syndrome and short QT syndrome
2720. Wenckebach phenomenon is defined as...
A. Progressive shortening of PR interval till a beat is dropped
B. *Progressive lengthening of PR interval till a beat is dropped
C. Irregular heart rate and PVC’s
D. Shortened QT intervals
E. Slurred QRS complex
2721. Giant a waves in JVP (“cannon” waves) are seen in
A. Sinus bradycardia
B. Atrial fibrillation
C. First-degree AV-block
D. *Complete heart block
E. Atrial ectopic
2722. All of the following findings are suggestive of LVH except:
A. (S in V1 + R in V5 or V6) > 35 mm
B. R in aVL > 11 mm
C. R in aVF > 20 mm
D. (R in I + S in III) > 25 mm
E. *R in aVR > 8 mm
2723. All of the following findings are suggestive of RVH except:
A. R in V1=7 mm (15 mm with RBBB)
B. (R in V1 + S in V5 or V6) > 10 mm
C. R < S in V6
D. R or R’ in V1
E. *S in V3 + R in aVL > 28 mm (20 mm for women)
2724. When present, each of the following heart sounds occurs shortly after S2 EXCEPT:
A. Opening snap
B. Third heart sound
C. *Ejection clic
D. Tumor plop
E. Pericardial knock
2725. Each of the following statements regarding splitting of the second heart sound is true EXCEPT:
A. Severe pulmonic valvular stenosis is associated with a softened P2
B. Delayed closure of the pulmonic valve with inspiration contributes to physiologic splitting of S2
C. Fixed splitting of S2 is the auscultatory hallmark of an ostium secundum atrial septal defect
D. Paradoxical splitting of S2 is expected in patients with a right ventricular electronically paced rhythm
E. *Right bundle branch block is associated with paradoxical splitting of S2
2726. The timing of an "innocent" murmur is usually:
A. Early systolic
B. Presystolic
C. *Midsystolic
D. Holosystolic
E. Early diastolic
2727. Which of the following pairs of medical conditions and antihypertensive medications would be
incorrect to use in a patient with essential hypertension?
A. Beta-blocker and a history of myocardial infarction
B. Alpha-blocker and prostatic hypertrophy
C. *Thiazide diuretic and gout
D. Amlodipine and heart failure
E. ACE-inhibitor and diabetes mellitus
2728. Thiazide diuretics can contribute to each of the following metabolic effects EXCEPT:
A. Hypomagnesemia
B. *Hypouricemia
C. Hypercalcemia
D. Hypercholesterolemia
E. Hyponatremia
2729. Which of the following statements regarding prevention of stroke is correct?
A. Treatment of systolic hypertension does not decrease the risk of stroke in patients older than age 60
B. Hypertension should not be a target of secondary prevention after an ischemic stroke because elevated
BP is desirable to maintain adequate cerebral perfusion
C. *Treatment with HMG-CoA reductase inhibitors reduces the risk of recurrent stroke
D. The combination of aspirin and clopidogrel is superior to aspirin alone for prevention of recurrent
stroke
E. None of the above
2730. You see a diabetic patient presents with BP readings that are 155/95 or higher. All of the following
statements about the treatment of this patient's hypertension are correct EXCEPT:
A. Pharmacologic blocade of the renin-angiotensin system reduces the risk of both microvascular and
macrovascular events
B. Aggressive BP control reduces cardiovascular events more in diabetics than in nondiabetics
C. *Calcium channel blockers show no benefit in reducing cardiovascular events
D. The goal BP for this patient is <130/80 mmHg
E. All the above statements are correct
2731. Which of the following antihypertensive agents is a known cause of autoimmune hemolytic anemia?
A. Metoprolol
B. *Methyldopa
C. Captopril
D. Losartan
E. Monoxidine
2732. All of the following characteristics are typical of hypertensive crisis EXCEPT:
A. Diastolic BP >120 mmHg
B. Retinal hemorrhages
C. *Constriction of cerebral arterioles with decreased vascular permeability
D. Proteinuria
E. Microangiopathic hemolytic anemia
2733. Which of the following pairs of medical conditions and antihypertensive medications would be
INCORRECT to use in a patient with essential hypertension?
A. Beta-blocker and a history of myocardial infarction
B. Alpha-blocker and prostatic hypertrophy
C. *Thiazide diuretic and gout
D. Amlodipine and heart failure
E. ACE-inhibitor and diabetes mellitus
2734. Which of the following statements regarding antihypertensive agents and atrial fibrillation (AF) is
TRUE?
A. *Losartan has been shown to decrease new-onset AF more effectively than atenolol
B. Valsartan has been shown to decrease new-onset AF more effectively than amlodipine
C. Atenolol has been shown to decrease new-onset AF more effectively than captopril
D. Nifedipine has been shown to decrease new-onset AF more effectively than diltiazem
E. All of the above are false
2735. Which lifestyle change has been shown to produce the biggest reduction in systolic blood pressure?
A. *10-kg weight loss
B. Dietary sodium reduction
C. Moderation of alcohol consumption
D. Change to a vegetarian diet
E. Magnesium supplement
2736. Which of these statements is true regarding renin-angiotensin system-blocking agents?
A. Less effective when combined with a diuretic than when used alone
B. More effective in patients of African descent than in white patients
C. *Preserve kidney function in addition to lowering blood pressure
D. Are the first-line antihypertensive medications in pregnancy
E. All of the above
2737. Which of the following antihypertensive agents is INCORRECTLY matched with the indication for
therapy?
A. ACE inhibitor – diabetic nephropathy
B. Beta-blocker – coronary artery disease
C. Calcium channel blocker – angina pectoris
D. * Hydrochlorthiazide – diabetes mellitus
E. Loop diuretic – heart failure
2738. Which of the following statements about microalbuminuria is TRUE?
A. To be of clinixal value, microalbuminuria must be measured in a timed 12- to 24-hour sample
B. *Microalbuminuria is a cardiovascular risk factor that is independent of traditional Framingham risk
factors
C. Microalbuminuria is a predictor of risk only in patients with diabetes
D. Microalbuminuria is present when the “spot” urine albumin-to-creatinine ration is >500 mg/g
E. All of the above statements are correct
2739. Renin-dependent hypertension includes:
A. Primary hyperaldosteronism
B. Essential hypertension
C. *Renovascular hypertension
D. Pheochromocytoma
E. Cushing syndrome
2740. Fundus picture of stage 2 of Keith-Wegener classification includes
A. *Copper wire AV nipping and focal spasm
B. Silver wire increased reflex
C. Exudate and hemorrhagic spots
D. Papilloedema
E. Retinal hemorrhages
2741. A young hypertensive patient has serum potassium 2.8 mEq/l and increased aldosterone level with
decreased plasma renin activity. The likely diagnosis is....
A. Renal artery stenosis
B. Ectopic ACTH syndrome
C. *Conn syndrome
D. Liddle syndrome
E. Cushing syndrome
2742. The proportion of essential (primary) hypertension among all hypertension causes is as high as
A. 25-30%
B. 40-45%
C. 60-65%
D. 70-75%
E. *90-95%
2743. All of the following are risk factors for hypertension, EXCEPT
A. Increased body weight
B. Family history of hypertension
C. Excessive intake of sodium
D. *Regular use of one glass of wine per day
E. Cigarette smoking
2744. All of the following statements concerning hypertension are true, EXCEPT
A. In a western adult population the prevalence of hypertension exceeds 20%
B. Hypertension is a major risk factor for cardiovascular and cerebrovascular disease
C. The pathophysiology of hypertension differs in black adults compared to South Asians and whites
D. *People of African descent commonly have a high renin type hypertension
E. An increase in potassium intake may significantly reduce blood pressure in hypertensive patients
2745. Optimal blood pressure (BP) is defined as a BP level of
A. *<120/80 mmHg
B. <130/80 mmHg
C. <140/90 mmHg
D. <160/100 mmHg
E. <125/80 mmHg
2746. Which parameter of arterial blood pressure (BP) yields the best prognostic information in patients at
risk for cardiovascular disease?
A. Systolic BP
B. Diastolic BP
C. Mean arterial BP
D. *Pulse pressure
E. None
2747. Which BP profile below identifies the patient with the highest risk for development of cardiovascular
complications?
A. 160/90 mmHg
B. *160/65 mmHg
C. 140/100 mmHg
D. 130/90 mmHg
E. 120/70 mmHg
2748. Physicians should diagnose isolated office hypertension (so-called “white-coat hypertension”)
whenever office BP is ≥140/90 mmHg at several visits, while 24-h ambulatory BP is
A. <160/100 mmHg
B. <140/90 mmHg
C. *<130/85 mmHg
D. <125/80 mmHg
E. <120/70 mmHg
2749. Target organs in hypertension include all of the following EXCEPT
A. Brain and eyes
B. Heart
C. Kidneys
D. Peripheral arteries
E. *Liver
2750. All of the following statements concerning target organ damage are true EXCEPT
A. Microalbuminuria is a sensitive marker of hypertension-induced renal damage
B. *An increase in serum creatinine when antihypertensive therapy is intensified is a sign of progressive
renal deterioration
C. Electrocardiography should be part of all routine assessment of subjects with high BP
D. Echocardiography is much more sensitive than electrocardiography in diagnosing left ventricular
hypertrophy
E. Grades 1 and 2 arteriolar retinal changes do not have a significant prognostic value
2751. The initial antihypertensive medication recommended for patients who have no compelling
indications or contraindications is
A. ACE-inhibitor
B. Calcium-channel blocker
C. Diuretics
D. Beta blocker
E. *Any of the above
2752. A 40-year-old diabetic patient presents with a blood pressure (BP) of 145/90 mmHg and proteinuria.
Which BP profile represents the best therapeutic goal for this patient?
A. <160/90
B. <140/90
C. *<130/85
D. <125/75
E. <140/85
2753. A 40-year-old diabetic patient presents with a blood pressure (BP) of 145/90 mmHg and proteinuria.
What is the best medication for the initial management of this patient’s hypertension?
A. Calcium channel blockers
B. Beta blockers
C. *ACE-inhibitors / angiotensin receptor blockers
D. Alpha blockers
E. Diuretics
2754. In patients with a history of stroke or TIA the preferred drug combination is:
A. *ACE-inhibitor and diuretic
B. Calcium-channel blocker and beta-blocker
C. Beta-blocker and diuretic
D. Angiotensin receptor blocker and beta blocker
E. Beta- and alpha blocker
2755. In the elderly with systolic hypertension antihypertensive therapy should be initiated if SBP
A. ≥120 mmHg
B. ≥130 mmHg
C. *≥140 mmHg
D. ≥160 mmHg
E. ≥170 mmHg
2756. The best medication for the treatment of isolated systolic hypertension in the elderly:
A. ACE-inhibitor and diuretic
B. *Dihydropiridine calcium-channel blocker and diuretic
C. Beta-blocker and diuretic
D. Non-dihydropiridine calcium-channel blocker and diuretic
E. Beta- and alpha blocker
2757. The following medications can be used in pregnancy EXCEPT
A. Methyldopa
B. Labetalol
C. *ACE-inhibitor / ARB
D. Hydralazine
E. Nifidipine
2758. In patients of African descent the drug of choice for initial treatment of hypertension is:
A. Verapamil
B. Atenolol
C. Perindopril
D. *Hydrochlorothiazide
E. Amlodipine
2759. Which of the following antihypertensive agents is INCORRECTLY matched with the indication for
therapy?
A. ACE inhibitor – diabetic nephropathy
B. Beta-blocker – coronary artery disease
C. Calcium channel blocker – angina pectoris
D. Diuretics – heart failure
E. *Loop diuretic – gout
2760. All the following patients should be evaluated for secondary causes of hypertension EXCEPT
A. *A 47-year-old male with strong family history hypertension who presents to your office with a BP of
150/100
B. A 26-year-old female with hematuria and a family history of early renal failure who has a BP of 160/90
C. A 73-year-old male with no past history with a BP of 165/90
D. A 58-year-old male with a history of hypertension since age 45 whose BP has become increasingly
difficult to control on four antihypertensive agents
E. A 31-year-old female with complaints of severe headaches, weight gain, and new-onset DM with a BP
of 145/90
2761. In a patient with suspected aortic coarctation which of the following diagnostic studies is most likely
to demonstrate the cause of the headaches?
A. MRI of the head
B. MRI of the kidney
C. *MRI of the thorax
D. 24-h urinary 5-HIAA
E. 24-h urinary free cortisol
2762. A 27-year-old female has hypertension that is difficult to control. She was diagnosed at age 25. Since
that time she has been on increasing amounts of medication. Her current regimen consists of labetalol 1000
mg bid, lisinopril 40 mg qd, clonidine 0.1 mg bid, and amlodipine 5 mg qd. On physical examination she
appears to be without distress. Her BP is 170/100, and HR is 84 bpm. Cardiac exam is unremarkable,
without rubs, gallops, or murmurs. She has good peripheral pulses and has no edema. Laboratory studies
reveal a potassium of 2.8 mEq/dL and a serum bicarbonate of 32 mEq/dL. Fasting blood glucose 114 mg/dL.
What is the likely diagnosis?
A. Congenital adrenal hyperplasia
B. Fibromuscular dysplasia
C. Cushing’s syndrome
D. *Conn’s syndrome
E. Pheochromocytoma
2763. A 27-year-old female has hypertension that is difficult to control. She was diagnosed at age 25. Since
that time she has been on increasing amounts of medication. Her current regimen consists of labetalol 1000
mg bid, lisinopril 40 mg qd, clonidine 0.1 mg bid, and amlodipine 5 mg qd. On physical examination she
appears to be without distress. Her BP is 170/100, and HR is 84 bpm. Cardiac exam is unremarkable,
without rubs, gallops, or murmurs. She has good peripheral pulses and has no edema. Laboratory studies
reveal a potassium of 2.8 mEq/dL and a serum bicarbonate of 32 mEq/dL. Fasting blood glucose 114 mg/dL.
What is the best way to diagnose this disease ?
A. Renal vein renin levels
B. 24-h urine collection for metanephrines
C. MRI of the renal arteries
D. 24-h urine collection for cortisol
E. *Plasma aldosterone/renin ratio
2764. A 20-year-old female is seen in the ED with symptoms of periodic headaches, sweating, and nausea
with vomiting. She also complains of feeling light-headed with standing. Her BP on presentation is 230/135,
with a HR of 92. On standing, the patient has BP of 205/100, with a HR of 136. On ophthalmologic
examination the patient has mild blurring of the optic discs without hemorrhage. The examination is
otherwise normal. What is the best medication for the management of this patient’s hypertension?
A. *Phentolamine
B. Fenoldopam
C. Esmolol
D. Nicardipine
E. Diazoxide
2765. A 20-year-old female is seen in the ED with symptoms of periodic headaches, sweating, and nausea
with vomiting. She also complains of feeling light-headed with standing. Her BP on presentation is 230/135,
with a HR of 92. On standing, the patient has BP of 205/100, with a HR of 136. On ophthalmologic
examination the patient has mild blurring of the optic discs without hemorrhage. The examination is
otherwise normal. What test would best determine the patient’s diagnosis?
A. Plasma catecholamines
B. 24-h urine collection for 5-HIAA
C. Abdominal CT scan
D. *24-h urine collection for metanephrines and vanilylmandelic acid
E. Adrenal vein sampling for renin levels
2766. What is the correct feature of the hypertensive urgency?
A. It is always a life-threatening situation
B. Patient must be hospitalized
C. *Patients can be managed as outpatients
D. Therapy should not be given orally
E. None of the above
2767. If there is suspicion of phaeochromocytoma, the first line drug is:
A. Sodium-nitroprusside
B. Nifedipine
C. *Phentolamine
D. Verapamil
E. Enalapril
2768. If you are in your office and a patient sees you with the following symptoms: long history of
hypertension, home BP usually 160/100 mmHg, suddenly developed severe headache, blurred vision,
weakness in the right arm, blood pressure: 200/110 mmHg. What will you do:
A. Give the patient 20 mg nifedipine orally, and check the BP in 15 min. If it is still >200/100 mmHg,
give another dose of nifedipine
B. Give the patient 20 mg nifedipine, and check the BP in 30 min. If it is 160/80 mmHg, send the patient
home and give another appointment in two weeks
C. Give the patient 10 mg amlodipine, send the patient home and give another appointment in two days
D. Call the ambulance and send the patient to the nearest hospital
E. *Give the patient some pain-killer drug (e.g. acetaminophen), call the ambulance and send the patient
to the nearest hospital with stroke-unit
2769. A 28-year-old male with type 1 diabetes mellitus is seen in the clinic for routine follow-up. The
patient has hypertension. He is being maintained on an insulin pump and lisinopril 5 mg daily. He has been
on this dose of lisinopril for approximately 6 weeks. He has normal creatinine and no proteinuria. BP is
140/90 mmHg. What do you recommend ?
A. Continue the current dose of lisinopril and return for follow-up in 3 months as the patient has received
his target BP of less than 140/90 mmHg
B. Ask a patient to return in 6 weeks; if his BP remains at this level, increase the dose of lisinopril
C. *Increase lisinopril to 10 mg daily to achieve a goal BP of less than 130/80 mmHg
D. Assure the patient that he is suffering from “white-coat hypertension” and plan no intervention
E. Switch the patient from lisinopril to losartan
2770. A 49-year-old female presents to you for an initial visit to establish health care. She has no past
medical history and denies a family history of diabetes mellitus, early cardiovascular disease, or renal
disease. Her BP on presentation is 185/110 mmHg. She denies headache or visual complaints. She has no
chest pain, shortness of breath, or change in urine color. Her physical examination is normal with the
exception of an S4 gallop heard at the 4th intercostal space just to the left of the sternum. An ECG is
significant for LVH and LAD. there are no signs of cardiac ischemia. Urinalysis shows no red blood cells or
proteinuria. What is your recommendation for therapy for this patient?
A. *Initiate therapy with hydrochlorothiazide in combination with a beta-blocker
B. Initiate therapy with hydrochlorothiazide alone
C. Initiate therapy with an ACE inhibitor alone
D. Advise the patient to go to the nearest emergency department for further treatment for a hypertensive
emergency
E. Advise the patient on lifestyle modifications and plan a return visit in 3 weeks
2771. What will you do if you are on duty in a hospital and a hypertensive patient, who has been treated
with long-acting ß-blocker, long-acting dihydropyridine calcium antagonist and long-acting a-1 adrenoceptor
blocker, is admitted with the following symptoms: headache, dyspnea, ankle-swelling, BP 180/110 mmHg,
pulmonary rales, LVH on ECG, echo: LVH, left ventricular diastolic dysfunction, ejection fraction 46%.
A. Amlodipine orally
B. Nifedipine orally
C. Enalaprilate iv
D. *Enalaprilate iv + furosemide iv
E. Esmolol iv
2772. Which of the following blood pressure levels in pregnancy should be considered an emergency
requiring hospitalization?
A. ≥140/90 mmHg
B. ≥150/95 mmHg
C. ≥160/100 mmHg
D. *≥170/110 mmHg
E. A rise in SBP ≥25 mmHg or a rise in DPB ≥15 mmHg compared to pre-pregnancy values, or those in
the first trimester
2773. The following drug combinations are effective to treat hypertension EXCEPT
A. Diuretics + beta-blockers
B. Beta-blockers + dihydropiridine calcium channel blockers
C. Diuretics + ACE-inhibitors/ARB
D. ACE-inhibitors/ARB + calcium channel blockers
E. *Beta-blockers + ACE-inhibitors/ARB
2774. Each of the following statements regarding hypertension is true EXCEPT:
A. Pure "white coat hypertension is found in 20 to 30% of patients
B. *When measuring BP, an inappropriately small cuff size results in a spuriously low systolic
measurement
C. Pseudohypertension may occur in patients with sclerotic brachial arteries
D. Chronic renal disease is the second most common cause of hypertension after essential hypertension
E. Coarctation of the aorta, Cushing disease, primary aldosteronism account for approximately 1% of all
hypertensive patients
2775. With respect to renovascular disease, all of the statements are true EXCEPT:
A. Worsening renal function with ACE-inhibitor therapy suggests bilateral renovascular disease
B. Atherosclerotic disease most commonly involves the proximal third of the main renal artery
C. The most common form of fibroplastic renovascular disease in adults involves the media
D. *The incidence of renovascular hypertension falls with increased age
E. Patients with severe, accelerated hypertension have the highest prevalence of renovascular disease
2776. A newly diagnosed diabetic patient presents with BP readings that are 155/95 or higher. All of the
following statements about the treatment of this patient's hypertension are correct EXCEPT:
A. Pharmacologic blockade of the renin-angiotensin system reduces the risk of both microvascular and
macrovascular events
B. Aggressive BP control reduces cardiovascular events more in diabetics than in nondiabetics
C. *Calcium channel blockers show no benefit in reducing cardiovascular events
D. The goal BP for this patient is <130/80 mmHg
E. All the above statements are correct
2777. All of the following statements regarding the association of oral contraceptive pills and hypertension
are correct EXCEPT:
A. The likelihood of developing hypertension is increased by significant alcohol consumption
B. The incidence of hypertension is about twice as great in pill users as in non-users
C. The likelihood of developing hypertension is dependent on the age of the user
D. *Hypertension resolves in almost all cases after discontinuation of oral contraceptives
E. The mechanism for contraceptive-induced hypertension likely involves renin-aldosterone-mediated
volume expansion
2778. Each of the following statements regarding pheochromocytoma is true EXCEPT:
A. Approximately 15% of pheochromocytomas are extra-adrenal
B. Ten percent of pheochromocytomas are malignant
C. Ten percent of adrenal pheochromocytomas are bilateral
D. *Hypertension related to pheochromocytoma is nearly always episodic and only rarely persistent
E. Pheochromocytoma can be inherited
2779. All of the following are features of renovascular hypertension due to fibromuscular hyperplasia, as
opposed to atherosclerosis, EXCEPT:
A. Age <50 years
B. Female gender
C. *More likely to progress to complete renal artery occlusion
D. No family history of hypertension
E. Absence of carotid bruits
2780. Which of the following ACE-inhibitors are not prodrugs?
A. Captopril, lisinopril, ramipril
B. Lisinopril, enalapril, banazepril
C. *Captopril, lisinopril, enalaprilat
D. Moexipril, captopril, lisinopril
E. Perindopril, ramipril, enalapril
2781. Each of the following statements regarding hypertension is true EXCEPT:
A. The prevalence of hypertension rises progressively with age in both men and women
B. Systolic and diastolic hypertension are each associated with an increased crisk of coronary heart
disease
C. *The target blood pressure for hypertensive patients with cardiovascular disease or diabetes is 140/90
mmHg
D. Patients with prehypertension (SBP 120-139 mmHg or DBP 80-89 mmHg) benefit from lifestyle
modifications incl. weight reduction, regular exercise, and smoking cessation
E. Drug therapy of hypertension benefits patients greater than 80 years of age
2782. All of the following interventions have a BP-lowering effect EXCEPT:
A. A diet that reduces caloric intake by 1000 calories per day
B. Reduction of dietary sodium
C. *Daily magnesium supplements
D. Tobacco cessation
E. Reduction of ethanol consumption to less than 30 ml/day
2783. Each of the following statements regarding hypertension is true EXCEPT:
A. Pure "white coat hypertension is found in 20 to 30% of patients
B. When measuring BP, an inappropriately small cuff size results in a spuriously low systolic
measurement
C. Pseudohypertension may occur in patients with sclerotic brachial arteries
D. Chronic renal disease is the second most common cause of hypertension after essential hypertension
E. Coarctation of the aorta, Cushing disease, primary aldosteronism account for approximately 1% of all
hypertensive patients
2784. Each of the following statements regarding hypertension is true EXCEPT:
A. Increased LV muscle mass in hypertension is an independent risk factor of cardiac mortality
B. The risk of ventricular arrhythmias in hypertension is increased 2-fold in the presence of LV
hypertrophy
C. Low birth weight is associated with an increased incidence of hypertension later in life
D. *Chronic caffeine consumption is associated with an increased risk of hypertension
E. In hypertensive adolescents, essential hypertension is the most likely etiology
2785. The following statements regarding the association of oral contraceptive pills and hypertension are
correct EXCEPT:
A. The likelihood of developing hypertension is increased by significant alcohol consumption
B. The incidence of hypertension is about twice as great in pill users as in non-users
C. The likelihood of developing hypertension is dependent on the age of the user
D. *Hypertension resolves in almost all cases after discontinuation of oral contraceptives
E. The mechanism for contraceptive-induced hypertension likely involves renin-aldosterone-mediated
volume expansion
2786. Thiazide diuretics can contribute to each of the following metabolic effects EXCEPT:
A. Hypomagnesemia
B. *Hypouricemia
C. Hypercalcemia
D. Hypercholesterolemia
E. Hyponatremia
2787. Which of the following statements regarding prevention of stroke is correct?
A. Treatment of systolic hypertension does not decrease the risk of stroke in patients older than age 60
B. Hypertension should not be a target of secondary prevention after an ischemic stroke because elevated
BP is desirable to maintain adequate cerebral perfusion
C. *Treatment with HMG-CoA reductase inhibitors reduces the risk of recurrent stroke
D. The combination of aspirin and clopidogrel is superior to aspirin alone for prevention of recurrent
stroke
E. None of the above
2788. Which of the following antihypertensive agents is a known cause of autoimmune hemolytic anemia?
A. Metoprolol
B. *Methyldopa
C. Captopril
D. Losartan
E. Monoxidine
2789. All of the following characteristics are typical of hypertensive crisis EXCEPT:
A. Diastolic BP >120 mmHg
B. Retinal hemorrhages
C. *Constriction of cerebral arterioles with decreased vascular permeability
D. Proteinuria
E. Microangiopathic hemolytic anemia
2790. Which of the following pairs of medical conditions and antihypertensive medications would be
INCORRECT to use in a patient with essential hypertension?
A. Beta-blocker and a history of myocardial infarction
B. Alpha-blocker and prostatic hypertrophy
C. *Thiazide diuretic and gout
D. Amlodipine and heart failure
E. ACE-inhibitor and diabetes mellitus
2791. Which of the following statements regarding antihypertensive agents and atrial fibrillation (AF) is
TRUE?
A. *Losartan has been shown to decrease new-onset AF more effectively than atenolol
B. Valsartan has been shown to decrease new-onset AF more effectively than amlodipine
C. Atenolol has been shown to decrease new-onset AF more effectively than captopril
D. Nifedipine has been shown to decrease new-onset AF more effectively than diltiazem
E. All of the above are false
2792. Which lifestyle change has been shown to produce the biggest reduction in systolic blood pressure?
A. *10-kg weight loss
B. Dietary sodium reduction
C. Moderation of alcohol consumption
D. Change to a vegetarian diet
E. Magnesium supplement
2793. Which of these statements is true regarding renin-angiotensin system-blocking agents?
A. Less effective when combined with a diuretic than when used alone
B. More effective in patients of African descent than in white patients
C. *Preserve kidney function in addition to lowering blood pressure
D. Are the first-line antihypertensive medications in pregnancy
E. All of the above
2794. Which of the following antihypertensive agents is INCORRECTLY matched with the indication for
therapy?
A. ACE inhibitor – diabetic nephropathy
B. Beta-blocker – coronary artery disease
C. Calcium channel blocker – angina pectoris
D. *Hydrochlorthiazide – diabetes mellitus
E. Loop diuretic – heart failure
2795. Which of the following statements about microalbuminuria is TRUE?
A. To be of clinixal value, microalbuminuria must be measured in a timed 12- to 24-hour sample
B. *Microalbuminuria is a cardiovascular risk factor that is independent of traditional Framingham risk
factors
C. Microalbuminuria is a predictor of risk only in patients with diabetes
D. Microalbuminuria is present when the “spot” urine albumin-to-creatinine ration is >500 mg/g
E. All of the above statements are correct
2796. Renin-dependent hypertension includes:
A. Primary hyperaldosteronism
B. Essential hypertension
C. *Renovascular hypertension
D. Pheochromocytoma
E. Cushing syndrome
2797. Fundus picture of stage 2 of Keith-Wegener classification includes
A. Copper wire AV nipping and focal spasm
B. *Silver wire increased reflex
C. Exudate and hemorrhagic spots
D. Papilloedema
E. Retinal hemorrhages
2798. A young hypertensive patient has serum potassium 2.8 mEq/l and increased aldosterone level with
decreased plasma renin activity. The likely diagnosis is....
A. Renal artery stenosis
B. Ectopic ACTH syndrome
C. *Conn syndrome
D. Liddle syndrome
E. Cushing syndrome
2799. The proportion of essential (primary) hypertension among all hypertension causes is as high as
A. 25-30%
B. 40-45%
C. 60-65%
D. 70-75%
E. *90-95%
2800. All of the following are risk factors for hypertension, EXCEPT
A. Increased body weight
B. Family history of hypertension
C. Excessive intake of sodium
D. *Regular use of one glass of wine per day
E. Cigarette smoking
2801. All of the following statements concerning hypertension are true, EXCEPT
A. In a western adult population the prevalence of hypertension exceeds 20%
B. Hypertension is a major risk factor for cardiovascular and cerebrovascular disease
C. The pathophysiology of hypertension differs in black adults compared to South Asians and whites
D. People of African descent commonly have a high renin type hypertension
E. *An increase in potassium intake may significantly reduce blood pressure in hypertensive patients
2802. Optimal blood pressure (BP) is defined as a BP level of
A. *<120/80 mmHg
B. <130/80 mmHg
C. <140/90 mmHg
D. <160/100 mmHg
E. <125/80 mmHg
2803. Which parameter of arterial blood pressure (BP) yields the best prognostic information in patients at
risk for cardiovascular disease?
A. Systolic BP
B. Diastolic BP
C. Mean arterial BP
D. *Pulse pressure
E. None
2804. Which BP profile below identifies the patient with the highest risk for development of cardiovascular
complications?
A. 160/90 mmHg
B. *160/65 mmHg
C. 140/100 mmHg
D. 130/90 mmHg
E. 120/70 mmHg
2805. Physicians should diagnose isolated office hypertension (so-called “white-coat hypertension”)
whenever office BP is ≥140/90 mmHg at several visits, while 24-h ambulatory BP is
A. <160/100 mmHg
B. <140/90 mmHg
C. *<130/85 mmHg
D. <125/80 mmHg
E. <120/70 mmHg
2806. Target organs in hypertension include all of the following EXCEPT
A. Brain and eyes
B. Heart
C. Kidneys
D. Peripheral arteries
E. *Liver
2807. All of the following statements concerning target organ damage are true EXCEPT
A. Microalbuminuria is a sensitive marker of hypertension-induced renal damage
B. *An increase in serum creatinine when antihypertensive therapy is intensified is a sign of progressive
renal deterioration
C. Electrocardiography should be part of all routine assessment of subjects with high BP
D. Echocardiography is much more sensitive than electrocardiography in diagnosing left ventricular
hypertrophy
E. Grades 1 and 2 arteriolar retinal changes do not have a significant prognostic value
2808. At a routine examination, an asymptomatic 46-year-old man is found to have a BP of 150/110
mmHg, but no other abnormalities are present. What do you do next?
A. Reassure the patient and repeat the physical examination in 12 months
B. Initiate antihypertensive therapy
C. *Obtain repeated BP recordings in your office and/or the patient’s home or work site
D. Hospitalize patient for renal arteriography
E. Order a 24-h ambulatory BP monitoring
2809. The initial antihypertensive medication recommended for patients who have no compelling
indications or contraindications is
A. ACE-inhibitor
B. Calcium-channel blocker
C. Diuretics
D. Beta blocker
E. *Any of the above
2810. In patients with a history of stroke or TIA the preferred drug combination is:
A. *ACE-inhibitor and diuretic
B. Calcium-channel blocker and beta-blocker
C. Beta-blocker and diuretic
D. Angiotensin receptor blocker and beta blocker
E. Beta- and alpha blocker
2811. In the elderly with systolic hypertension antihypertensive therapy should be initiated if SBP
A. ≥120 mmHg
B. ≥130 mmHg
C. *≥140 mmHg
D. ≥160 mmHg
E. ≥170 mmHg
2812. The best medication for the treatment of isolated systolic hypertension in the elderly:
A. ACE-inhibitor and diuretic
B. *Dihydropiridine calcium-channel blocker and diuretic
C. Beta-blocker and diuretic
D. Non-dihydropiridine calcium-channel blocker and diuretic
E. Beta- and alpha blocker
2813. The following medications can be used in pregnancy EXCEPT
A. Methyldopa
B. Labetalol
C. *ACE-inhibitor / ARB
D. Hydralazine
E. Nifidipine
2814. In patients of African descent the drug of choice for initial treatment of hypertension is:
A. Verapamil
B. Atenolol
C. Perindopril
D. *Hydrochlorothiazide
E. Amlodipine
2815. Which of the following antihypertensive agents is INCORRECTLY matched with the indication for
therapy?
A. ACE inhibitor – diabetic nephropathy
B. Beta-blocker – coronary artery disease
C. Calcium channel blocker – angina pectoris
D. Diuretics – heart failure
E. *Loop diuretic – gout
2816. All the following patients should be evaluated for secondary causes of hypertension EXCEPT
A. *A 47-year-old male with strong family history hypertension who presents to your office with a BP of
150/100
B. A 26-year-old female with hematuria and a family history of early renal failure who has a BP of 160/90
C. A 73-year-old male with no past history with a BP of 165/90
D. A 58-year-old male with a history of hypertension since age 45 whose BP has become increasingly
difficult to control on four antihypertensive agents
E. A 31-year-old female with complaints of severe headaches, weight gain, and new-onset DM with a BP
of 145/90
2817. Which of the following is an effective adjunct to increase the rate of smoking cessation?
A. Aldosterone
B. *Bupropion
C. Sildenafil
D. Cimetidine
E. Buspirone
2818. With which of the following is sildenafil acetate (Viagra) most likely to interact adversely?
A. Fibrinolytic therapy
B. Primary PCI
C. *Nitrates
D. Aspirin
E. Beta-blockers
2819. Bupropion hydrochloride (Zyban) is contraindicated in patients with a history of which of the
following?
A. *Seizures
B. Insulin-dependent diabetes mellitus
C. Severe chronic obstructive pulmonary disease (with reversible component)
D. Longer than 40-year history of tobacco use
E. Recent MI
2820. In which of the following has the high-sensitivity C-reactive protein (CRP) been shown to be
predictive of risk?
A. Acute MI
B. Acute coronary syndromes
C. Chronic stable angina
D. Peripheral vascular disease
E. *All of the above
2821. Which of the following has been shown to decrease the level of the high-sensitivity CRP?
A. Unopposed estrogen
B. Amlodipine besylate (Norvasc)
C. *Simvastatin
D. All of the above
E. None of the above
2822. All of the following statements regarding myocardial stunning are true EXCEPT:
A. *Stunning is a state of depressed myocardial function due to chronic hypoperfusion
B. Stunning can be global or regional
C. Stunning can follow cardiac surgery with cardiopulmonary bypass
D. Oxygen-free radicals and excess intracellular calcium likely contribute to stunning
E. Stunning affects both systolic and diastolic function
2823. What is the most common coexisting congenital anomaly in patients with coarctation of the aorta?
A. Cleft mitral valve
B. *Bicuspid aortic valve
C. Ebstein's anomaly
D. VSD
E. PDA
2824. All of the following are characteristic findings of ostium primum atrial septal defect (ASD)
EXCEPT:
A. Precordial heave
B. Fixed split S2
C. *Right axis deviation
D. Systolic ejection murmur
E. Prominent pulmonary vascular markings on CXR
2825. Choose the adult congenital disorder corresponding to the following physical examination fingings:
RV lift with a loud systolic ejection murmur along the left sternal border, with a single S2
A. Eisenmenger's syndrome
B. Coarctation of the aorta
C. PDA
D. Ebstein's anomaly
E. *Tetralogy of Fallot
2826. Choose the adult congenital disorder corresponding to the following physical examination fingings:
loud S1, holosystolic murmur in left sternal border, systolic ejection click, and hepatomegaly
A. Eisenmenger's syndrome
B. Coarctation of the aorta
C. PDA
D. *Ebstein's anomaly
E. Tetralogy of Fallot
2827. Choose the adult congenital disorder corresponding to the following physical examination fingings:
weak or delayed femoral pulses, harsh systolic murmur in the back, and a systolic ejection click in the aortic
area
A. Eisenmenger's syndrome
B. *Coarctation of the aorta
C. PDA
D. Ebstein's anomaly
E. Tetralogy of Fallot
2828. Choose the adult congenital disorder corresponding to the following physical examination fingings:
cyanosis, digital clubbing, loud P2, and a variable Graham-Steel murmur
A. *Eisenmenger's syndrome
B. Coarctation of the aorta
C. PDA
D. Ebstein's anomaly
E. Tetralogy of Fallot
2829. Choose the adult congenital disorder corresponding to the following physical examination fingings:
wide pulse pressure, prominent LV impulse, and a continuous machinery murmur enveloping S2
A. Eisenmenger's syndrome
B. Coarctation of the aorta
C. *Patent ductus arteriosus
D. Ebstein's anomaly
E. Tetralogy of Fallot
2830. Bacterial endocarditis prophylaxis is indicated in all adults who have the following congenital heart
disease EXCEPT:
A. VSD
B. Coarctation of the aorta
C. *Secundum ASD
D. Hypertrophic obstructive cardiomyopathy
E. PDA
2831. Besides pulmonary valve stenosis, which of the following is the most common associated cardiac
defect present in patients with PA stenosis?
A. *VSD
B. ASD
C. Coarctation of the aorta
D. PDA
E. Tetralogy of Fallot
Situational tasks
1.
The ovaries of a 55-year-old woman are producing only minimal amounts of estrogen. What effect
should the physician expect this decreased estrogen secretion to have on hypothalamic production of
gonadotropin-releasing hormone (Gn-RH) and anterior pituitary production of follicle-stimulating hormone
(FSH)?
A.
*Increased Gn-RH, increased FSH
B.
Increased Gn-RH, decreased FSH
C.
Decreased Gn-RH, increased FSH
D.
Decreased Gn-RH, decreased FSH
E.
Non of above
2.
What symptoms or problems should the physician expect in a patient who is receiving a treatment
that has a side effect of increasing the synthesis and release of aldosterone?
A.
Hypertension, hyperkalemia
B.
*Hypertension, hypokalemia
C.
Hypotension, hyperkalemia
D.
Hypotension, hypokalemia
E.
Non of above
3.
The patient has been taking an oral cortisol preparation for 2 years to manage an autoimmune disease.
What effects does the physician expect this therapy to have on this patient's circulating levels of ACTH and
aldosterone?
A.
Increased ACTH, decreased aldosterone
B.
Increased ACTH, increased aldosterone
C.
Decreased ACTH, increased aldosterone
D.
*Decreased ACTH, decreased aldosterone
E.
Non of above
4.
What would be the expected clinical manifestation for a patient who has excessive production of
melanocyte-stimulating hormone?
A.
Hypoglycemia and hyperkalemia
B.
Irritability and insomnia
C.
Increased urine output
D.
*Darkening of the skin
E.
Non of above
5.
Which endocrine gland function is most important to assess in the 45-year-old woman who has
bilateral patchy areas of skin depigmentation on her arms and face?
A.
*Adrenal gland
B.
Thyroid gland
C.
Pancreas
D.
Ovary
E.
Non of above
6.
If a patient has a low dietary intake of iodine (iodide), which hormone would be most profoundly
affected and how would it be affected?
A.
Deficiency of parathyroid hormone
B.
Excess of parathyroid hormone
C.
*Deficiency of thyroid hormones
D.
Excess of thyroid hormones
E.
Non of above
7.
A 35-year-old female patient has been diagnosed with a deficiency of most anterior pituitary
hormones. Which fact reported in her history should the physician explore further with regard to her pituitary
problem?
A.
The patient’s mother and sister have adult-onset diabetes mellitus.
B.
*The patient experienced a postpartum hemorrhage 5 years ago.
C.
The patient has a severe allergy to shellfish and iodine.
D.
The patient has used oral contraceptives for 5 years.
E.
Non of above
8.
The patient thought to have a problem with the pituitary gland is given 25 units of regular insulin. A
short time later, blood analysis reveals elevated levels of growth hormone and adrenocorticotropic hormone
(ACTH). What is the physician’s interpretation of this finding?
A.
The patient has pituitary hyperfunction.
B.
The patient has pituitary hypofunction.
C.
The patient has pituitary-induced diabetes mellitus.
D.
*The patient has a normal pituitary response to insulin.
E.
Non of above
9.
Which clinical manifestations alert the physician to the possibility of anterior pituitary
hyperfunction?
A.
*Enlarged hands and feet, heat intolerance
B.
Non of above
C.
Bradycardia, hypotension, and somnolence
D.
Chronic constipation and darkening of the skin
E.
Hyponatremia, hyperkalemia, and hypercalcemia
10.
The patient, a 45-year-old woman, has acromegaly as a result of a pituitary adenoma found and
removed when she was 16 years old. During a physical assessment before surgery for a knee replacement,
the physician discovers that she has a moderately enlarged liver. What is the physician’s best action?
A.
Counsel the patient on the health risks of alcoholism.
B.
Determine whether the patient is jaundiced.
C.
*Document the finding as the only action.
D.
Request liver function blood tests.
E.
Non of above
11.
What is the priority nursing diagnosis in a 35-year-old man being treated for hyperpituitarism in
which excessive amounts of prolactin are secreted?
A.
Risk for Injury related to seizure activity
B.
*Disturbed body image related to gynecomastia
C.
Constipation related to anorexia and decreased metabolism
D.
Decreased cardiac output related to electrolyte imbalances
E.
Non of above
12.
Which patient responses demonstrate to the physician that treatment for diabetes insipidus is
effective?
A.
Urine output is increased; specific gravity is increased.
B.
Urine output is increased; specific gravity is decreased.
C.
*Urine output is decreased; specific gravity is increased.
D.
Urine output is decreased; specific gravity is decreased.
E.
Non of above
13.
Which of the following clinical manifestations alerts the physician to the possibility of side effects of
desmopressin acetate (DDAVP) therapy, taken nonparenterally by a patient with diabetes insipidus?
A.
Fibrosis at the injection site
B.
Orthostatic hypotension
C.
Decreased urine output
D.
*Nasal ulceration
E.
Non of above
14.
Which patient is at greatest risk for the development of the syndrome of inappropriate antidiuretic
hormone secretion (SIADH)?
A.
48-year-old man with an acute myocardial infarction
B.
38-year-old woman taking oral contraceptives
C.
68-year-old woman with diabetes mellitus
D.
*68-year-old man with chronic emphysema
E.
Non of above
15.
Which medication should the physician be prepared to administer to a patient with the syndrome of
inappropriate antidiuretic hormone (SIADH) secretion?
A.
Morphine
B.
*Demeclocycline
C.
Dextrose 5% in water
D.
Non of above
E.
Tricyclic antidepressants
16.
What dietary alterations should the physician make for the patient with Cushing's disease?
A.
High protein, high carbohydrate, low potassium
B.
*Low carbohydrate, calories to maintain BMI < 25, low sodium
C.
Low protein, high carbohydrate, low calcium
D.
High carbohydrate, low potassium, fluid restriction
E.
Non of above
17.
The patient is being admitted with acute adrenal insufficiency (addisonian crisis). What medication
should the physician prepared to administer?
A.
Sodium bicarbonate
B.
Pancuronium bromide
C.
*Hydrocortisone sodium succinate
D.
Insulin 20 units and dextrose 20%
E.
Non of above
18.
Which clinical manifestation indicates to the physician that the patient's adrenocortical insufficiency
is of primary origin rather than secondary origin?
A.
Red, beefy tongue
B.
Weight loss
C.
Orthostatic hypotension
D.
*Increased skin pigmentation
E.
Non of above
19.
Which drug(s) would the physician expect to administer to a patient to prevent gastrointestinal
ulceration from hypercortisolism?
A.
*Omeprazole (Prilosec)
B.
Metoclopramide (Reglan)
C.
Clarithromycin (Biaxin)
D.
Vitamins and iron
E.
Non of above
20.
The patient with hyperaldosteronism is being treated with spironolactone therapy. What precautions
should the physician teach this patient?
A.
*“Avoid salt substitutes.”
B.
“Avoid adding salt to food.”
C.
“Avoid excessive exposure to sunlight.”
D.
“Avoid acetaminophen and acetaminophen-containing products.”
E.
Non of above
21.
Which clinical manifestation change indicates to the physician that the therapy for the patient with
hyperaldosteronism is effective?
A.
The urine output has decreased from 25 mL/hr to 15 mL/hr.
B.
The serum calcium level (total) has increased from 8.6 to 9.0 mg/dL.
C.
*The systolic blood pressure has decreased by 24 mm Hg.
D.
The fasting blood glucose level is 86 mg/dL.
E.
Non of above
22.
Which assessment maneuver should the physician avoid performing with a patient suspected of
having a pheochromocytoma?
A.
Having the patient attempt to touch the chin to the chest
B.
Attempting to dorsiflex the feet
C.
Inflating the blood pressure cuff above 200 mm Hg
D.
*Palpating the abdomen
E.
Non of above
23.
Which clinical manifestation alerts the physician to the possibility of Graves’ disease as the cause of
hyperthyroidism?
A.
Weight loss
B.
*Menstrual irregularities
C.
Increased heart rate and blood pressure
D.
Non of above
E.
All of above
24.
The patient with hyperthyroid symptoms is having hormone studies done to confirm the diagnosis.
Which set of values indicates non–Graves’ disease hyperthyroidism?
A.
*Elevated T3, elevated T4, high TSH levels
B.
Elevated T3, normal T4, low TSH levels
C.
Elevated T3, low T4, low TSH levels
D.
Low T3, normal T4, high TSH levels
E.
Non of above
25.
In collaboration with the dietician, what dietary modification should the physician suggest for the
patient with hyperthyroidism?
A.
Decrease calories and proteins and increase carbohydrates.
B.
Eliminate carbohydrates and increase proteins and fats.
C.
*Increase calories, proteins, and carbohydrates.
D.
No dietary modification is needed.
E.
Non of above
26.
For which patient with hyperthyroidism is radioactive iodine therapy contraindicated?
A.
18-year-old man with asthma
B.
*28-year-old woman who is pregnant
C.
48-year-old man with type 2 diabetes mellitus
D.
68-year-old woman with mild congestive heart failure
E.
Non of above
27.
The patient with hyperthyroidism is taking lithium carbonate to inhibit thyroid hormone release.
Which of the following patient reports alerts the physician to side effects of this therapy?
A.
Blurred vision
B.
*Increased thirst and urination
C.
Increased sweating and diarrhea
D.
Decreased attention span and insomnia
E.
Non of above
28.
Which medication should the physician be prepared to administer to a patient with bradycardia as a
result of hypothyroidism?
A.
Atropine sulfate
B.
*Levothyroxine sodium
C.
Propranolol
D.
Epinephrine
E.
Non of above
29.
Which clinical manifestation indicates to the physician that treatment for the patient with
hypothyroidism is effective?
A.
The patient is thirsty.
B.
The patient’s weight has been the same for 3 weeks.
C.
The patient’s total white blood cell count is 6000 cells/mm3.
D.
*The patient has had a bowel movement every day for 1 week.
E.
Non of above
30.
Which patient is at greatest risk for hyperparathyroidism?
A.
28-year-old patient with pregnancy-induced hypertension
B.
*45-year-old patient receiving dialysis for end-stage renal disease
C.
55-year-old patient with moderate congestive heart failure after myocardial infarction
D.
60-year-old patient on home oxygen therapy for chronic obstructive pulmonary disease
E.
Non of above
31.
The diabetic patient asks the physician why it is necessary to maintain blood glucose levels no lower
than about 74 mg/dL. What is the physician’s best response?
A.
“Glucose is the only fuel form used by body cells to produce energy needed for physiologic activity.”
B.
*“The central nervous system, which cannot store glucose, requires a continuous supply of glucose
for fuel.”
C.
“Without a minimum level of glucose circulating in the blood, erythrocytes cannot produce ATP.”
D.
“The presence of glucose in the blood counteracts the formation of lactic acid and prevents acidosis.”
E.
Non of above
32.
Which assessment finding in the patient with diabetes mellitus indicates that the disease is damaging
the kidneys?
A.
The presence of ketone bodies in the urine during acidosis
B.
The presence of glucose in the urine during hyperglycemia
C.
*The presence of protein in the urine during a random urinalysis
D.
The presence of white blood cells in the urine during a random urinalysis
E.
Non of above
33.
The 30-year-old woman whose father has type 1 diabetes mellitus asks the physician what her
chances are of developing diabetes because of her father's disease. What is the physician’s best response?
A.
*“You have a greater susceptibility for developing the disease, with a 1 in 20 to a 1 in 50 chance.”
B.
Non of above
C.
“Your risk is the same as the general population, because there is no genetic risk for development of
type 1 diabetes.”
D.
“Type 1 diabetes is inherited in an autosomal dominant pattern. Therefore, the risk for becoming
diabetic is 50%.”
E.
“Because you are a woman and your father is the parent with the diabetes, your risk is not increased
for eventual development of the disease; however, your brothers will become diabetic.”
34.
The 30-year-old woman whose mother has type 2 diabetes mellitus asks the physician what her
chances are of developing diabetes because of her mother's disease. What is the physician’s best response?
A.
“You have a greater susceptibility for developing the disease, with a 1 in 20 to a 1 in 50 chance.”
B.
“Your risk is the same as the general population, because there is no genetic risk for development of
type 2 diabetes.”
C.
“Type 2 diabetes is inherited in an autosomal dominant pattern. Therefore, your risk for becoming
diabetic is 50%.”
D.
*“Children of people with type 2 diabetes have a 15% chance of developing the disease, but
environmental factors, such as obesity, also influence your risk.”
E.
Non of above
35.
With which patient should the physician be alert for undiagnosed diabetes mellitus?
A.
25-year-old white male
B.
45-year-old African American man
C.
25-year-old African American woman
D.
*45-year-old Native American woman
E.
Non of above
36.
The patient with type 2 diabetes had been taking the oral antidiabetic agents glyburide and
metformin. These medications have been discontinued and he has now been prescribed to take Glucovance.
He asks why he only needs one medication. What is the physician’s best response?
A.
“Glucovance is more effective than glyburide and metformin.”
B.
*“Glucovance contains a combination of glyburide and metformin.”
C.
“Glucovance is a new oral insulin and replaces all other oral antidiabetic agents.”
D.
“Your diabetes is improving and you now only need one drug for blood glucose control.”
E.
Non of above
37.
The patient with type 1 diabetes mellitus is switching from an animal-source regular insulin to a
synthetically derived human regular insulin. Which precaution should the physician explain to this patient?
A.
“Human insulin should only be administered in the umbilical area.”
B.
“Do not mix human regular insulin with any other type of insulin.”
C.
*“Adjustments in insulin timing may be needed with the human regular insulin.”
D.
“You may notice the need to increase the dose to achieve the same level of glucose control.”
E.
Non of above
38.
The patient with diabetes who is just starting on insulin therapy wants to know why more than one
injection of insulin each day will be required. What is the physician’s best response?
A.
“You need to start with multiple injections until you become more proficient at self-injection.”
B.
Non of above
C.
*“A single dose of insulin each day would not match your blood insulin levels and your food intake
patterns closely enough.”
D.
“A regimen of a single dose of insulin injected each day would require that you could eat no more
than one meal each day.”
E.
“A single dose of insulin would be too large to be absorbed predictably, so you would be in danger of
unexpected insulin shock.”
39.
The patient on an intensified insulin regimen consistently has a fasting blood glucose between 70 and
80 mg/dL, a postprandial blood glucose level below 200 mg/dL, and a hemoglobin A1c level of 5.5%. What
is the physician’s interpretation of these findings?
A.
The patient is at increased risk for developing hypoglycemia.
B.
The patient is at increased risk for developing hyperglycemia.
C.
The patient is demonstrating signs of insulin resistance.
D.
*The patient is demonstrating good control of blood glucose.
E.
Non of above
40.
Which action should the physician suggest to reduce insulin needs in the patient with diabetes
mellitus?
A.
Reducing intake of water and other liquids to no more than 2 L/day
B.
Eating animal organ meats high in insulin
C.
Taking two 1-hour naps daily
D.
*Walking 1 mile each day
E.
Non of above
41.
The patient newly diagnosed with type 2 diabetes tells the physician that since he has increased his
intake of fiber, he is having loose stools, flatulence, and abdominal cramping. What is the physician’s best
response?
A.
“Decrease your intake of water and other fluids.”
B.
*“Decrease your intake of fiber now and gradually add high-fiber foods back into your diet.”
C.
“You must have allergies to high-fiber foods and will need to avoid them in the future.”
D.
“Taking an antacid 1 hour before meals or 2 hours after meals should reduce the intensity of your
bowel problems.”
E.
Non of above
42.
The patient getting ready to engage in a 30-minute, moderate-intensity exercise program performs a
self-assessment. Which data indicate that exercise should be avoided at this time?
A.
*Ketone bodies in the urine
B.
Blood sugar level of 155 mg/dL
C.
Pulse rate of 66 beats/min
D.
Weight 1 pound higher than the week before
E.
Non of above
43.
Three hours after surgery, the physician note that the breath of the patient who is a type 1 diabetic has
a “fruity” odor. What is the physician’s best first action?
A.
Document the finding as the only action.
B.
Increase the IV fluid flow rate.
C.
*Test the urine for ketone bodies.
D.
Perform oral care.
E.
Non of above
44.
The patient with type 1 diabetes has a blood glucose level of 160 mg/dL on arrival at the operating
room. What is the physician’s best action?
A.
*Document the finding as the only action.
B.
Administer regular insulin.
C.
Cancel the surgery.
D.
Notify the physician.
E.
Non of above
45.
Which change in clinical manifestations in a patient with long-standing diabetes mellitus alerts the
physician to the possibility of renal dysfunction?
A.
The presence of ketone bodies in the urine
B.
Loss of tactile perception
C.
The presence of glucose in the urine
D.
*A sustained increase in blood pressure from 130/84 to 150/100
E.
Non of above
46.
For the diabetic patient with microalbuminuria, what dietary modification should the physician
suggest?
A.
Decreased percentage of total calories derived from carbohydrates
B.
*Decreased percentage of total calories derived from proteins
C.
Decreased percentage of total calories derived from fats
D.
Decreased total caloric intake
E.
Non of above
47.
Which statement made by the diabetic patient who has a urinary tract infection indicates correct
understanding regarding antibiotic therapy?
A.
“If my temperature is normal for 3 days in a row, the infection is gone and I can stop taking my
medicine.”
B.
Non of above
C.
“If my temperature goes above 100° F (37.8° C) for 2 days, I should take twice as much medicine.”
D.
*“Even if I feel completely well, I should take the medication until it is gone.”
E.
“When my urine no longer burns, I will no longer need to take the antibiotics.”
48.
The physician is rapidly infusing insulin to a patient with extreme hyperglycemia. Which electrolyte
abnormality indicates that the infusion is too rapid?
A.
Serum chloride level of 90 mmol/L
B.
Serum calcium level of 8.0 mg/dL
C.
Serum sodium level of 132 mmol/L
D.
*Serum potassium level of 2.5 mmol/L
E.
Non of above
49.
Which clinical manifestation indicates to the physician that the therapy for the patient with
hyperglycemic, hyperosmolar, nonketotic syndrome (HHNS) needs to be adjusted?
A.
The patient's serum potassium level increased from 2.8 mEq/L to 3.2 mEq/L.
B.
The patient's blood osmolarity has decreased from 350 mOsm to 330 mOsm.
C.
*The patient's score on the Glasgow Coma Scale is unchanged from 3 hours ago.
D.
The patient's urine has remained negative for ketone bodies for the past 3 hours.
E.
Non of above
50.
The physician is caring for a patient receiving total parenteral nutrition (TPN) at 125 mL/hr. The
patient complains of excessive urination. The patient’s finger stick reveals an elevated blood sugar level. The
physician realizes the patient is at risk for which of the following?
A.
Rebound hypoglycemia
B.
*Hypovolemic shock
C.
Hyperkalemia
D.
Hypernatremia
E.
None of above
51.
A patient is being treated for obesity with orlistat (Xenical). What statement made by the patient
indicates an understanding of the medication regimen?
A.
“This medication will make me feel full.”
B.
*“I may have loose stools with this medication.”
C.
“This medication will increase my metabolic rate.”
D.
“This medication will turn my urine a bright yellow color.”
E.
None of above
52.
What dietary discharge instructions should be given to the patient who has undergone gastroplasty for
treatment obesity?
A.
*“Your diet will be limited to liquids or pureed foods for 6 weeks.”
B.
“You will be placed on a low-protein diet for the first 2 months after surgery.”
C.
“You will be gradually progressed to eating three meals per day during your hospitalization.”
D.
“You will need to continue taking the anorectic drug prescribed for you until your stomach shrinks.”
E.
None of above
53.
The ovaries of a 55-year-old woman are producing only minimal amounts of estrogen. What effect
should the physician expect this decreased estrogen secretion to have on hypothalamic and anterior pituitary
production?
A.
Increased Gn-RH
B.
Increased LH
C.
Increased FSH
D.
*All of them
E.
None of above
54.
Which tissue(s) or organ(s) should be evaluated in a man who begins to have fluid secretion from the
breast?
A.
Posterior pituitary and testes
B.
Adrenal medulla and adrenal cortex
C.
*Hypothalamus and anterior pituitary
D.
Parathyroid and islets of Langerhans
E.
None of above
55.
What symptoms or problems should the physician expect in a patient who is receiving a treatment
that has a side effect of increasing the synthesis and release of aldosterone?
A.
Hypertension
B.
Hypokalemia
C.
Hypernatremia
D.
*All of them
E.
None of above
56.
The patient who is taking corticosteroids daily for severe asthma now has an elevated blood glucose
level. He asks the physician if he is now considered diabetic. What is the physician’s best response?
A.
“Yes, the corticosteroids have destroyed the ability of the pancreas to synthesize insulin.”
B.
“Yes, whenever blood glucose levels are abnormally high, the condition is called diabetes.”
C.
*“No, the blood glucose level is elevated because corticosteroids increase the synthesis of glucose.”
D.
None of above
E.
“No, the lack of insulin is temporary and will return to a normal level when the asthma is better.”
57.
The patient has been taking an oral cortisol preparation for 2 years to manage an autoimmune disease.
What side effects does the physician expect of this therapy?
A.
Headache
B.
Weight gain
C.
Nervousness
D.
*All of them
E.
None of above
58.
Which hormone level is most important to assess in the 45-year-old woman who has bilateral patchy
areas of skin depigmentation on her arms and face?
A.
*Cortisol
B.
Thyroxine
C.
Elastase
D.
Estradiol
E.
None of above
59.
The patient has a deficiency of all the following pituitary hormones. Which one should be addressed
first?
A.
Growth hormone
B.
Luteinizing hormone
C.
*Thyroid-stimulating hormone
D.
None of above
E.
Follicle-stimulating hormone
60.
A 35-year-old female patient has been diagnosed with a deficiency of most anterior pituitary
hormones. Which of next hormones deficiency is due to her pituitary problem?
A.
*TSH.
B.
ADH.
C.
Cortisol.
D.
TRH.
E.
None of above
61.
The male patient with hypopituitarism asks the physician how long he will have to take testosterone
hormone replacement therapy. What is the physician’s best answer?
A.
None of above
B.
“When your blood levels of testosterone are normal, the therapy is no longer needed.”
C.
*“When your beard thickens and your voice deepens, the dose is decreased but must continue
forever.”
D.
“When your sperm count is high enough to demonstrate fertility, you will no longer need this
therapy.”
E.
“When you start to have undesirable side effects, the dose is decreased to the lowest possible level
and continued until you are 50 years old.”
62.
The patient thought to have a problem with the pituitary gland is given 25 units of regular insulin. A
short time later, blood analysis reveals that the patient has a normal pituitary response to insulin. What find
the physician in lab findings?
A.
Decreased level of growth hormone and elevated level of adrenocorticotropic hormone (ACTH).
B.
Decreased levels of growth hormone and adrenocorticotropic hormone (ACTH).
C.
Elevated level of growth hormone and decreased level of adrenocorticotropic hormone (ACTH).
D.
*Elevated levels of growth hormone and adrenocorticotropic hormone (ACTH).
E.
None of above
63.
The patient, a 45-year-old woman, has acromegaly as a result of a pituitary adenoma found and
removed when she was 16 years old. During a physical assessment before surgery for a knee replacement,
the physician discovers her liver. What is the physician can find?
A.
*Enlargement of her liver.
B.
Decreased of her lever
C.
Liver cirrhosis.
D.
All of them.
E.
None of above
64.
The patient just diagnosed with hyperpituitarism and acromegaly is scheduled for a hypophysectomy.
Which statement made by the patient indicates a need for clarification regarding this treatment?
A.
“I will drink whenever I feel thirsty after surgery.”
B.
“I'm glad there will be no visible incision from this surgery.”
C.
*“I hope I can go back to wearing size 8 shoes instead of size 12.”
D.
“I will wear slip-on shoes after surgery so I don't have to bend over.”
E.
None of above
65.
What is the priority nursing diagnosis in a 35-year-old man being treated for hyperpituitarism in
which excessive amounts of prolactin are secreted?
A.
Risk for Injury related to seizure activity
B.
*Disturbed Body Image related to gynecomastia
C.
Constipation related to anorexia and decreased metabolism
D.
Decreased Cardiac Output related to electrolyte imbalances
E.
None of above
66.
The patient who has been taking high-dose corticosteroid therapy for a month to treat a severe
inflammatory condition, which has now resolved, asks the physician why she needs to continue taking the
corticosteroids. What is the physician’s best response?
A.
“It is possible for the inflammation to recur when corticosteroid therapy is halted.”
B.
“Corticosteroids are a type of hormone. Once you have been started on a replacement hormone, you
must continue the hormone replacement therapy for the rest of your life.”
C.
None of above
D.
*“The drug suppressed your own adrenal gland secretion of corticosteroids. Slowly decreasing the
dose over time allows your adrenal glands to start adequate secretion again.”
E.
“The drug suppressed your immune system while you were taking it. Slowly decreasing the dose over
time prevents your immune system from starting up too quickly and initiating allergic reactions.”
67.
The patient is being admitted with acute adrenal insufficiency (addisonian crisis). What medication
should be prepared the physician to administer, EXEPT?
A.
Dextrose IV
B.
Saline solution IV
C.
Hydrocortisone sodium succinate
D.
*Insulin 20 units and dextrose 20%
E.
None of above
68.
The patient is taking exogenous cortisol, in the form of prednisone, daily for a temporary pulmonary
inflammation. She tells the physician that she is upset about her moon-shaped face and fat body. What is the
physician’s best response?
A.
“Don't worry; this is only water retention, not true fat.”
B.
“Increasing your exercise will help change the shape of your face.”
C.
*“When you come off the drug, your body fat will change back to normal over time.”
D.
“Obviously, you are not following the dietary restrictions recommended to you.”
E.
None of above
69.
The patient with adrenal hyperfunction screams at her husband, bursts into tears, and throws her
water pitcher against the wall. She then tells the physician, “I feel like I am going crazy.” What is the
physician’s best response?
A.
“I will ask for a psychiatric consult for you.”
B.
*“You feel this way because of your high hormone levels. I will order an antianxiety drug for you.”
C.
“You feel this way because you are frightened about having a chronic disease. Would you like me to
give you information about a support group?”
D.
“You must learn to control your behavior. Because you disturbing others, I am going to keep the door
to your room closed and restrict your visitors.”
E.
None of above
70.
The patient with hyperthyroid takes treatment with carbimasole 10 mg a day. Suddenly he start
complains with nausea, jaundice, vomiting, diarrhoea, abdominal pain, heart rate 140 beats per minute.
Which cause can be of this state?
A.
Infection
B.
Myocardial infarction or stroke
C.
Recent thyroid surgery
D.
Trauma
E.
*All of above
71.
The patient with hyperthyroid symptoms is having hormone studies done to confirm the diagnosis.
Which set of values indicates Graves’ disease hyperthyroidism?
A.
Elevated T3, elevated T4, high TSH levels
B.
*Elevated T3, normal T4, low TSH levels
C.
Elevated T3, low T4, high TSH levels
D.
Low T3, normal T4, high TSH levels
E.
None of above
72.
The patient with hyperthyroidism is taking lithium carbonate to inhibit thyroid hormone release.
Which of the following patient reports can not alerts the physician to side effects of this therapy?
A.
Increased sweating and diarrhea
B.
Blurred vision
C.
*Decreased attention span and insomnia
D.
All of above
E.
None of above
73.
The patient with hypothyroidism as a result of Hashimoto’s thyroiditis asks the physician how long
she will have to take thyroid medication. What is the physician’s best response?
A.
“You will need to take the thyroid medication until the goiter is completely gone.”
B.
None of above
C.
“The thyroiditis will be cured with antibiotics, and then you will no longer need the thyroid
medication.”
D.
*“You will need thyroid replacement hormone therapy for the rest of your life because the thyroid
gland function will not return.”
E.
“When your thyroid function studies indicate a normal blood level of thyroid hormones, you will be
able to discontinue the medication.”
74.
When taking the blood pressure of a patient after a parathyroidectomy, the physician notes that the
patient's hand has gone into flexion contractions. What is the physician’s interpretation of this observation?
A.
Hypokalemia
B.
Hyperkalemia
C.
Hyponatremia
D.
*Hypocalcemia
E.
None of above
75.
The patient diabetic patient asks the physician why it is necessary to maintain blood glucose levels no
lower than about 74 mg/dL. What is the physician’s best response?
A.
“Glucose is the only fuel form used by body cells to produce energy needed for physiologic activity.”
B.
*“The central nervous system, which cannot store glucose, requires a continuous supply of glucose
for fuel.”
C.
“Without a minimum level of glucose circulating in the blood, erythrocytes cannot produce ATP.”
D.
“The presence of glucose in the blood counteracts the formation of lactic acid and prevents acidosis.”
E.
None of above
76.
The patient is a 28-year-old man newly diagnosed with type 1 diabetes mellitus. He wears glasses for
myopia and asks the physician how frequently he should see his ophthalmologist now. What is the
physician’s best answer?
A.
“At your age, you do not need to change your usual patterns for visiting the ophthalmologist.”
B.
“See your ophthalmologist whenever you have a vision problem and yearly after you are 40 years
old.”
C.
“Your vision will change more quickly now, and you should see the ophthalmologist whenever you
find that your glasses are not strong enough to allow you to read comfortably.”
D.
*“The disease increases your risk for cataracts, glaucoma, and retinal blood vessel changes, so you
should see the ophthalmologist yearly, even when you do not have a new vision problem.”
E.
None of above
77.
The 30-year-old woman whose father has type 1 diabetes mellitus asks the physician what her
chances are of developing diabetes because of her father's disease. The risk for becoming diabetic is:
A.
*20-50%
B.
0%
C.
50%
D.
50-80%
E.
None of above
78.
The 30-year-old woman whose mother has type 2 diabetes mellitus asks the physician what her
chances are of developing diabetes because of her mother's disease. The risk for becoming diabetic is:
A.
20-50%
B.
0%
C.
100%
D.
*15-20%
E.
50-80%
79.
The patient with type 2 diabetes had been taking the oral antidiabetic agents glyburide and
metformin. Which medications can you prescribe to increase the compliance to treatment?
A.
*Glucovance
B.
Phenformin
C.
Intensive insulin therapy
D.
Insulin therapy and metformin
E.
None of above
80.
The patient with type 1 diabetes mellitus is switching from an animal-source regular insulin to a
synthetically derived human regular insulin. Which precaution should the physician explain to this patient?
A.
“Human insulin should only be administered in the umbilical area.”
B.
“Do not mix human regular insulin with any other type of insulin.”
C.
*“Adjustments in insulin timing may be needed with the human regular insulin.”
D.
“You may notice the need to increase the dose to achieve the same level of glucose control.”
E.
None of above
81.
The patient with diabetes mellitus 2 type who is just starting on insulin therapy wants to know why
more than one injection of insulin each day will be required. What can you prescribe to him that takes one
injection of insulin each day?
A.
Novo Rapid
B.
Actrapid
C.
Protaphane
D.
*Lantus
E.
None of above
82.
The patient on an intensified insulin regimen consistently has a fasting blood glucose between 270
and 280 mg/dL, a postprandial blood glucose level below 200 mg/dL, ketones in urine ++and a hemoglobin
A1c level of 8.5%. What is the physician’s interpretation of these findings?
A.
*The patient has developing night hypoglycemia.
B.
The patient is at increased risk for developing hyperglycemia.
C.
The patient is demonstrating signs of insulin resistance.
D.
The patient is demonstrating good control of blood glucose.
E.
None of above
83.
The patient with diabetes is visually impaired and wants to know if syringes can be prefilled and
stored for use later. What is the physician’s best response?
A.
*“Yes, prefilled syringes can be stored for up to 3 weeks in the refrigerator in a vertical position with
the needle pointing up.”
B.
“Yes, prefilled syringes can be stored for up to 3 weeks in the refrigerator, placed in a horizontal
position.”
C.
“Insulin reacts with plastic, so prefilled syringes must be made of glass.”
D.
“No, insulin cannot be stored for any length of time outside of the container.”
E.
None of above
84.
The patient newly diagnosed with type 2 diabetes tells the physician that since he has increased his
intake of fiber, he is having loose stools, flatulence, and abdominal cramping. What is the physician’s best
response?
A.
“Decrease your intake of water and other fluids.”
B.
*“Decrease your intake of fiber now and gradually add high-fiber foods back into your diet.”
C.
“You must have allergies to high-fiber foods and will need to avoid them in the future.”
D.
“Taking an antacid 1 hour before meals or 2 hours after meals should reduce the intensity of your
bowel problems.”
E.
None of above
85.
The 45-year-old diabetic patient has proliferative retinopathy, nephropathy, and peripheral
neuropathy. What should the physician teach this patient about exercise?
A.
“The type of exercise that would most efficiently help you to lose weight, decrease insulin
requirements, and maintain cardiovascular health would be jogging for 20 minutes 4 to 7 days each week.”
B.
“Considering the complications you already have, vigorous exercise for an hour each day is needed to
prevent progression of disease.”
C.
“Considering the complications you already have, you should avoid engaging in any form of
exercise.”
D.
*“Swimming or water aerobics 30 minutes each day would be the safest exercise routine for you.”
E.
None of above
86.
The patient tells the physician that he enjoys having a glass of wine on Saturdays when dining out
with friends. He asks if having type 1 diabetes will prohibit him from this activity. What is the physician’s
best response?
A.
“Insulin activity is dramatically reduced under the influence of alcohol and drinking even one glass of
wine will increase your insulin requirements.”
B.
“Diabetics have decreased kidney function and should avoid ingesting alcohol in all forms at all
times.”
C.
“You shouldn't drink any alcohol because it is likely to increase your sense of hunger and make you
overeat.”
D.
*“One glass of wine can be ingested with a meal and is counted as two fat exchanges.”
E.
None of above
87.
The patient getting ready to engage in a 30-minute, moderate-intensity exercise program performs a
self-assessment. Which data indicate that exercise should be avoided at this time?
A.
*Ketone bodies in the urine
B.
Weight 1 pound higher than the week before
C.
Blood sugar level of 155 mg/dL
D.
Pulse rate of 66 beats/min
E.
None of above
88.
A patient who has long-standing diabetes mellitus and severe, burning pain in the feet and hands as a
result of peripheral neuropathy asks the physician why an antidepressant has been prescribed. What is the
physician’s best response?
A.
“Many people experiencing chronic pain become depressed.”
B.
*“The antidepressants may counteract the chemicals causing your pain.”
C.
“You are less likely to become addicted from using antidepressants than you are from using other
types of pain killers.”
D.
“The antidepressants also have strong anti-inflammatory properties and can reduce the pain you have
from inflammation.”
E.
None of above
89.
The physician is rapidly infusing insulin IV to a patient with extreme hyperglycemia. Which insulin
he used?
A.
*Actrapid
B.
NovoMix30
C.
Protaphane
D.
Lantus
E.
Humulin NPH
90.
A 32-year-old father of two is involved in a head-on motor vehicle accident and is admitted to your
hospital. He has sustained a blunt force trauma to the upper thorax but injuries appear only to be several
cracked ribs and a minor concussion. Appropriate therapy and observation is begun and he begins to recover
gradually. On hospital day 3, he reports the uncontrollable urge to drink copious amounts of water, states
that he has been urinating much more frequently and that his urine is very thin in appearancThe next step
you take is to obtain blood work and a urinanalysis, which are found to be normal except for high plasma
osmolarity and low urine osmolarity. You decide to do a water deprivation test. Thus for 24 hours, the
patient is restricted to a sip of water every hour, and on day 2 of the test is clinically judged to be
dehydrateVasopressin (ADH) is then given by IV and his urine osmolarity is seen to rise above a baseline
dehydrated state reading. What is the most likely diagnosis?
A.
Syndrome of inappropriate antidiuretic hormone (SIADH).
B.
*Central diabetes insipidus.
C.
Nephrogenic diabetes insipidus.
D.
Cushing's syndrom
E.
Addison's diseas
91.
A 32-year-old father of two is involved in a head-on motor vehicle accident and is admitted to your
hospital. He has sustained a blunt force trauma to the upper thorax but injuries appear only to be several
cracked ribs and a minor concussion. Appropriate therapy and observation is begun and he begins to recover
gradually. On hospital day 3, he reports the uncontrollable urge to drink copious amounts of water, states
that he has been urinating much more frequently and that his urine is very thin in appearancThe next step
you take is to obtain blood work and a urinanalysis, which are found to be normal except for high plasma
osmolarity and low urine osmolarity. You decide to do a water deprivation test. Thus for 24 hours, the
patient is restricted to a sip of water every hour, and on day 2 of the test is clinically judged to be
dehydrateVasopressin (ADH) is then given by IV and his urine osmolarity is seen to rise above a baseline
dehydrated state reading. The most likely diagnosis is central diabetes insipidus. What is the next step in the
management of this patient?
A.
Hydrochlorothiazide (HCTZ).
B.
Furosemide (Lasix).
C.
Hydrocortisone orally.
D.
Aggressive fluid replacement.
E.
*Desmopressin (DDAVP).
92.
A 47-year-old female presents to the health clinic for symptoms of weight she can't seem to lose
despite strict dieting and concerns of menstrual irregularity. She states she also has developed multiple
abdominal stretch marks and has noticed a deepening of her voice and appearance of facial hair. As part of
her initial work up, you obtain a 24-hour urine cortisol level, which is returned as 120 ng/ml (N 20 100
ng/ml). What is the most likely diagnosis in this patient?
A.
Metabolic syndrome.
B.
Diabetes mellitus type II.
C.
*Cushing's syndrom
D.
Addison's diseas
E.
Conn's syndrom
93.
A 47-year-old female presents to the health clinic for symptoms of weight she can't seem to lose
despite strict dieting and concerns of menstrual irregularity. She states she also has developed multiple
abdominal stretch marks and has noticed a deepening of her voice and appearance of facial hair. As part of
her initial work up, you obtain a 24-hour urine cortisol level, which is returned as 120 ng/ml (N 20 100
ng/ml). The most likely diagnosis in this patient is Cushing's syndrome.What is the most appropriate next
step in diagnosis of this patient?
A.
Cosyntropin (ACTH) stimulation test.
B.
*Dexamethasone suppression test.
C.
Cosyntropin (ACTH) suppression test.
D.
Dexamethasone stimulation test.
E.
Urine catecholamine collection over a 24-hour perio
94.
You are currently a physician on a medical charity trip to Botswana and as part of the work you and
your group are doing, you are holding a clinic for the village children in a school building. You are brought
an 8-year-old girl who her mother states is not eating well lately and has had chronic diarrheOn physical
exam, she appears chronically malnourished with thin arms and legs although she has a large, distended
abdomen. Her liver is palpable one inch below the right costal margin and there appears to be ascetic fluid
within her abdomen. She does have 2+ dependent edema in her ankles and a generalized flaky skin rash. Her
mother has two other children who appear in similar states of health. What dietary deficiency is this patient
likely suffering from?
A.
*Protein deficiency.
B.
Carbohydrate deficiency.
C.
Fat deficiency.
D.
Calcium deficiency.
E.
Vitamin K deficiency.
95.
The patient has clinical signs which may suggest acromegaly. Which of the following is the most
specific test to diagnose acromegaly?
A.
Magnetic resonance imaging of the pituitary gland
B.
Measurement of growth hormone (GH) levels during an oral glucose tolerance test
C.
*Measurement of insulin-like growth factor I serum levels
D.
Measurement of random GH blood levels
E.
An octreotide scan
96.
A 35-year-old woman with amenorrhea is found to have an enlarged pituitary glan. Her prolactin
level is 80 ng/L (normal less than 20 ng/L), and her thyrotropin level is 100 mlU/ml (normal, 0.5 to 4.5
mlU/ml). Which of the following is the best treatment option for this patient?
A.
Administration of bromocriptine
B.
*Administration of L-thyroxine
C.
Irradiation of the pituitary gland
D.
Resection of the pituitary gland
E.
Use of oral contraceptives
97.
The patient has clinical signs which may suggest primary hyperparathyroidism. Which of the
following is the most common presentation of primary hyperparathyroidism?
A.
*Asymptomatic hypercalcemia
B.
Bone fracture
C.
Kidney stones
D.
Osteoporosis
E.
Renal failure
98.
A 60-yr-old woman is found to have a large toxic nodular goiter. Choose the most likely
management.
A.
Radioactive iodine
B.
*Subtotal thyroidectomy
C.
Propanolol
D.
Carbimazole
E.
Thyroxine
99.
A previously healthy 45-year-old man is hospitalized in the intensive care unit for almost a month,
following a car accident that caused serious head trauma. During evaluation of the patient for tachycardia, he
was found to have a thyrotropin level of 0.5 mlU/ml (normal, 0.5 to 4.5 mlU/ml) and a serum thyroxine level
of 5.7 pmol/L (normal, 10 to23 pmol/L). What is the most likely diagnosis for this patient?
A.
Central hypothyroidism
B.
*Euthyroid sick syndrome
C.
Hyperthyroidism
D.
Pituitary apoplexy
E.
Primary hypothyroidism
100. 19 year old girl presented with obesity, round plethoric face, hirsutism and short stature. On
examination her blood pressure was high and her obesity was truncal. Of the following laboratory finding is
expected to find?
A.
Metabolic acidosis
B.
Hyperkalemia
C.
Hypoglycemia
D.
Hypokalemia and metabolic alkalosis
E.
*Increased cortisol level
101. A 27-yr-old pregnant woman is found to have thyrotoxicosis due to Grave's disease during second
trimester of her pregnancy. Choose the most likely management.
A.
Radioactive iodine
B.
Subtotal thyroidectomy
C.
Propanolol
D.
*Carbimazole
E.
Thyroxine
102. A 10-yr-old girl presents with thyrotoxicosis. A radioisotope scan shows an enlarged thyroid with
uniform uptake throughout. Choose the most likely management.
A.
Radioactive iodine
B.
Subtotal thyroidectomy
C.
Propanolol
D.
*Carbimazole
E.
Thyroxine
103. A 30-yr-old woman is found to have Graves' disease. She remains thyrotoxic after treatment with
Carbimazole for 1 yr. Choose the most likely management.
A.
Radioactive iodine
B.
*Thyroidectomy
C.
Propanolol
D.
Carbimazole
E.
Thyroxine
104. A 50-yr-old woman presents with thyroid enlargement. Thyroid function tests are normal. Needle
biopsy confirms the diagnosis of Hashimoto's thyroiditis. Choose the most likely management.
A.
Radioactive iodine
B.
Subtotal thyroidectomy
C.
Propanolol
D.
Carbimazole
E.
*Thyroxine
105. A 40-yr-old man presents with hypertension, palpitations and sweating; 24 hr urinary VMA is
elevated. Choose the most likely diagnosis.
A.
Parathyroid adenoma
B.
Parathyroid hyperplasia
C.
Prolactinoma
D.
Insulinoma
E.
*Pheochromocytoma
106. A 61-yr-old woman presents with stiff joints, myopathy and constipation. Plain radiographs reveal a
right calculus and evidence of osteitis fibrosa. Choose the most likely diagnosis.
A.
*Parathyroid adenoma
B.
Parathyroid hyperplasia
C.
Prolactinoma
D.
Insulinoma
E.
Phaeochromocytoma
107. A 40-yr-old woman presents with a solitary nodule in the right thyroid lob. FNAC suggests follicular
adenoma. Choose the most likely management.
A.
*Total thyroid lobectomy
B.
Ablative dose of radioactive iodine
C.
External beam radiation
D.
Chemotherapy
E.
Reassure and repeat FNAC in 1 yr
108. A 20-yr-old woman presents with a 4 cm solid mass in the left thyroid lob. FNAC reveals papillary
carcinoma. Choose the most likely management.
A.
Total thyroid lobectomy
B.
Ablative dose of radioactive iodine
C.
External beam radiation
D.
Propranolol
E.
*Total thyroidectomy and removal of central group of lymph nodes
109. A 15-yr-old boy presents with a 1 cm solitary thyroid nodule and diarrhea. FNAC is reported as
malignant. Serum calcitonin is raise. Choose the most likely management.
A.
Total thyroid lobectomy
B.
Ablative dose of radioactive iodine
C.
External beam radiation
D.
Propranolol
E.
*Total thyroidectomy and removal of central group of lymph nodes
110. A 30-yr-old woman presents with a 2 cm thyroid nodule. FNAC suggests a colloid nodule. Choose
the most likely management.
A.
Total thyroid lobectomy
B.
Ablative dose of radioactive iodine
C.
External beam radiation
D.
Chemotherapy
E.
*Reassure and repeat FNAC in 1 yr
111. A 50-yr-old woman presents with a thyroid goiter. A core biopsy reveals evidence of lymphoma.
Choose the most likely management.
A.
Total thyroid lobectomy
B.
Thyroxine
C.
*External beam radiation
D.
Propranolol
E.
Reassure and repeat FNAC in 1 yr
112. A 27-yr-old woman presents with fever, sore throat and dysphagia. On examination she has a fine
tremor and a diffusely tender thyroid. Radioisotope scan shows no uptake. Choose the most likely diagnosis
A.
Thyroglossal cyst
B.
*De Quervain's cyst
C.
Hypothyroidism
D.
Lymphoma
E.
Simple goiter
113. A 30-yr-old woman presents with weight gain, constipation, lethargy and flaky rash. Choose the most
likely diagnosis
A.
Thyroglossal cyst
B.
De Quervain's cyst
C.
*Hypothyroidism
D.
Lymphoma
E.
Simple goiter
114. A 37-yr-old woman presents with weight loss, muscular weakness, oligomenorrhea, diarrhea and
blurring of vision. On examination, there is exophthalmos and proximal myopathy. Choose the most likely
diagnosis
A.
Thyroglossal cyst
B.
*Graves' disease
C.
Hypothyroidism
D.
Lymphoma
E.
Simple goiter
115. A 19-yr-old student presents with a neck swelling. On examination the swelling moves up with
swallowing and protrusion of the tongue. Choose the most likely diagnosis
A.
*Thyroglossal cyst
B.
De Quervain's cyst
C.
Hypothyroidism
D.
Graves' disease
E.
Simple goiter
116. A 49-yr-old woman presents with goiter. On examination, the thyroid is firm and rubbery. Thyroid
microsomal antibodies are positive in high titer. Choose the most likely diagnosis
A.
Thyroglossal cyst
B.
*Hashimoto's thyroiditis
C.
Hypothyroidism
D.
Graves' disease
E.
Simple goiter
117. In the biochemical profile of patient S., 43 yr-old, it is elevated serum T4 and increased radioactive
iodine uptake. Choose the most likely diagnosis.
A.
Non-toxic goitre
B.
Hashimoto's thyroiditis
C.
Subacute thyroiditis
D.
Hypothyroidism
E.
*Graves' disease
118. In the biochemical profile of patient B., 29 yr-old, it is elevated serum T4 and low radioactive iodine
uptake. Choose the most likely diagnosis.
A.
Non-toxic goitre
B.
Hashimoto's thyroiditis
C.
*Subacute thyroiditis
D.
Hypothyroidism
E.
Graves' disease
119. In the biochemical profile of patient J., 38 yr-old, who has a neck mass, it is normal T3 and T4.
Choose the most likely diagnosis.
A.
*Non-toxic goitre
B.
Hashimoto's thyroiditis
C.
Subacute thyroiditis
D.
Hypothyroidism
E.
Graves' disease
120. In the biochemical profile of patient L., 56 yr-old., it is normal TSH, free T4 and T3. Decreased
serum total T4. Choose the most likely diagnosis.
A.
Non-toxic goitre
B.
Hashimoto's thyroiditis
C.
*Thyroid binding globulin deficiency
D.
Hypothyroidism
E.
Graves' disease
121. Patient N., 46 yr-old, complained on restlessness, confusion, irritability, seizures and developed
coma. Choose the most likely diagnosis.
A.
Hypokalaemia
B.
*Hyponatraemia
C.
Hypervitaminosis A
D.
Hyperkalaemia
E.
Hyperglycaemia
122. Patient D., 56 yr-old, complains on muscle weakness, bradycardia and hypotension. ECG shows tall
peaked T waves. Choose the most likely diagnosis.
A.
Hypokalaemia
B.
Hyponatraemia
C.
Hypervitaminosis A
D.
*Hyperkalaemia
E.
Hypoglycaemia
123. Patient J., 38 yr-old, complains on sweating, palpitations, tremors, drowsiness and fatigue. Choose
the most likely diagnosis.
A.
Hypokalaemia
B.
Hyponatraemia
C.
Hypervitaminosis A
D.
Hyperkalaemia
E.
*Hypoglycaemia
124. Patient G., 38 yr-old, complains on muscle weakness and ectopic beats. ECG shows flattened or
inverted T waves. Choose the most likely diagnosis.
A.
*Hypokalaemia
B.
Hyponatraemia
C.
Hypervitaminosis A
D.
Hyperkalaemia
E.
Hypoglycaemia
125. Patient S., 43 yr-old, has perioral paraesthesia, carpopedal spasm and generalised seizures. Choose
the most likely diagnosis.
A.
Hypokalaemia
B.
Hyponatraemia
C.
*Hypocalcaemia
D.
Hyperkalaemia
E.
Hypoglycaemia
126. Patient U., 44 yr-old, complains on severe abdominal pain, nausea, vomiting, constipation, polyuria
and polydipsia. Choose the most likely diagnosis.
A.
*Hypercalcaemia
B.
Hyponatraemia
C.
Hypervitaminosis A
D.
Hyperkalaemia
E.
Hypoglycaemia
127. A 40-yr-old man presents to his GP complaining of change in appearance and headaches. His brow is
more prominent and his nose had broadened. He states that his shoes are too small and he has tingling in
certain fingers worse at night. Choose the most likely management.
A.
Propanolol
B.
Calciferol
C.
Carbimazole
D.
Thyroxine
E.
*Octreotide (somatostatin)
128. A 50-yr-old woman presents to her GP for fatigue, depression and weight gain. She also complains of
constipation and poor memory. On examination, she has puffy face and coarse facial features, thin eyebrows
and a large tongue. Choose the most likely management
A.
Propanolol
B.
Calciferol
C.
Carbimazole
D.
*Thyroxine
E.
Octreotide (somatostatin)
129. A 35-yr-old pregnant woman presents to her GP with anxiety. On examination she is nervous woman
with exophthalmos, warm peripheries and atrial fibrillation. Choose the most likely management.
A.
Propanolol
B.
Calciferol
C.
*Propylthiouracil
D.
Thyroxine
E.
Octreotide (somatostatin)
130. A 50-yr-old obese woman presents to casualty with rib fractures and bruising following a fall in the
bathroom. She is noted to be hypertensive and have glycosuria. Choose the most likely management
A.
Propanolol
B.
Calciferol
C.
Carbimazole
D.
Thyroxine
E.
*Metyrapone (anticortisol)
131. A 70-yr-old woman presents to her GP with weight loss and depression. On examination she is noted
to have buccal pigmentation and pigmented scars. She appears dehydrated. Her BP is 100/60 mm Hg.
Choose the most likely management.
A.
*Long-term replacement with glucocorticoids and mineralocorticoids
B.
Calciferol
C.
Carbimazole
D.
Thyroxine
E.
Octreotide (somatostatin)
132. A 16-yr-old girl presents with anterior neck mass. It moves upward upon protrusion of tongue.
Thyroid radionucleotide scan shows no uptake in the midline. Choose the most likely diagnosis.
A.
*Thyroglossal cyst
B.
Hashimoto's thyroiditis
C.
Toxic multinodular goiter
D.
Thyroid storm
E.
Graves' disease
133. A 40-yr-old woman presents with a hard, nodular midline neck mass. Blood tests reveal the presence
of antibodies to thyroglobulin. Choose the most likely diagnosis.
A.
Thyroglossal cyst
B.
*Hashimoto's thyroiditis
C.
Toxic multinodular goiter
D.
Thyroid storm
E.
Graves' disease
134. A 45-yr-old woman presents with a diffuse swelling of the thyroid gland. On examination she has a
stare, lid lag, and lid retraction. On the dorsum of her legs she has areas of raised peau d'orange-like
thickened skin. Blood tests reveal thyroid-stimulating immunoglobulins against the TSH receptor site.
Choose the most likely diagnosis.
A.
Thyroglossal cyst
B.
Hashimoto's thyroiditis
C.
Toxic multinodular goiter
D.
Thyroid storm
E.
*Graves' disease
135. A 50-yr-old woman presents with fever, tachycardia restlessness, hypertension and vomiting. On
examination she has diffuse swelling of the thyroid gland and strabismus with diplopia. Choose the most
likely diagnosis.
A.
Thyroglossal cyst
B.
Hashimoto's thyroiditis
C.
Toxic multinodular goiter
D.
*Thyroid storm
E.
Graves' disease
136. A 35-yr-old man presents with a hard, nodular midline neck mass that moves upward on swallowing.
Thyroid radionucleotide scan shows cold spots. Choose the most likely diagnosis.
A.
Thyroglossal cyst
B.
*Thyroid carcinoma
C.
Toxic multinodular goiter
D.
Thyroid storm
E.
Graves' disease
137. A 67-yr-old man is noted to have a glucose level of 37 mmol/l and a Na+ of 163 mmol/l. He has no
prior history of diabetes and has been on IV fluids for a week. His other medications include IV Cefuroxime,
Metronidazole, and Dexamethasone. Choose the most likely management.
A.
Insulin sliding scale, Heparin and 0.9% saline
B.
*Insulin sliding scale, Heparin and 0.45% saline
C.
Sugary drink
D.
Chest X-ray
E.
50 ml of 50% dextrose IV
138. A 60-yr-old man is brought to A&E in an unconscious state. His glucose is 35 mmol/l. His arterial
blood gas shows a pH of 7.2 and a PaCO2 of 28 mm Hg. Serum Na is 140, K is 3.0, Cl is 100 and HCO3" is
– 18 mEq/L. Choose the most likely management.
A.
Sugary drink
B.
Insulin sliding scale, Heparin and 0.45% saline
C.
*Insulin sliding scale, 0.9% NS and potassium replacement
D.
Chest X-ray
E.
50 ml of 50% dextrose IV
139. A 40-yr-old diabetic actor is started on Propanolol for stage fright. He collapses after a day shooting.
He has not changed his insulin regimen. Choose the most likely management.
A.
Sugary drink
B.
Insulin sliding scale, Heparin and 0.45% saline
C.
Insulin sliding scale, 0.9% NS and potassium replacement
D.
Chest X-ray
E.
*50 ml of 50% dextrose IV
140. A 50-yr-old diabetic presents in a coma. He is febrile with diminished breath sounds on auscultation.
He has warm extremities. His glucose is 20 mmol/l. His white cell count is 22 x 109/l with increased
neutrophils. Choose the most likely management.
A.
Sugary drink
B.
Insulin sliding scale, Heparin and 0.45% saline
C.
Insulin sliding scale, 0.9% NS and potassium replacement
D.
*Chest X-ray
E.
50 ml of 50% dextrose IV
141. A 78-yr-old woman was diagnosed with diabetes after she was found to have a high blood glucose
during an admission to hospital with a fall. Despite following appropriate dietary advice, her HbAlc remains
elevated at 11%. She is visually impaired and finds it impossible to test her blood glucose at home. She is
not obese. Choose the most likely management.
A.
Metformin
B.
No change in treatment required
C.
IV insulin sliding scale
D.
*Gliclazide
E.
Dietary adjustment
142. A 27-yr-old woman was found to have glycosuria at a routine antenatal clinic visit. A glucose
tolerance test confirmed diagnosis of gestational diabetes. Choose the most likely management.
A.
Metformin
B.
No change in treatment required
C.
IV insulin sliding scale
D.
Gliclazide
E.
*Dietary adjustment
143. A 65-yr-old man has had type 2 diabetes for 4 years, for which he was taking Chlorpropamide. He
presents with an acute MI and his laboratory blood glucose is 11 mmol/l. Choose the most likely
management.
A.
Metformin
B.
No change in treatment required
C.
*IV insulin sliding scale
D.
Gliclazide
E.
Dietary adjustment
144. A 58-yr-old man was diagnosed with diabetes at a routine medical examination 3 months ago. His
BMI is 32 despite losing 5 kg by following the dietician's advice. His home blood glucose readings range
from 7 to 11 and his HbAlc is 10%. Choose the most likely management.
A.
*Metformin
B.
No change in treatment required
C.
IV insulin sliding scale
D.
Gliclazide
E.
Dietary adjustment
145. A 32-yr-old woman has had type 1 diabetes for 15 yr. She injects Isophane insulin twice a day and
rarely tests her blood glucose at home. She attends the diabetic clinic for the first time in over a year and
informs you that she is 12 weeks pregnant. Choose the most likely management.
A.
Metformin
B.
No change in treatment required
C.
IV insulin sliding scale
D.
*One long-acting and 3 short- acting insulin
E.
Dietary adjustment
146. A 14-yr-old girl presents with lethargy and weight gain. She is depressed and sensitive to cold. She
would like something done about her excessive weight. The other children abuse her. Choose the most likely
investigation.
A.
Chest X-ray
B.
Serum cortisol
C.
Dexamethasone suppression
D.
Water deprivation test
E.
*T3, T4 and TSH
147. An obese 12-yr-old is brought in by his mother who complains about his weight, lack of energy. He
has polyuria, polydipsia. His height is normal for his age. Choose the most likely investigation
A.
*CT scan of the skull
B.
Serum cortisol
C.
Dexamethasone suppression
D.
Water deprivation test
E.
T3, T4 and TSH
148. A 52-yr-old man has been gaining weight. He complains of a chronic cough, acne and bruising. On
physical examination you find his legs and arms to be abnormally thin. Choose the most likely investigation
A.
Chest X-ray
B.
Serum cortisol
C.
*Dexamethasone suppression test
D.
Water deprivation test
E.
T3, T4 and TSH
149. A teenage boy presents with obesity, greasy skin and acne. His face is round and his cheeks are red.
The past year he has suffered recurrent bouts of bronchitis.
A.
Chest X-ray
B.
Serum cortisol
C.
*Dexamethasone suppression test
D.
Water deprivation test
E.
T3, T4 and TSH
150. An 18-yr-old girl complains of her appearance. She is much too fat, she says. She also complains of
missed periods and hairiness. On physical examination you find her to be 10 kg overweight. Choose the most
likely investigation
A.
Abdominal ultrasonography
B.
Serum cortisol
C.
*Dexamethasone suppression test
D.
Water deprivation test
E.
T3, T4 and TSH
151. A 52-yr-old man presents with painless lump in the neck and a chronic cough. Physical examination
finds tachycardia and pallor. He feels that he has lost weight, but he is not certain. He does not smoke or
drink. Choose the most likely diagnosis.
A.
Toxic adenoma
B.
Adrenal hyperplasia
C.
Hyperthyroidism
D.
Hypothyroidism
E.
*Follicular carcinoma
152. On a routine blood examination a 43-yr-old woman is found to have very high serum calcium level.
She has complained recently of bouts of abdominal pain and recurrent UTI. On physical examination you
find an enlarged thyroid gland. Choose the most likely diagnosis.
A.
Toxic adenoma
B.
Adrenal hyperplasia
C.
Hyperthyroidism
D.
Hypothyroidism
E.
*Parathyroid carcinoma
153. A young woman is warm, even when resting. She turns the central heating off, opens the windows
and annoys her family. Her pulse rate is high and her skin is moist. Choose the most likely diagnosis.
A.
Toxic adenoma
B.
Adrenal hyperplasia
C.
*Hyperthyroidism
D.
Hypothyroidism
E.
Follicular carcinoma
154. A middle-aged woman complains of irritability and weight loss. She says she has palpitations. On
physical examination you find mild tachycardia and goiter. There are no eye changes. A thyroid scan
determines a single hot nodule. Choose the most likely diagnosis.
A.
*Toxic adenoma
B.
Adrenal hyperplasia
C.
Hyperthyroidism
D.
Hypothyroidism
E.
Follicular carcinoma
155. A young man presents with a neck lump. It is painless and had been bothering him for the past 4
months. He has no other symptoms or signs. On palpation you find the lump to be single discrete not
particularly hard and confined to the thyroid gland itself. His cervical lymph nodes are enlarged. Choose the
most likely diagnosis.
A.
Toxic adenoma
B.
Adrenal hyperplasia
C.
Hyperthyroidism
D.
Hypothyroidism
E.
*Papillary carcinoma
156. A 12-yr-old boy presents with 4 weeks of weight loss, polyuria and polydipsia. Choose the most
likely investigation
A.
Dexamethasone suppression
B.
ACTH stimulation test
C.
*Fasting blood glucose
D.
T3, T4 and TSH levels
E.
Water deprivation test
157. A 44-yr-old woman presents with tachycardia, atrial fibrillation, double vision and swelling above
her ankles. She has lid lag on examination. Choose the most likely investigation
A.
Dexamethasone suppression test
B.
ACTH stimulation test
C.
Fasting blood glucose
D.
*T3, T4 and TSH levels
E.
Water deprivation test
158. A 42-yr-old man has hypertension, hyperglycaemia, myopathy, thinning of the skin, buffalo hump
and truncal obesity. Choose the most likely investigation
A.
*Dexamethasone suppression test
B.
ACTH stimulation test
C.
Fasting blood glucose
D.
T3, T4 and TSH levels
E.
Water deprivation test
159. A 34-yr-old man presents with insidious onset weakness and weight loss. On examination, he has
hyperpigmentation of the palmar creases and postural hypotension. Choose the most likely investigation
A.
Dexamethasone suppression test
B.
*ACTH stimulation test
C.
Fasting blood glucose
D.
T3, T4 and TSH levels
E.
Water deprivation test
160. A 48-yr-old man is admitted for investigation and glycosuria. His wife comments that his appearance
has changed over the last few years and "everything seems to have got bigger". He also complains of tingling
in his left hand and excessive sweating. Choose the most likely investigation
A.
Dexamethasone suppression test
B.
*Look at old radiographs
C.
Fasting blood glucose
D.
T3, T4 and TSH levels
E.
Water deprivation test
161. A 55-yr-old presented with following reports on a routine screen: Calcium-2.85 mmol/l, phosphate—
0.8, ALP—110, PTH—raised, 25-OH vitamin D—low-normal. Choose the most likely diagnosis
A.
*Primary hyperparathyroidism
B.
Tertiary hyperparathyroidism
C.
Hypoparathyroidism
D.
Hyperthyroidism
E.
Paget's disease of bone
162. A 35-yr-old woman with bone pain, drowsiness and thirst: Calcium—3.3, phosphate—0.75, ALP—
190, PTH—low-normal, PTH—activity high, glucose—6 mmol/l. Choose the most likely diagnosis
A.
Primary hyperparathyroidism
B.
Tertiary hyperparathyroidism
C.
Hypoparathyroidism
D.
Hyperthyroidism
E.
*Hyperparathyroidism with ectopic PTH
163. A 65-yr-old patient is becoming increasingly confused. She has periods where her confusion seems to
be stable and then seems to rapidly deteriorate in a stepwise progression. On examination there are extensor
plantars but leg reflexes are diminished. Choose the most likely diagnosis
A.
Coronary artery disease
B.
*Cerebrovascular disease
C.
Nephropathy
D.
Retinopathy
E.
Polyneuropathy
164. A 20-yr-old patient complains of worsening vision over several months. On examination fundoscopy
is difficult even through dilated pupils. There appears to be opacity of the lens. Choose the most likely
diagnosis
A.
Coronary artery disease
B.
Cerebrovascular disease
C.
Nephropathy
D.
*Cataracts
E.
Polyneuropathy
165. A 50-yr-old patient complains of burning pain in the feet, worse at night or on walking. He describes
the sensation as like walking on hot coals. Choose the most likely diagnosis
A.
Coronary artery disease
B.
Cerebrovascular disease
C.
Nephropathy
D.
Retinopathy
E.
*Polyneuropathy
166. A 45-yr-old patient complains of giddiness and falls. She also suffers with intermittent vomiting and
sweating and occasional faecal incontinence at night. Choose the most likely diagnosis
A.
Coronary artery disease
B.
Cerebrovascular disease
C.
Nephropathy
D.
Retinopathy
E.
*Autonomic neuropathy
167. Patient N., 52 years old, suffers from diabetes during 8 years, gets Glibenclamide 15 mg a day. He
complains on shortness of breath and pitting edema. On examination: BP - 180/110 mm Hg. Laboratory:
glycemia - 6.2 mmol / L, creatinine - 0.15 mmol / L, glomerular filtration - 62 ml / min. What diagnosis will
you expose the patient?
A.
diabetes mellitus 1 type, moderate, stage subcompensation.
B.
diabetes mellitus 1 type, severe, stage subcompensation.
C.
diabetes mellitus 2 type, moderate, stage subcompensation.
D.
*diabetes mellitus 2 type, severe, stage subcompensation.
E.
diabetes mellitus 1 type, severe, stage of decompensation.
168. Patient R., 54 years old, suffers from diabetes for 10 years, gets Glibenclamide 15 mg a day.
Complains of shortness of breath and pitting edema. On examination: BP - 180/110 mmHg Laboratory:
glycemia - 6.2 mmol / L, creatinine - 0.15 mmol / L, glomerular filtration - 62 ml / min. What complications
of diabetes occurs in the patient?
A.
Diabetic nephropathy III stage.
B.
Diabetic nephropathy IV stage.
C.
*Diabetic nephropathy V stage.
D.
Diabetic cardiopathy, heart failure IIA stage
E.
Diabetic cardiopathy, heart failure III stage
169. The patient, 51 years old, suffers from diabetes 7 years, gets glibenclamide 15 mg a day. Complains
of shortness of breath and pitting edema. On examination of blood pressure - 180/110 mm Hg. Laboratory:
glycemia - 6.2 mmol / L, creatinine - 0.15 mmol / L, glomerular filtration - 62 ml / min. What medication
will you recommend to the patient?
A.
Gliclazide
B.
*Gliquidone
C.
Glimepiride
D.
Pioglitazone
E.
Repaglinide
170. Patient L, 41 years is suffering from diabetes type 1 for 16 years, receiving insulin therapy. He
complaints of swelling and shortness of breath. On examination: glycemia - 7.1 mmol / l, cholesterol - 6.2
mmol / l, Creatinine - 0.21 mmol / l. What will you recommend the patient first of all?
A.
Increase the dose of insulin
B.
ACE inhibitors
C.
Statins
D.
Lipoic acid
E.
*Enterosorbents
171. Patient K., 43 years old, suffers from diabetes for 10 years, receives insulin therapy, complains of
burning pain in the foot, shin muscle cramps, numbness of toes. On examination: hypotension and muscle
atrophy, decreased reflexes, raised all kinds of sensitivity as "socks" and "gloves". What will you
recommend?
A.
Multivitamin complexes
B.
Gentle hot tub
C.
*Lipoic acid
D.
Unloading feet
E.
Muscle relaxants
172. Patient S., 48, suffers from diabetes for 8 years, receives glibenclamide 15 mg a day, complains of
burning pain in the foot, shin muscle cramps, numbness of toes. On examination: hypotension and muscle
atrophy, decreased reflexes, raised all kinds of sensitivity as "socks" and "gloves". What will you
recommend?
A.
B vitamins
B.
gabapentin
C.
lipoic acid
D.
pregabalin
E.
*all of the above
173. Patient R., 22 years old, complains of blurred vision. He suffers from diabetes for 14 years. In
anamnesis -- frequent ketosis. On examination: blood pressure 150/95 mm Hg, pitting edema. Urinalysis:
specific gravity 1.009, sugar - 0.5, protein - 0,66 g / l. Blood sugar fasting 7.8 mmol / l. Ophthalmoscopy:
hemorrhages multiple point, degenerative foci, enlarged veins. What is your diagnosis?
A.
diabetes mellitus 1 type, moderate, stage of subcompensation. Diabetic nephropathy III. Diabetic
retinopathy nonproliferative.
B.
diabetes mellitus 1 type, moderate, stage of subcompensation. Diabetic nephropathy IVst. Diabetic
retinopathy nonproliferative.
C.
*diabetes mellitus 1 type, severe, stage of subcompensation. Diabetic nephropathy IVst. Diabetic
preproliferative retinopathy.
D.
diabetes mellitus 1 type, severe, stage of subcompensation. Diabetic nephropathy Vst. Diabetic
retinopathy nonproliferative.
E.
diabetes mellitus 1 type, severe, stage of decompensation. Diabetic nephropathy IVst. Diabetic
preproliferative retinopathy.
174. Patient L., 27 years old complains of dizziness. He suffers from diabetes for 15 years. In anamnesis –
a history of sometimes loss of consciousness at the rapid change in body position. On examination: pulse
114/min., rhythmic, BP 150/95 mm Hg in the supine position and 100/60 in the sitting position. ECG - sinus
tachycardia. What is your diagnosis?
A.
*Diabetic autonomic cardiopathy
B.
Diabetic myocardiodystrophy
C.
Dysmetabolic cardiomyopathy
D.
Coronary heart disease
E.
Vegetative - vascular dystonia
175. Patient L., 57 years, complains of dizziness. He is suffering from diabetes for 18 years. In anamnesis
– a history of sometimes loss of consciousness at the rapid change in body position. On examination: pulse
104/min., rhythmic, BP 160/95 mmHg in the supine position and 90/60 in the sitting position. ECG - sinus
tachycardia. Preparations which groups need not assign this patient?
A.
lipoic acid
B.
b-blockers
C.
ACE inhibitors
D.
methabolic drugs
E.
*cardiac glycosides
176. Patient B., 49 years, height 171 cm, weight 91 kg. Diabetes mellitus has been diagnosed 11 years ago.
He is receives metformin. Glucose 7.2 mmol / l, blood creatinine - 0.082 mmol / l, cholesterol - 7.8 mmol / l.
He has hypertension during 7 years, is suffering from exertional angina for 3 years. He is suffering from pain
in the calf muscles when he is rapidly walking. Pain pass when the patient stops. What the complications of
diabetes are prevalent in the patient?
A.
peripheral polyneuropathy
B.
autonomic cardiopathy
C.
microangiopathy
D.
*multiple macroangiopathy
E.
no complications due to diabetes
177. One worker in the surgical center is found to be culture-positive for streptococcal microorganisms in
his nasal mucosa but has no symptoms of infection. What is this patient's status in the chain of infection?
A.
*Reservoir
B.
Portal of entry
C.
Portal of exit
D.
Susceptible host
E.
Non of above
178. 20-years-old man suffers from bronchial asthma, the attacks of dyspnea arise 3-4 times a day. Night
attacks are present 2 times a week. FEV1 - 70 %, its variability during day is 30 %. What is your diagnosis?
A.
*Moderate persistent BA
B.
Status asthmaticus
C.
Intermittent BA
D.
Severe persistent BA
E.
Mild persistent BA
179. 47-year-old man is evaluated because of cough that has persisted for 6 months. He has no postnasal
drip, wheeze, or heartburn. His physical examination, chest radiograph, and spirometry are normal. He
receives no benefit from a 3-month trial of twice-daily proton-pump inhibitors, intranasal corticosteroids,
and antihistamines. He has a family history of allergies. Which of the following would suggest the diagnosis
of this patient?
A.
*Methacholine challenge testing
B.
24-hour esophageal pH monitoring
C.
CT scan of the sinuses
D.
Bronchoscopy
E.
CT scan of the chest
180. 60-yr-old male non-smoker demonstrates such results of his pulmonary function tests: FEV1—1.4 l,
FVC—3.5 l, FEV1/FVC—40%. After bronchodilator trial—FEV1/FVC—59%. After 2 weeks of
prednisolone 30 mg daily FEV1/FVC = 72%. What is the diagnosis?
A.
*Bronchial asthma
B.
COPD
C.
Emphysema
D.
Tracheal compression
E.
Pulmonary fibrosis
181. A 10-yr-old boy presents with wheezing attacks and episodic shortness of breath. His PEFR is 400
l/min. What is the best method of treatment?
A.
*β2 agonist
B.
Erythromycin
C.
Plasmaphoresis
D.
Cyclophosphamide
E.
Co-trimoxazole
182. A 17-year-old previously healthy man presents with a history of shortness of breath on exertion,
particularly during basketball season, when he sometimes needs to sit down during practice to catch his
breath. He does not notice any shortness of breath with routine activity. There is no family history of asthma.
On physical examination, he is in no respiratory distress. His lungs are clear, with no wheezing during either
tidal breathing or forced expiration. His heart is normal. Baseline spirometry is normal. What is the next
diagnostic step?
A.
*Exercise testing with postexercise spirometry
B.
Allergy testing
C.
Methacholine challenge testing
D.
Overnight oximetry
E.
Repeat lung volumes and diffusing capacity
183. A 20-yr-old men presents with wheezing and shortness of breath. His PEFR is 400 l/min.
Spirography: FEV1, FVC and Typhno index are low. What is the diagnosis?
A.
*Asthma
B.
COPD
C.
Emphysema
D.
Pulmonary embolism
E.
Pneumonia
184. A 20-yr-old woman is too breathless to speak. Her pulse is 120/min, respiratory rate is 30/min and
PEFR is 100 l/min. Examination reveals a very quiet chest and CXR is normal. Choose the single most likely
management:
A.
*Nebulised salbutamol
B.
Pleural aspiration
C.
Heparin
D.
Intramuscular adrenaline
E.
Rapid infusion of saline
185. A 25-year-old man is evaluated because of a 3-month history of episodic dyspnea at rest. The
episodes occur approximately three times per week and are accompanied by cough. His symptoms awaken
him at night approximately three times per month. He had asthma as a child, which resolved. His
temperature is 36.5 C, pulse rate 85/min and regular, respiration rate 14/min, and blood pressure 125/75 mm
Hg. The only abnormality noted on physical examination is bilateral wheezing without crackles. Chest
radiograph is normal. Spirometry shows a forced expiratory volume in 1 sec (FEV1) 78% of predicted, and a
forced vital capacity 93% of predicted. He has a 17% (430 ml) improvement in FEV1 after using
bronchodilators. Which of the following is the most appropriate treatment regimen for this patient?
A.
*Albuterol and a low-dose inhaled corticosteroid
B.
Albuterol
C.
Albuterol and a long-acting β2-agonist
D.
A long-acting β2-agonist
186. A 28 years old patient, complaints on cough with small amount of colourless sputum, pain in the right
half of thorax during breathing, shortness of breath, increase of temperature to 39 °С. Felt ill rapidly. Used
aspirin. Objectively: herpes on lips. In lower lobe of right lung there is dull percussion sound, bronchial
breathing. X-ray: there is homogeneous infiltration of right lower lobe. What is the most possible etiology of
pneumonia?
A.
*Pneumococcus
B.
Staphylococcus
C.
Mycoplasma
D.
Legionella
E.
Klebsiella
187. A 34-year-old woman is evaluated because of a 1-year history of increased dyspnea on exertion. She
has no symptoms at rest but has to stop about 15 minutes into her aerobics class because of dyspnea and
occasional cough. She usually recovers fully in about an hour. One year ago she was able to do aerobics for
45 minutes without difficulty. Her vital signs are normal, and her physical examination is normal, including
clear breath sounds. Baseline spirometry is also normal. Which of the following would be best next step in
the management of this patient?
A.
*Inhaled albuterol prior to exercise
B.
Oral leukotriene inhibitors
C.
Long-acting theophylline
D.
Inhaled ipratropium bromide prior to exercise
E.
Inhaled corticosteroids
188. A 37-year-old man with asthma is evaluated because he continues to have frequent attacks and now
feels his short-acting β2-agonist is not providing relief. He states he is using his medications, including a
long-acting β2-agonist inhaler, inhaled high-dose corticosteroids, and a short-acting β2-agonist inhaler as
rescue medication. He has symptoms daily and nocturnal symptoms about twice per week. On physical
examination, he is in mild respiratory distress. He is afebrile. Pulse rate is 90/min and regular, respiration
rate is 18/min, and blood pressure is 140/85 mm Hg. He has bilateral wheezing. Spirometry shows a forced
expiratory volume in 1 sec (FEV1) 65% of predicted; it improves with bronchodilators to 85% of predicted.
He has no history of recent viral upper respiratory infections or rhinitis or symptoms of gastroesophageal
reflux disease. Which of the following is the best next step in this patient’s management?
A.
*Observe the patient using the metered-dose inhaler
B.
Add a leukotriene inhibitor
C.
Switch to an oral β2-agonist and have the patient return for a pill count
D.
Initiate oral prednisone therapy and have the patient return for a pill count
E.
Have the patient return with a symptom and treatment log.
189. A 38 years old patient, who drunk a lot of alcohol, has severe pneumonia. His condition got worse,
the temperature of body rose to 39-40 °С, an unpleasant smell appeared from a mouth, increased amount of
purulent sputum; increased ESR and amount of band leucocytes. On the X-ray - in the lower lobe of right
lung there is massive infiltration with light area in a center. What complication is it necessary to suspect?
A.
*Acute pulmonary abscess
B.
Bronchiectasis
C.
Infarction-pneumonia
D.
Gangrene of lungs
E.
Empyema of pleura
190. A 40-year-old woman, a nurse, is evaluated because of worsening asthma symptoms. She has had
mild, intermittent asthma since college, for which she has been using an albuterol inhaler as needed, usually
less than once a month. During the past 3 months, she has experienced cough, tightness of the chest, and
wheezing, which improve after the use of inhaled albuterol. She uses the inhaler twice a day on average and
has awakened at least twice a week with nocturnal cough. She works three consecutive 12-hour day shifts,
and the cough is regularly worse at the end of each shift. During her days off, she has fewer asthma
symptoms and feels significantly better by the time she returns to work. She has a history of allergic rhinitis
that has also recently become more symptomatic. Approximately 6 months ago, she acquired a kitten that
sleeps in the bedroom. She has lived in her home for 6 years, and it is carpeted and has heavy draperies.
Chest examination is notable for good air entry. There are scattered end-expiratory wheezes. In addition to
treatment with inhaled corticosteroids, which of the following interventions is most likely to benefit this
patient?
A.
*Avoiding exposure to latex products
B.
Treatment with an oral antihistamine
C.
Getting rid of the kitten
D.
Removing the carpets and draperies from her home
E.
Treatment with a leukotriene-modifying drug
191. A 46-year-old woman who works as a nurse is evaluated because of a 2-year history of episodic
wheezing and a squeaky voice. This past spring, her symptoms worsened, requiring her to seek medical
attention; she was placed on a short-acting β2-agonist that did not provide much relief. She has no history of
wheezing and says that these changes began after a severe influenza infection 3 years ago. Currently she
feels well and has had no symptoms for several months; she is not taking any medications. Physical
examination shows no abnormalities, and baseline spirometry is normal. What is the best test to evaluate this
patient’s condition?
A.
*Methacholine challenge testing
B.
Bronchoscopy to evaluate her trachea
C.
Exercise echocardiogram
D.
CT scan of the sinuses
E.
Non of above
192. A 48 years old patient, complaints on weakness, dyspnea, pain in the left half of thorax, permanent
cough with viscid sputum, in which particles of blood are sometimes determined. For the last 3 months lost 5
kg of body mass. On the X-ray of lungs there is total homogeneous shade determined from the left side.
Organs of mediastinum are displaced to the left. What diagnosis is possible?
A.
*Lung athelectasis
B.
Lung gangrene
C.
Total exudative pleurisy
D.
Pneumonia
E.
Empyema of pleura
193. A 53-year-old woman with a history of mild persistent asthma is evaluated because of a recent
increase in her symptoms, with dyspnea and cough occurring daily and a cough that awakens her once a
week. She is currently using low-dose inhaled corticosteroids. She has no symptoms of rhinitis or
gastroesophageal reflux. On physical examination, she has intermittent wheezing bilaterally. Which of the
following is the most appropriate change in her therapy?
A.
*Add a long-acting β2-agonist
B.
Initiate azithromycin therapy
C.
Add a nebulized short-acting β2-agonist
D.
Add inhaled ipratropium bromide
E.
Add a leukotriene inhibitor
194. A 55-year-old man is evaluated in the emergency department because of an acute, severe asthma
attack; he is hospitalized in the intensive care unit for aggressive medical therapy and monitoring. He is
expectorating thick greenish sputum. His medical history includes hypertension, cholecystectomy and
glaucoma. Chest radiograph reveals hyperinflation only. Medical therapy in the emergency department
included repeated doses of aerosolized albuterol and ipratropium, as well as methylprednisolone, 125mg
administered intravenously. Peak expiratory flow rate is unimproved at 80 l/min. Which of the following is
the most appropriate next step in this patient’s management?
A.
*Intravenous magnesium sulfate
B.
Nebulized ipratropium bromide administered by face mask
C.
Broad-spectrum antibiotics targeting community-acquired respiratory pathogens
D.
Inhaled corticosteroids
195. A 57-year-old man with severe persistent asthma is evaluated on routine follow-up. He states that his
asthma has been under good control for the last 3 months on high-dose inhaled corticosteroids and a longacting β2-agonist. He uses a short-acting β2-agonist only three times per week as a rescue medication, and
he has nocturnal symptoms very rarely. Peak expiratory flows have been stable. His physical examination is
normal, including clear breath sounds, and spirometry is normal. It is decided that he will keep using the
short-acting β2-agonist as a rescue medication. Which of the following is the best next step in this patient’s
management?
A.
*Continue the long-acting β2-agonist and decrease the dose of inhaled corticosteroid
B.
Stop the long-acting β2-agonist and decrease the dose of inhaled corticosteroid
C.
Continue current therapy and have the patient return in 6 months
D.
Stop the long-acting β2-agonist and maintain the dose of inhaled corticosteroid
196. A patient who is 2 days postoperative from a bowel resection tells her physician that she is having a
hard time “catching her breath,” feels nauseated, and has chest pains when she inhales. The physician
suspects that she is having a pulmonary embolism. What intervention should the physician perform before
notifying the physician?
A.
Increase the IV flow rate
B.
*Apply oxygen by mask or nasal cannula at 5 l/min
C.
Assess the chest and axillary area for the presence of petechiae
D.
Place the patient in shock position, with her head and neck flat and her legs elevated
E.
Non of above
197. A young woman complains of wheeze, dyspnoea and cough. She cannot sleep at night because of a
chronic cough. She and her mother love animals and together they have 14 cats. Her PEFR is normal but her
CXR suggests hyperinflation. What is the previous diagnosis?
A.
*Bronchial asthma
B.
Bronchogenic carcinoma
C.
Emphysema
D.
Respiratory failure
E.
Bronchitis
198. After emotional exertion patient of 24 y.o. developed condition with dyspnea and prolonged
expiration, distantional wheezes, frequent night symptoms. All that limited his physical activity. PEV and
FEV1 < 60 %, daily variability -30 %. What diagnosis is possible?
A.
*Severe persistent bronchial asthma
B.
Mild persistent bronchial asthma
C.
Persistent bronchial asthma of moderate severity
D.
Intermittent bronchial asthma
E.
Bronchospastic syndrome of allergic origin
199. Drug abuser, a 41-yr-old man, presents with fever, cough and breathlessness. This was preceded by
viral influenza. Chest radiograph shows multiple abscesses. What is the most possible etiology of disease?
A.
*Staphylococcus aureus
B.
Cryptococcus
C.
Streptococcus pneumoniae
D.
Legionella pneumonia
E.
Mycobacterium avium
200. Female B., 44 years old, complains on cough with mucous sputum, increase of temperature to 39 °С,
weakness, dyspnea, sweating. Breathing rate - 26/min, skin is moist. Below left scapula there is shortening of
percussion sound. Breathing during auscultation is weakened, moist rales. Blood test: L - 11х109/l, ESR - 29
mm/h. Your previous diagnosis?
A.
*Left-side lower lobe pneumonia
B.
Gangrene of lungs
C.
Left-side exudative pleurisy
D.
Cancer of left side lower lobe
E.
Pulmonary abscess
201. Female patient K., 46 years old, after decreasing of fever after flue noticed pain appeared in a thorax,
cough with yellow-green sputum (amount-150 ml a day), sometimes with some blood. Objectively:
breathing rate - 36/min. In lungs from the right side lower scapula there is dull sound during percussion, hard
breathing, and moist rales. Blood test: L - 18,6х109/l, ESR -64 mm/h. Analysis of sputum: L -80-100 , Er 40-50, elastic fibers, cocci. X-ray: lung roots are enlarged, from the right side lower lobe is heterogeneously
infiltrated with two lighter areas. What is the most possible previous diagnosis?
A.
*Right-side pneumonia with abscesses
B.
Peripheral cancer
C.
Infiltrative tuberculosis in the phase of disintegration
D.
Exudative pleurisy
E.
Infarction-pneumonia
202. Female, 34 years old, has an increase of body temperature to 38 °С, cough with purulent sputum,
weakness, dyspnea, pain in a thorax during breathing. During percussion there is shortening of sound in the
lower part of left lung, during auscultation – moist rales. What method of investigation is the decisive one to
confirm diagnosis?
A.
*X-ray examination
B.
Bacteriological analysis of sputum
C.
Spirometry
D.
Pneumotachometry
E.
Bronchography
203. Girl, 18 y.o., pets seller, complaints mainly during working time on the attacks of dry cough, feeling
of running nose. She often ills with viral respiratory infections. Her mother is ill with bronchial asthma.
Objectively: breathing rate - 18/min. Heart rate - 80/min, BP - 110/70. In lungs vesicular breathing, dry
wheezes are heard in distance. Tones of heart are weaker than normally. Test with berotec showed
reversibility of bronchial obstruction. What tactic will be the best for the patient?
A.
*To change job
B.
To use intal
C.
To use monteleucast
D.
To use berotec constantly
E.
To use antihystaminic preparations
204. Girl, 23 y.o., for 2 years is ill with bronchial asthma. Recently attacks of dyspnea became more
frequent and started to arise 4-5 times a week, night attacks - 2-3 times a month. She used salbutamol to
remove the symptoms. Test with the antigen of home dust is positive. Objectively: condition is satisfactory.
Breathing rate - 20/min. Heart rate - 76/min, BP -120/80. In lungs breathing is vesicular. Tones of heart are a
little weak, rhythm is normal. What mechanism is desicive in development of bronchial obstruction in this
case?
A.
*Hyperreactivity of bronchi
B.
Тrachео-bronchial dyskinesia
C.
Violation of metabolism of arachidonic acid
D.
Adrenergic disorders
E.
Activity of the parasympathetic nervous system is increased
205. Male patient F., 48 years old, during a week stayed at home with diagnosis of respiratory viral
infection. Doctor noticed complaints on cough with small amount of mucus-purulent sputum, weakness.
Objectively: condition is relatively satisfactory. T - 37,2 °С. Breathing rate - 18/min., pulse - 80/min., BP 110/70. In lungs there is vesicular breathing, with a hard tint, single dry wheezes. Tones of heart are a little
dull, rhythm is correct. What is the treatment tactic?
A.
*To prescribe antibacterial therapy
B.
To stay at home for some more days
C.
To go to work
D.
To send patient to pulmonologist
E.
To hospitalize patient to the pulmonological department
206. Male patient G., 56 years old, complaints on permanent pain in a thorax which disturbs for last 2
months. Pain is not connected with breathing. There is also cough with particles of blood in sputum.
Weakness, fatigue are present. On the chest X-ray in the lower lobe of right lung there is spherical shadow,
size 4x6 cm, related to the lungs rhadicis. What is the most possible diagnosis?
A.
*Perypheral lung cancer
B.
Tuberculoma
C.
Metastasis
D.
Pulmonary abscess
E.
Pneumonia
207. Man, 32 y.o., complaints on attack of expiratory dyspnea, which lasts for 48 hours, cough with small
amount of sputum. He is ill with bronchial asthma for 5 years, was treated with glucocorticosteroids, used
inhalers. Objectively: condition is severe, patient sits. Diffuse cyanosis, pulse -110/min, BP - 110/70. Tones
of heart are weak, II tone is louder above the pulmonary artery. During percussion in lungs there is “bangbox” sound, large amount of dry wheezes. In blood there is eosinophylia - 18 %. What medicines are drugs
of choice for this patient?
A.
*Corticosteroids
B.
β2-agonists
C.
Theophyllin
D.
Cholynolytics
E.
Antihystamines
208. Man, 39 y.o., 8 last years is ill with bronchial asthma. Rapidly during physical work he felt
worsening of breathing, cough, distance wheezes appeared and dyspnea began to increase. Medicine of what
pharmacological group is it better to recommend for the patient to remove such attacks of dyspnea?
A.
*Agonists of β2-adrenoreceptors
B.
Metilxantines
C.
β2-adrenoblockers
D.
Inhalated glucocorticoids
E.
Oral glucocorticoids
209. Man, 46 y.o., suffers for the last 10 years from bronchial asthma. Rapidly during physical work he
felt worsening of breathing, cough, distance wheezes appeared and dyspnea began to increase. Medicine of
what pharmacological group is it better to recommend for the patient to remove such attacks of dyspnea?
A.
*Agonists of β2-adrenoreceptors
B.
Atropine
C.
Intal
D.
Epinephrine
E.
Monteleucast
210. Man, 43 y.o., complaints on dyspnea during physical activity. Objectively: temperature 36,4 °С,
breathing rate - 20/min, pulse - 78/min, BP-125/80. Emphysematous form of thorax. In lungs – weak
vesicular breathing. What test should be passed by patient at home to decide question about efficiency of
prescribed broncholytics?
A.
*Peakflowmetry
B.
Spirography
C.
ECG-control of overload of right chambers of heart
D.
Bronchoscopy
E.
Analysis of sputum (amount and microscopy)
211. Patient, 28 y.o., has running nose, attacks of dyspnea at night once a week. Felt ill after viral
respiratory infection which was treated with acetilsalicylic acid. Eosynophylia was founded in blood and
sputum. What disease may be suspected?
A.
*Aspirin bronchial asthma
B.
Eosinophylic infiltrate of lungs
C.
Bronchial asthma of physical exertion
D.
Allergic rhinitis
E.
Bronchial asthma, exogenous form
212. Patient, 30 y.o., after a viral infection has daily symptoms of dyspnea, which causes lowering of
activity and bad sleep; night symptoms are more frequent then once a week. PEV and FEV1 - 60-80 %, day
variability > 30 %. There is a necessity of daily usage of β2-agonists of short action. What is the diagnosis?
A.
*Persistent bronchial asthma of moderate severity
B.
Mild persistent bronchial asthma
C.
Intermittent bronchial asthma
D.
Severe persistent bronchial asthma
E.
Status asthmaticus
213. Patient, 42 years old, complaints on attacks of dyspnea, every time uses 1-2 doses of salbutamol.
These attacks are accompanied with cough and minimal amount of viscid glassy sputum. He is ill for 8
years. Objectively: temperature - 36,7 C; breathing rate – 21/min.; pulse-90/min.; BP - 130/80; FEV1 - 77 %.
In lungs – solitary dry wheezes. Blood test: eosinophyles - 6 %. What medicines are “basic” in the treatment
of this patient?
A.
*Antiinflammatory
B.
Cholynolytics
C.
Mucolytics
D.
Antihystaminic
E.
β2-agonists
214. Patient, 44 y.o., complaints on attack of dyspnea, which arises suddenly at night. Connects this attack
with overcooling. He is ill for more than 10 years. Thorax of emphysematous form. During percussion in
lungs – “bang-box” sound. During auscultation there is plenty of dry wheezes. In blood: moderate
leucocytosis, eosinophylia - 10 %. On the chest X-ray film – increased pneumatization of pulmonary tissue.
What diagnosis is the most possible?
A.
*Bronchial asthma, exacerbation phase
B.
Bronchiectasis, exacerbation phase
C.
COPD, exacerbation phase
D.
Chronic bronchitis
E.
Eosinophylic pulmonary vasculitis
215. Patient J., 45 y.o., complaints on dyspnea during insignificant physical exertion, cough with minimal
amount of “glass-like” sputum, attacks of dyspnea up to 3 times a day, more often at night, sweating. She is
ill for more than 5 years. Has an allergy on dust, cockroaches. For treatment uses bekotid for near the year.
Diagnosis?
A.
*Bronchial asthma
B.
Eosinophylic pulmonary infiltrate
C.
COPD
D.
Bronchiectasis with bronchial spasm
E.
Pulmonary vasculitis (syndrome of Charg - Stross)
216. Patient G., 47 y.o., with long history of bronchial asthma, has developed more frequent attacks of
dyspnea. Inhalations of astmopent and berotec are not effective. From prescription of what medicine is it
better to begin the intensive treatment?
A.
*Glucocorticoids
B.
Oxygen therapy
C.
Bronchodylators
D.
Infusion therapy
E.
Heart glycosides
217. Patient Y.,49 y.o., complaints on dyspnea, cough. Sputum is absent. Used many puffs of salbutamol,
intal, but without any efficacy. Objectively: sits, leaning on a table. Total cyanosis of the body. Peripheral
edema is absent. Breathing is superficial, dyspnea, during auscultation breathing cannot be heard in some
areas of lungs; wheezes are diffuse, expiration is considerably prolonged. Tones of heart are weak,
tachycardia. Pulse - 112/min, BP - 110/70. Liver is near the edge of costal arch. What is the previous
diagnosis?
A.
*Status asthmaticus
B.
Bronchial asthma of moderate severity
C.
COPD
D.
Aspiration of foreign body
E.
Heart asthma
218. Patient A., 35 y.o., noticed infrequent (rarer than 1 time a week) attacks of dyspnea, which are easily
removed with inhalations of β2-agonists of short action. During attack in lungs are heard dry wheezes,
between attacks FEV1 is more than 80 % from normal. What is the diagnosis?
A.
*Intermittent bronchial asthma
B.
Persistent bronchial asthma of moderate severity
C.
Mild persistent bronchial asthma
D.
Severe persistent bronchial asthma
E.
Given information is not enough for determination of severity of bronchial asthma
219. Patient B., 25 years old engineer, appeared during a fire in the area of high concentration of CO (an
industrial accident). In hospital delivered in the irresponsible state. What laboratory tests are the early criteria
of estimation of severety of the state?
A.
Estimation of blood viscosity
B.
Anemia
C.
Leucocytosis
D.
*Carboxihemoglobinemia
E.
Methemoglobinemia
220. Patient complaints on attacks of dyspnea, which arise 1-2 times a week, night symptoms - 2 times a
month and even more frequently. For a patient night sleep is violated as a result of attacks of dyspnea. FEV1
> 80 % from normal. What diagnosis would you suspect?
A.
*Mild persistent BA
B.
Severe persistent BA
C.
Intermittent BA
D.
Moderate persistent BA
E.
Status asthmaticus
221. Patient E., 43 years old, worker of coal mine, complaints on expiratory dyspnea, cough with dark
sputum. On roentgenogram there are the linear-reticulated diffuse changes. Doctor thinks about anthracosis.
Which characteristic is peculiar for the coal dust pneumoconiosis?
A.
*Raised activity of phagocytosis
B.
Stimulation of carcinogenesis
C.
To cause the unspecific allergic reactions
D.
To cause the considerable mechanical irritation
E.
To form colloid precipitates
222. Patient F., 46 years old, was hospitalized urgently with acute attack of dyspnea. Last 5 years he has
been working on poultry farm. During examination bronchial asthma was diagnosed. What additional
diagnostic methods are necessary to confirm the professional genesis of asthma?
A.
sanitary-hygienic characteristics of the work conditions
B.
echocardioscopy
C.
*allergic and immunological tests
D.
investigation of the function of external breath
E.
roentgenography of pulmonary system
223. Patient G., 36 years old, works on a poultry factory. She was emergently hospitalized with acute
attack of dyspnea. During observation bronchial asthma was diagnosed. What additional methods of research
must be conducted above all things to confirm the professional genesis of bronchial asthma?
A.
roentgenologic research of breathing organs
B.
professional route of patient
C.
sanitary-hygienic characteristic of work conditions
D.
research of external breathing function
E.
*allergic and immunological tests
224. Patient H., a 52 years old man, is evaluated because of a 2-month history of nonproductive cough,
myalgias, and low-grade fever. When his illness began, a chest radiograph showed bilateral alveolar
infiltrates, and a presumptive diagnosis of community-acquired pneumonia was made. He was treated with
oral azithromycin without effect, followed by a 10-day course of levofloxacin, also without effect. During
the course of his illness he has lost 4.5 kg without significant anorexia. He is a lifetime nonsmoker and
works as an office manager. He has no pets and no unusual hobbies. On physical examination, his vital signs
are normal, except of respiration rate of 22/min. He is in mild respiratory distress on exertion. On
examination of the chest, bilateral crackles are audible, without wheezing. Chest radiograph shows bilateral
alveolar infiltrates, which are changed in location from those seen on his original radiographs. Pulmonary
function tests show forced expiratory volume in 1 sec (FEV1) 75% of predicted, forced vital capacity (FVC)
72% of predicted, total lung capacity 80% of predicted, and diffusing lung capacity for carbon monoxide
65% of predicted. Arterial blood gas values, with the patient breathing room air, are PO2 62 mm Hg, PCO2
42 mm Hg, and pH 7.39.Which of the following is the most likely diagnosis?
A.
*Cryptogenic organizing pneumonitis
B.
Hypersensitivity pneumonitis
C.
Resistant pneumococcal pneumonia
D.
Chlamydia pneumonia
E.
Bronchoalveolar cell carcinoma
225. Patient has severe attack of bronchial asthma which lasts more than 1 hour. Usage of beta-agonists in
inhalation, euphylline intravenously and cholynolytics was not effective. What medicines are necessary for
emergency therapy?
A.
*Glucocorticosteroids intravenously
B.
Beta-agonists intravenously
C.
Inhaled glucocorticosteroids
D.
Antihystaminic
E.
Nonsteroid anti-inflammatory medicines
226. Patient I., a 50 years old man, is evaluated in the emergency department because of fever,
nonproductive cough and 2-day history of myalgia and headache. He has also had nausea and diarrhea. He is
a heavy smoker. On physical examination, he is slightly disoriented. Temperature is 38.9 C, pulse rate is
110/min, respiration rate is 32/min. Chest radiograph shows fluffy infiltrates to the right upper and lower
lobes. Results of laboratory testing show serum sodium of 128 meq/L, blood urea nitrogen of 42 mg/dL,
serum creatinine of 2.2 mg/dL, and serum creatine kinase of 250 U/L. Which one of the following is best
next step in the management of this patient’s pneumonia?
A.
*Initiate empiric antibiotic therapy for Legionella
B.
Order direct fluorescent antibody testing of the sputum for Legionella
C.
Order serologic testing for Legionella
D.
Send a urine specimen for measurement of Legionella antigen
E.
All of the above
227. Patient J., 36 y.o., complains for fever (39 C), pain in the left part of the chest. Pleuropneumonia was
diagnosed in the patient. What onset is typical for pleuropneumonia?
A.
*Acute
B.
Latent
C.
Fulminant
D.
Gradual
E.
Non of the above
228. Patient K., 27 y.o., complains for dry cough, hoarseness, general weakness, sweating, increase of
body temperature up to 37,5 С. Data of physical examination: vocal fremitus is not changed, resonant
pulmonary sound is heard above the lungs. Preliminary diagnosis?
A.
Acute purulent bronchitis
B.
*Acute catarrhal bronchitis
C.
Pneumonia
D.
Pulmonary emphysema
E.
Pleural empyema
229. Patient K., 37 years old, worker of the factory, during a fire appeared in the area of high
concentration of CO. Into the clinic he was delivered without consciousness. In a blood test: Er - 4,5 x
1012/l, НЬ - 136 g/l, Le - 17,2 x 109/l, Eos - 0 %, band neutrophils - 5 %, segments - 65 %, lymphocytes 22 %, monocytes - 3 %, ESR - 3 mm/h, carboxyhaemoglobin in blood - 52 %. What criterion is the most
important for determination the severity degree of the patient’s state?
A.
Results of ECG and spirography
B.
Presence of violations of breathing
C.
*Duration of unconsciousness
D.
Prevalence of trophic violations
E.
Development of vascular violations
230. Patient M., 30 years old, during last 3 years works as a nurse in manipulations cabinet. Last year
during the contact with penicilline she started tp complain on discomfort in throat, sneezing, attack of cough
and dyspnea which disappear after inhalation of salbutamol. During last months attacks of dyspnea became
more severe and occurred only at contact with penicilline. During the life she had not any diseases including
allergic. She hadn’t received antibiotics. Can we consider the bronchial asthma is professional in this
patient?
A.
no, we can’t
B.
yes, we can if we have conclusion about attacks of bronchial asthma
C.
yes, we can if we have conclusion about appearance of bronchial asthma attacks after contact with
penicilline
D.
yes, we can
E.
*yes, we can, if allergic and immunological tests are positive
231. Patient M., 39 years old, complains on attacks of cough with yellow-brown sputum, pain in a right
side, related to the deep breathing, sweating. He is ill for 6 days, after overcooling. Used aspirin.
Objectively: T - 39,6 °С, breathing rate - 26/min, pulse - 110/min, BP -110/70. In lower part of right lung moist loud rales. X-ray: in right lower lobe there is massive unhomogeneous infiltration with lighter areas,
sinus is not changed. What complication of disease is the most possible?
A.
*Abscesses
B.
Dry pleurisy
C.
Empyema of pleura
D.
Spontaneous pneumothorax
E.
Pulmonary athelectasis
232. Patient N., 45 y.o., complains for fever (38 C), sweating, dry caugh and general weakness.
Bronchopneumonia was diagnosed in the patient. What onset is typical for bronchopneumonia?
A.
Acute
B.
Latent
C.
Fulminant
D.
*Gradual
E.
Non of the above
233. Patient O., 29-yr-old male prostitute, has felt generally unwell for 2 months with some weight loss.
Over the last 3 weeks he has noticed a dry cough with increasing breathlessness. Two courses of antibiotics
from the GP have produced no improvement. The CXR shows bilateral interstitial infiltrates. What is the
most possible etiology of disease?
A.
*Pneumocystis carinii
B.
Streptococcus pneumoniae
C.
Mycoplasma pneumoniae
D.
Fungi
E.
Legionella pneumoniae
234. Patient of 23, during viral respiratory infection used 1 gram of aspirin, after that he received an attack
of severe dyspnea with prolonged expiration, prescription of euphylline was necessary. There were no
allergic diseases in his history. He had two operations for the treatment of nasal poliposis. What is your
diagnosis?
A.
*Aspirin asthma
B.
Symptomatic bronchial spasm
C.
Intermittent bronchial asthma
D.
Persistent bronchial asthma
E.
Asthma of physical exertion
235. Patient of 44, episodically in spring has dyspnea with worsening of expiration, wheezes in lungs.
Brief daily symptoms are rarer than once a week, night symptomes occur less than 2 times a month. PEV and
FEV1 - 80 %. Between exacerbations wheezes in lungs are absent. What is the possible diagnosis?
A.
*Intermittent bronchial asthma
B.
Easy persistent bronchial asthma
C.
Middle persistent bronchial asthma
D.
Severe persistent bronchial asthma
E.
COPD
236. Patient of 51, with 10-years history of bronchial asthma, develpos more frequent attacks of dyspnea
and inhalations of astmopent and berotec are not effective. From prescription of what medicine is it better to
begin intensive treatment?
A.
*Glucocorticoids
B.
Oxygen therapy
C.
Bronchodylators
D.
Infusion therapy
E.
Heart glycosides
237. Patient of 54, complaints on dyspnea during insignificant physical exertion, cough with minimal
amount of sputum. Objectively: diffuse cyanosis. Thorax of emphysematous form. In lungs breathing is
vesicular, weakened with prolonged expiration, dry wheezes are heard. BP -140/80. Pulse - 92/min,
rhythmic. Spirography: FVC – 72 %, FEV1/FVC - 50 %. What is the type of respiratory failure in this
patient?
A.
*Obstructive
B.
Mixed type with prevalence of obstruction
C.
Restrictive
D.
Mixed type with prevalence of restriction
E.
Respiratory insufficiency is absent
238. Patient P., a young male homosexual with Kaposi's sarcoma, complains of increasing breathlessness
and a dry cough. He has a 3-day history of shivering, general malaise and productive cough. The X-ray
shows right lower lobe consolidation. What is the most possible etiology of disease?
A.
*Pneumocystis carinii
B.
Mycoplasma tuberculosis
C.
Haemophilus influenzae
D.
Chlamydia trachomatis
E.
Klebsiella pneumoniae
239. Patient Q., 37 years old, was operated in the surgical department because of appendicitis. After 4
days appeared the recidive of chills, cough, dyspnea, fever 38,5 °С, returned leucocytosis with shift to the
left in leucocyte formula. On X-ray there is infiltration of lower right lobe. What is the diagnosis?
A.
*Nosocomial pneumonia
B.
Pulmonary abscess
C.
Infarction pneumonia
D.
Community-acquired pneumonia
E.
Tuberculosis
240. Patient R., 48 y.o., complains for sharp pain in the right part of his chest at deep breathing and cough.
Pain in the chest which relates to disease of respiratory system, is typical for
A.
Bronchiectatic disease
B.
Asthma
C.
Emphysema
D.
Exudative pleurisy
E.
*Dry pleurisy
241. Patient S., a 25-year-old male has just returned from holiday abroad, presents with flu-like illness,
headaches, high fever. Prior to this, he had complained of abdominal pain, vomiting, diarrhea associated
with blood per rectum. What is the most possible etiology of disease?
A.
*Legionella pneumoniae
B.
Streptococcus pneumoniae
C.
Mycoplasma pneumoniae
D.
Pneumocystis carinii
E.
Fungi
242. Patient V., a 24 years old barman, presents with a dry cough of sudden onset. He complains of a chest
pain and rusty sputum. He also has a very high fever, rapid breathing, cyanosis and crepitations. Pneumonia
was suspected. What is the most nesessary method of investigation?
A.
*Chest X-ray
B.
Spirography
C.
Analysis of sputum
D.
General blood analysis
E.
General urine analysis
243. Patient W., 62 y.o., suffers with morning cough with expectoration of large volume of greenish
sputum. Sputum is better expelled in a certain position of patient’s body. Such kind of cough is typical for
A.
*bronchiectatic disease
B.
asthma
C.
emphysema
D.
pneumonia
E.
pleurisy
244. Patient W., 67 years old, during the epidemic of influenza after decreasing of fever noticed pain that
appeared in a thorax, cough with yellow-green sputum (amount-100 ml a day), sometimes with some blood.
Objectively: breathing rate - 36/min. In lungs from the right side lower scapula there is dull sound during
percussion, hard breathing, and moist rales. Blood test: L - 18,6х109/l, ESR -64 mm/h. Analysis of sputum:
L -80-100 , Er - 40-50, elastic fibres, cocci. X-ray: rhadicis are enlarged, from the right side lower lobe is
heterogeneously infiltrated with two lighter areas. What is the most possible previous diagnosis?
A.
*Right-side pneumonia with abscesses
B.
Peripheral cancer
C.
Infiltrative tuberculosis in the phase of disintegration
D.
Exudative pleurisy
E.
Infarction-pneumonia
245. Patient X., 55 years old, was admitted to the hospital recently. He complaints on cough with very
small amount of mucous-purulent sputum, significant weakness, increased temperature, which is
accompanied with chill, dizziness. Objectively: t - 38°С. Breathing rate - 22/min. Heart rate - 90/min, BP 110/70. From the right side below scapula the vocal fremitus is increased, percussion sound is shortened,
vesicular breathing is weakened, small amount of moist rales. Tones of heart are dull, rhythm is correct,
moderate tachycardia. Doctor suspected pneumonia. The presence of what syndrome let to suspect such
diagnosis?
A.
*Pulmonary tissue infiltration
B.
Intoxication
C.
Inflammation
D.
Bronchial obstruction
E.
Respiratory insufficiency
246. Patient Z., a 33-yr-old car mechanic, is brought to casualty by his girlfriend. She describes a 2-day
history of rigors, sweats and intermittent confusion. On examination he is agitated, sweaty and pyrexial with
38.6° C. He is hyperventilating and cyanosed despite receiving O2 by face mask. There is dullness to
percussion and bronchial breathing at the left lung base. What method of investigation is necessary?
A.
*Chest X-ray
B.
Spiral CT with contrast
C.
Arterial blood gases
D.
Blood count and film
E.
Urea and electrolytes of blood
247. Previously healthy 28-year-old man is evaluated in the emergency department because of fever,
productive cough, and shortness of breath. His temperature is 40 C, pulse is 120/min, respiration rate is
32/min, and blood pressure is 100/70 mm Hg. Measurement of arterial blood gases with the patient breathing
room air shows PO2 of 55 mm Hg, PCO2 of 30 mm Hg, pH of 7.41. Chest radiograph reveals bilateral
alveolar infiltrates with no effusions. Gram stain of the sputum reveals gram-positive diplococci. Which of
the following is the most appropriate for this patient?
A.
*Hospitalize him
B.
Treat him as an outpatient with oral therapy
C.
Treat him as an outpatient with intravenous therapy
D.
Hospitalize the patient in the intensive care unit
E.
All of the above
248. Previously healthy 32-yr-old woman presents with general malaise, severe cough and breathlessness,
which has not improved with a 7 day-long course of Amoxycillin. There is nothing significant to find on
examination. The X-ray shows patchy shadowing throughout the lung fields. The blood film shows clumping
of red cells with suggestion of cold agglutinins. What is the most possible etiology of disease?
A.
*Mycoplasma pneumoniae
B.
Legionella pneumonia
C.
Haemophilus influenzae
D.
Chlamydia trachomatis
E.
Klebsiella pneumoniae
249. The 60-year-old patient tells you that she smoked three packs of cigarettes per day since she was 15
years old until she was 40, and then smoked two packs per day. How many pack-years should you record in
the patient's history?
A.
45
B.
80
C.
90
D.
*115
E.
Non of above
250. The 82-year-old patient has a pulmonary infection. Which action addresses the age-related change of
increased vascular resistance to blood flow through pulmonary vasculature in this patient?
A.
Encouraging the patient to turn, cough, and deep breathe every hour
B.
*Assessing the patient's level of consciousness
C.
Raising the head of the bed
D.
Humidifying the oxygen
E.
Non of above
251. The patient has broken ribs that penetrated through the skin as a result of a motor vehicle crash 3 days
ago. The patient now complains of increased pain, shortness of breath, and fever. Which assessment finding
alerts the physician to the possibility of a pleural effusion and empyema?
A.
Wheezing on exhalation on the side with the broken ribs
B.
*Absence of fremitus at and below the site of injury
C.
Crepitus of the skin around the site of injury
D.
Absence of gastric motility
E.
Non of above
252. The patient has severe nasal congestion, headache, and sneezing but no rhinorrhea, watery eyes, sore
throat, or fever. Which statement made by the patient alerts the physician to the possibility of rhinitis
medicamentosa?
A.
“I have been taking two aspirins every 6 hours for this headache.”
B.
*“My nose doesn't stay open even though I'm using nasal spray every hour.”
C.
“I have been taking a lot of vitamin C this year to keep from getting so many colds.”
D.
“The only way I can get to sleep with this nasal congestion is by taking an over-the-counter
antihistamine at night.”
E.
Non of above
253. The patient is 34 years old and has been diagnosed with COPD as a result of being homozygous for a
mutation of the alpha1-antitrypsin (AAT) gene alleles. His wife has two normal AAT gene alleles. He is
concerned that his two children may develop this problem. What is your best response?
A.
“Because neither of your parents have COPD and your wife does not have the abnormal gene alleles,
your children will not be affected.”
B.
“Because your wife is not affected nor is or a carrier, your children will have normal levels of AAT
and their risk is the same as for the general population.”
C.
“Because you have the mutations and your wife does not, your son will be at an increased risk for
developing COPD but your daughter will only be a carrier.”
D.
*“Because both of your AAT gene alleles are mutated, your children will each have one abnormal
gene and their risk for COPD is only increased if they smoke or are chronically exposed to other
precipitating factors.”
E.
Non of above
254. The patient is a 42-year-old man recently diagnosed with new-onset asthma. What specific
patiental/demographic information should you obtain related to this diagnosis?
A.
Previous diagnosis of pneumonia or tuberculosis
B.
Known allergies and hypersensitivities
C.
Nutritional intake and diet history
D.
*Occupation and hobbies
E.
Non of above
255. The patient with hospital-acquired (nosocomial) pneumonia caused by a bacterial infection with a
gram-negative microorganism is receiving treatment with intravenous amikacin (Amikin). In addition to
frequent respiratory assessment, what other assessment should the physician routinely perform to identify a
common complication of this medication?
A.
*Monitor urine output every shift
B.
Perform neuro checks every 2 hours
C.
Examine the stool and vomitus for the presence of blood
D.
Monitor the complete white blood cell count and differential daily
E.
Non of above
256. Woman 36 y.o., complaints on dry cough, dyspnea. Felt ill after viral respiratory infection 2 years
ago. Objectively: breathing rate - 16/min, pulse -68/min, BP - 130/90. In lungs during percussion - clear
pulmonary sound. Auscultation – diffuse dry wheezes. To check reversibility of bronchial obstruction it is
necessary to provide test with:
A.
*Salbutamol
B.
Forced expiration
C.
Obzidan
D.
Physical exertion
E.
Oxygen
257. Woman 45 y.o., is ill with bronchial asthma for 20 years. She came to pulmonologist to discuss plan
of treatment in different cases. Now attacks arise 2-3 times a week, she uses intal, ventolin if necessary.
What medicine would be useful in case of arising night attacks?
A.
*Inhaled glucocorticosteroids
B.
To use intal
C.
To continue usual therapy
D.
To use prednisole
E.
To add atrovent to usual treatment
258. Woman 58 y.o., is ill with bronchial asthma, entered the hospital with complaints on dyspnea and
palpitation. Objectively: condition is severe, breathing is noisy with participation of additional breathing
muscles, periodically cramps are present, diffuse cyanosis. In lungs – diffuse dry wheezes, breathing is
weaker in the lower parts of lungs. Pulse - 100/min, liver is a little lower from the edge of costal arch,
edema, 3 extrasystoles/min, BP - 140/100, РаO2 - 45, pH - 7,3. What syndrome is the most severe for this
patient?
A.
*Respiratory failure
B.
Blood hypertension
C.
Tachycardia
D.
Arrhythmia
E.
Heart failure
259. Woman of 34, for 15 years is ill with bronchial asthma. Some time ago increased frequency of attacks
of dyspnea, they arised 4-5 times a week, night attacks - 2-3 times a month. Used salbutamol to remove
attacks. Objectively: condition is satisfactory. Breathing rate - 20/min. Heart rate - 76/min, BP - 120/80. In
lungs there is vesicular breathing. Tones of heart are a little weak, rhythm is normal. What medicine must be
used for the prophylaxis of attacks of bronchial asthma on the first stage?
A.
*Corticosteroids in inhalations
B.
Corticosteroids in injections
C.
Regular usage of salbutamol
D.
Corticosteroids orally
E.
Cromoglicat sodium
260. Woman of 62, is ill with bronchial asthma. Recently appeared pain behind the sternum, interruptions
in work of heart. Objectively: t - 36,6 °С, pulse -78/min, extrasystoles are present, BP -160/95, breathing rate
- 18/min. In lungs during auscultation - breathing with prolonged expiration, diffuse dry wheezes are present.
What preparations are not indicated in this situation?
A.
*β-blockers
B.
Ca-channel blockers
C.
Nitrosorbid
D.
Sustak
E.
Anticoagulants
261. Woman, 68 years old, with moderate emphysema is evaluated during a routine visit. She has chronic
dyspnea on exertion but has no cough or sputum production. She uses supplemental oxygen, 2 l/min, when
sleeping and on exertion. She currently uses albuterol and ipratropium four times per day, and salmeterol and
theophylline twice per day. She is currently enrolled in a pulmonary rehabilitation program and is concerned
about “catching a cold” from other people enrolled in the pulmonary rehabilitation program. What is the best
advice for this patient?
A.
*Practice good hand washing, attempt to avoid close prolonged contact with ill persons, and take
pneumococcal and annual influenza vaccine
B.
Avoid any social functions where there will be large crowds
C.
Discontinue attendance at the pulmonary rehabilitation program
D.
Take a daily antibiotic (long-term suppressive antibiotic therapy) to prevent pneumonia
E.
All of the above
262. Woman, a 76 years old resident of a nursing home, is evaluated in the emergency department because
of decreasing mental status and hypothermia. She has a history of stroke and is currently taking only aspirin.
She has been able to eat on her own and there have been no witnessed aspirations. She has not been treated
recently with antibiotics. Her leukocyte count is 12,000/l, and her hemoglobin is 120 g/l. Serum electrolytes
are within normal limits and she has mild chronic renal insufficiency. Chest radiograph shows a small
interstitial infiltrate in the right lower lung field. She receives traditional empiric treatment for communityacquired pneumonia. Therapy for which of the following should also be considered?
A.
*Enteric gram-negative organisms
B.
Pseudomonas aeruginosa
C.
Anaerobic bacteria
D.
Aspergillus fumigatus
E.
Mycobacterium tuberculosis
263. 67-year-old man with chronic obstructive pulmonary disease is evaluated because of chronic
dyspnea, minimally productive cough, and limited exercise tolerance. He thinks his dyspnea on exertion has
worsened. He stopped smoking cigarettes 8 years ago and is currently using an ipratropium inhaler four
times per day and salmeterol discus twice per day. His body mass index, which 6 months ago was 21, is now
19. On physical examination, he is afebrile, his pulse rate is 94/min and regular, and respiration rate is
20/min. His breathing is unlabored at rest. He has signs of chest hyperinflation and decreased breath sounds
without wheezing. He has no peripheral edema. The remainder of his examination is normal; results of a
fecal occult blood test are negative. Baseline spirometry is unchanged. Forced expiratory volume in 1 sec
(FEV1) 35% of predicted Forced vital capacity (FVC) 85% of predicted FEV1/FVC ratio50% PO2 62 mm
Hg PCO2 45 mm Hg pH 7.38 (with the patient breathing room air) Chest radiograph reveals only
hyperinflation. What is the best way to manage this patient’s weight loss?
A.
*Refer him for pulmonary rehabilitation with exercise and nutritional counseling.
B.
Provide dietary instructions to increase his caloric intake.
C.
Treat him with anabolic steroids
D.
Prescribe oxygen supplementation to improve his oxygen consumption.
E.
Add inhaled corticosteroids to his medical regimen.
264. 68-year-old man with severe chronic obstructive pulmonary disease (forced expiratory volume in 1
sec 32% of predicted) is evaluated because of severe dyspnea and the inability to carry out his activities of
daily living. He is on maximal bronchodilator and oxygen therapy. Which of the following might pulmonary
rehabilitation improve?
A.
*Exercise tolerance
B.
Forced expiratory volume in 1 sec
C.
Oxygenation
D.
Survival
E.
Non of above
265. A 35-yr-old accountant presents with a chronic cough, dyspnea and wheezing. He produces copious
sputum. His arterial carbon dioxide is low and his arterial oxygen is normal. Chest X-ray: high
pneumatization of lungs. What is your diagnosis?
A.
*Emphysema
B.
Bronchogenic carcinoma
C.
Pulmonary embolism
D.
Pneumonia
E.
Tuberculosis
266. A 53-yr-old smoker with chronic cough and copious yellow sputum presents in a state of agitation.
He is confused. His pulse is bounding. He has a terrible headache and you find papilloedema on fundoscopy.
What is the cause?
A.
*Emphysema
B.
Bronchogenic carcinoma
C.
Pneumonia
D.
Cardiac failure
E.
Bronchial asthma
267. A 53-yr-old smoker with chronic cough and copious yellow sputum presents in a state of agitation.
He is confused. His pulse is bounding. He has a terrible headache and you find papilloedema on fundoscopy.
What may be found on chest X-ray?
A.
*Signs of fibrosis and hyperpneumatization
B.
Infiltration
C.
Round shadow
D.
Round shadow with horizontal level of fluid
E.
Signs of fibrosis and local infiltration
268. A 55-year-old man is evaluated in the emergency department because of a 5-day history of increased
dyspnea and cough productive of yellow-green mucus. Nine months ago, he required prolonged mechanical
ventilation for an exacerbation of chronic obstructive pulmonary disease. His medical history includes
hypertension. On recent pulmonary function testing, the forced expiratory volume in 1 sec (FEV1) was 38%
of predicted. His temperature is 38.1 C, pulse rate is 135/min and irregular, respiration rate is 25/min, and
blood pressure is 90/65 mm Hg. He is mildly lethargic but arousable and oriented. He has a weak cough with
pooling of secretions in the oral cavity and hypopharynx. Electrocardiogram demonstrates multifocal atrial
tachycardia. Chest examination reveals accessory muscle use, coarse rhonchi, and decreased breath sounds at
the right base. Leukocyte count is 1 7,000/?L. Chest radiograph shows right lower lobe consolidation. With
the patient breathing 4 L oxygen, arterial blood gases show a PO2 of 50 mm Hg, a PCO2 of 65 mm Hg, and
a pH of 7.25. Therapy with methylprednisolone sodium succinate, 125mg intravenously every 6 h, is
initiated, along with nebulized albuterol and ipratropium bromide every 4 h, and azithromycin, 500 mg
administered intravenously daily. Which of the following is the most appropriate additional management?
A.
*Intubate and begin mechanical ventilation
B.
Initiate mucloytic therapy, chest physiotherapy, and oral-tracheal suctioning
C.
Administer a helium-oxygen mixture of 70%:30%, delivered by face mask
D.
Initiate bilevel noninvasive positive-pressure ventilation by face mask
E.
Administer 40% oxygen by Venturi mask
269. A 55-year-old man is evaluated in the emergency department because of an acute, severe asthma
attack; he is hospitalized in the intensive care unit for aggressive medical therapy and monitoring. He is
expectorating thick greenish sputum. His medical history includes hypertension, cholecystectomy, and
glaucoma. Chest radiograph reveals hyperinflation only. Medical therapy in the emergency department
included repeated doses of aerosolized albuterol and ipratropium, as well as methylprednisolone, 125mg
administered intravenously. Peak expiratory flow rate is unimproved at 80 L/min. Which of the following is
the most appropriate next step in this patient’s management?
A.
*Intravenous magnesium sulfate
B.
Nebulized ipratropium bromide administered by face mask
C.
Broad-spectrum antibiotics targeting community-acquired respiratory pathogens
D.
Inhaled corticosteroids
E.
Non of above
270. A 56-yr-old man wheezes and coughs. He has tried to give up smoking, but he finds it very difficult.
He is thin and healthy looking with a rounded chest. His breathing is noisy. His cough is unproductive. What
method of investigation is not useful?
A.
*Ultrasound examination
B.
Chest X-ray
C.
Spirography
D.
General blood analysis
E.
Sputum analysis
271. A 56-yr-old man wheezes and coughs. He has tried to give up smoking, but he finds it very difficult.
He is thin and healthy looking with a rounded chest. His breathing is noisy. His cough is unproductive. What
treatment has to be prescribed?
A.
*Salbutamol
B.
Amoxycillin
C.
Prednisolone
D.
ACC
E.
Bronchial lavage
272. A 56-yr-old man wheezes and coughs. He has tried to give up smoking, but he finds it very difficult.
He is thin and healthy looking with a rounded chest. His breathing is noisy. His cough is unproductive. What
is the previous diagnosis?
A.
*Emphysema
B.
Bronchogenic carcinoma
C.
Pneumonia
D.
Asthma
E.
Bronchitis
273. A 57-year-old man with advanced chronic obstructive pulmonary disease (COPD) and systemic
hypertension is evaluated because of a 6-day history of productive cough and shortness of breath. He uses
inhaled albuterol and ipratropium bromide, a long-acting theophylline preparation, and lisinopril. He uses
supplemental oxygen at night and during ambulation. Ciprofloxacin is prescribed for an exacerbation of
COPD. Three days later, having had nausea for a day, the man is brought to the emergency department after
he is found nearly unconscious. Arterial oxygen saturation is 89%, with the patient breathing room air.
Electrocardiogram shows normal sinus rhythm with nonspecific ST-T changes in the lateral chest leads.
Which of the following is likely to have interacted with ciprofloxacin and caused the symptoms that brought
the man to the emergency department?
A.
*Theophylline
B.
Albuterol
C.
Ipratropium bromide
D.
Lisinopril
E.
Oxygen
274. A 59-year-old man with advanced chronic obstructive pulmonary disease is evaluated because of a
daily cough productive of white or yellow sputum, dyspnea after climbing one flight of stairs, and a recent
4.5-kg weight loss with no associated change in appetite or food intake. The patient stopped smoking 4 years
ago. On physical examination, he has diminished breath sounds throughout all lung fields. Arterial oxygen
saturation measured by pulse oximetry with the patient at rest, breathing room air, is 87%. Chest radiograph
suggests hyperinflation of the lungs but shows no pulmonary infiltrates or abnormalities of the cardiac
silhouette. Pulmonary function studies show a forced expiratory volume in 1 sec 39% of predicted and
forced vital capacity 78% of predicted. Which of the following may prolong life in this patient?
A.
*Supplemental oxygen
B.
Albuterol
C.
Ipratropium bromide
D.
Theophylline
E.
Lisinopril
275. A 60-year-old woman is hospitalized for an exacerbation of chronic obstructive pulmonary disease.
She is treated with ipratropium bromide by nebulizer every 4 hours; intravenous azithromycin, 500 mg/d;
methylprednisolone, 125 mg intravenously every 6 hours; and oxygen by nasal cannula. During the first 2
hospital days, her condition remains unchanged. On the 3rd hospital day, she develops increased dyspnea
and a cough productive of sputum. On physical examination, she is awake and alert and in moderate
respiratory distress. Her temperature is 36.7 C, pulse rate is 110/min, respiration rate is 20/min, and blood
pressure is 150/90 mm Hg. Her lungs are hyperresonant to percussion, with accessory muscle use, poor air
movement, mild wheezing, and no crackles. Chest radiograph demonstrates hyperinflation, with no other
abnormalities. Leukocyte count is 16,000/L. Arterial blood gas measurements, with the patient breathing 1.5
L oxygen, show PO2 of 55 mm Hg, PaCO2 of 55 mm Hg, and pH of 7.32. She is transferred to the intensive
care unit for close observation and possible assisted ventilation. Which of the following is most appropriate
additional management for this patient?
A.
*Add albuterol to ipratropium bromide by nebulizer every 4 hours
B.
Discontinue azithromycin and begin levofloxacin
C.
Begin intravenous aminophylline
D.
Increase methylprednisolone to 250 mg every 6 hours
E.
Increase oxygen by nasal cannula to 3 L/min
276. A 66-year-old woman with chronic obstructive pulmonary disease is evaluated because of chronic
cough and dyspnea. She currently uses a long-acting bronchodilator twice per day, an inhaled corticosteroid
twice per day, ipratropium four times per day, and albuterol four to six times per day. She smokes 1 pack of
cigarettes per day. On physical examination, her vitals signs are normal. Her oxygen saturation at rest and
with exertion is 94%. She has diminished breath sounds, a prolonged expiratory-to-inspiratory phase, and no
wheezes. Her heart rate and rhythm are regular, with physiologically split S2, and no murmurs or rubs. Chest
radiograph reveals hyperinflation, increased retrosternal airspace, and flattened hemidiaphragms bilaterally.
Which of the following should be initiated at this time to address this patient’s cough and dyspnea?
A.
*Discuss techniques to help her to quit smoking
B.
Increase her use of the long-acting bronchodilator.
C.
Prescribe supplemental oxygen
D.
Provide emergency treatment for a tension pneumothorax
E.
Increase her dosage of inhaled corticosteroid.
277. A 67-year-old man is evaluated because of a 3-week history of cough productive of blood-streaked
sputum. A chest radiograph shows an infiltrate in the right upper lobe. He is treated with antibiotics for 2
weeks, but the blood-streaked sputum persists. A CT scan of the chest shows a mass obstructing the right
upper lobe and evidence of postobstructive pneumonitis. Examination of the mediastinum shows enlarged
lymph nodes in the right paratracheal space. A positron emission tomography (PET) scan shows uptake in
the mass itself and in the lymph nodes in the right paratracheal space. Fiberoptic bronchoscopy is performed;
an endobronchial lesion is identified and a sample is taken for biopsy. The biopsy shows squamous cell
carcinoma. Which of the following is the best next step in this patients management?
A.
*Perform immediate mediastinoscopy
B.
Refer him for radiation therapy
C.
Refer him for surgery
D.
Perform a repeat positron emission tomography scan
E.
Non of above
278. A 70-year-old man is evaluated because of shortness of breath. He has noted progression of his
symptoms, primarily with exertion over the past 6 months. He is unable to walk one flight of stairs or two
blocks on level ground without becoming short of breath. He has no chest pain, paroxysmal nocturnal
dyspnea, orthopnea, or lower extremity edema. He has a 40-pack-year history of cigarette smoking, but
stopped smoking 10 years ago. He worked in a naval shipyard 50 years ago but has spent most of his
working life as a schoolteacher. On physical examination, his respiration rate is 18/min but he does not
appear short of breath. Examination of the chest shows an increased anterior-posterior diameter. On
auscultation he has diffusely decreased breath sounds and a prolonged expiratory phase with no wheezing.
He has no cyanosis or clubbing. Spirometry shows forced expiratory volume in 1 sec 55% of predicted and
forced vital capacity 80% of predicted with a ratio of FEV1 to FVC 60%. Which of the following is the best
test to evaluate this patients condition?
A.
*Lung volumes and diffusing capacity
B.
Echocardiography
C.
Exercise spirometry
D.
Methacholine challenging testing
E.
Non of above
279. A 71-year-old woman is evaluated because of progressive fatigue, weakness, and dyspnea on
exertion. A former smoker, she has a history of advanced emphysema and is on continuous long-term
oxygen therapy. She currently uses a long-acting bronchodilator twice per day, theophylline and an inhaled
corticosteroid twice per day, ipratropium four times per day, and albuterol four to six times per day. Physical
examination is normal. Heart and lung examinations are consistent with long-standing, advanced
emphysema. Her laboratory and radiographic findings are unremarkable. Which of the following is the best
next step in this patient’s management?
A.
*Refer her to a multidisciplinary rehabilitation program
B.
Prescribe an antidepressant medication
C.
Prescribe an empiric course of “pulse dose” corticosteroids at 500 mg/d for 3 consecutive days
D.
Refer her for surgery
E.
Non of above
280. A middle-aged smoker presents with chronic cough and phlegm. His sputum is tenacious but not
yellow or blood stained. His chest is hyperinflated. His arterial carbon dioxide is high and is arterial oxygen
is low. What is the previous diagnosis?
A.
*Emphysema
B.
Bronchitis
C.
Bronchogenic carcinoma
D.
Pneumonia
E.
Respiratory failure
281. For 8 years patient is disturbed with cough in the morning with little amount of sputum, shortness of
breath. He is a smoker for more than 10 years. Objective examination: cyanosis, increased duration of
expiration, dry wheezes. Possible diagnosis is:
A.
*COPD
B.
Pneumonia
C.
Idiopathic alveolitis
D.
Bronchiectasis
E.
Bronchial asthma
282. Man 39 y.o., driver, complaints on the shortness of breath during physical exertion, cough with the
small amount of light sputum mostly in the morning. For a long time is ill with COPD. He is a smoker, uses
alcohol episodically. Objectively: temperature - 36,5 °С, breathing rate - 24/min, pulse - 90/min, BP - 120/
80. During auscultation breathing is hard, moderate amount of dry wheezes. FEV1 - 68 % of normal index.
What methods are necessary to prevent the disease?
A.
*To stop smoking
B.
To change the job
C.
To avoid alcohol
D.
Sanation of chronic infection
E.
To change region of living
283. Man 60 y.o., complains on expiration dyspnea which increases at the physical exertion, cough with
small amount of mucus-purulent sputum mostly in the morning. He is ill with COPD. Objectively:
temperature - 36,0 °С, breathing rate - 22/min, pulse - 84/min, BP - 110/70. Skin is moist, diffuse cyanosis.
Auscultation: breathing is hard, diffuse dry and moist wheezes are present. FEV - 62 %; pharmacological
test with atrovent showed 5 % increasing of this index. What mechanism of bronchial obstruction
development is the most possible in this case?
A.
*Diffuse sclerotic changes
B.
Hypercrynia
C.
Inflammatory edema
D.
Bronchial spasm
E.
Mucostasis
284. Man of 43 complaints on dyspnea during physical activity. Objectively: temperature 36,4 °С,
breathing rate - 20/min, pulse - 78/min, BP-125/80. Emphysematous form of thorax. In lungs – weak
vesicular breathing. What research must be provided by patient at home to decide question about efficiency
of prescribed broncholytics?
A.
*Peakflowmetry
B.
Spirography
C.
ECG-control of overload of right departments of heart
D.
Bronchoscopy
E.
Analysis of sputum
285. Patient 44 y.o., complaints on attack of dyspnea, which arises suddenly at night. Connects this attack
with overcooling. He is ill for more than 10 years. Thorax of emphysematous form. During percussion in
lungs – “box” sound. During auscultation there is plenty of dry wheezes. In blood: moderate leucocytosis,
eosinophylia - 10 %. On the X-ray film – increased pneumatization of pulmonary tissue. What diagnosis is
the most possible one?
A.
*Bronchial asthma, exacerbation phase
B.
Bronchiectasis, exacerbation phase
C.
COPD, exacerbation phase
D.
Chronic bronchitis
E.
Eosinophylic pulmonary vasculitis
286. Patient 47 y.o., complaints on cough, dyspnea during physical exertion, local pain in the heart region,
general weakness. Suffers with COPD for 10 years. During auscultation of lungs were founded disseminated
dry wheezes. Systolic blood pressure in the pulmonary artery is 50. It is the most important to prescribe for
treatment:
A.
*Euphyllin
B.
Bromhexin
C.
Caffeine
D.
Prednisone
E.
Atropin
287. Patient complaints on attacks of dyspnea, which arises 1-2 times a week, night symptoms - 2 times a
month and even more frequent. For a patient night sleep is broken as a result of attacks of dyspnea. FEV1 >
80 % from normal. What diagnosis would you suspect?
A.
*Mild persistent BA
B.
Severe persistent BA
C.
Intermittent BA
D.
Moderate persistent BA
E.
Status asthmaticus
288. A 14-yr-old student with cystic fibrosis rapidly deteriorated and developed acute respiratory failure
while in hospital. Which infection is the most possible cause of deterioration of his state?
A.
*Pseudomonas aeroginosa
B.
Pneumocystis carinii
C.
Chlamydia psittaci
D.
Ecoli
E.
Mycobacterium tuberculosis
289. A 20-yr-old male IV drug abuser presents with breathlessness and cough. CXR reveals patchy areas
of consolidation with abscess formation. Which infection is the most possible cause of his state?
A.
*Staphylococcus aureus
B.
Chlamydia psittacci
C.
Coxiella burnetti
D.
Pneumocystis carinii
E.
Pseudomonas aeroginosa
290. A 20-yr-old previously healthy woman presents with general malaise, severe cough and
breathlessness, which has not improved with a 7 day course of Amoxycillin. There is nothing significant to
find on examination. The X-ray shows patchy shadowing throughout the lung fields. The blood film shows
clumping of red cells with suggestion of cold agglutinins. Which infection is the most possible cause of her
state?
A.
*Mycoplasma pneumoniae
B.
Legionella pneumonia
C.
Haemophilus influenzae
D.
Chlamydia trachomatis
E.
Klebsiella pneumoniae
291. A 22-year-old previously healthy man is evaluated in the emergency department because of fever,
productive cough, and shortness of breath. His temperature is 40 C, pulse is 120/min, respiration rate is
32/min, and blood pressure is 100/70 mm Hg. Measurement of arterial blood gases with the patient breathing
room air shows PO2 of 55 mm Hg, PCO2 of 30 mm Hg, pH of 7.41. Chest radiograph reveals bilateral
alveolar infiltrates with no effusions. Gram stain of the sputum reveals gram-positive diplococci. Which of
the following is the most appropriate for this patient?
A.
*Hospitalize him
B.
Treat him as an outpatient with oral therapy
C.
Treat him as an outpatient with intravenous therapy
D.
Hospitalize the patient in the intensive care unit
E.
All of the above
292. A 22-yr-old barman presents with a dry cough of sudden onset. He complains of a chest pain and
rusty sputum. He also has a very high fever, rapid breathing, cyanosis and crepitations. Pneumonia was
suspected. What is the most necessary method of investigation?
A.
*Chest X-ray
B.
Spirography
C.
Analysis of sputum
D.
General blood analysis
E.
General urine analysis
293. A 22-yr-old barman presents with a dry cough of sudden onset. He complains of a chest pain and
rusty sputum. He also has a very high fever, rapid breathing, cyanosis and crepitations. What is your
previous diagnosis?
A.
*Pneumonia
B.
Asthma
C.
Lung abscess
D.
COPD
E.
Lung cancer
294. A 24-yr-old car mechanic is brought to casualty by his girlfriend. She describes a 2-day history of
rigors, sweats and intermittent confusion. On examination he is agitated, sweaty and pyrexial with 38.6° C.
He is hyperventilating and cyanosed despite receiving O2 by face mask. There is dullness to percussion and
bronchial breathing at the left lung basse. What method of investigation is nesessary?
A.
*Chest X-ray
B.
Spiral CT with contrast
C.
Arterial blood gases
D.
Blood count and film
E.
Urea and electrolytes
295. A 24-yr-old man presents dry cough, skin manifestations and bone and muscle aches. His chest
radiograph shows widespread patchy shadows. Blood tests show evidence of haemolysis. Which infection is
the most possible cause of his state?
A.
*Mycoplasma pneumoniae
B.
Pneumocystis carinii
C.
Chlamydia psittaci
D.
coli
E.
Pseudomonas aeroginosa
296. A 27-yr-old male patient has just returned from holiday abroad presents with flu-like illness,
headaches, high fever prior to this, he had complained of abdominal pain, vomiting, diarrhoea associated
with blood per rectum. Which infection is the most possible cause of his state?
A.
*Legionella pneumoniae
B.
Streptococcus pneumoniae
C.
Mycoplasma pneumoniae
D.
Pneumocystis carinii
E.
Fungi
297. A 27-yr-old male prostitute has felt generally unwell for 2 months with some weight loss. Over the
last 3 weeks he has noticed a dry cough with increasing breathlessness. Two courses of antibiotics from the
GP have produced no improvement. The CXR shows bilateral interstitial infiltrates. Which infection is the
most possible cause of his state?
A.
*Pneumocystis carinii
B.
Streptococcus pneumoniae
C.
Mycoplasma pneumoniae
D.
Fungi
E.
Legionella pneumoniae
298. A 28 years old patient, complaints on cough with small amount of colourless sputum, pain in the right
half of thorax during breathing, shortness of breath, increase of temperature to 39 °С. Felt ill rapidly. Used
aspirin. Objectively: herpes on lips. In lower lobe of right lung there is dull percussion sound, bronchial
breathing. Chest X-ray: there is homogeneous infiltration of right lower lobe. What is the most possible
etiology of pneumonia?
A.
*Pneumococcus
B.
Staphylococcus
C.
Mycoplasma
D.
Legionella
E.
Klebsiella
299. A 30-yr-old man with AIDS presents with fever, dry cough and dyspnoea. CXR shows diffuse
bilateral alveolar and interstitial shadowing beginning in the perihilar regions and spreading outward. Which
infection is the most possible cause of his state?
A.
*Pneumocystis carinii
B.
Chlamydia psittacci
C.
Coxiella burnetti
D.
Staphylococcus aureus
E.
Pseudomonas aeroginosa
300. A 35-yr-old previously healthy man returned from holiday five days ago. He smokes 10 cigarettes per
day. He presents with mild confusion, a dry cough and mild pyrexia. His chest is normal. The X-ray shows
widespread upper zone shadowing. Which infection is the most possible cause of his state?
A.
*Legionella pneumoniae
B.
Haemophilus influenzae
C.
Chlamydia trachomatis
D.
Pneumocystis carinii
E.
Klebsiella pneumoniae
301. A 38 years old patient, who drunk a lot of alcohol, has severe pneumonia. His condition was
worsened, the temperature of body rose to 39-40 °С, an unpleasant smell appeared from a mouth, increased
amount of purulent sputum; increased ESR and amount of band leucocytes. On the X-ray - in the lower lobe
of right lung there is massive infiltration with bright area in a center. What complication is it necessary to
suspect?
A.
*Acute pulmonary abscess
B.
Bronchiectasis
C.
Infarction-pneumonia
D.
Gangrene of lungs
E.
Empyema of pleura
302. A 40-yr-old man who works in an abattoir presents with sudden onset of fever, myalgia, headache,
dry cough and chest pain. CXR shows patchy consolidation of the right lower lobe giving a ground glass
appearance. Which infection is the most possible cause of his state?
A.
*Coxiella burnetti
B.
Chlamydia psittacci
C.
Staphylococcus aureus
D.
Pneumocystis carinii
E.
Aspergillosis
303. A 44-yr-old travelling insurance salesman presents with high fever myalgia abdominal pain and
haemoptysis. CXR shows diffuse patchy lobar shadows. The cough progresses from a modest nonproductive
cough to producing mucopurulent sputum. The fever persists for 2 weeks. Which infection is the most
possible cause of his state?
A.
*Legionella pneumophilia
B.
Mycoplasma pneumoniae
C.
Actinomycosis
D.
Tuberculosis
E.
Streptococcus pneumoniae
304. A 48 years old patient, complaints on weakness, dyspnea, pain in the left half of thorax, permanent
cough with viscid sputum, in which particles of blood are sometimes determined. For the last 3 months lost 5
kg of body mass. On the X-ray of lungs there is total homogeneous shade is determined from the left side.
Organs of mediastinum are displaced to the left. What diagnosis is possible?
A.
*Lung athelectasis
B.
Lungs gangrene
C.
Total exudative pleurisy
D.
Pneumonia
E.
Empyema of pleura
305. A 48-yr-old man presents with fever, rigors, headache and diarrhea. He recently had been on a
holiday abroad. CXR shows consolidation. Which infection is the most possible cause of his state?
A.
*Legionella pneumoniae
B.
Staphylococcus aureus
C.
Crytococcus
D.
Streptococcus pneumoniae
E.
Mycobacterium avium
306. A 50-year-old man is evaluated in the emergency department because of fever, a nonproductive
cough, and a 2-day history of myalgia and headache. He has also had some nausea and diarrhea. He is a
heavy smoker. On physical examination, he is slightly disoriented. Temperature is 38.9 0C, pulse rate is
110/min, respiration rate is 32/min. Chest radiograph shows fluffy infiltrates to the right upper and lower
lobes. Results of laboratory testing show serum sodium of 128 meq/L, blood urea nitrogen of 42 mg/dL,
serum creatinine of 2.2 mg/dL, and serum creatine kinase of 250 U/L. Which one of the following is best
next step in the management of this patient’s pneumonia?
A.
*Initiate empiric antibiotic therapy for Legionella
B.
Order direct fluorescent antibody testing of the sputum for Legionella
C.
Order serologic testing for Legionella
D.
Send a urine specimen for measurement of Legionella antigen
E.
All of the above
307. A 50-yr-old man presents with shortness of breath and dry cough. CXR shows widespread pulmonary
shadowing. He takes Azathioprine for resistant rheumatoid arthritis. Choose the most appropriate treatment.
A.
*Co-trimoxazole
B.
Erythromycm
C.
Tetracycline
D.
Flucoxacillin
E.
Isoniazid
308. A 52-year-old man is evaluated because of a 2-month history of nonproductive cough, myalgias, and
low-grade fever. When his illness began, a chest radiograph showed bilateral alveolar infiltrates, and a
presumptive diagnosis of community-acquired pneumonia was made. He was treated with oral azithromycin
without effect, followed by a 10-day course of levofloxacin, also without effect. During the course of his
illness he has lost 4.5 kg without significant anorexia. He is a lifetime nonsmoker and works as an office
manager. He has no pets and no unusual hobbies. On physical examination, his vitals signs are normal,
except for a respiration rate of 22/min. He is in mild respiratory distress on exertion. On examination of the
chest, bilateral crackles are audible, without wheezing. Chest radiograph shows bilateral alveolar infiltrates,
which are changed in location from those seen on his original radiographs. Pulmonary function tests show
forced expiratory volume in 1 sec (FEV1) 75% of predicted, forced vital capacity (FVC) 72% of predicted,
total lung capacity 80% of predicted, and diffusing lung capacity for carbon monoxide (DL CO) 65% of
predicted. Arterial blood gas values, with the patient breathing room air, are PO2 62 mm Hg, PCO2 42 mm
Hg, and pH 7.39. Which of the following is the most likely diagnosis?
A.
*Cryptogenic organizing pneumonitis
B.
Hypersensitivity pneumonitis
C.
Resistant pneumococcal pneumonia
D.
Chlamydia pneumonia
E.
Bronchoalveolar cell carcinoma
309. A 60-year-old man with chronic obstructive pulmonary disease is evaluated because of persistent
cough and sputum production. He reports no fevers or weight loss. He has had increased sputum production
over the past 6 to 9 months and moderately severe reduction in forced expiratory volume in 1 sec (FEV1
55% of predicted). His symptoms have been unresponsive to antibiotics. He has an 80-pack-year history of
cigarette smoking. Chest radiograph reveals multiple small nodules in the left upper lung zone without
infiltrate. Serologic testing for HIV is negative. Sputum cultures are negative for bacteria. A test for acid-fast
bacillus is negative, but culture grows Mycobacterium avium-intracellulare with 1+ growth. Which of the
following is the best next step in this patient’s management?
A.
*Order a sputum culture for Mycobacterium avium-intracellulare
B.
Treat him for Mycobacterium avium-intracellulare without further testing
C.
Perform bronchoscopy with bronchioalveolar lavage and transbronchial biopsy
D.
Order a HRCT scan of the chest
E.
All of the above
310. A 62-year-old woman with moderate emphysema is evaluated during a routine visit. She has chronic
dyspnea on exertion but has no cough or sputum production. She uses supplemental oxygen, 2 L/min, when
sleeping and on exertion. She currently uses albuterol and ipratropium four times per day, and salmeterol and
theophylline twice per day. She is currently enrolled in a pulmonary rehabilitation program and is concerned
about “catching a cold” from other people enrolled in the pulmonary rehabilitation program. What is the best
advice for this patient?
A.
*Practice good hand washing, attempt to avoid close prolonged contact with ill persons, and take
pneumococcal and annual influenza vaccine
B.
Avoid any social functions where there will be large crowds
C.
Discontinue attendance at the pulmonary rehabilitation program
D.
Take a daily antibiotic (long-term suppressive antibiotic therapy) to prevent pneumonia
E.
All of the above
311. A 65-yr-old man currently undergoing chemotherapy of chronic leukaemia has felt unwell with fever
and unproductive cough for 2 weeks despite treatment with broad-spectrum IV antibiotics. The CXR shows
an enlarging right sided midzone consolidation. Which infection is the most possible cause of his state?
A.
*Fungi
B.
Streptococcus pneumoniae
C.
Mycoplasma pneumoniae
D.
Pneumocystis carinii
E.
Legionella pneumoniae
312. A 67-year-old woman is evaluated in the emergency department because of a 2-day history of fever
and a cough productive of purulent sputum. She is intubated for hypoxic respiratory failure due to
pneumonia and hospitalized in the intensive care unit. Her medical history is remarkable only for
hypertension for which she takes a calcium channel blocker. She has never worked outside the home, has no
pets, and has not traveled recently. Which of the following would be appropriate initial antibiotic therapy?
A.
*A second-generation cephalosporin and macrolide
B.
A second-generation cephalosporin
C.
A third-generation cephalosporin, aminoglycoside, and macrolide
D.
An antipseudomonal β-lactam/β-lactamase combination and fluoroquinolone
E.
All of the above
313. A 70-year-old man is ready to be discharged in October from the hospital after treatment of an
exacerbation of his chronic obstructive pulmonary disease. His only other medical problems include stable
angina and hypertension. He has normal renal function and is well nourished. He quit smoking 2 years ago.
He received pneumonia vaccine 2 years ago and influenza vaccine 1 year ago. Which of the following is true
of his vaccination status?
A.
*He should receive influenza vaccine
B.
He should receive pneumonia vaccine and influenza vaccine
C.
He should receive pneumonia vaccine
D.
He should receive neither pneumonia vaccine nor influenza vaccine
E.
Nothing of the above
314. A 70-year-old woman resident of a nursing home is evaluated in the emergency department because
of decreasing mental status and hypothermia. She has a history of stroke and is currently taking only aspirin.
She has been able to eat on her own and there have been no witnessed aspirations. She has not been treated
recently with antibiotics. Her leukocyte count is 12,000/L, and her hemoglobin is 120 g/L. Serum
electrolytes are within normal limits and she has mild chronic renal insufficiency. Chest radiograph shows a
small interstitial infiltrate in the right lower lung field. She receives traditional empiric treatment for
community-acquired pneumonia. Therapy for which of the following should also be considered?
A.
*Enteric gram-negative organisms
B.
Pseudomonas aeruginosa
C.
Anaerobic bacteria
D.
Aspergillus fumigatus
E.
Mycobacterium tuberculosis
315. A 72-year-old woman with a history of rheumatoid arthritis is evaluated because of persistent cough,
purulent sputum production, and occasional scant hemoptysis. She reports no fever, but says she has
difficulty gaining weight. She has never smoked cigarettes. She is hospitalized because of tachypnea and
hypoxia. Spirometry shows moderately severe obstructive impairment. Chest radiograph shows tramlines
without infiltrate. Gram stain of the sputum shows numerous leukocytes but no bacteria; culture results are
pending. Previous IgE levels were normal. There is no eosinophilia. An immediate skin test for Aspergillus
is negative. A tuberculin skin test performed last year was nonreactive. Which of the following is appropriate
therapy for this patient?
A.
*Intravenous fluoroquinolone and aminoglycoside
B.
A β-lactam/β-lactamase combination
C.
Itraconazole
D.
A new-generation macrolide
E.
All of the above
316. A businessman, had a rest during summer on Cyprus in 5-stars hotel. After 3 weeks of rest suddenly,
without any cause, has got severe headache, pain in muscles, joints, dry cough, moderate dyspnea, pain in a
thorax. X-ray: infiltration of lung tissue. In a blood test: absolute lymphopenia with moderate leucocytosis.
Most possible etiological factor of pneumonia:
A.
*Legionella
B.
Mycoplasma
C.
Pneumocysts
D.
Enteroviruses
E.
Pneumococcus
317. A patient who is 2 days postoperative from a bowel resection tells her physician that she is having a
hard time “catching her breath,” feels nauseated, and has chest pains when she inhales. The physician
suspects that she is having a pulmonary embolism. What intervention should the physician perform before
notifying the physician?
A.
Increase the IV flow rate
B.
*Apply oxygen by mask or nasal cannula at 5 L/min
C.
Assess the chest and axillary area for the presence of petechiae
D.
Place the patient in shock position, with her head and neck flat and her legs elevated
E.
Non of above
318. A pet shop owner presents with high swinging fever, cough and malaise. He has scanty rose spots
over his abdomen. CXR reveals diffuse pneumonia. Which infection is the most possible cause of his state?
A.
*Chlamydia psittacci
B.
Coxiella burnetti
C.
Staphylococcus aureus
D.
Pneumocystis carinii
E.
Aspergillosis
319. A previously healthy 18-yr-old girl has had influenza for the last 2 weeks. She is deteriorating and
has a swinging fever. She is coughing up copious purulent sputum. CXR shows cavitating lesions. Which
infection is the most possible cause of his state?
A.
*Staphylococcus aureus
B.
Streptococcus pneumoniae
C.
Mycoplasma pneumoniae
D.
Pneumocystis carinii
E.
Legionella pneumoniae
320. During the epidemic of influenza patient G., 59 y.o., after decreasing of fever noticed pain appeared
in a thorax, cough with yellow-green sputum (amount-100 ml a day), sometimes with some blood.
Objectively: breathing rate - 36/min. In lungs from the right side lower scapula there is dull sound during
percussion, hard breathing, and moist rales. Blood test: L - 18,6х109/л, ESR -64 mm/h. Analysis of sputum:
L -80-100 , Er - 40-50, elastic fibres, cocci. X-ray: rhadicis are enlarged, from the right side lower lobe is
heterogeneously infiltrated with two lighter areas. What is the most possible previous diagnosis?
A.
*Right-side pneumonia with abscesses
B.
Peripheral cancer
C.
Infiltrative tuberculosis in the phase of disintegration
D.
Exudative pleurisy
E.
Infarction-pneumonia
321. Female B., 44 years old, complains on cough with mucous sputum, increase of temperature to 39 °С,
weakness, dyspnea, sweating. Breathing rate - 26/min., skin is moist. Below left scapula there is shortening
of percussion sound. Breathing during auscultation is weaker, moist rales. Blood test: L - 11х109/l, ESR - 29
mm/h. Your previous diagnosis?
A.
*Left-side lower lobe pneumonia
B.
Gangrene of lungs
C.
Left-side exudative pleurisy
D.
Cancer of left side lower lobe
E.
Pulmonary abscess
322. Female patient K., 46 years old, after decreasing of fever after flu noticed pain appeared in a thorax,
cough with yellow-green sputum (amount-150 ml a day), sometimes with some blood. Objectively:
breathing rate - 36/min. In lungs from the right side lower scapula there is dull sound during percussion, hard
breathing, and moist rales. Blood test: L - 18,6х109/l, ESR -64 mm/h. Analysis of sputum: L -80-100 , Er 40-50, elastic fibres, cocci. X-ray: rhadicis are enlarged, from the right side lower lobe is heterogeneously
infiltrated with two lighter areas. What is the most possible previous diagnosis?
A.
*Right-side pneumonia with abscesses
B.
Peripheral cancer
C.
Infiltrative tuberculosis in the phase of disintegration
D.
Exudative pleurisy
E.
Infarction-pneumonia
323. Female, 34 years old, has an increase of body temperature to 38 °С, cough with purulent sputum,
weakness, dyspnea, pain in a thorax during breathing. During percussion there is shortening of sound in the
lower part of left lung, during auscultation – moist rales. What method of investigation is the decisive one to
confirm diagnosis?
A.
*Chest X-ray examination
B.
Bacteriological analysis of sputum
C.
Spirometry
D.
Pneumotachometry
E.
Bronchography
324. Male patient F., 48 years old, during a week was at home with diagnosis of respiratory viral infection.
Doctor noticed complaints on cough with small amount of mucus-purulent sputum, weakness. Objectively:
condition is relatively satisfactory. T - 37,2 °С. Breathing rate - 18/min, pulse - 80/min, BP - 110/70. In
lungs there is vesicular breathing, with a hard tint, single dry wheezes. Tones of heart are a little dull, rhythm
is correct. What is the treatment tactic?
A.
*To prescribe antibacterial therapy
B.
To stay at home for some more days
C.
To go to work
D.
To send patient to pulmonologist
E.
To hospitalize patient to the pulmonological department
325. Male patient G., 56 years old, complaints on permanent pain in a thorax which disturbs for last 2
months. Pain is not connected with breathing. There is also cough with particles of blood in sputum.
Weakness, fatigue are present. On the X-ray of thorax in the lower lobe of right lung there is spherical
shadow, size 4x6 cm, related to the lungs rhadicis. What is the most possible diagnosis?
A.
*Perypheral lung cancer
B.
Tuberculoma
C.
Metastasis
D.
Pulmonary abscess
E.
Pneumonia
326. Man 40 y.o., is ill with attacks of cough with yellow-brown sputum, pain in a right side, related to the
deep breathing, sweating. He is ill for 6 days, after overcooling. Used aspirin. Objectively: T - 39,6 °С,
breathing rate - 26/min, pulse - 110/min, BP -110/70. In lower part of right lung - moist loud rales. Chest Xray: in right lower lobe there is massive unhomogeneous infiltration with lighter areas, sinus is not changed.
What complication of disease is the most probable?
A.
*Abscesses
B.
Dry pleurisy
C.
Empiema of pleura
D.
Spontaneous pneumothorax
E.
Pulmonary athelectasis
327. On return to university, a 20-yr-old student presented with the onset of fever, malaise and a dry
cough. The student health service gave him Amoxycilline. After a week he felt no better and his CXR
showed patchy bilateral consolidation. Which infection is the most possible cause of his state?
A.
*Mycoplasma pneumoniae
B.
Streptococcus pneumoniae
C.
Pneumocystis carinii
D.
Fungi
E.
Legionella pneumoniae
328. Patient B., 26 years old, who used to drink alcohol, has right-side lower-lobe pneumonia. On the
chest X-ray film there are infiltrative changes from the right side – in S 6 and S 10. He has no diseases of the
respiratory tract before. From which medicine is it better to begin antibacterial therapy?
A.
*Amoxyclav
B.
Biseptol
C.
Ofloxacin
D.
Tetracyclin
E.
Ceporin
329. Patient K., 27 years old, has dry cough and pain, related to breathing, in the right half of thorax, fever
- to 39,5 °С. In the right lung lower from scapula there is dullness during percussion, bronchial breathing.
What is the most possible diagnosis for a patient?
A.
*Right-side lower lobe pneumonia
B.
An abscess of lower lobe of right lung
C.
Bronchiectasis
D.
An athelectasis of lower lobe of right lung
E.
Exudative pleurisy
330. Patient F., 38 years old, complaints on increase of temperature to 37,9 °С, cough with small amount
of mucus-purulent sputum, pain in right part. He is ill for 5 days, after overcooling. Objectively:
acrocyanosis. Pulse - 96/min. BP - 120/80. From the right side below scapula there is increased vocal
fremitus, shortened percussion sound, moist rales. What is the most possible diagnosis?
A.
*Community-acquired right side lobar pneumonia
B.
Right lung athelectasis
C.
Right-side exudative pleurisy
D.
Right lung athelectasis
E.
Infiltrative tuberculosis
331. Patient of 53 y.o., complaints on cough with mucous sputum, increase of temperature to 39 °С,
weakness, dyspnea, sweating. Breathing rate - 26/min, skin is moist. Below left scapula there is shortening of
percussion sound. Breathing during auscultation is weaker, moist rales. Blood test: L - 11х109/l, ESR - 29
mm/h. Your previous diagnosis?
A.
*Left-side lower lobe pneumonia
B.
Gangrene of lungs
C.
Left-side exudative pleurisy
D.
Cancer of left side lower lobe
E.
Pulmonary abscess
332. Patient of 56 years old, complaints on permanent pain in a thorax which disturbs for last 2 months.
Pain is not connected with breathing. There is also cough with particles of blood in sputum. Weakness,
fatigue are present. On the X-ray of thorax in the lower lobe of right lung there is spherical shadow, size 4x6
sm, related to the lungs rhadicis. What is the most possible diagnosis?
A.
*Perypheral lung cancer
B.
Tuberculoma
C.
Metastasis
D.
Pulmonary abscess
E.
Pneumonia
333. Patient G., 36 years, works on a poultry factory. Her emergency hospitalized with acute attack of
dyspnoea. During observation was diagnosed bronchial asthma. What additional methods of research must
be conducted above all things for confirmation of professional genesis of bronchial asthma?
A.
roentgenologic research of breathing organs
B.
professional route of patient
C.
sanitary-hygienic characteristic of work conditions
D.
research of function of the external breathing
E.
*allergic and immunologic tests
334. Patient G., 59 y.o., after decreasing of fever after flu noticed pain appeared in a thorax, cough with
yellow-green sputum (amount-150 ml a day), sometimes with some blood. Objectively: breathing rate 36/min. In lungs from the right side lower scapula there is dull sound during percussion, hard breathing, and
moist rales. Blood test: L - 18,6х109/l, ESR -64 mm/h. Analysis of sputum: L -80-100 , Er - 40-50, elastic
fibres, cocci. X-ray: rhadicis are enlarged, from the right side lower lobe is heterogeneously infiltrated with
two lighter areas. What is the most possible previous diagnosis?
A.
*Right-side pneumonia with abscesses
B.
Peripheral cancer
C.
Infiltrative tuberculosis in the phase of disintegration
D.
Exudative pleurisy
E.
Infarction-pneumonia
335. Patient I., a 50 years old man is evaluated in the emergency department because of fever, a
nonproductive cough, and a 2-day history of myalgia and headache. He has also had some nausea and
diarrhea. He is a heavy smoker. On physical examination, he is slightly disoriented. Temperature is 38.9 C,
pulse rate is 110/min, respiratory rate is 32/min. Chest radiograph shows fluffy infiltrates to the right upper
and lower lobes. Results of laboratory testing show serum sodium of 128 meq/L, blood urea nitrogen of 42
mg/dL, serum creatinine of 2.2 mg/dL, and serum creatine kinase of 250 U/L. Which one of the following is
best next step in the management of this patient’s pneumonia?
A.
*Initiate empiric antibiotic therapy for Legionella
B.
Order direct fluorescent antibody testing of the sputum for Legionella
C.
Order serologic testing for Legionella
D.
Send a urine specimen for measurement of Legionella antigen
E.
All of the above
336. Patient K. 38 y.o., who is in a hospital, after week noticed temperature increased to 39 0С, cough
appeared with “ferruginous” sputum, pain in a thorax, related to the act of breathing, breathing rate is 26 per
minute. On X-ray there is infiltration in the lower lobe of the left lung. What medicine is useful?
A.
*Cephalosporin III generation
B.
Tetracyclin
C.
Penicillin
D.
Erythromycin
E.
Streptomycin
337. Patient M., 39 years old, is ill with attacks of cough with yellow-brown sputum, pain in a right side,
related to the deep breathing, sweating. He is ill for 6 days, after overcooling. Used aspirin. Objectively: T 39,6 °С, breathing rate - 26/min., pulse - 110/min., BP -110/70. In lower part of right lung - moist loud rales.
X-ray: in right lower lobe there is massive unhomogeneous infiltration with lighter areas, sinus is not
changed. What complication of disease is the most possible?
A.
*Abscesses
B.
Dry pleurisy
C.
Empiema of pleura
D.
Spontaneous pneumothorax
E.
Pulmonary athelectasis
338. Patient N., 46 years old, was admitted into the surgical department and operated because of
appendicitis. After 4 days appeared she developed chills, cough, dyspnea, fever 38,5 °С, leucocytosis with
shift of leucocyte formula to the left. On chest X-ray there is infiltration of lower right lobe. What is the
diagnosis?
A.
*Nosocomial pneumonia
B.
Pulmonary abscess
C.
Infarction pneumonia
D.
Community-acquired pneumonia
E.
Tuberculosis
339. Patient O., 29-yr-old male prostitute, has felt generally unwell for 2 months with some weight loss.
Over the last 3 weeks he has noticed a dry cough with increasing breathlessness. Two courses of antibiotics
from the GP have produced no improvement. The CXR shows bilateral interstitial infiltrates. What is the
most possible etiology of disease?
A.
*Pneumocystis carinii
B.
Streptococcus pneumoniae
C.
Mycoplasma pneumoniae
D.
Fungi
E.
Legionella pneumoniae
340. Patient of 51 y.o., complaints on weakness, dyspnea, pain in the left half of thorax, permanent cough
with viscid sputum, in which particles of blood are sometimes determined. For the last 3 months he lost 5 kg
of body weight. On the X-ray of lungs there is total homogeneous shade is determined from the left side.
Organs of mediastinum are displaced to the left. What diagnosis is possible?
A.
*Lung athelectasis
B.
Lungs gangrene
C.
Total exudative pleurisy
D.
Pneumonia
E.
Empyema of pleura
341. A 38 years old patient, who drunk a lot of alcohol, has severe pneumonia. His condition got worse,
the temperature of body rose to 39-40 °С, an unpleasant smell appeared from a mouth, increased amount of
purulent sputum; increased ESR and amount of band leucocytes. On the X-ray - in the lower lobe of right
lung there is massive infiltration with light area in a center. What complication is it necessary to suspect?
A.
*Acute pulmonary abscess
B.
Bronchiectasis
C.
Infarction-pneumonia
D.
Gangrene of lungs
E.
Empyema of pleura
342. Patient of 43, complaints on cough with small amount of colourless sputum, pain in the right half of
thorax during breathing, shortness of breath, increase of temperature to 39 °С. Felt ill rapidly. Used aspirin.
Objectively: gerpes on lips. In lower lobe of right lung there is dull percussion sound, bronchial breathing.
X-ray: there is homogeneous infiltration of right lower lobe. What is the most possible etiology of
pneumonia?
A.
*Pneumococcus
B.
Staphylococcus
C.
Mycoplasma
D.
Legionella
E.
Klebsiella
343. Patient P., a young male homosexual with Kaposi's sarcoma, complains of increasing breathlessness
and a dry cough. He has a 3-day history of shivering, general malaise and productive cough. The X-ray
shows right lower lobe consolidation. What is the most possible etiology of disease?
A.
*Pneumocystis carinii
B.
Mycoplasma tuberculosis
C.
Haemophilus influenzae
D.
Chlamydia trachomatis
E.
Klebsiella pneumoniae
344. Patient Q., 37 years old, was admitted into the surgical department and operated because of
appendicitis. After 4 days appeared she developed chills, cough, dyspnea, fever 38,5 °С, leucocytosis with
shift of leucocyte formula to the left. On chest X-ray there is infiltration of lower right lobe. What is the
diagnosis?
A.
*Nosocomial pneumonia
B.
Pulmonary abscess
C.
Infarction pneumonia
D.
Community-acquired pneumonia
E.
Tuberculosis
345. Patient S., a 25-year-old male has just returned from holiday abroad, presents with flu-like illness,
headaches, high fever. Prior to this, he had complained of abdominal pain, vomiting, diarrhea associated
with blood per rectum. What is the most possible etiology of disease?
A.
*Legionella pneumoniae
B.
Streptococcus pneumoniae
C.
Mycoplasma pneumoniae
D.
Pneumocystis carinii
E.
Fungi
346. Patient V., a 24 years old barman, presents with a dry cough of sudden onset. He complains of a chest
pain and rusty sputum. He also has a very high fever, rapid breathing, cyanosis and crepitations. Pneumonia
was suspected. What is the most nesessary method of investigation?
A.
*Chest X-ray
B.
Spirography
C.
Analysis of sputum
D.
General blood analysis
E.
General urine analysis
347. Patient W., 67 years old, during the epidemic of influenza after decreasing of fever noticed pain that
appeared in a thorax, cough with yellow-green sputum (amount-100 ml a day), sometimes with some blood.
Objectively: breathing rate - 36/min. In lungs from the right side lower scapula there is dull sound during
percussion, hard breathing, and moist rales. Blood test: L - 18,6х109/l, ESR -64 mm/h. Analysis of sputum:
L -80-100 , Er - 40-50, elastic fibres, cocci. X-ray: rhadicis are enlarged, from the right side lower lobe is
heterogeneously infiltrated with two lighter areas. What is the most possible previous diagnosis?
A.
*Right-side pneumonia with abscesses
B.
Peripheral cancer
C.
Infiltrative tuberculosis in the phase of disintegration
D.
Exudative pleurisy
E.
Infarction-pneumonia
348. Patient X., 55 years old, was admitted to the hospital recently. He complaints on cough with very
small amount of mucous-purulent sputum, significant weakness, increased temperature, which is
accompanied with chill, dizziness. Objectively: t - 38°С. Breathing rate - 22/min. Heart rate - 90/min, BP 110/70. From the right side below scapula the vocal fremitus is increased, percussion sound is shortened,
vesicular breathing is weakened, small amount of moist rales. Tones of heart are dull, rhythm is correct,
moderate tachycardia. Doctor suspected pneumonia. The presence of what syndrome let to suspect such
diagnosis?
A.
*Pulmonary tissue infiltration
B.
Intoxication
C.
Inflammation
D.
Bronchial obstruction
E.
Respiratory insufficiency
349. Patient Z., a 33-yr-old car mechanic is brought to casualty by his girlfriend. She describes a 2-day
history of rigors, sweats and intermittent confusion. On examination he is agitated, sweaty and pyrexial with
38.6° C. He is hyperventilating and cyanosed despite receiving O2 by face mask. There is dullness to
percussion and bronchial breathing at the left lung base. What method of investigation is necessary?
A.
*Chest X-ray
B.
Spiral CT with contrast
C.
Arterial blood gases
D.
Blood count and film
E.
Urea and electrolytes
350. Previously healthy 28-year-old man is evaluated in the emergency department because of fever,
productive cough, and shortness of breath. His temperature is 40 C, pulse is 120/min, respiration rate is
32/min, and blood pressure is 100/70 mm Hg. Measurement of arterial blood gases with the patient breathing
room air shows PO2 of 55 mm Hg, PCO2 of 30 mm Hg, pH of 7.41. Chest radiograph reveals bilateral
alveolar infiltrates with no effusions. Gram stain of the sputum reveals gram-positive diplococci. Which of
the following is the most appropriate for this patient?
A.
*Hospitalize him
B.
Treat him as an outpatient with oral therapy
C.
Treat him as an outpatient with intravenous therapy
D.
Hospitalize the patient in the intensive care unit
E.
All of the above
351. Previously healthy 32-yr-old woman presents with general malaise, severe cough and breathlessness,
which has not improved with a 7 day-long course of Amoxycillin. There is nothing significant to find on
examination. The X-ray shows patchy shadowing throughout the lung fields. The blood film shows clumping
of red cells with suggestion of cold agglutinins. What is the most possible etiology of disease?
A.
*Mycoplasma pneumoniae
B.
Legionella pneumonia
C.
Haemophilus influenzae
D.
Chlamydia trachomatis
E.
Klebsiella pneumoniae
352. Female, 34 years old, has an increase of body temperature to 38 °С, cough with purulent sputum,
weakness, dyspnea, pain in a thorax during breathing. During percussion there is shortening of sound in the
lower part of left lung, during auscultation – moist rales. What method of investigation is the decisive one to
confirm diagnosis?
A.
*X-ray examination
B.
Bacteriological analysis of sputum
C.
Spirometry
D.
Pneumotachometry
E.
Bronchography
353. Female B., 44 years old, complains on cough with mucous sputum, increase of temperature to 39 °С,
weakness, dyspnea, sweating. Breathing rate - 26/min, skin is moist. Below left scapula there is shortening of
percussion sound. Breathing during auscultation is weakened, moist rales. Blood test: L - 11х109/l, ESR - 29
mm/h. Your previous diagnosis?
A.
*Left-side lower lobe pneumonia
B.
Gangrene of lungs
C.
Left-side exudative pleurisy
D.
Cancer of left side lower lobe
E.
Pulmonary abscess
354. The 60-year-old patient tells you that she smoked three packs of cigarettes per day since she was 15
years old, until she was 40, and then smoked two packs per day. How many pack-years should you record in
the patient's history?
A.
45
B.
80
C.
90
D.
*115
E.
Non of above
355. The 82-year-old patient has a pulmonary infection. Which action addresses the age-related change of
increased vascular resistance to blood flow through pulmonary vasculature in this patient?
A.
Encouraging the patient to turn, cough, and deep breathe every hour
B.
*Assessing the patient's level of consciousness
C.
Raising the head of the bed
D.
Humidifying the oxygen
E.
Non of above
356. The patient diagnosed with moderate stage COPD says there is no sense in stopping smoking now
because the damage is done. Which response is the best rationale for encouraging this patient to stop
smoking?
A.
“The damage will be reversed.”
B.
*“The COPD will progress more slowly.”
C.
“Your risk for asthma development, which would further reduce your lung function, will be
decreased.”
D.
“You will be less likely to lose excessive amounts of weight and will have a more normal
appearance.”
E.
Non of above
357. The patient has broken ribs that penetrated through the skin as a result of a motor vehicle crash 3 days
ago. The patient now complains of increased pain, shortness of breath, and fever. Which assessment finding
alerts the physician to the possibility of a pleural effusion and empyema?
A.
Wheezing on exhalation on the side with the broken ribs
B.
*Absence of fremitus at and below the site of injury
C.
Crepitus of the skin around the site of injury
D.
Absence of gastric motility
E.
Non of above
358. The patient has severe nasal congestion, headache, and sneezing but no rhinorrhea, watery eyes, sore
throat, or fever. Which statement made by the patient alerts the physician to the possibility of rhinitis
medicamentosa?
A.
“I have been taking two aspirins every 6 hours for this headache.”
B.
*“My nose doesn't stay open even though I'm using nasal spray every hour.”
C.
“I have been taking a lot of vitamin C this year to keep from getting so many colds.”
D.
“The only way I can get to sleep with this nasal congestion is by taking an over-the-counter
antihistamine at night.”
E.
Non of above
359. The patient is 34 years old and has been diagnosed with COPD as a result of being homozygous for a
mutation of the alpha1-antitrypsin (AAT) gene alleles. His wife has two normal AAT gene alleles. He is
concerned that his two children may develop this problem. What is your best response?
A.
“Because neither of your parents have COPD and your wife does not have the abnormal gene alleles,
your children will not be affected.”
B.
“Because your wife is not affected nor is or a carrier, your children will have normal levels of AAT
and their risk is the same as for the general population.”
C.
“Because you have the mutations and your wife does not, your son will be at an increased risk for
developing COPD but your daughter will only be a carrier.”
D.
*“Because both of your AAT gene alleles are mutated, your children will each have one abnormal
gene and their risk for COPD is only increased if they smoke or are chronically exposed to other
precipitating factors.”
E.
Non of above
360. The patient with hospital-acquired (nosocomial) pneumonia caused by a bacterial infection with a
gram-negative microorganism is receiving treatment with intravenous amikacin (Amikin). In addition to
frequent respiratory assessment, what other assessment should the physician routinely perform to identify a
common complication of this medication?
A.
*Monitor urine output every shift
B.
Perform neuro checks every 2 hours
C.
Examine the stool and vomitus for the presence of blood
D.
Monitor the complete white blood cell count and differential daily
E.
Non of above
361. A 35-year-old man is evaluated because of a 2-week history of low-grade fevers, fatigue, cough,
pleuritic chest pain, and increasing dyspnea on exertion. He is a construction worker and is having difficulty
performing his usual tasks. He has a 10-pack-year history of cigarette smoking. On physical examination, he
has right chest pain but no respiratory distress at rest. Temperature is 38.2 C, pulse rate is 112/min and
regular, and respiration rate is 20/min. There is evidence of a right pleural effusion and no other
abnormalities. Peripheral blood leukocyte count is 9,000/L, with 80% neutrophils and 15% lymphocytes.
Liver function test results are normal. Chest radiograph shows a moderate right pleural effusion with
minimal contralateral shift and no parenchymal infiltrates. Thoracentesis yields minimally turbid yellow
fluid with results as follows: Pleural fluid nucleated cell count 3000/L with 5% neutrophils, 85%
lymphocytes, and 1% macrophages Pleural fluid total protein 5.2 g/dL Pleural fluid serum lactate
dehydrogenase 230 U/L Pleural fluid glucose 80 mg/dL Pleural fluid pH 7.36 Pleural fluid Gram and acidfast bacilli stains are negative. Tuberculin skin test is negative. Cytologic evaluation for malignant cells is
negative. What is the most likely diagnosis?
A.
*Tubercular pleurisy
B.
Lung cancer
C.
Parapneumonic effusion
D.
Pulmonary embolism
E.
Benign asbestos pleural effusion
362. A 37-yr-old man who has had recurrent chest infections since a serious bout of influenza 3-yr ago
presents with chronic productive cough. His sputum is tenacious and blood stained. On auscultation you find
crackling. What treatment is necessary?
A.
*Postural drainage
B.
Surgical excision
C.
Prednisolone
D.
Ipratropium
E.
Pleurectomy
363. A 38 years old patient, who drunk a lot of alcohol, has severe pneumonia. His condition was
worsened, the temperature of body rose to 39-40 °С, an unpleasant smell appeared from a mouth, increased
amount of purulent sputum; increased ESR and amount of band leucocytes. On the X-ray - in the lower lobe
of right lung there is massive infiltration with light area in a center. What complication is it necessary to
suspect?
A.
*Acute pulmonary abscess
B.
Bronchiectasis
C.
Infarction-pneumonia
D.
Gangrene of lungs
E.
Empyema of pleura
364. A 48 years old patient, complaints on weakness, dyspnea, pain in the left half of thorax, permanent
cough with viscid sputum, in which particles of blood are sometimes determined. For the last 3 months he
lost 5 kg of body mass. On the X-ray of lungs there is total homogeneous shade determined from the left
side. Organs of mediastinum are displaced to the left. What diagnosis is possible?
A.
*Lung athelectasis
B.
Lung gangrene
C.
Total exudative pleurisy
D.
Pneumonia
E.
Empyema of pleura
365. A 56-year-old construction worker, a heavy smoker, sustains severe trauma to his left chest. Chest
pain is severe for several minutes but subsides over the next hour. Because the chest pain does not resolve
completely, he is evaluated in the emergency department 2 hours later, where results of a chest radiograph
and complete blood count with differential are normal. The next day, he leaves on a week-long vacation to
South America. During that time, he has intermittent chest discomfort and gradually increasing dyspnea with
exertion. Upon returning home, he sees his physician because of dyspnea. Chest radiograph shows a large
left-sided pleural effusion with minimal contralateral shift. At thoracentesis, 500 cc of brownish-colored
fluid is removed and analyzed with the following results. Pleural fluid nucleated cell count 4000/mL with
10% neutrophils, 30% lymphocytes, 15% macrophages, and 45% eosinophils Pleural fluid hematocrit 10%
Pleural fluid total protein 4 g/dL Pleural fluid serum lactate dehydrogenase 200 UIL Pleural fluid glucose 80
mg/dL Pleural fluid pH 7.35 Cytology test results are negative. Pain medication is prescribed for the patient.
When he returns 14 days later, complete blood count shows a leukocyte count of 9000/?L with 20%
eosinophils and chest radiograph shows that the pleural effusion has decreased substantially. Which of the
following is the most likely diagnosis?
A.
*Post-traumatic hemothorax
B.
Benign asbestos pleural effusion
C.
Paragonimiasis
D.
Lung cancer
E.
Pulmonary infarction
366. A 60-year-old man is evaluated because of a 6-week history of progressive dyspnea on exertion,
fatigue, a decrease in appetite, and a weight loss of 1.8 kg. He has a 30-pack-year history of cigarette
smoking and drinks two or three cocktails every evening. He has no gastrointestinal complaints and no
history of a febrile illness. On physical examination, he is afebrile with normal vital signs. The only
abnormalities noted on chest examination are findings compatible with a right pleural effusion. Chest
radiograph confirms a pleural effusion occupying 40% of the right hemithorax without evidence of
loculation. There are no obvious parenchymal lesions and no mediastinal adenopathy. Results of pleural
fluid analysis are as follows. Pleural fluid nucleated cell count 2800/?L with 10% neutrophils, 50%
lymphocytes, 30% macrophages, and 10% mesothelial cells Pleural fluid total protein 3.8 g/dL (pleural
fluid/serum ratio 0.60) Pleural fluid serum lactate dehydrogenase 210 U/L (ratio of pleural fluid to upper
limits of normal serum lactate dehydrogenase 0.72) Pleural fluid amylase 30 mg/dL (pleural fluid/serum
ratio 0.5) Pleural fluid glucose 50 mg/dL Pleural fluid pH 7.26 Which of the following is the most likely
diagnosis?
A.
*Malignant effusion
B.
Complicated parapneumonic effusion
C.
Esophageal rupture
D.
Rheumatoid pleurisy
E.
Acute pancreatitis
367. A 60-year-old man with a history of alcohol abuse is evaluated because of the insidious onset of
dyspnea over the course of 6 weeks. He has no cough, chest or abdominal pain, or hemoptysis. He smoked
1.5 packs of cigarettes per day for 25 years and stopped smoking 4 years ago. He has a moderate-sized right
pleural effusion. Chest radiograph shows the effusion with minimal contralateral mediastinal shift and is
otherwise normal. Pleural fluid analysis shows clear yellow fluid with 500 nucleated cells/L, 10%
neutrophils, 25% lymphocytes, 60% macrophages, and 5% mesothelial cells. Pleural fluid values are as
follows: Total protein 1.1 g/dL Serum lactate dehydrogenase 4l U/L Serum amylase 20 U/L Glucose 100
mg/dL pH 7.45 Ratio of pleural fluid to serum (PF/S) total protein 0.2 Ratio of pleural fluid to serum lactate
dehydrogenase (upper limits of normal) 0.35 Other laboratory results show a serum albumin of 2.4 g/dL, an
INR of 1 .5, and a normal urinalysis. Electrocardiogram is normal. Which of the following is the most likely
diagnosis?
A.
*Hepatic hydrothorax
B.
Congestive heart failure
C.
Chronic pancreatitis
D.
Lung cancer
E.
Nephrotic syndrome
368. A 72-year-old woman is evaluated because of morning headaches and swelling in the lower
extremities that worsens as the day progresses. She is able to sleep supine, using one pillow at night, and
notices shortness of breath walking distances greater than 20 feet. On physical examination, significant
findings include diminished breath sounds, distant heart sounds, with pulmonic valve component equal in
intensity to aortic valve component, and paradoxical splitting of the S2. Laboratory finding include a Pa
O2of 59mm Hg, Pa CO2 of 44 mm Hg, and pH of 7.41. Electrocardiogram shows right ventricular
hypertrophy with cor pulmonale and right axis deviation. Which of the following is the best rationale for
long-term oxygen therapy for this patient?
A.
*Evidence of cor pulmonale and a PaO2 between 55 and 60 mm Hg
B.
The PaO2 is less than or equal to 65 mm Hg
C.
Her morning headaches
D.
Dyspnea
E.
All of the above
369. A man of 38 years old felt ill 2 weeks ago with cough, weakness, increased temperature up to 38,0
°С. His condition suddenly got worse after 7 days of the disease, when chills and sweating appeared, and
evening temperature increased up to 39,0 °С. 2 days prior to hospitalization the patient after cough
expectorated a large amount of stinky bloody sputum; after this he felt better. Objectively: pulse - 80/min,
brteathing rate - 20/min, t - 37,6 °С. What changes are possible on the chest X-ray?
A.
*Presence of cavity with horizontal level of fluid
B.
Displacement of mediastinum to the side of homogeneous shade
C.
The homogeneous rounded shade is in the pulmonary tissue
D.
Shade in a lower lobe with diagonal upper border
E.
Darkening of lung lobe
370. A student complaints on cough with production of mucus-purulent sputum, sometimes with particles
of blood, t - 37,6 °С, weakness, sweating. Since childhood he was often ill with cold, for last several years
there were exacerbations of COPD twice. Doctor suspected bronchiectatic disease in this patient. What
method of investigation may confirm this diagnosis?
A.
*Bronchography
B.
Anamnesis morbi
C.
Physical examination of lungs
D.
Scintigraphy of lungs
E.
Tomography of lungs
371. Female patient K., 46 years old, after decreasing of fever after flue noticed pain appeared in a thorax,
cough with yellow-green sputum (amount-150 ml a day), sometimes with some blood. Objectively:
breathing rate - 36/min. In lungs from the right side lower scapula there is dull sound during percussion, hard
breathing, and moist rales. Blood test: L - 18,6х109/l, ESR -64 mm/h. Analysis of sputum: L -80-100 , Er 40-50, elastic fibers, cocci. X-ray: lung roots are enlarged, from the right side lower lobe is heterogeneously
infiltrated with two lighter areas. What is the most possible previous diagnosis?
A.
*Right-side pneumonia with abscesses
B.
Peripheral cancer
C.
Infiltrative tuberculosis in the phase of disintegration
D.
Exudative pleurisy
E.
Infarction-pneumonia
372. Female, 34 years old, has an increase of body temperature up to 38 °С, cough with purulent sputum,
weakness, dyspnea, pain in a thorax during breathing. During percussion there is shortening of sound in the
lower part of left lung, during auscultation – moist rales. What method of investigation is the decisive one to
confirm diagnosis?
A.
*X-ray examination
B.
Bacteriological analysis of sputum
C.
Spirometry
D.
Pneumotachometry
E.
Bronchography
373. Male patient G., 56 years old, complaints on permanent pain in a thorax which disturbs for the last 2
months. Pain is not connected with breathing. There is also moist cough with particles of blood in sputum.
Weakness, fatigue are present. On the chest X-ray in the lower lobe of right lung there is spherical shadow,
with size of 4x6 cm, related to the lung’s rhadicis. What is the most possible diagnosis?
A.
*Perypheral lung cancer
B.
Tuberculoma
C.
Metastasis
D.
Pulmonary abscess
E.
Pneumonia
374. Man of 26 years old after measles (in childhood) constantly suffers from cough with production of
mucus-purulent sputum up to 200 ml, mainly in the morning. There is periodical increase of temperature up
to 38 °С. At auscultation - dry, and in lower parts - moist rales. On chest X-ray - there is “web-like”
deformation of pulmonary picture. Your diagnosis?
A.
*Bronchiectasis
B.
COPD
C.
Bronchial asthma
D.
Tuberculosis
E.
Pneumosclerosis
375. Man of 26 years old, complaints on prickly pain during breathing, cough, dyspnea. Objectively: t 37,5 °С, breathing rate - 19/min, heart rate and pulse - 92/min; BP - 120/80. Breathing is vesicular. From the
left side in lateral and lower regions of thorax in the phase of inspiration and expiration there is sound which
increases during pressure with stethoscope and it is preserved after cough. EKG - without pathological
changes. Your diagnosis?
A.
*Acute pleurisy
B.
Spontaneous pneumothorax
C.
Intercostal neuralgia
D.
Subcutaneous emphysema
E.
Dry pericarditis
376. Man of 50 y.o. complaints on dyspnea, cough, pain in the left part of thorax, increasing of
temperature up to 37,5 °С. He felt ill three weeks ago. He smokes for 30 years. Objectively: low feeding,
cyanosis of lips, emphysematous thorax, on the left side – from the 6th rib and below a dull sound is present.
Breathing is absent between scapulas from the left side. X-ray: intensive homogeneous darkening from the
left side, shade of heart is shifted to the right. Your diagnosis?
A.
*Exudative pleurisy
B.
Athelectasis of lungs
C.
Infiltrative tuberculosis
D.
Dry pleurisy
E.
Pneumonia
377. Man of 56 years old, alcoholic, felt ill suddenly: temperature increased to 40 °С, weakness, cough
appeared with production of dark sputum. Objectively: condition is severe. T - 39,5 °С. Breathing rate 30/min. Heart rate is 100/min., BP - 110/70. In lungs from the right side there are moist rales. Tones of heart
are dull, rhythm is correct, tachycardia. On the chest X-ray film of lungs there is infiltration of right upper
lobe. What complication is the most possible?
A.
*Pulmonary abscess
B.
Endocarditis
C.
Bronchiectasis
D.
Pericarditis
E.
Pulmonary bleeding
378. Man of 54 y.o., complaints on pain in a thorax, dyspnea, cough with bloody sputum. In anamnesis:
long history of cough with purulent sputum up to 200 ml per day, mostly in the morning, periodically
increasing of temperature up to 37,8 °С, sweating, chills. He smokes since 14. Objectively: low weight, skin
with grey tint, edema of face, fingers has shape of "drumsticks", in lungs - pulmonary and bang-box sound,
in some parts dull sound, dry and moist rales. In blood: leucocytosis, moderately increased ESR. What is the
most possible cause of pulmonary bleeding in this case?
A.
*Bronchiectasis
B.
Tuberculosis
C.
Chronic bronchitis
D.
Pulmonary abscess
E.
Pulmonary cancer
379. On the X-ray of 46 y.o. patient, which complaints on increase of temperature and cough with sputum,
in the 10th segment of left lung was founded ring-shape shade with a diameter 8 cm with thick walls and
horizontal level. Other pulmonary parts - without changes. Diagnosis?
A.
*Pulmonary abscess
B.
Bronchiectasis
C.
Tubercular cavern
D.
Tuberculoma
E.
Gangrene of lungs
380. Patient N., 35 y.o., complaints on cough with large amount of purulent sputum in the morning,
sometimes with particles of blood, general weakness, loss of weight. The general condition is satisfactory,
thorax of emphysematous form, breathing rate - 18/min. During auscultation there is rough breathing, diffuse
dry rales. What instrumental method of investigation is the decisive one for diagnosis?
A.
*Bronchography
B.
Chest X-ray
C.
Bronchoscopy with biopsy
D.
Spirometry
E.
Scintigraphy
381. Patient of 19 years old, complaints on increase of temperature up to 39 °С at evenings, severe cough,
production of large amount of sputum with an unpleasant smell. He is ill for several years, the last
exacerbation is related to overcooling. During comparative percussion of lungs – in right lower parts
pulmonary sound is dull, during auscultation – moist rales. What disease is the most possible?
A.
*Bronchiectasis
B.
COPD
C.
Abscess of lungs
D.
Gangrene of lungs
E.
Community-acquired pneumonia
382. Patient of 20 years old complaints on increase of temperature up to 37,5 °С, dyspnea, cough with
purulent sputum, sometimes with particles of blood. Objectively: fingers has shape of "drumsticks". During
auscultation breathing is hard, dry and moist wheezes are present. On X-ray – pulmonary fibrosis. What is
the diagnosis?
A.
*Bronchiectasis
B.
Bronchial asthma
C.
COPD
D.
Abscess of lungs
E.
Canceromatosis of lungs
383. Patient of 25 years old complaints on pain in the left part of thorax, absence of appetite, severe
sweating; after 7-days of permanent fever during an attack of cough he produced 150 ml of yellow sputum.
Objectively: temperature - 38,7 C; breathing rate – 22/min; pulse-96/min; BP 110/70. Chest X-ray: from the
left side there is shade of round form with clear borders, with light areas in a center. What auscultative sign
will confirm the diagnosis of this patient?
A.
*Amphoryc breathing
B.
Crepitation
C.
Moist rales
D.
Bronchial breathing
E.
Dry rales
384. Patient of 28 years old, complaints on severe prickly pain in the left part of thorax, connected with
breathing, dry cough, subfebrile temperature. He is ill for 3 days, after overcooling. Objectively: position is
on the right side, left part of thorax falls behind during breathing, there is clear pulmonary sound at
percussion, vesicular breathing is decreased from the left side, in the phase of inspiration and expiration
there is sound which increases during pressure with stethoscope and it doesn’t disappear saved after cough.
On chest X-ray pulmonary fields are without changes, left sinus is opened incompletely. The most possible
diagnosis:
A.
*Dry pleurisy
B.
Pneumonia
C.
Acute bronchitis
D.
Spontaneous pneumothorax
E.
COPD
385. Patient of 50 years old after overcooling developed a fever up to 40 °С, dyspnea and pain in the right
part of thorax are present. Approximately 100 ml of purulent sputum with bloody particles and unpleasant
smell was producted. On chest X-ray in the right lung there is unhomogeneous massive infiltration with two
bright areas. Mycobacteria tuberculosis and atypical cells in sputum were not founded. Diagnosis?
A.
*Acute abscess of lungs
B.
Gangrene of lungs
C.
Infiltrative tuberculosis with destruction
D.
Tumor of lungs with destruction
E.
Empyema of pleura
386. Patient of 50 years old, entered hospital on the 9th day of disease with complaints on increase of
temperature up to 38,5 °С, acute weakness, pain in the area of right scapula during breathing, dry cough.
Objectively: Breathing rate -28/min. Pulse - 100/min, signs of intoxication. Under the area of right scapula
there is dull percussion sound, bronchial breathing, single moist rales and crepitation. Three days later there
was an attack of cough with production of 200 ml of purulent sputum, after that temperature of body
decreased. On the chest X-ray film under the corner of scapula an infiltration of lungs of round shape with
horizontal level of fluid was found. Diagnosis?
A.
*Acute pulmonary abscess
B.
Pulmonary cyst
C.
Cancer of lungs with destruction
D.
Bronchiectasis
E.
Empyema of pleura
387. Patient came to doctor with exacerbation of COPD. During examination were founded signs of
respiratory insufficiency of II degree. What symptom is the main clinical sign of respiratory insufficiency of
the II degree?
A.
*Presence of dyspnea during usual physical exertion
B.
Worsening of external breathing functions
C.
Presence of arterial hypoxemia as cyanosis
D.
Hypertrophy of muscles of neck and abdominal press
E.
Perypheral oedema
388. Patient complaints on severe cough with production of 600 ml a day purulent sputum of chocolate
color with a putrid smell. Felt ill suddenly, temperature - 39 °С. On the chest X-ray film there is an area of
darkening with cavities in a center, with unclear contours and with the level of fluid. What disease may be
suspected?
A.
*Pulmonary gangrene
B.
Tubercular cavern
C.
Pulmonary abscess
D.
Bronchiectasis
E.
Cancer of lungs with destruction
389. Patient has fever, temperature - 39 °С, there is cough with production of sputum with unpleasant
smell and particles of blood. During auscultation of lungs in right lower lobe there is amphoric breathing,
moist rales. On the chest X-ray film: in the right lower lobe there is a cavity 4 cm in diameter, with the level
of fluid. What is the possible diagnosis?
A.
*Pulmonary abscess
B.
Gangrene of lungs
C.
Infiltrative tuberculosis
D.
Cancer of lungs
E.
Pneumonia
390. Patient M., 39 years old, is ill with attacks of cough with yellow-brown sputum, pain in a right side,
related to the deep breathing, sweating. He is ill for 6 days, after overcooling. Used aspirin. Objectively: T 39,6 °С, breathing rate - 26/min., pulse - 110/min., BP -110/70. In lower part of right lung - moist loud rales.
X-ray: in right lower lobe there is massive unhomogeneous infiltration with lighter areas, sinus is not
changed. What complication of disease is the most possible?
A.
*Abscesses
B.
Dry pleurisy
C.
Empyema of pleura
D.
Spontaneous pneumothorax
E.
Pulmonary athelectasis
391. Patient N., 31 y.o., complaints on cough with production of mucus-purulent sputum with an
unpleasant smell by “full mouth”, subfebrile temperature, dyspnea, loss of body mass. He is ill since
childhood. Objectively: skin is pale, fingers are changed as "drumsticks", nails - "sentinel glass", percussion
sound is mosaical, breathing is hard, places of moist rales in the places of dull percussion sound. Your
diagnosis:
A.
*Bronchiectasis
B.
Viral pneumonia
C.
COPD in the phase of exacerbation
D.
Abscess of lungs
E.
Hypoplasia of lungs
392. Patient of 54 years old, complaints on shortness of breath during mild physical exertion, cough with
some amount of sputum. Objectively: diffuse cyanosis. Thorax of emphysematous form. In lungs there is a
bit weakened vesicular breathing with prolonged expiration, dry wheezes. BP -140/80. Pulse - 92/min,
rhythmical. Spyrometry: FVC – 36 %, FEV1 – 49 %, FEV1/FVC - 50 %. What is the type of respiratory
insufficiency in this patient?
A.
*Mixed
B.
Restrictive
C.
Obstructive
D.
Respiratory insufficiency is not present
E.
It is impossible to make conclusion
393. Patient of 32 y.o. complaints on dyspnea, pain in the right side of thorax during breathing, cough with
ferruginous sputum, fever with chills, weakness. Objectively: breathing rate - 24/min, vocal fremitus is
increased in the back-lower region of the right part of thorax, during percussion sound is dull, at auscultation
– crepitation is heard. After 5 days of treatment dyspnea increased, vocal fremitus became weaker, and it is
almost impossible to hear vesicular breathing. What complication developed in this patient?
A.
*Exudative pleurisy
B.
Athelectasis
C.
Carnification of lungs
D.
Abscesses
E.
Pneumothorax
394. Patient of 32 y.o., who drunk a lot of alcohol, has severe pneumonia. His condition was worsened, the
temperature of body rose to 39-40 °С, an unpleasant smell appeared from a mouth, increased amount of
purulent sputum; increased ESR and amount of band leucocytes. On the X-ray - in the lower lobe of right
lung there is massive infiltration with bright area in a center. What complication is it necessary to suspect?
A.
*Acute pulmonary abscess
B.
Bronchiectasis
C.
Infarction-pneumonia
D.
Gangrene of lungs
E.
Empyema of pleura
395. Patient of 51 y.o. complaints on severe dyspnea, pain in the left part of thorax. Objectively: breathing
rate - 30/min, heart rate - 108/min. Above the left part of thorax vocal fremitus is significantly decreased,
during percussion sound is dull, at auscultation – weak vesicular breathing. On chest X-ray - homogeneous
shade up to the level of the 2nd rib with diagonal upper border, organs of mediastinum are displaced to the
right. Method of choice in the treatment of this patient is:
A.
*Pleural punction
B.
Intravenous usage of large doses of glucocorticosteroids
C.
Emergent bronchoscopy
D.
Inhalation of b2-agonists of short action
E.
Intravenous antibiotics
396. Patient of 52 y.o. came to the doctor with complaints on temperature 38,6 С, weakness, sweating,
cough with production of purulent sputum (up to 100 ml a day) with an unpleasant smell, pain in the right
part of thorax during breathing. On the chest X-ray it was found “ring-like” shade with the level of fluid.
When abscess may be called “chronic”?
A.
*In case of absence of healing signs after a monthly treatment
B.
In case of presence of complications
C.
In the case of development of chronic bronchitis
D.
In the case of development of diffuse pneumosclerosis
E.
In the case of development of pulmonary insufficiency
397. Patient V. complaints on dyspnea at rest, fever, sweating, pain in a thorax. During examination the
right part of thorax is slowed down in the act of breathing, percussion - dull sound, auscultation - absence of
respiratory sounds. On the X-ray: homogeneous darkening of 2/3 of right lung. The most informative
diagnostic method in this case:
A.
*Pleural punction
B.
Pneumotachometry
C.
Bronchography
D.
Bronchoscopy
E.
Spyrometry
398. Student of 17 years old, in childhood was often ill with respiratory diseases. In a period between
respiratory viral diseases, cough was present with the production of sputum. Once noticed particles of blood
in sputum. In lungs, especially from the right side, there are different moist rales. Sputum is mucouspurulent, up to 50 ml per day. On the chest X-ray film – fibrosis and “web” picture mostly in the right lower
lobe. What is the most possible diagnosis?
A.
*Bronchiectasis
B.
Chronic pulmonary disease
C.
Chronic bronchitis
D.
Metapneumonic pneumosclerosis
E.
Metatubercular pneumosclerosis
399. Patient of 36 years old, complaints on dyspnea, feeling of pressure in the right half of thorax,
increasing of temperature up to 38,7 °С, cough with production of small amount of mucus-purulent sputum.
He is ill for a week, after overcooling. Objectively: light acrocyanosis of lips, pulse is rhythmic, 90/min, BP 140/85. The right half of thorax is slowed down in the act of breathing. Percussion – from the right side
below corner of scapula there is dull sound. In this region breathing sounds are not heard. What is the most
possible diagnosis?
A.
*Right side exudative pleurisy
B.
Bronchiectasis
C.
Right side lower lobe pneumonia
D.
Right side pulmonary athelectasis
E.
Right side abscess of lungs
400. Patient of 68 years old, complaints on dry cough, elevation of temperature up to 37,5 °С, pain in a
thorax during breathing. 5 days ago he had trauma of thorax. Objectively: pallor, lag of right half of thorax
during breathing. Auscultation – weakened breathing and sound of friction of pleura in right lower region. In
blood: leucocytosis, increased ESR. X-ray - pulmonary fields are not changed. Your clinical diagnosis?
A.
*Dry pleurisy
B.
Pneumonia
C.
Exudative pleurisy
D.
Thraumatic pneumothorax
E.
Cancer of lungs
401. Patient D., 47 years old, came to the doctor with complaints on fever up to 39 °С, sweating, dry
cough, dyspnea, pain in the right part of thorax during deep breathing and cough. Felt ill suddenly six days
ago. Objectively: condition is severe, skin is pale, breathing rate - 28/min, breathing sounds are not heard in
the lower lobe of right lung, during percussion there is dull sound, during auscultation - breathing is
significantly weakened. Your previous diagnosis:
A.
*Right-side exudative pleurisy
B.
Pneumonia
C.
Acute bronchitis
D.
Thromboembolia of pulmonary artery
E.
Right-side hydrothorax
402. Patient L., 26 years old, with left-side lower-lobe pneumonia during cough feels acute pain in the left
part of thorax. Objectively: diffuse cyanosis, dilation of the left half of thorax, at percussion - tympanitis,
auscultation - absence of respiratory sounds above the left part of thorax. Displacement of right border of
heart to the medioclavicular line. What method of investigation will be the most informative in this case:
A.
*Chest X-ray
B.
Bronchoscopy
C.
Bronchography
D.
Pneumotachymetry
E.
Spyrometry
403. Student of 22, felt ill suddenly. He has fever up to 39 °С, cough, pain under the right scapula during
inspiration and cough. In 3 days the dyspnea increased, respiratory rate is up to 32/min. Below the corner of
right scapula there is dull percussion sound, vocal fremitus is decreased, breathing sounds are not
auscultated. The most effective method of treatment is:
A.
*Pleural punction
B.
Prescribing of furosemid
C.
Bed regimen
D.
Prescribing of cephalosporin
E.
Physiotherapy
404. The patient who has experienced blunt trauma to the chest is at risk for developing a hemothorax.
Which would the physician expect to find in a patient with a hemothorax?
A.
Hemoptysis
B.
Paradoxical chest movements
C.
*Percussion dullness on affected side
D.
Hypertympanic sound on affected side
E.
Non of above
405. Woman of 55 years old, complaints on dull pain in the right half of thorax, unproductive cough,
dyspnea. In anamnesis – rheumatic fever. During examination of lungs from the right side from the third rib
and below there is dull sound at percussion, breathing in this area is significantly weakened. On the chest Xray: homogeneous darkening with diagonal upper border from the 3rd rib and below. Analysis of received
exudate: Rivalt test - positive, microscopy revealed lymphocytes. Previous diagnosis?
A.
*Exudative serous pleurisy
B.
Right-side hydrothorax
C.
Empyema of pleura
D.
Chylothorax
E.
Mezotelioma of pleura
406. Woman of 58 years old, entered the hospital with complaints on dyspnea and palpitation.
Objectively: condition is severe, nervous, breathing is noisy with participation of additional breathing
muscles, periodical cramps, diffuse cyanosys. In lungs – diffuse dry rales, in lower parts of lungs breathing is
significantly weaker. Pulse - 100/min, oedema, 3 extrasystoles/min, BP - 140/100, Ра O2 - 45, pH - 7,3.
What is the main syndrome in this case?
A.
*Respiratory insufficiency
B.
Blood hypertension
C.
Tachycardia
D.
Arrhythmia
E.
Heart failure
407. A 27-year-old male presents to the health center for right upper quadrant abdominal pain, generalized
pruritus, and jaundice for 3 days. He states that the pain came on gradually and awoke him early on the
morning of presentation. His past medical history is pertinent for ulcerative colitis, although he has not taken
any medication in 4 years. His temperature is (39.2°C) and physical exam shows pain in the right subcostal
region with deep inspiration, and generalized jaundice. What is the most serious complication of the most
likely diagnosis?
A.
Perforation of rectum.
B.
Perforation of stomach wall.
C.
Infarction of small bowel.
D.
*Cholangiocarcinoma.
E.
Pancreatic pseudocyst formation.
408. A 38-year-old man who works as a reporter for a travel magazine comes to his physician because of
the acute onset of jaundice, malaise, and temperatures to 38.5 С .He had returned from Burma 2 weeks ago,
where he spent 4 weeks. He says that he abstains from alcohol beverages and does not take any medications.
Laboratory studies show elevated serum aminotransferases, high bilirubin (both total and direct), and
negative serology for hepatitis A virus (HAV) and С virus (HCV) infection. He was vaccinated for hepatitis
В virus (HBV) 3 years ago and is now positive for anti-HBsAg antibodies. Which of the following serologic
markers should be tested as the most appropriate next step in diagnosis?
A.
Anti-HCV IgG antibodies by RIBA
B.
Anti-HDV IgG antibodies
C.
*Anti-HEV IgM antibodies
D.
Anti-HGV IgG antibodies
E.
HBsAg
409. A 40-year-old male with long-standing alcohol abuse complains of abdominal swelling, which has
been progressive over several months. He has a history of gastrointestinal bleeding. On physical exam, there
are spider angiomas and palmar erythema. Abdominal collateral vessels are seen around the umbilicus.
There is shifting dullness, and bulging flanks are noted. An important first step in the patient’s evaluation is
A.
*Diagnostic paracentesis
B.
UGI series
C.
Ethanol level
D.
CT scan
E.
Esophagogastroduodenoscopy
410. A 40-year-old white female complains of pruritus. She has an elevated alkaline phosphatase and
positive antimitochondrial antibody test. What is the most likely disease?
A.
*Primary biliary cirrhosis
B.
Sclerosing cholangitis
C.
Anaerobic liver abscess
D.
Hepatoma
E.
Hepatitis C
411. A 40-year-old white female complains of pruritus. She has an elevated alkaline phosphatase and
positive antimitochondrial antibody test. What is the most likely disease?
A.
*Primary biliary cirrhosis
B.
Sclerosing cholangitis
C.
Anaerobic liver abscess
D.
Hepatoma
E.
Hemochromatosis
412. A 40-year-old white female complains of pruritus. She has an elevated alkaline phosphatase and
positive antimitochondrial antibody test. What is the most likely disease?
A.
*Primary biliary cirrhosis
B.
Sclerosing cholangitis
C.
Anaerobic liver abscess
D.
Hepatitis D
E.
Hemochromatosis
413. A 40-year-old white female complains of pruritus. She has an elevated alkaline phosphatase and
positive antimitochondrial antibody test. What is the most likely disease?
A.
*Primary biliary cirrhosis
B.
Sclerosing cholangitis
C.
Hepatitis C
D.
Hepatitis D
E.
Hemochromatosis
414. A 40-year-old white female complains of pruritus. She has an elevated alkaline phosphatase and
positive antimitochondrial antibody test. What is the most likely disease?
A.
*Primary biliary cirrhosis
B.
Hepatoma
C.
Hepatitis C
D.
Hepatitis D
E.
Hemochromatosis
415. A 41-year-old morbidly obese female comes to the emergency department with colicky abdominal
pain in her right upper abdomen. She complains that this is similar to, yet more severe than, the pain that
often occurs after meals for the past 4 months. Her past medical history is positive for diabetes mellitus type
2, hypertension, hyperlipidemia, and smoking. On physical exam, her temperature is 100.5°F (38.1°C) and
her sclera appear mildly icteric. What imaging modality may be limited in this patient?
A.
MRCP.
B.
CT scan.
C.
*Ultrasonography.
D.
Esophogastroduodenoscopy (EGD).
E.
HIDA scan.
416. A 45 year-old man undergoes a routine physical examination with screening blood studies. Physical
examination is notable for an increased liver diameter; the liver edge is palpable and without irregularities.
Blood studies show elevated liver enzymes. The clinician suspects alcoholic hepatitis. Which of the
following findings would tend to support this diagnosis?
A.
Alanine aminotransferase = 2000 U/L
B.
*Aspartate aminotransferase (AST)/alanine aminotransferase (ALT) ratio = 2.5
C.
Gamma-glutamyl transferase (GGT) = 20 U/L (norm
D.
Mean corpuscular volume (MCV) = 65 urn3
E.
Platelet count = 600,000/mm3
417. A 45-year-old female with long-standing alcohol abuse complains of abdominal swelling, which has
been progressive over several months. On physical exam, there are spider angiomas and palmar erythema.
Abdominal collateral vessels are seen around the umbilicus. A paracentesis is performed. The serum albumin
minus ascitic fluid albumin equals 1.4 g/dL. The most likely diagnosis is
A.
*Portal hypertension
B.
Pancreatitis
C.
Tuberculous peritonitis
D.
Hepatoma
E.
No ascitis
418. A 47-yr-old agricultural worker complains of a chronic cough, purulent sputum and abdominal
distention. He has just arrived in England from Spain where he was picking grapes. Choose the single most
likely diagnosis from the list of options above.
A.
*Tuberculosis
B.
Cirrhosis
C.
Malabsorption
D.
Pancreatitis
E.
Peptic ulcer
419. A 51-year-old female comes to the emergency department complaining of left lower quadrant
abdominal pain. She describes an acute illness accompanying the pain with subjective fever and diarrhea
over the last 8 hours. Abdominal exam shows tenderness in the LLQ of the abdomen, no rebound tenderness
at McBurney's point, and negative Murphy's sign. What imaging modality is most appropriate for this
patient?
A.
Ultrasound of the abdomen.
B.
*CT scan with and without contras
C.
Colonoscopy.
D.
Barium enema.
E.
Plain upright abdominal x-ray.
420. A 60-year-old alcoholic man is admitted to the emergency department with hematemesis. His pulse is
110/min, blood pressure is 100/60 mm Hg, and respirations are 19/min. He has multiple spider angiomata on
his back and chest, with bilateral gynecomastia. Abdominal examination is significant for
hepatosplenomegaly, and his abdomen is distended and tympanic on percussion; a fluid level is easily
detectable. His testicles are small, and a rectal examination produces guaiac-negative stool. His hematocritis
23%. After placement of a nasogastric tube, 400 mL of bright red blood is evacuated. After initial fluid
resuscitation, which of the following is the most appropriate next step in management?
A.
Barium swallow
B.
Esophageal balloon tamponade
C.
*Esophagogastroscopy
D.
Exploratory celiotomy
E.
Selective angiography
421. A 60-year-old alcoholic man is admitted to the emergency department with hematemesis. His pulse is
110/min, blood pressure is 100/60 mm Hg, and respirations are 19/min. He has multiple spider angiomata on
his back and chest, with bilateral gynecomastia. Abdominal examination is significant for
hepatosplenomegaly, and his abdomen is distended and tympanic on percussion; a fluid level is easily
detectable. His testicles are small, and a rectal examination produces guaiac-negative stool. His hematocritis
23%. After placement of a nasogastric tube, 400 mL of bright red blood is evacuated. After initial fluid
resuscitation, which of the following is the most appropriate next step in management?
A.
Barium swallow
B.
Esophageal balloon tamponade
C.
*Esophagogastroscopy
D.
Exploratory celiotomy
E.
Transjugular intrahepatic portosystemic shunt
422. A 67-year- male presents with a complaint of fatigue. There is no history of alcohol abuse or liver
disease. Scleral icterus is noted on physical exam. The liver and spleen are nonpalpable. The patient is noted
to have a normocytic, normochromic anemia. The first step in evaluation of this patient is
A.
CT scan of the abdomen
B.
Hepatitis profile
C.
*Liver function tests, including direct versus indirect bilirubin and urine bilirubin
D.
Abdominal ultrasound
E.
Percutaneous transhepatic Cholangiography
423. A 67-year-old male presents with conjugated hyperbilirubinema, with bilirubin detected in the urine.
Serum bilirubin is 12 mg/dL, AST and ALT are in normal range, and alkaline phosphatase is 300 U/L (3
times normal). The next step in evaluation is
A.
*Ultrasound or CT scan
B.
Hepatitis profile
C.
Reticulocyte count
D.
Family history for hemochromatosis
E.
Colonoscopy
424. A 68-year-old male presents with a complaint of fatigue. . There is no history of alcohol abuse or
liver disease. Scleral icterus is noted on physical exam. The liver and spleen are nonpalpable. The patient is
noted to have conjugated hyperbilirubinema, with bilirubin detected in the urine. Serum bilirubin is 12
mg/dL, AST and ALT are in normal range, and alkaline phosphatase is 300 U/L (3 times normal). The next
step in evaluation is
A.
*Ultrasound or CT scan
B.
Hepatitis profile
C.
Reticulocyte count
D.
Family history for hemochromatosis
E.
Esophagogastroduodenoscopy
425. A 70-year-old male presents with a complaint of fatigue. There is no history of alcohol abuse or liver
disease; the patient is on no medication. Scleral icterus is noted on physical exam. There is no evidence for
chronic liver disease on physical exam, and the liver and spleen are nonpalpable. The patient is noted to have
a normocytic, normochromic anemia. The first step in evaluation of this patient is
A.
CT scan of the abdomen
B.
Hepatitis profile
C.
*Liver function tests, including direct versus indirect bilirubin and urine bilirubin
D.
Abdominal ultrasound
E.
Esophagogastroduodenoscopy
426. A 71-year-old male presents with a complaint of fatigue. There is no history of alcohol abuse or liver
disease; the patient is on no medication. Scleral icterus is noted on physical exam. The patient is noted to
have conjugated hyperbilirubinema, with bilirubin detected in the urine. Serum bilirubin is 12 mg/dL, AST
and ALT are in normal range, and alkaline phosphatase is 300 U/L (3 times normal). The next step in
evaluation is
A.
*Ultrasound or CT scan
B.
Hepatitis profile
C.
Reticulocyte count
D.
Family history for hemochromatosis
E.
Esophagogastroduodenoscopy
427. A 72-year-old male presents with a complaint of fatigue. There is no history of alcohol abuse or liver
disease; the patient is on no medication. Scleral icterus is noted on physical exam. The patient is noted to
have conjugated hyperbilirubinema, with bilirubin detected in the urine. Serum bilirubin is 12 mg/dL, AST
and ALT are in normal range, and alkaline phosphatase is 300 U/L (3 times normal). The next step in
evaluation is
A.
*Ultrasound of abdomen
B.
Hepatitis profile
C.
Reticulocyte count
D.
Family history for hemochromatosis
E.
Esophagogastroduodenoscopy
428. A 73-year-old male presents with a complaint of fatigue. There is no history of alcohol abuse or liver
disease; the patient is on no medication. Scleral icterus is noted on physical exam. The patient is noted to
have conjugated hyperbilirubinema, with bilirubin detected in the urine. Serum bilirubin is 12 mg/dL, AST
and ALT are in normal range, and alkaline phosphatase is 300 U/L (3 times normal). The next step in
evaluation is
A.
*Ultrasound or CT scan
B.
Hepatitis profile
C.
Reticulocyte count
D.
Family history for hemochromatosis
E.
Colonoscopy
429. A 77-year- male presents with a complaint of fatigue. . There is no history of alcohol abuse or liver
disease. Scleral icterus is noted on physical exam. The liver and spleen are nonpalpable. The patient is noted
to have conjugated hyperbilirubinema, with bilirubin detected in the urine. Serum bilirubin is 12 mg/dL,
AST and ALT are in normal range, and alkaline phosphatase is 300 U/L (3 times normal). The next step in
evaluation is
A.
*Ultrasound of abdomen
B.
Hepatitis profile
C.
Reticulocyte count
D.
Percutaneous transhepatic Cholangiography
E.
Esophagogastroduodenoscopy
430. A 78-year-old male presents with a complaint of fatigue. There is no history of alcohol abuse or liver
disease; the patient is on no medication. Scleral icterus is noted on physical exam. Scleral icterus is noted on
physical exam. The liver and spleen are nonpalpable. The patient is noted to have conjugated
hyperbilirubinema, with bilirubin detected in the urine. Serum bilirubin is 12 mg/dL, AST and ALT are in
normal range, and alkaline phosphatase is 300 U/L (3 times normal). The next step in evaluation is
A.
*Ultrasound or CT scan
B.
Percutaneous transhepatic Cholangiography
C.
Reticulocyte count
D.
Esophagogastroduodenoscopy
E.
Colonoscopy
431. A nursing student has just completed her hepatitis B vaccine series. On reviewing her laboratory
studies (assuming she has no prior exposure to hepatitis B), you expect
A.
Positive test for hepatitis B surface antigen
B.
*Antibody against hepatitis B surface antigen (anti-HBS) alone
C.
Antibody against hepatitis core antigen (anti-HBC)
D.
Antibody against both surface and core antigen
E.
Antibody against hepatitis E antigen
432. A paracentesis is performed on 43-year-old patient with long-standing alcohol abuse. On physical
exam, there are spider angiomas and palmar erythema. Abdominal collateral vessels are seen around the
umbilicus. The serum albumin minus ascitic fluid albumin equals 1.4 g/dL. The most likely diagnosis is
A.
*Portal hypertension
B.
Pancreatitis
C.
Tuberculous peritonitis
D.
Hepatoma
E.
No ascitis
433. A patient presents to a physician with severe jaundice. Physical examination reveals a nodular,
enlarged liver. In addition to the generalized nodularity of the liver, the physician can feel one nodule that is
much larger than the others. CT of the abdomen confirms multinodular cirrhosis and demonstrates a 7-cm
mass near the lower border of the liver. CT-guided biopsy of this mass shows a malignant tumor derived
from hepatic parenchymal cells. Which of the following risk factors is most strongly associated with the
development of this tumor?
A.
Anatoxin exposure
B.
Hemochromatosis
C.
*Hepatitis В virus infection
D.
Opistharchis infection
E.
Thorotrast exposure
434. Over a 2-month period, a 50-year-old woman with a history of polycythemia vera develops
abdominal pain and gross ascites. Physical examination demonstrates smooth hepatomegaly and mild
jaundice. Pressure applied over the liver fails to distend the jugular veins. The abdominal wall is grossly
edematous and shows a tortuous venous pattern. Edema of the legs is prominent. Which of the following is
the most likely diagnosis?
A.
Hepatocellular carcinoma
B.
Primary sclerosing cholangitis
C.
Steatosis
D.
*Budd-Chiari syndrome
E.
Hepatic cirrhosis
435. A 76-year-old man, who is in a rehabilitation facility after fracturing a leg, develops acute diarrhea.
He has no history of intestinal disorders. The patient has regular colonoscopic screenings. His last
colonoscopy, done 2 years ago, was normal. He has not received antibiotics in the past year. His roommate is
taking oral metronidazole for a diarrheal syndrome that he developed while in the rehabilitation facility. Use
of which of the following would most likely have prevented development of the patients diarrheal
syndrome?
A. *Good hand-washing technique
B. Prophylactic antibiotics
C. Prophylactic probiotic agents
D. Prophylactic loperamid
E. None of above
436. A 68-year-old man is evaluated because of nausea, vomiting, and upper abdominal pain and
distention of 2 days duration. He has no fever, chills, or jaundice. On physical examination, he appears
uncomfortable and has orthostatic hypotension. Abdominal examination discloses distention, tympany on
percussion, and rushes on auscultation. Serum aspartate aminotransferase is 0,34 U/L, serum alanine
aminotransferase is 0,61 U/L, and serum total bilirubin is 19,6 mcmol/L. Plain radiographs of the abdomen
show pneumobilia with multiple air-fluid levels in the jejunum. No free air is seen. Abdominal
ultrasonography shows four gallstones measuring 2 to 4 cm. Because of pneumobilia, the biliary tree cannot
be further visualized. Which of the following is the most appropriate next step in this patient’s management?
A. *Exploratory laparotomy for bowel obstruction
B. Cholecystectomy
C. Endoscopic retrograde cholangiopancreatography
D. CT scan of the abdomen
E. Magnetic resonance cholangiopancreatography
437. A 31-year-old man has a 2-month history of abdominal pain and bloating. An upper gastrointestinal
series with small bowel follow-through shows ulcerations and inflammatory changes in the distal 12 cm of
the terminal ileum. Colonoscopy is normal except for erythema and small linear ulcerations seen on
cannulation of the terminal ileum. Which of the following is the most appropriate therapy for this patient at
this time?
A. *pH-release mesalamine
B. Mesalamine enemas
C. Balsalazide
D. Olsalazine
E. Sulfasalazine
438. A 27-year-old Hispanic woman who is 13 weeks pregnant has had recurrent episodes of biliary colic
for the past 6 months. The episodes are now increasing in frequency, and one episode was associated with
mild pancreatitis. Abdominal ultrasonography shows multiple gallstones. Results of routine laboratory
studies are normal. When is the most appropriate time for this patient to undergo laparoscopic
cholecystectomy?
A. *During the second trimester
B. Immediately
C. During the third trimester
D. Post partum
E. Conservative therapy is recommended
439. A 55-year-old male alcoholic has recurrent attacks of severe mid-epigastric pain after eating. Serum
amylase determinations after such attacks have been in the normal range. The examination reveals mild
cachexia but is otherwise unremarkable. On further questioning, the patient states that he has been sober for
the past 10 years but prior to that time had multiple episodes of alcohol-induced pancreatitis. He is currently
taking pancreatic replacement enzymes by mouth. An ERCP reveals a stricture of the pancreatic duct but is
otherwise unremarkable. Computed tomography of the abdomen reveals calcifications in the pancreas but
does not show any evidence of malignancy.
A. The patient is taking 30 mg of continuous-release morphine sulfate twice a day. The best strategy at this
point would be to
B. *resect the head of the pancreas
C. double the dose of pancreatic replacement enzymes
D. double the dose of morphine
E. institute a low-fat diet
F. begin a continuous search for other causes ofabdominal pain
440. A 44-yr-old mother of five complained of yellow skin and abdominal pain, especially after meals.
She was overweightand she said that she did not like going out to restaurants because of embarrassing
flatulence. Choose the single most likely diagnosis from the list of options above.
A. *Gallstones
B. Hepatitis
C. Carcinoma of bile duct
D. Alcoholism
E. Carcinoma pancreas
441. A 44-yr-old woman complains of intense pruritus and yellowing of her skin. On physical
examination you notice xanthomata and skin pigmentation. Choose the single most likely diagnosis from
the list of options above.
A. *Primary biliary cirrhosis
B. Hepatitis
C. Carcinoma of bile duct
D. Alcoholism
E. Haemolytic anaemia
442. A 21-yr-old man: Bilirubin—45 mcmol/l—conjugated—7, unconjugated—38, alkaline phosphatasenorm, ALT—Norm, Blood film reported as normal. Choose the single most likely diagnosis from the list of
options above.
A. *Gilbert's syndrome
B. Alcoholic cirrhosis
C. Hepatitis
D. Carcinoma of head of pancreas
E. Primary biliary cirrhosis
443. A 49-yr-old man who enjoys drinking presents with pallor, epistaxis and bleeding. On physical
examination you find the spleen to be enlarged and the liver to be slightly enlarged. Choose the single most
likely investigation from the list of options above.
A. *Liver function test
B. Abdominal USG
C. Bone marrow biopsy
D. Blood culture
E. White cell count
444. A 40-yr-old woman has a long history of pruritus, arthralgia and mild jaundice. She presents with
haematemesis and is found to have splenomegaly. Endoscopy shows oesophageal varices. Choose the single
most likely diagnosis from the list of options above. Choose the single most likely diagnosis from the list of
options above.
A. *Portal hypertension
B. Myelofibrosis
C. Infective endocarditis
D. None of above
E. Gilbert's syndrome
445. 44-year-old woman for 1 year complained of attacks of right subcostal pain after fatty meal. Last
week the attacks have repeated every day and become more painful. What diagnostic study would you
recommend?
A. *Ultrasound examination of the gallbladder
B. Blood cell count
C. Ultrasound study of the pancreas
D. X-ray examination of the gastrointestinal tract
E. None of above
446. A 42-year-old patient suffering from alcoholism has advanced liver disease with ascites.
Examination reveals asterixis of the hands, ankle clonus, and spider angiomas on the face and chest.
Precipitating factors to look for include all of the following EXCEPT:
A. *heart insufficiency
B. bleeding esophageal varices
C. excessive diuretic therapy
D. non-compliance with lactulose therapy
E. spontaneous bacterial peritonitis
447. A 20-year-old female exchange student from Paris has had bouts of jaundice, fever, malaise,
arthralgias, and marked elevation of hepatic transaminases over the past 6 months. The patient was not
exposed to hepatotoxic drugs. Hypergammaglobulinemia has been noted. Serologic evaluation for infection
with hepatitis A, B, and C has been negative, as have tests for systemic lupus. Liver biopsy now reveals
bridging necrosis. Which of the following tests will be most helpful in confirming the diagnosis?
A. *Antibodies to liver and kidney microsomal Antigens
B. Hemoglobin electrophoresis
C. Rheumatoid factor
D. Antibodies to hepatitis D virus
E. Antibodies to hepatitis E virus
448. A 38-year-old former hemodialysis nurse is seen because of a 6-month history of fatigue and
amenorrhea. On examination she has scleral icterus, a mildly tender liver, and a tibial rash consistent with
erythema nodosum. ALT and AST levels are both in the range increased, while alkaline phosphatase and
serum albumin levels are normal. Hepatitis serologic testing detects HBsAg and IgG anti-HBcAg. Liver
biopsy discloses a mononuclear cell portal infiltrate and hepatocyte destruction at the periphery of lobules.
Which of the following therapeutic strategies is best?
A. *Administration of interferon, 10 million units three times per week for 4 months
B. Administration of prednisone, 20 to 40 mg/d for 2 months and then a taper based on the response
C. Administration of prednisone, 10 mg every other day for 3 months
D. Administration of acyclovir, 400 mg every 6 h for 2 weeks
E. Administration of low-dose cyclophosphamide, 50 mg/d for 2 months
449. A 60-year-old man with biopsy-proven hepatic cirrhosis is hospitalized because of massive ascites
and pedal edema. There is no evidence of respiratory compromise or hepatic encephalopathy. Bed rest,
sodium and water restriction, and the administration of spironolactone produce no significant weight change
after 5 days. Which of the following therapeutic measures would be most appropriate at this time?
A. *Therapeutic paracentesis
B. None of them
C. Oral acetazolamide, 250 mg/d
D. Placement of a peritoneovenous shunt
E. Intravenous furosemide, 80 mg now
450. A 56-year-old patient with cirrhosis of the liver presents with massive hemetemesis. Two large-bore
intravenous lines are placed; somatostatin, fluids, and blood products are administered; and the patient is
intubated. Emergency endoscopy reveals bleeding esophageal varices. The patient becomes stable
hemodynamically but is still bleeding. The most appropriate next step is
A. *endoscopic variceal band ligation
B. intravenous vasopressin
C. balloon tamponade
D. endoscopic injection sclerotherapy
451. intravenous propranolol
452. A 35-year-old woman who underwent successful therapy for acute leukemia 10 years ago is
seropositive for hepatitis C. Polymerase chain reaction testing confirms the presence of virus in a blood
sample. She has mildly elevated serum transaminases. A liver biopsy discloses moderately severe chronic
hepatitis without fibrosis. The most appropriate therapy is
A. *interferon plus ribavirin
B. acyclovir
C. ribavirin
D. propranolol
E. no therapy is indicated
453. Patient A., 45 years old, suffers from chronic cholecystits during last 5 years. She was admitted to
the emergency department with acute pain in right hypochondriac area and high temperature. Laboratory
findings reveal leucocytosis, high ESR. Put preliminary diagnosis.
A. *Chronic cholecystitis, acute phase.
B. Chronic cholecystitis, subacute phase.
C. Chronic cholecystitis, phase of remission.
D. Dyskinezia of bile ducts.
E. Rotor’s syndrome.
454. A 59-yr-old man presents with obstructive jaundice. USG shows no gallstones. The liver appears
normal and the common bile duct measures 12 mm in diameter. His past medical history includes partial
gastrectomy 15 yr ago for peptic ulcer. Choose the single most likely investigation from the list of options
above.
A. Percutaneous transhepatic
B. *Cholangiography
C. MRI scan
D. Barium follow through
E. CTscan
455. A 59-yr-old man presents with obstructive jaundice. USG shows no gallstones. The liver appears
normal and the common bile duct measures 12 mm in diameter. His past medical history includes partial
gastrectomy 15 yr ago for peptic ulcer. Choose the single most likely investigation from the list of options
above.
A. Percutaneous transhepatic
B. *Cholangiography
C. MRI scan
D. CTscan
E. Oral cholecystogram
456. A 22-yr-old man who was driving was involved with a high speed collision. He was wearing his
seat belt and now complains of upper abdominal pain. His CXR is normal. Choose the single most likely
investigation from the list of options above.
A. *Serum amylase
B. Arterial blood gases
C. Barium enema and colonic biopsy
D. ESR
E. Serum electrolytes
457. A 48-year-old woman develops fevers, chills, and icteric sclera. In addition to a fever of 39.2_C, the
physical examination is remarkable for an ill-appearing jaundiced female with right upper quadrant pain.
Ultrasonography reveals a dilated common bile duct with stones in the gallbladder and in the duct itself. The
patient is placed on broad-spectrum antibiotics to cover organisms known to infect the biliary tract. The
procedure most appropriate now is
A. *endoscopic retrograde cholangiopancreatography
B. laparoscopic cholecystectomy
C. placement of an external stent for bilary drainage
D. laparotomy to canulate the common bile duct, remove the stone, and perform a cholecystectomy
E. antibiotics for several days
458. A 45-year-old woman complains of right upper quadrant pain, which occurs after she eats a large
meal. Occasionally the episodes are accompanied by nausea and vomiting. A plain x-ray of the abdomen
discloses gallstones. Ultrasonography reveals gallstones and a normalsized common bile duct. The patient’s
blood chemistry and CBC are normal. The most therapeutic maneuver at this time would be
A. *laparoscopic cholecystectomy
B. observation
C. ursodeoxycholic acid
D. shock wave lithotripsy
E. ursodeoxycholic acid and shock wave lithotripsy
459. A 50-year-old man without significant past medical history or recent exposure to alcohol presents
with midepigastric abdominal pain, nausea, and vomiting. The physical examination is remarkable for the
absence of jaundice and any other specific physical findings. Which of the following is the best strategy for
screening for acute pancreatitis?
A. Measurement of both serum amylase and serum
B. *Lipase
C. Measurement of serum lipase
D. Measurement of serum amylase
E. Isoamylase level analysis
F. None of above
460. A 50-year-old man without significant past medical history or recent exposure to alcohol presents
with midepigastric abdominal pain, nausea, and vomiting. The physical examination is remarkable for the
absence of jaundice and any other specific physical findings. Which of the following is the best strategy for
screening for acute pancreatitis?
A. Measurement of both serum amylase and serum
B. *Lipase
C. Measurement of serum amylase
D. Isoamylase level analysis
E. None of above
461. A 31-year-old woman with ulcerative colitis has been taking prednisone for the past year. Each time
the prednisone is tapered below 20 mg/d, her symptoms return. She is subsequently started on 6mercaptopurine, 50 mg/d. Three days after beginning the new drug, she develops worsening abdominal pain
with radiation to her back. She does not have a rash. Her leukocyte count is 3200/μL. Which of the following
is the most likely cause of this patients new symptoms?
A. *Pancreatitis due to initiation of 6-mercaptopurine
B. Pancreatitis due to continuation of prednisone
C. Flare of ulcerative colitis
D. An abdominal and psoas abscess secondary to 6-mercaptopurine-induced neutropenia
E. None of above
462. A 61-year-old white man has a 2-month history of dysphagia for solid foods. He has lost 6.7 kg
during this time. The patient has chronic heartburn that is relieved by antacids. He also has hypertension for
which he takes atenolol and diltiazem. Which of the following is the most likely diagnosis
A. Esophageal adenocarcinoma*
B. Pill-induced esophagitis
C. Esophageal web
D. diffuse esophageal spasm
E. None of above
463. A 37-year-old woman has a 4-month history of epigastric discomfort and heartburn. Symptoms are
usually exacerbated postprandially, especially if she eats spicy foods. The patient denies dysphagia, weight
loss, and decreased appetite. Treatment with a proton pump inhibitor, once daily for 4 weeks, resulted in
only minimal improvement. Increasing the medication to twice daily for an additional 4 weeks did not
improve her symptoms, and the patient wants to know what other management options are available.
Referral for which of the following procedures is most appropriate at this time?
A. *Ambulatory 24-hour esophageal pH monitoring
B. Upper endoscopy with esophageal dilation
C. Barium swallow
D. Surgical fundoplication
E. USD
464. A 51-year-old woman undergoes colorectal cancer screening. He feels well, has no significant
medical history, takes no medications, and has no family history of colorectal cancer. Physical examination
and complete blood count are normal.
Which of the following is the most appropriate screening program for colorectal cancer in this patient?
A. *Colonoscopy every 10 years
B. Flexible sigmoidoscopy every 2 to 3 years
C. Barium enema examination every 3 years
D. Fecal occult blood testing every 2 to 3 years
E. CT colonography (virtual colonoscopy) every 10 years
465. A 67-year-old woman had her first colonoscopy 1 month ago for routine colorectal cancer screening.
A 6-mm tubular adenoma of the sigmoid colon was removed. She has no family history of colorectal cancer.
She asks what can be done to decrease her risk of developing colorectal cancer.
Which of the following is the most appropriate surveillance for this patient
A. *Repeat colonoscopy in 5 year
B. Repeat colonoscopy in 1 years
C. Aspirin, 81 mg daily
D. A high-fiber, low-fat diet
E. None of above
466. A 75-year-old man has been deferring colon cancer screening because she is afraid to undergo
colonoscopy. She learned of a new technique called virtual colonoscopy that she thinks may be more
tolerable and asks you about the relative merits of this procedure. Which of the following statements is true
regarding virtual colonoscopy?
A. *It detects colorectal cancers and large adenomas quite well, but may miss small polyps
B. It is more acceptable to patients because it does not require any bowel preparation
C. It is a noninvasive procedure that images the colon using ultrasound
D. Its sensitivity and specificity for detecting colon cancers and polyps is similar to that of conventional
colonoscopy
E. It does not require any instrumentation of the bowel
467. A 34-year-old woman is scheduled to undergo flexible sigmoidoscopy because of bloody bowel
movements of 4 days duration. She has had diarrhea for about 10 days, but 4 days ago she developed
frequent, low-volume bloody stools that are associated with urgency. The patient has had episodic diarrhea
for about 3 months, during which time she noted and ignored bloody bowel movements on four occasions.
Stool specimens obtained 3 days ago are negative for pathogens. Sigmoidoscopic findings include
inflammation from the rectum to 20 cm with no areas of normal intervening mucosa. Which of the following
is the most likely diagnosis?
A. *Ulcerative proctocolitis
B. Crohn’s disease
C. Irritable bowel syndrome
D. lschemic colitis
E. None of above
468. A 29 year-old man has Crohns ileocolitis complicated by perianal fistulae. Ileocecal resection was
performed 5 years ago. Approximately 4 months ago, he developed abdominal pain, cramping, and a new
enterocutaneous fistula. An upper gastrointestinal series with small bowel follow-through showed 4 cm of
inflammatory changes at the neoterminal ileum plus an enterocutaneous fistula originating from this area.
After treatment with metronidazole and mesalamine, his abdominal pain improved and the fistula appeared
to resolve. Which of the following is most appropriate for maintaining remission in this patient?
A. *Start 6-mercaptopurine
B. Continue metronidazole
C. Start budesonide
D. Start prednisone
E. tart oral cyclosporine
469. A 45-year-old man for 1 month has complained of epigastric and right subcostal aching pain,
pruritus, indigestion, dark color of the urine and acholic stool, fever, and significant weight loss. On exam:
jaundice, presence of Curvuasier’s sign. US scan did not reveal stones in the gallbladder and choledochus.
What is the most likely diagnosis?
A. *Cancer of the pancreas head
B. Gallbladder stones
C. Chronic pancreatitis
D. Chronic cholangitis
E. Chronic hepatitis
470. 55-yr-old woman underwent upper GI endoscopy for assessment of dysphagia. Three hours later he
complained of severe chest pain. On examination there was crepitus in the neck. Choose the single most
likely investigation from the list of options.
A.
Barium swallow
B.
Oesophageal manometry
C.
Upper GI endoscopy and biopsy
D.
CT scan chest
E.
*Chest radiography
471. A 27-year-old man complains to a physician of chronic gastric pain. The pain is sometimes relieved
by food. EGDS shows antral erosive gastritis, biopsy of antral mucous presents Hеlicobacter Pylori. What
can be diagnosed in this case?
A.
*Gastritis of type B
B.
Menetrier’s disease
C.
Rigid antral gastritis
D.
Gastritis of A type
E.
Reflux – gastritis
472. A 27-year-old man is evaluated because of two episodes of painless melena, the last of which
occurred 6 hours ago. He has been taking naproxen for the past 6 weeks for a sports-related shoulder injury.
Medical history is otherwise noncontributory. On physical examination, temperature is 36.0 C (96.8 F), pulse
rate is 98/min and regular, respiration rate is 18/min, and blood pressure is 104/89 mm Hg without
orthostatic changes. Digital rectal examination discloses very dark stool that is positive for occult blood.
Hemoglobin is 9.3 g/dL, leukocyte count is 10,800/?L, platelet count is 250,000/?L, and other routine
laboratory studies are normal. Nasogastric aspirate is negative. Which of the following should be done next?
A.
Order red blood cell transfusion to achieve a hemoglobin value above 10 g/dL
B.
Establish access with two intravenous lines and infuse isotonic saline, 500 mL/h for 6 to 8 hours; then
reassess the hemoglobin value
C.
Transfuse 6 units of platelets to correct the platelet dysfunction caused by naproxen
D.
*Perform upper endoscopy
E.
Order an upper gastrointestinal series
473. A 32 -year-old woman complained on epigastric pain just after meal, heartburn, and nausea. Stomach
endoscopy revealed a large amount of mucus, hyperemia, erosions of mucous membrane of stomach.
Helicobacter infection test is positive.
A.
Chronic type A gastritis
B.
Peptic ulcer of the stomach
C.
Chronic type C gastritis
D.
Menetrier’s disease
E.
*Chronic type B gastritis
474. A 32-year-old woman has a 4-month history of hoarseness and throat clearing. Evaluation by an
otolaryngologist disclosed laryngeal inflammation suggestive of gastroesophageal reflux disease, and the
patient is referred to you. She is otherwise asymptomatic and denies heartburn, regurgitation, dysphagia, and
weight loss. The patient maintains an active lifestyle and currently takes no medications. Physical
examination and routine laboratory studies are normal. Which of the following should be done next?
A.
Upper endoscopy
B.
Esophageal manometry
C.
Ambulatory 24-hour esophageal pH monitoring
D.
Barium swallow
E.
*Trial of acid suppressive therapy
475. A 32-yr-old male presents with nausea and acute abdominal pain boring through to the back. The
epigastrium is very tender. He has had three similar bouts in the past 18 months. A barium meal is normal.
Choose the single most likely diagnosis
A.
Gastric atrophy
B.
Gastric ulcer
C.
Basal pneumonia
D.
Chronic cholecystitis
E.
*Non-ulcer dyspepsia
476. A 34-year-old male presents with substernal discomfort. The symptoms are worse after meals,
particularly a heavy evening meal, and are sometimes associated with hot/sour fluid in the back of the throat
and nocturnal awakening. The patient denies difficulty swallowing, pain on swallowing, or weight loss. The
symptoms have been present for 6 weeks; the patient has gained 20 lb in the past 2 years. Your initial
approach is
A.
*A therapeutic trial of ranitidine
B.
Exercise test with thallium imaging
C.
Esophagogastroduodenoscopy
D.
CT scan of the chest
E.
Barium contrast study is indicated
477. A 34-year-old woman is hospitalized after a minor episode of hematemesis. She denies current use of
nonsteroidal anti-inflammatory drugs. Two years ago, a duodenal ulcer and Helicobacter pylori infection
were diagnosed by upper endoscopy, and bismuth subsalicylate, metronidazole, tetracycline, and ranitidine
were prescribed for 14 days. However, the patient took the medications for only 8 days because of nausea.
Upper endoscopy performed during the current admission shows a clean-based duodenal ulcer, and rapid
urease testing of an endoscopic mucosal biopsy specimen is positive for H. pylori. Which of the following is
the most appropriate management at this time?
A.
Bismuth subsalicylate, metronidazole, tetracycline, and ranitidine for 4 days
B.
Bismuth subsalicylate, metronidazole, amoxicillin, and ranitidine for 14 days
C.
A proton pump inhibitor for 14 days
D.
*A proton pump inhibitor, clarithromycin, and amoxicillin for 14 days
E.
Ranitidini (H2-histaminoblokers)
478. A 34-yr-old woman points to an area of acute epigastric pain with his right index finger. The pain is
worse at night and taking food relieves him. Taking antacids also relieves it. Choose the single most likely
diagnosis
A.
Gastric atrophy
B.
Acute pancreatitis
C.
Basal pneumonia
D.
Chronic cholecystitis
E.
*Oesophagitis
479. A 35 -year-old man complained of squeezed epigastric pain 1 hour after meal and heartburn. She had
been ill for 2 years. On palpation, there was moderate tenderness in pyloroduodenal area. Antral gastritis was
revealed on gastroscopy. Which study can establish genesis of the disease?
A.
USD of abdomen
B.
Gastrin level in blood
C.
Examination of stomach secretion
D.
Examination of stomach motor function
E.
*Revealing of Helicobacter infection in gastric mucosa
480. A 35-year-old man complains to a physician of chronic vague gastric pain of several years' duration.
The pain is sometimes relieved by food. Which of the following is the most appropriate test to noninvasively
determine the H. pylori?
A.
Culture of gastric biopsy
B.
Rapid urease test
C.
Repeat qualitative IgA and IgG antibodies against H. pylori
D.
Repeat quantitative IgA and IgG antibodies against H. pylori
E.
*Urea breath test
481. A 35-year-old man consults a gastroenterologist because of chronic heartburn for several years. The
heartburn tends to be worse at night, and he frequently tastes refluxed gastric contents when he goes to bed.
He found that his symptoms were a little better when he avoided his customary late evening alcoholic drink;
however, this modest improvement has subsequently deteriorated. The gastroenterologist performs
esophageal manometry with pH monitoring, which demonstrates decreased pressure of the lower esophageal
sphincter and the presence of acid in the esophagus. Biopsy of the proximal end of one of these fingers
shows surface epithelium with regular columnar cells with small, ovoid nuclei admixed with goblet cells.
Which of the following is the most likely diagnosis?
A.
Achalasia
B.
*Barrett esophagus
C.
Corrosive esophagitis
D.
Esophageal adenocarcinoma
E.
Zenker diverticulum
482. A 35-year-old patient with alcohol abuse and abnormal diet complains of pain in epigastrium that
occurs in 1-1,5 hours after having meal. Esophagogastroduodeno-scopy: marked hyperemia, small defects,
easy appearance of sores on the mucous membrane in the antral section of the stomach. What is the most
probable reason for this pathology?
A.
*Helicobacter pуlori infection
B.
Presence of antibodies to parietal cells
C.
Alimentary factor
D.
Nervous overstrain
E.
Toxic action of alcohol
483. A 37-year-old man complains of pains in epigastrium which are relieved by food intake. EGDS
shows antral erosive gastritis, biopsy of antral mucous presents Hеlicobacter Pylori. Diagnosis is:
A.
Gastritis of type A
B.
Reflux - gastritis
C.
Menetrier's gastritis
D.
Rigid antral gastritis
E.
*Gastritis of type B
484. A 37-year-old patient complains of epigastric pain that occurs in 1-1,5 hours after having meal.
Esophagogastroduodeno-scopy: marked hyperemia, small defects, easy appearance of sores on the mucous
membrane in the antral section of the stomach. What is the most probable reason for this pathology?
A.
*Helicobacter pуlori infection
B.
Presence of antibodies to parietal cells
C.
Alimentary factor
D.
Nervous overstrain
E.
Toxic action of alcohol
485. A 37-year-old woman has a 4-month history of epigastric discomfort and heartburn. Symptoms are
usually exacerbated postprandially, especially if she eats spicy foods. The patient denies dysphagia, weight
loss, and decreased appetite. Treatment with a proton pump inhibitor, once daily for 4 weeks, resulted in
only minimal improvement. Increasing the medication to twice daily for an additional 4 weeks did not
improve her symptoms, and the patient wants to know what other management options are available.
Referral for which of the following procedures is most appropriate at this time?
A.
Upper endoscopy with esophageal dilation
B.
Barium swallow
C.
Surgical fundoplication
D.
USD
E.
*Ambulatory 24-hour esophageal pH monitoring
486. A 38-year-old male insurance agent with a benign past medical history presents to his primary care
physician complaining of indigestion. He notes that “heartburn” has occurred weekly for about 1 year,
especially after eating a heavy meal. He has no risk factors for coronary artery disease and does not complain
of weight loss, vomiting, dysphagia, or bleeding. Physical and routine laboratory examinations are
unrevealing. Which of the following is the most appropriate next step?
A.
Upper gastrointestinal barium radiography
B.
Upper gastrointestinal endoscopy
C.
Ambulatory esophageal pH testing
D.
*Serology for H. Pylori
E.
Prescribe omeprazole
487. A 38-year-old male insurance agent with a benign past medical history presents to his primary care
physician complaining of indigestion. He notes that “heartburn” has occurred weekly for about 1 year,
especially after eating a heavy meal. He has no risk factors for coronary artery disease and does not complain
of weight loss, vomiting, dysphagia, or bleeding. Physical and routine laboratory examinations are
unrevealing. Which of the following is the most appropriate next step?
A.
Upper gastrointestinal endoscopy
B.
Ambulatory esophageal pH testing
C.
Upper gastrointestinal barium radiography
D.
Prescribe omeprazole
E.
*Serology for H. Pylori
488. A 38-year-old man complains of chronic heartburn for several years. The heartburn tends to be worse
at night, and he frequently tastes refluxed gastric contents when he goes to bed. He found that his symptoms
were a little better when he avoided his customary late evening alcoholic drink; however, this modest
improvement has subsequently deteriorated. The esophageal manometry with pH monitoring demonstrates
decreased pressure of the lower esophageal sphincter and the presence of acid in the esophagus. Biopsy of
the proximal end of esophagus shows surface epithelium with regular columnar cells with small, ovoid
nuclei admixed with goblet cells. Which of the following is the most likely diagnosis?
A.
Achalasia
B.
*Barrett esophagus
C.
Corrosive esophagitis
D.
Esophageal adenocarcinoma
E.
Zenker diverticulum
489. A 38-year-old woman with rheumatoid arthritis presents to her physician because of increased joint
pain. On physical examination, both passive and active range of motion of the hips are decreased. Her
physician increases her dose of nonsteroidal anti-inflammatory drugs (NSAIDs). Which of the following
should be prescribed as well to prevent peptic ulcer disease?
A.
Cimetidine
B.
Clarithromycin
C.
*Misoprostol
D.
Omeprazole
E.
Sucralfate
490. A 39-year-old woman has a 4-month history of epigastric discomfort and heartburn. Symptoms are
usually exacerbated postprandially, especially if she eats spicy foods. The patient denies dysphagia, weight
loss, and decreased appetite. Previous treatment with a proton pump inhibitor did not improve her symptoms,
and the patient wants to know what other management options are available. Referral for which of the
following procedures is most appropriate at this time?
A.
Upper endoscopy with esophageal dilation
B.
Barium swallow
C.
Surgical fundoplication
D.
USD
E.
*Ambulatory 24-hour esophageal pH monitoring
491. A 40-year-old cigarette smoker complains of epigastric pain, well localized, nonradiating, and
described as burning. The pain is partially relieved by eating. There is no weight loss. He has not used
nonsteroidal antiinflammatory agents. The pain has gradually worsened over several months. The most
sensitive way to make a specific diagnosis is
A.
Barium x-ray
B.
*Endoscopy
C.
Serologic test for Helicobacter pylori
D.
Serum gastrin
E.
Ultrasound
492. A 40-year-old cigarette smoker complains of epigastric pain, well localized, nonradiating, and
described as burning. The pain is partially relieved by eating. There is no weight loss. He has not used
nonsteroidal antiinflammatory agents. The pain has gradually worsened over several months. The most
sensitive way to make a specific diagnosis is
A.
Barium x-ray
B.
*Endoscopy
C.
Serologic test for Helicobacter pylori
D.
Serum gastrin
E.
Ultrasound
493. A 40-year-old man comes to the emergency department because of a 1-month history of intermittent
epigastric pain that abruptly increased several hours ago. He also reports intractable nausea and vomiting and
had one episode of hematemesis this morning. Medical history is significant only for ankylosing spondylitis,
for which he takes indomethacin. On physical examination, the patient is lying quietly on his side with his
legs flexed at the knees and hips. Temperature is 38.7 C (101 .7 F). Abdominal examination discloses
tympany and diffuse tenderness to palpation and percussion in all four quadrants. A stool specimen is brown
and is positive for occult blood. Which of the following is the most likely diagnosis?
A.
Gastropathy due to nonsteroidal anti-inflammatory drugs (NSAIDs)
B.
An actively bleeding duodenal ulcer
C.
A penetrating duodenal ulcer with pancreatitis
D.
A pyloric channel ulcer with gastric outlet obstruction
E.
*A perforated peptic ulcer
494. A 42-year-old man has a 6-week history of epigastric pain following meals and at night. The patient
does not smoke cigarettes but does drink alcoholic beverages socially. He denies use of nonsteroidal antiinflammatory drugs (NSAIDs) and aspirin. Physical examination and routine laboratory studies are normal.
A proton pump inhibitor is begun. One week later, upper endoscopy is performed, which shows a small,
clean-based ulcer in the duodenal bulb. Rapid urease testing of an endoscopic gastric antral biopsy specimen
is negative. Which of the following is the most likely cause of this patient’s ulcer?
A.
Zollinger-Ellison syndrome
B.
*Helicobacter pylori infection
C.
Duodenal adenocarcinoma
D.
Alcohol consumption
E.
Surreptitious NSAID use
495. A 43-year-old cigarette smoker complains of epigastric pain, well localized, nonradiating, and
described as burning. The pain is partially relieved by eating The pain has gradually worsened over several
months. The most sensitive way to make a specific diagnosis is
A.
Barium x-ray
B.
*Endoscopy
C.
Serologic test for Helicobacter pylori
D.
Serum gastrin
E.
Ultrasound
496. A 44-year-old cigarette smoker complains of epigastric pain, well localized, nonradiating, and
described as burning. The most sensitive way to make a specific diagnosis is
A.
Barium x-ray
B.
*Endoscopy
C.
Serologic test for Helicobacter pylori
D.
Serum gastrin
E.
Ultrasound
497. A 45-year-old man presents with sharp epigastric pain relieved by antacids and food. Barium study of
the upper gastrointestinal tract reveals a crater in the proximal portion of the duodenal bulb. Which of the
following statements concerning therapeutic alternatives is correct?
A.
Atropine or related anticholinergic agents are effective
B.
Sucralfate is effective in eradicating Helicobacter
C.
Cimetidine or other H -receptor antagonists are 2
D.
*Sucralfate can significantly reduce the bioavailability
E.
Omeprazole, a specific inhibitor of parietal cell
498. A 45-year-old woman consults a physician because of chronic fatigue. A review of systems reveals
long-standing stomach problems characterized by slow digestion and delayed emptying of her stomach. A
complete blood count demonstrates a moderately severe megaloblastic anemia. Serum vitamin B12 level is
85 pg/mL; serum folate is 3 ng/mL; and serum iron is 105 mg/dL. Autoantibodies to intrinsic factor are
detected in the serum. A biopsy of the stomach is most likely to show which of the following?
A.
Acute erosive gastritis
B.
*Gastric atrophy
C.
Linitis plastica
D.
Menetrier disease
E.
Peptic ulcer
499. A 45-year-old woman consults a physician because of chronic fatigue. A review of systems reveals
long-standing stomach problems characterized by slow digestion and delayed emptying of her stomach. A
complete blood count demonstrates a moderately severe megaloblastic anemia. A biopsy of the stomach is
most likely to show which of the following?
A.
Acute erosive gastritis
B.
*Gastric atrophy
C.
Linitis plastica
D.
Menetrier disease
E.
Peptic ulcer
500. A 45-year-old woman is evaluated because of epigastric fullness, bloating, and nausea of several
months’ duration. Antacids and an over-the-counter H2-receptor antagonist do not control her symptoms.
Physical examination, routine laboratory studies, upper endoscopy, and abdominal ultrasonography are
normal. Rapid urease testing of gastric mucosal biopsy specimens obtained at endoscopy is positive. Which
of the following should be done next?
A.
Confirmation of the positive urease test by serologic studies for Helicobacter pylori
B.
*A course of therapy for eradication of H.pylori
C.
A trial of a proton pump inhibitor
D.
A trial of metoclopramide
E.
A radionuclide gastric emptying study
501. A 45-year-old woman undergoes upper endoscopy for evaluation of epigastric pain and weight loss.
Gastric erythema and mild gastric nodularity are observed, and endoscopic mucosal biopsy specimens show
Helicobacter pylori and a mucosa-associated lymphoid tissue (MALT) lymphoma. CT scan of the abdomen
reveals gastric wall thickening, and endoscopic ultrasonography shows that the tumor is confined to the
gastric mucosa. Which of the following is the most appropriate next step in the management of this patient?
A.
Referral for subtotal gastrectomy
B.
Referral to an oncologist for systemic chemotherapy
C.
*H. pylori eradication therapy with follow-up endoscopy in 6 weeks
D.
Referral for staging laparoscopy
E.
Serological tests
502. A 45-year-old woman undergoes upper endoscopy for symptoms of dyspepsia. The endoscopic
examination is normal. Physical examination, routine laboratory studies, serologic tests forHelicobacter
pylori, and abdominal ultrasonography are also normal. The patient has not received any medications for her
symptoms. Which of the following is the most appropriate initial empiric therapy for this patient?
A.
*A proton pump inhibitor
B.
Alosetron
C.
Ondansetron
D.
Tegaserod
E.
Sumatriptan
503. A 46-year-old man presents with sharp epigastric pain relieved by antacids and food. Barium study of
the upper gastrointestinal tract reveals a crater in the proximal portion of the duodenal bulb. Which of the
following statements concerning therapeutic alternatives is correct?
A.
Atropine or related anticholinergic agents are effective in improving the symptoms
B.
Sucralfate is effective in eradicating Helicobacter pylori colonization
C.
*H2-receptor antagonists are more effective than sucralfate in promoting healing
D.
Sucralfate can significantly reduce the bioavailability of fluoroquinolone antibiotics
E.
Omeprazole, a specific inhibitor of parietal cell H+, K+-ATPase, is contraindicated in routine
situations because of its carcinogenic potential
504. A 47-year-old woman consults a physician because of chronic fatigue. A review of systems reveals
long-standing stomach problems characterized by slow digestion and delayed emptying of her stomach. A
complete blood count demonstrates a moderately severe megaloblastic anemia. Autoantibodies to intrinsic
factor are detected in the serum. A biopsy of the stomach is most likely to show which of the following?
A.
Acute erosive gastritis
B.
*Gastric atrophy
C.
Linitis plastica
D.
Menetrier disease
E.
Peptic ulcer
505. A 47-year-old woman presents with a history of crushing nonradiational chest pain.
Electrocardiography and exercise stress testing reveal no evidence of cardiac edema. A more detailed history
is taken, and the patient states that he has had a sensation of sticking after swallowing. He notes this
sensation equally whether he is eating solids or liquids. The diagnosis that would most likely account for
these symptoms is
A.
achalasia
B.
lower esophageal (Schatzki) ring
C.
esophageal carcinoma
D.
Zenker’s diverticulum
E.
*diffuse esophageal spasm
506. A 47-yr-old alcoholic presents with haematemesis and melaena. He is stable after being transfused.
Choose the single most likely investigation from the list of options.
A.
Oesophageal manometry
B.
Motility studies
C.
Ultrasound scan
D.
Barium meal
E.
*Upper GI endoscopy
507. A 49-year-old cigarette smoker complains of burning epigastric pain, well localized, nonradiating.
The pain is partially relieved by eating. There is no weight loss. He has not used nonsteroidal
antiinflammatory agents. The pain has gradually worsened over several months. The most sensitive way to
make a specific diagnosis is
A.
Barium x-ray
B.
*Endoscopy
C.
Serologic test for Helicobacter pylori
D.
Serum gastrin
E.
Ultrasound
508. A 49-yr-old alcoholic presents with haematemesis and melaena. Choose the single most likely
investigation from the list of options above.
A.
Oesophageal manometry
B.
Motility studies
C.
Ultrasound scan
D.
Barium meal
E.
*Upper GI endoscopy
509. A 49-yr-old alcoholic presents with haematemesis and melaena. He is stable after being transfused.
Choose the single most likely investigation from the list of options above.
A.
CT scan abdomen
B.
Motility studies
C.
Barium enema
D.
*Upper GI endoscopy
E.
Ultrasound scan
510. A 50-year-old black male with a history of alcohol and tobacco abuse has complained of difficulty
swallowing solid food for the past 2 months. More recently, swallowing fluids has also become a problem.
He has noted black, tarry stools on occasion. The patient has lost 10 lb. Which of the following statements is
correct?
A.
The patient’s prognosis is good
B.
*Barium contrast study is indicated
C.
The most likely diagnosis is peptic ulcer disease
D.
The patient has achalasia
E.
Chest X-ray
511. A 50-year-old black male with a history of alcohol and tobacco abuse has complained of difficulty
swallowing solid food for the past 2 months. He has noted black, tarry stools on occasion. Which of the
following statements is correct?
A.
The patient’s prognosis is good
B.
*Barium contrast study is indicated
C.
The most likely diagnosis is peptic ulcer disease
D.
The patient has achalasia
E.
Chest X-ray
512. A 51-year-old black male with a history of alcohol and tobacco abuse has complained of difficulty
swallowing solid food for the past 2 months. Which of the following statements is correct?
A.
The patient’s prognosis is good
B.
*Barium contrast study is indicated
C.
The most likely diagnosis is peptic ulcer disease
D.
The patient has achalasia
E.
Chest X-ray
513. A 51-year-old Hispanic man is evaluated because of severe epigastric pain, nausea, and vomiting.
Abdominal examination discloses epigastric tenderness. On rectal examination, a stool specimen is positive
for occult blood. Laboratory studies: Hemoglobin 15.3 g/dL Leukocyte count 10,400/?L Serum calcium 9.8
mg/dL Serum electrolytes: Sodium 136 meq/L Potassium 3.1 meq/L Chloride 90 meq/L Bicarbonate 35
meq/L Serum aspartate aminotransferase 104 U/L Serum alanine aminotransferase 78 U/L Serum alkaline
phosphatase 216 U/L Serum lipase 14 U/L Serum total bilirubin 0.4 mg/dL Serum albumin 3.7 g/dL A CT
scan of the abdomen shows thickened gastric mucosa. Upper endoscopy discloses duodenal ulcerations
extending into the second portion of the duodenum and thickened gastric folds in the fundus. A fasting
gastrin level is 520 pg/mL Basal acid output is markedly elevated. In addition to starting a proton pump
inhibitor, which of the following should be done next?
A.
Endoscopic ultrasonography
B.
*Octreotide scan
C.
Exploratory laparotomy
D.
Mesenteric venous sampling for gastrin
E.
Pentagastrin stimulation of maximal acid output
514. A 51-year-old man who has complained of heartburn over the past 12 years undergoes endoscopy.
The endoscopist notes a change in the appearance of the epithelium in the distal esophagus. Biopsy reveals
erosion of the squamous mucosa and replacement with metaplastic columnar epithelium. Which is most
likely diagnosis?
A.
Gallstones
B.
Reflux - gastritis
C.
Diffuse esophageal spasm
D.
Gastritis of type A
E.
*Barrett’s esophagus
515. A 53-year-old male comes to your local health center while on a trip to the lower 48 states. He states
that he always feels "full" and has been losing weight recently. On physical exam, he is a cachetic-appearing
man who appears much older than stated age. What physical exam finding would support the most likely
diagnosis?
A.
Point tenderness in the abdominal epigastrum.
B.
Positive Murphy's sign.
C.
Palpable splenomegaly.
D.
*Enlarged supraclavicular lymph nodes.
E.
Enlarged axillary lymph nodes.
516. A 53-year-old man with a history of recurrent alcoholic pancreatitis develops hematemesis and
melena. On physical examination, pulse rate is 136/min, and blood pressure is 82 mm Hg systolic.
Abdominal examination is normal. Hemoglobin is 8.2 g/dL, INR is 1.1, and liver chemistry studies are
normal. Nasogastric lavage shows fresh blood. The patient is stabilized with intravenous fluids and packed
red blood cells, following which upper endoscopy is performed. Endoscopic findings include varices of the
gastric fundus and cardia and fresh blood in the stomach. No esophageal varices or other sources of bleeding
are identified. Which of the following is most appropriate at this time?
A.
Repeat upper endoscopy with banding of varices
B.
Transjugular intrahepatic portosystemic shunt (TIPS)
C.
*Splenectomy
D.
Administration of ?-blockers
E.
Surgical portacaval shunt
517. A 55-year-old male smoker presents with burning epigastric pain several hours after a meal, which is
relieved by antacids. Upper gastrointestinal endoscopy discloses an ulcer with a well-demarcated border at
the duodenal bulb. Histologic examination of a biopsy specimen of the ulcer crater reveals eosinophilic
necrosis with surrounding fibrosis without evidence of malignancy. Furthermore, analysis of a histologic
section involving the gastric mucosa reveals invasion with a gram-negative rod. Which of the following is
the most appropriate therapy?
A.
Mylanta
B.
Ranitidine
C.
Omeprazole
D.
Bismuth subsalicylate plus metronidazole
E.
*Omeprazole plus clarithromycin plus
518. A 55-year-old woman with chronic alcoholism has a 2-day history of nausea, vomiting, and upper
abdominal discomfort. The patient drinks about 6 or 7 alcoholic beverages each day. She denies using
prescription medications but recently has been taking several acetaminophen tablets each day for a right
rotator cuff injury suffered 1 week ago. Physical examination shows epigastric and right upper quadrant
abdominal tenderness to deep palpation. Laboratory studies: Complete blood count Normal INR 1.4 Serum
creatinine 1.9 mg/dL Serum alkaline phosphatase 175 U/L Serum aspartate aminotransferase 15,750 U/L
Serum alanine aminotransferase 12,500 U/L Serum total bilirubin 2.5 mg/dL Serum albumin 3.5 g/dL
Administration of which of the following is most appropriate at this time?
A.
An antiemetic agent
B.
A corticosteroid
C.
Pentoxifylline
D.
*N-acetylcysteine
E.
Multivitamins
519. A 55-yr-old obese woman complains of severe epigastric pain radiating to his back. The pain is
relieved by eating and is worse at night. Choose the single most likely investigation from the list of options
above.
A.
Rectal examination
B.
USG abdomen
C.
Barium meal
D.
Sigmoidoscopy
E.
*Upper GI endoscopy
520. A 57-year-old man is evaluated because of epigastric pain. Upper endoscopy discloses a gastric ulcer,
and endoscopic biopsy specimens of the ulcer show gastric adenocarcinoma. Which of the following is most
likely to be associated etiologically with this patients gastric adenocarcinoma?
A.
Gastric hyperplastic polyps
B.
Tyrosine kinase activity
C.
*Helicobacter pylori infection
D.
Cytomegalovirus infection
E.
Amyloid deposition
521. A 59-yr-old obese woman complains of severe epigastric pain radiating to his back. The pain is
relieved by eating and is worse at night. Choose the single most likely investigation from the list of options
above.
A.
Rectal examination
B.
USG abdomen
C.
Barium meal
D.
Sigmoidoscopy
E.
*Upper GI endoscopy
522. A 59-yr-old obese woman complains of severe epigastric pain radiating to his back. The pain is
relieved by eating and is worse at night. Choose the single most likely investigation from the list of options
above.
A.
Rectal examination
B.
USG abdomen
C.
Barium meal
D.
Sigmoidoscopy
E.
*Upper GI endoscopy
523. A 59-yr-old with 9-week history of dysphagia undergoes a barium swallow. It shows bird peak
deformity of the distal oesophagus with proximal dilatation. Choose the single most likely investigation from
the list of options.
A.
Barium swallow
B.
Chest radiography
C.
CT scan chest
D.
None of above
E.
*Motility studies
524. A 61-year-old white man has a 2-month history of dysphagia for solid foods. He has lost 6.7 kg
during this time. The patient has chronic heartburn that is relieved by antacids. He also has hypertension for
which he takes atenolol and diltiazem. Which of the following is the most likely diagnosis
A.
Drug-induced esophagitis
B.
Esophageal web
C.
diffuse esophageal spasm
D.
achalasia
E.
*Esophageal adenocarcinoma
525. A 62-yr-old man complains of chronic abdominal pain around the umbilicus. She often suffers from
indigestion, abdominal pain and vomiting. She had a thyroidectomy 12 yr ago and is on replacement regime.
A barium meal shows an absence of mucosal folds. Choose the single most likely diagnosis
A.
Gastric ulcer
B.
Non-ulcer dyspepsia
C.
Basal pneumonia
D.
A.Chronic cholecystitis
E.
*Gastric atrophy
526. A 63-year-old white man is hospitalized because of a 1-month history of severe epigastric pain. The
pain initially occurred 4 weeks ago but then abated, and the patient did not seek medical attention. However,
the pain recurred last week and is more severe now than initially. It is described as stabbing with radiation to
the back, is worse after eating, and is not relieved by ranitidine. The patient also has nausea and vomiting.
He denies fever, chills, weight loss, and early satiety. Medical and family history are noncontributory, and he
does not drink alcoholic beverages. On physical examination, temperature is 36.6 C (97.9 F), pulse rate is
110/min, and blood pressure is 146/92mm Hg. There is no jaundice. Abdominal examination shows
epigastric tenderness. The liver and spleen are not enlarged. Laboratory studies: Hematocrit 44% Leukocyte
count 7000/?L (normal differential) Serum calcium 8.6 mg/dL Serum triglycerides 256 mg/dL Serum total
bilirubin 0.7 mg/dL Serum albumin 4.0 g/dL Serum alkaline phosphatase 106 U/L Serum aspartate
aminotransferase 19 U/L Serum alanine aminotransferase 30 U/L Serum amylase 123 U/L Serum lipase 852
U/L A CT scan of the abdomen shows an enlarged pancreatic head with peripancreatic inflammation and a
small amount of fluid in the perirenal space. The body of the pancreas is atrophic with dilation of the main
pancreatic duct to 5 mm. The splenic vein is not seen. A 7.2 X 7.8-cm collection of fluid and debris
surrounded by a thin rim is present at the junction of the pancreatic head and neck. The patient responds
rapidly to intravenous fluids and narcotics. He tolerates a low-fat diet, although he continues to have some
bloating and nausea. Which of the following is the most appropriate management at this time?
A.
Refer the patient for surgical drainage of the cyst
B.
Refer the patient for surgical excision of the cyst
C.
Perform fine-needle aspiration of the cyst
D.
*Repeat the CT scan in 6 weeks
E.
Begin parenteral nutrition
527. A 66-year-old man has a 6-month history of dyspepsia without dysphagia or weight loss. Although
his symptoms previously responded to antacids and over-the-counter H2-receptor antagonists, these
medications have been ineffective for the past 2 months. Physical examination and complete blood count are
normal. Which of the following is the most appropriate next step in the management of this patient?
A.
Serologic testing for Helicobacter pylori
B.
CT scan of the abdomen
C.
*Upper endoscopy
D.
Radionuclide gastric emptying study
E.
An 8-week trial of a proton pump inhibitor
528. A 66-yr-old alcoholic and heavy smoker presents with a 5-month history of progressive dysphagia
and weight loss. Choose the single most likely investigation from the list of options above.
A.
Oesophageal manometry
B.
Chest radiography
C.
Motility studies
D.
CT scan chest
E.
*Barium swallow
529. A 69-yr-old man has been on medication for gastric ulcer for 12-weeks. A repeat upper gastrograffin
series shows moderate shrinkage of the ulcer. Choose the single most likely investigation from the list of
options above.
A.
Oesophageal manometry
B.
Barium swallow
C.
*Upper GI endoscopy and biopsy
D.
CT scan chest
E.
Oesophageal pH testing
530. A 71-yr-old man has been on medication for gastric ulcer for 14-weeks. A repeat upper gastrograffin
series shows moderate shrinkage of the ulcer. Choose the single most likely investigation from the list of
options above.
A.
Barium swallow
B.
Chest radiography
C.
Oesophageal manometry
D.
Upper GI endoscopy and biopsy
E.
*CT scan chest
531. A 74-yr-old alcoholic and heavy smoker presents with a 4-month history of progressive dysphagia
and weight loss. Choose the single most likely investigation from the list of options above.
A.
Oesophageal manometry
B.
Chest radiography
C.
Motility studies
D.
CT scan chest
E.
*Barium swallow
532. A 74-yr-old alcoholic and heavy smoker presents with a 4-month history of progressive dysphagia
and weight loss. Choose the single most likely investigation from the list of options above.
A.
Oesophageal manometry
B.
Chest radiography
C.
Motility studies
D.
CT scan chest
E.
*Barium swallow
533. A 74-yr-old alcoholic and heavy smoker presents with a 4-month history of progressive dysphagia
and weight loss. Choose the single most likely investigation from the list of options.
A.
Oesophageal manometry
B.
Chest radiography
C.
Motility studies
D.
CT scan chest
E.
*Barium swallow
534. A 74-yr-old alcoholic and heavy smoker presents with a 4-month history of progressive dysphagia
and weight loss. Choose the single most likely investigation from the list of options.
A.
Oesophageal manometry
B.
Chest radiography
C.
Motility studies
D.
CT scan chest
E.
*Barium swallow
535. A 75-year-old woman is evaluated because of dyspepsia. The patient has advanced rheumatoid
arthritis treated with diclofenac and several disease-modifying anti-rheumatic drugs. Upper endoscopy
reveals a 1-cm gastric ulcer. Biopsy specimens of the ulcer show no evidence of malignancy, and tests for
Helicobacter pylori are negative. The patient refuses to stop taking the diclofenac because this is the only
drug that helps her function despite the arthritis. Assuming that diclofenac is continued, which of the
following is the most appropriate treatment for healing of this patient’s gastric ulcer?
A.
Sucralfate
B.
Misoprostol
C.
Ranitidine
D.
*A proton pump inhibitor
E.
Partial gastrectomy
536. A 75-year-old woman with a history of aspirin-induced gastritis 5 years ago now has severe knee and
hip pain that is thought to be due to osteoarthritis. She requires treatment with nonsteroidal antiinflammatory agents. Which of the following agents would be most helpful for prophylaxis against recurrent
gastrointestinal bleeding?
A.
Omeprazole
B.
*Misoprostol
C.
Nizatidine
D.
Sucralfate
E.
Atropine
537. A middle-aged man complains of persistent abdominal pain. He says antacids and food help him with
the pain but it will not go away. Sometimes he feels sick and vomits and that helps. Choose the single most
likely diagnosis
A.
Gallstones
B.
Crohn's disease
C.
Duodenitis
D.
Carcinoma of tail of pancreas
E.
*Gastric ulcer
538. A young lawyer presents with stomach pain. Over the past year he has gained weight because eating
relieves his hunger pangs, but now the pain are almost constant. He does not have a fever or jaundice and a
physical examination shows nothing abnormal. Choose the most likely diagnosis
A.
Carcinoma colon
B.
Chronic pancreatitis
C.
Chronic cholecystitis
D.
Acute pancreatitis
E.
*Hiatus hernia
539. An otherwise healthy 24-year-old woman has a 5-month history of epigastric discomfort and
heartburn. Symptoms are usually exacerbated postprandially, especially after eating spicy foods. The patient
denies dysphagia, weight loss, and decreased appetite. He has an active lifestyle and takes no medications.
Physical examination is normal except for mild epigastric tenderness. Routine laboratory studies are normal.
Which of the following is most appropriate at this time?
A.
Esophageal manometry
B.
Ambulatory 24-hour esophageal pH monitoring
C.
Barium swallow
D.
None of above
E.
*Trial of acid suppressive therapy
540. Four months ago, a 36-year-old man with a peptic ulcer underwent a Billroth II anastomosis,
antrectomy, vagotomy, and gastrojejunostomy. He now returns for evaluation of a stomal (anastomotic)
ulcer. Fasting serum gastrin level is 350 ng/L; 5 min after the intravenous infusion of secretin the serum
gastrin level is 200 ng/L. The man should be advised that the most appropriate treatment for his condition is
A.
total vagotomy
B.
total gastrectomy
C.
*resection of the distal antrum attached to the duodenal
D.
laparotomy to search for a gastrin-producing tumor
E.
medical therapy with liquid antacids
541. A 19-yr-old man presents with sudden severe upper abdominal pain after being tackled during a
rugby practice. He was recently diagnosed with glandular fever. choose the single most likely diagnosis from
the list of options above.
A.
*Pancreatic pseudocyst
B.
Splenic rupture
C.
Acute pancreatitis
D.
Sigmoid volvulus
E.
Haemolytic uraemic syndrome
542. A 25 yrs women presents with bloody diarrhea & is diagnosed as a case of Ulcerative colitis. Which
of the following condition is not associated :
A.
Iritis
B.
Ankylosing spondylitis
C.
Sclerosing cholengitis
D.
Erythema nodosum
E.
*Pancreatitis
543. A 25-yr-old man who was driving was involved with a high speed collision. He was wearing his seat
belt and now complains of upper abdominal pain. His CXR is normal. Choose the single most likely
investigation from the list of options above.
A.
*Serum amylase
B.
Serum electrolytes
C.
Anti-DNA antibodies
D.
Arterial blood gases
E.
Jejunal biopsy
544. A 26-year-old Hispanic woman who is 12 weeks pregnant has had recurrent episodes of biliary colic
for the past 6 months. The episodes are now increasing in frequency, and one episode was associated with
mild pancreatitis. Abdominal ultrasonography shows multiple gallstones. Results of routine laboratory
studies are normal. When is the most appropriate time for this patient to undergo laparoscopic
cholecystectomy?
A.
Immediately
B.
*During the second trimester
C.
During the third trimester
D.
Post partum
E.
During the first trimester
545. A 26-year-old woman with ulcerative colitis has been taking prednisone for the past year. Each time
the prednisone is tapered below 20 mg/d, her symptoms return. She is subsequently started on 6mercaptopurine, 50 mg/d. Three days after beginning the new drug, she develops worsening abdominal pain
with radiation to her back. She does not have a rash. Her leukocyte count is 3200/?L. Which of the following
is the most likely cause of this patients new symptoms?
A.
Flare of ulcerative colitis
B.
Pancreatitis due to continuation of prednisone
C.
*Pancreatitis due to initiation of 6-mercaptopurine
D.
An abdominal and psoas abscess secondary to 6-mercaptopurine-induced neutropenia
E.
An allergic reaction to 6-mercaptopurine
546. A 27-year-old male presents to the health center for right upper quadrant abdominal pain, generalized
pruritus, and jaundice for 3 days. He states that the pain came on gradually and awoke him early on the
morning of presentation. His past medical history is pertinent for ulcerative colitis, although he has not taken
any medication in 4 years. His temperature is 102.5°F (39.2°C) and physical exam shows pain in the right
subcostal region with deep inspiration, and generalized jaundice. What is the most serious complication of
the most likely diagnosis?
A.
Perforation of rectum.
B.
Perforation of stomach wall.
C.
Infarction of small bowel.
D.
*Cholangiocarcinoma.
E.
Pancreatic pseudocyst formation.
547. A 27-year-old man with AIDS develops right-sided abdominal pain and nausea. His last CD4 cell
count was 180/L. On physical examination, he is afebrile. Abdominal examination discloses hepatomegaly
and moderate epigastric and right upper quadrant tenderness. Laboratory studies: Serum aspartate
aminotransferase 57 U/L Serum alanine aminotransferase 59 U/L Serum alkaline phosphatase 590 U/L
Serum total bilirubin 2.8 mg/dL Serum albumin 3.3 g/dL Abdominal ultrasonography shows common bile
duct dilatation to 12 mm. No stones are seen in the common bile duct or gallbladder. Which of the following
diagnostic studies should be done next?
A.
Liver biopsy
B.
CT scan of the abdomen
C.
*Endoscopic retrograde cholangiopancreatography
D.
Percutaneous transhepatic cholangiography
E.
Ultrasound examination
548. A 27-year-old woman is hospitalized because of acute-onset epigastric pain. The pain is worse after
eating, is nonradiating, and is described as a dull ache that increases over a 30-minute period before reaching
a plateau and then lasts for 2 to 3 hours before abating. This pain pattern has recurred eight times in the last
72 hours and is associated with nausea and vomiting but not with fever or chills. The patients eyes have been
yellow for the past 24 hours. Medical history is noncontributory. On physical examination, temperature is
37.3 C (99.2 F), pulse rate is 110/min, and blood pressure is 132/90 mm Hg. The sclerae are icteric.
Abdominal examination discloses epigastric tenderness without hepatosplenomegaly, peritoneal signs, or
Murphy’s sign. Laboratory studies: Leukocyte count 10,000/ ? L (normal differential) Serum total bilirubin
10.4 mg/dL Serum direct bilirubin 5.8 mg/dL Serum alkaline phosphatase 446 U/L Serum aspartate
aminotransferase 361 U/L Serum alanine aminotransferase 389 U/L Serum y-glutamyltransferase 485 U/L
(normal: 2-30 U/L) Serum amylase 42 U/L Serum lipase 60 U/L Abdominal ultrasonography shows multiple
stones in the gallbladder, a common bile duct measuring 5 mm in diameter, a distal common hepatic duct
measuring 9 mm in diameter, dilated bilateral intrahepatic ducts, and no stones visualized in the bile duct.
The gallbladder wall is not thickened. There is no pericholecystic fluid or sonographic Murphy’s sign (a
positive Murphy’s sign that is elicited by palpating the abdomen with an ultrasound transducer). Which of
the following is the most likely explanation for this patient’s jaundice?
A.
Choledocholithiasis
B.
Acute cholecystitis
C.
*Mirizzi’s syndrome
D.
Ascending cholangitis
E.
Biliary duct dyskinesia
549. A 29-year-old woman is evaluated because of vague epigastric and right upper quadrant abdominal
pain that is occasionally worse after eating. Initial laboratory studies, including liver chemistry tests and
measurement of serum amylase and lipase, are normal. Abdominal ultrasonography suggests the presence of
focal dilatation of the common bile duct. No stones are seen in the duct or gallbladder. Magnetic resonance
cholangiopancreatography (MRCP) shows an area of dilated common bile duct measuring 2 cm in length by
1 .2 cm in diameter. The bile duct diameter above and below this dilated area measures 4 to 5 mm. MRCP
findings are compatible with a choledochal cyst. There is no intrahepatic ductal dilatation. Which of the
following is the most appropriate management at this time?
A.
Endoscopic retrograde cholangiopancreatography with papillotomy
B.
*Surgical resection and biliary tree reconstruction
C.
Administration of ursodeoxycholic acid (ursodial) and annual follow-up MRCP
D.
No further diagnostic studies or treatment
E.
CT scan of the abdomen
550. A 32-yr-old obese female presents with fever, vomiting and right upper quadrant abdominal pain.
The pain is worse on inspiration. Choose the single most likely investigation from the list of options above.
A.
Barium enema
B.
Abdominal X-ray
C.
Upright Chest X-ray
D.
Sigmoidoscopy and biopsy
E.
*Abdominal USG
551. A 35-year-old alcoholic male is admitted for nausea, vomiting, and abdominal pain that radiates to
the back. The laboratory value that suggests a poor prognosis in this patient is
A.
Elevated serum lipase
B.
Elevated serum amylase
C.
*Leukocytosis of 20,000/µm
D.
Diastolic blood pressure greater than 90 mmHg
E.
anemia less than 90 g/l
552. A 35-year-old woman complains of right upper quadrant pain, which occurs after she eats a large
meal. Occasionally the episodes are accompanied by nausea and vomiting. A plain x-ray of the abdomen
discloses gallstones. Ultrasonography reveals gallstones and a normalsized common bile duct. The patient’s
blood chemistry and CBC are normal. The most therapeutic maneuver at this time would be
A.
observation
B.
*laparoscopic cholecystectomy
C.
ursodeoxycholic acid
D.
shock wave lithotripsy
E.
ursodeoxycholic acid and shock wave lithotripsy
553. A 35-yr-old obese female presents with fever, vomiting and right upper quadrant abdominal pain.
The pain is worse on inspiration. Choose the single most likely investigation from the list of options
A.
Upper GI endoscopy
B.
Barium meal
C.
*Ultrasound scan
D.
Supine abdominal X-ray
E.
Oesophageal manometry
554. A 35-yr-old obese female presents with fever, vomiting and right upper quadrant abdominal pain.
The pain is worse on inspiration. Choose the single most likely investigation from the list of options above.
A.
Sigmoidoscopy and biopsy
B.
Barium enema
C.
Abdominal X-ray
D.
Upright Chest X-ray
E.
*Abdominal USG
555. A 37-year-old man with chronic alcoholism is admitted to the hospital with acute pancreatitis. On the
third hospital day sudden, complete blindness develops in the left eye. The most likely explanation is
A.
alcohol withdrawal symptoms
B.
transient ischemic attack (transient monocular blindness)
C.
occlusion of the retinal vein
D.
acute glaucoma
E.
*Purtscher’s retinopathy
556. A 38-yr-old investment broker comes to A&E with severe upper abdominal pain. His breath smells of
alcohol. He is pale and sweaty. On physical examination you find boardlike rigidity. Choose the single most
likely investigation from the list of options above
A.
*Acute pancreatitis
B.
Gastric carcinoma
C.
Acute appendicitis
D.
Peptic ulcer
E.
Chronic cholecystitis
557. A 39-yr-old marketing executive presents with acute epigastric pain. The pain is continuous and it
has been increasing in intensity over the past day. It radiates to the right hypochondrium. Choose the single
most likely investigation from the list of options above
A.
*Acute cholecystitis
B.
Acute pancreatitis
C.
Duodenal peptic ulcer
D.
Oesophagitis
E.
Duodenitis
558. A 40-year-old white female complains of pruritus. She has an elevated alkaline phosphatase and
positive antimitochondrial antibody test. What is the most likely disease?
A.
*Primary biliary cirrhosis
B.
Sclerosing cholangitis
C.
Anaerobic liver abscess
D.
Hepatoma
E.
Hepatitis C
559. A 40-year-old white female complains of pruritus. She has an elevated alkaline phosphatase and
positive antimitochondrial antibody test. What is the most likely disease?
A.
*Primary biliary cirrhosis
B.
Sclerosing cholangitis
C.
Anaerobic liver abscess
D.
Hepatitis D
E.
Hemochromatosis
560. A 40-year-old white female complains of pruritus. She has an elevated alkaline phosphatase and
positive antimitochondrial antibody test. What is the most likely disease?
A.
*Primary biliary cirrhosis
B.
Hepatoma
C.
Hepatitis C
D.
Hepatitis D
E.
Hemochromatosis
561. A 40-year-old woman comes to the physician because of fever and chills, jaundice, and right upper
abdominal pain radiating to the shoulder for 24 hours. At present, the patient's temperature is 39° C, blood
pressure is 100/60 mm Hg, pulse is 110/min, and respirations are 20/min. She is admitted for further
diagnostic evaluation. Serum chemistry studies show: Alkaline phosphatase 800 U/L, Bilirubin: Total 141
mEq/L. White blood cell count is 12,000/mm3, with 70% neutrophils. Which of the following is the most
likely diagnosis?
A.
Acute cholecystitis
B.
Acute hepatitis
C.
Acute pancreatitis
D.
*Choledocholithiasis with cholangitis
E.
Cystic duct syndrome
562. A 40-yr-old man presents with acute abdominal pain that radiates through to the back. The pain is
severe and causes him to feel sick and vomit repeatedly. On physical examination you find the abdomen to
be tender. His serum amylase is five times greater than normal. Choose the single most likely diagnosis
A.
Gastric atrophy
B.
Non-ulcer dyspepsia
C.
Basal pneumonia
D.
Gastric ulcer
E.
*Acute pancreatitis
563. A 40-yr-old patient presents with acute epigastric pain. The pain is continuous and it has been
increasing in intensity over the past day. It radiates to the right hypochondrium. Choose the single most
likely diagnosis
A.
Gastric atrophy
B.
Non-ulcer dyspepsia
C.
Basal pneumonia
D.
Gastric ulcer
E.
*Acute cholecystitis
564. A 41-year-old morbidly obese female comes to the emergency department with colicky abdominal
pain in her right upper abdomen. She complains that this is similar to, yet more severe than, the pain that
often occurs after meals for the past 4 months. Her past medical history is positive for diabetes mellitus type
2, hypertension, hyperlipidemia, and smoking. On physical exam, her temperature is 100.5°F (38.1°C) and
her sclera appear mildly icteric. What imaging modality may be limited in this patient?
A.
MRCP.
B.
CT scan.
C.
*Ultrasonography.
D.
Esophogastroduodenoscopy (EGD).
E.
HIDA scan.
565. A 45 -year-old woman for 2 year complained of attacks of right subcostal pain after fatty meal. Last 2
weeks the attacks have repeated every day and become more painful. What diagnostic study would you
recommend?
A.
Urine analysis
B.
Ultrasound study of the pancreas
C.
X-ray examination of the gastrointestinal tract
D.
Liver function tests
E.
*Ultrasound examination of the gallbladder
566. A 45-year-old man is evaluated because of fatigue and pruritus of 1 years duration. The pruritus is
generalized and is not associated with a rash. He has no allergies, and antihistamines have not improved the
itching. The patient has ulcerative colitis that is well controlled with mesalamine. Physical examination is
normal except for excoriations on his extremities and back from scratching. Laboratory studies: Complete
blood count Normal Serum alkaline phosphatase 760 U/L Serum aspartate aminotransferase 15 U/L Serum
alanine aminotransferase 20 U/L Serum y-glutamyltransferase 350 U/L Serum total bilirubin 2.2 mg/dL
Serum direct bilirubin 1.5 mg/dL Serum total protein 6.4 g/dL Serum albumin 4.0 g/dL Which of the
following tests is most likely to establish the diagnosis?
A.
CT scan of the abdomen
B.
Hepatitis C virus RNA (HCV RNA)
C.
Hepatitis B surface antigen (HBSAg)
D.
*Endoscopic retrograde cholangiopancreatography
E.
Smooth muscle antibody titer
567. A 45-year-old woman presents with a yellowish discol¬oration of her body, first noted by her
husband last week. Since then, she has been having severe itching at night, which disturbs her sleep, and
complains of a tin¬gling sensation of her hands and feet. On examination, xanthelasmas are seen around the
eyes. The liver is firmly palpable 4 cm below the costal margin. Scratch marks are noted on her abdomen
and limbs. Clubbing is observed in all the digits. Serum creatinine is 0.9 mg/dL, bilirubin is 2.3 mg/dL,
albumin is 4.3 g/dL, ala¬nine aminotransferase is 92 U/L, and alkaline phosphatase is 410 U/L. Which of the
following is the most appropriate next step in diagnosis?
A.
*Anti-mitochondrial antibody assay
B.
Anti-smooth muscle antibody assay
C.
Endoscopic retrograde cholangiopancreatography (ERCP)
D.
Serum protein electrophoresis
E.
Technetium (99mTc) liver-spleen scan
568. A 45-yr-old woman presents with upper abdominal pain and obstructive jaundice. The gallbladder is
not palpable clinically. USG shows gallstones and a dilated common bile duct. Choose the single most likely
investigation from the list of options.
A.
ERCP
B.
*Percutaneous transhepatic Cholangiography
C.
Tc99 iodide scan
D.
CTscan
E.
Oral cholecystogram
569. A 45-yr-old woman presents with upper abdominal pain and obstructive jaundice. The gallbladder is
not palpable clinically. USG shows gallstones and a dilated commo